1) A catalog retailer has identified African Americans professionals between the ages of 35 and 45 as the group of customers    within its larger market as a potential market for its products. The retailer has identified a:  

@ market segment  
  marketing mix  
  product mix  
  mass market  
  market aggregation  

2) A catalog retailer has identified African American professionals between the ages of 35 and 45 as the market segment
   toward which it will direct its marketing efforts. This group has become its:  

  market position  
@ target market  
  mass market  
  market aggregation  
  CRM  
 
3) A consumer who is considering purchasing a house would undoubtedly use:  

  habitual decision making  
  stimulus generalization  
  operant decision making  
  limited problem solving  
@ extended problem solving

4) A market position refers to:  

@ how the organization wants its products perceived in relation to the competition's products  
    
5) A retailer that was interested in customer relationship management (CRM) would:  

  encourage its employees to spend no more than three minutes with each customer  
  assume its customers will never visit the store again  
  have a selling orientation  
@ learn the names of its customers and what products they prefer  
  none of the above 

6) A _____ orientation is a management philosophy that focuses on ways to satisfy customers' needs and wants.  

@ consumer  
  promotion  
  selling  
  production  
  New Era  

================

According to your instructor, what is the difference between marketing and sales  

  Sales is a subset of marketing  
  Sales involves exchanges, marketing does not involve exchanges.  
  Sales is primarily person-to-person, marketing is primarily mass-media communication.  
@ Sales focuses on creating transactions, marketing focuses on making it possible for transactions to happen.  

7) According to your instructor, which of the following is a major trend driving the evolution of marketing  

  The back to nature movement of the late 18th century.  
@ The growth of the world population.  
  The recent downturn in the stock market associated with the failure of many dot-com companies.  
  The collapse of the soviet union in 1989.

8) According to the instructor, the "law of large numbers" means...  

  The more people in a market, the more similar they become.  
  You must have a large number of customers in order to develop effective marketing strategies.  
  You have to develop more than one marketing plan for a very large group.  
@ You can't say for sure what any one person will do, but you can predict the behavior of groups of people. 

9) According to the material discussed in class, which of these is a way that marketing affects your life?  
  We are consumers of products. Understanding marketing can help us become smarter consumers.  
  You are a product and understanding marketing will help you position yourself in the eyes of a potential employer.  
  Understanding how buying and selling work can improve your relationships with other people.  
@ All of the above are ways marketing affects your life.  

================

10) Amazon.com where Marta can order books online and Waldenbooks, a store at the mall where she can purchase the same books,     are both examples of:  

  buying centers  
  virtual reality malls  
  demand centers  
@ marketplaces  
  market convergence

=================

11) An advertisement featuring "no money down" and "free delivery" is probably targeted at consumers in the ____________     stage of the consumer buying process?  

  Need recognition  
  Identify alternatives  
  Evaluate alternatives  
@ Decision  

12) An advertisement featuring Barry Bonds talking about life insurance would be most effective for baseball fans who are in      the ___________ stage of the consumer buying process.  

  Need Recognition  
  Identify Alternatives  
@ Evaluate alternatives  
  Postpurchase behavior 
     
13) An individual at a crafts fair who makes funnel cakes and sprinkles them with powdered sugar creates _____ utility.  

@ form  
  time  
  possession  
  marketing  
  creation  
  
14) An organic farmer has identified three distinct groups for his products-vegetarians, people who are concerned about chemicals in their food, and people who consider themselves innovators and trendsetters. These three groups identified within a larger group are examples of:  
 
@ market segments  
  marketing mixes  
  product mixes  
  mass markets  
  market aggregations  

15) A(n) _____ consists of all the consumers who share a common need that can be satisfied by a specific good or service and
    who have the resources, willingness, and authority to make the exchange.  

  demand center  
  mass market segment  
  exchange location  
  buying center  
@ market  

  A(n) _____ occurs when Ronda Albers trades Max Lynch three hours of child care for his three-year-old daughter for fixing the hole in her porch roof.  

  monetary trade  
  buying center  
@ exchange  
  reciprocal benefitization  
  virtual trade  
 
==================

16) By definition, a consumer is a person...  

@ Who purchases products for personal consumption.
  Who purchases products for resale to other people.  
  Who purchases products for use in a business.  
  Who creates products for personal consumption.  

17) Before the 1950s, marketing was: 
 
  based on a patronage system  
@ basically a means of making production more efficient  
  concerned more with customers than with products  
  using customer relationship management extensively  
  driven by an emphasis on international sales  

================

18) Companies that have a _____ orientation tend to be more successful at making one-time sales than they are at building         repeat business.  

  consumer  
  marketing  
@ selling  
  production  
  New Era 

  Consumers often rely on decision guidelines when weighing the claims companies make. These rules are referred to as:  
 
  synergistic criteria  
  core competencies  
  buying plans  
@ heuristics  
  analogies  
 


19) Conveniently located vending machines around your college or university campus make it easy for you to buy a soda and a
    snack between classes. This is an example of the _____ element of the marketing mix.  

  production  
@ place  
  price  
  product  
  promotion 

=================

20) Donna wants to buy a new coat. During the _______________ stage of her purchase decision, she will ask her friends to 
    recommend a store and/or a style of coat. She will search the newspapers for coat sales, and she will visit some nearby 
    stores to see what is available in her price range.  
 
  product evaluation  
  situational analysis  
  need recognition  
  problem screening  
@ identify alternatives 

=================

21) Employees at the Six Flags amusement parks are told to treat everyone who purchased a ticket to get into the parks as a  
    guest. These guests are actually:  

  producers  
@ consumers  
  target marketers  
  distributors  
  service providers 

22) Every morning at 10 a.m., Horatio buys a bottle of Diet Coke and a Three Musketeers candy bar from a vending machine that
    is located in a room near his office cubicle. Because there is no thought or effort put into this purchase decision, it 
    would be an example of:  

@ habitual decision making  
  stimulus generalization  
  nonrandom decision making  
  limited problem solving  
  extended problem solving 

=========================

2) How do fashion marketers stimulate the need-recognition phase of the consumer buying process?  

  By reducing prices on outdated merchandise in order to encourage fashion conscious customers to stock up on merchandise.  
  By creating "timeless" fashions encouraging fashion-conscious consumers to gradually accumulate merchandise over the long-    term.  
@ By creating new "looks" every season, making fashion-conscious customers dissatisfied with old, outdated merchandise.  
  By stressing the quality of the workmanship and the durability of the merchandise in order to entice value
  conscious
  customers to purchase a particular brand of fashionable merchandise.  
    
3) How does habitual decision making differ from extended problem solving  

  Extended problem solving uses behavioral learning, and habitual decision making uses cognitive learning.  
  Consumers engaged in extended problem solving perceive less risk than consumers involved in habitual decision making.  
@ The level of involvement for habitual decision making is lower than for extended problem solving.  
  Consumers who are buying an anniversary cruise would most likely use habitual decision making because the level of     involvement is higher than with extended problem solving.  
  All of the above statements about how habitual decision making differs from extended problem solving are true.  

================

IBM manager Celia Moore coordinates IBM's long-standing "Reinventing Education" program, which involves intensive research into how educational institutions can use the fruits of new technologies to transform what they do. In the process, the program is actually helping to shape a market of significant interest to IBM. "We see the program very much as an investment, rather than handing out money," says Moore. The existence of this program indicates IBM adheres to:  

@ the social marketing concept  
  the concept of managed relationships (CRM)  
  transactional philosophy  
  a total bottom-line orientation  
  the iceberg principle  

4) In class, we discussed the characteristics of a market. Based on that discussion, why would prisoners in a prison not be    considered customers for prison uniforms?  

  They don't use the product.  
  They receive no benefits from the product.  
@ They don't have the authority to make the exchange.  
  Because uniforms are a business to business transaction.  

5) In which kind of market would a product orientation be most successful?  

  in a buyer's market when supply exceeds demand  
  in a market where there are more sellers than buyers  
@ in a seller's market when demand exceeds supply  
  in a market that sells only intangible products  
  There is no market in which a product orientation can be used successfully 

6) In which of the following examples does an exchange occur?  

  An elementary student wants to trade her corned beef sandwich for something sweet, but no one is interested.  
  Jane's mother-in-law demands that Jane give her an expensive birthday dinner each year even though Jane and her husband are   saving to buy a house.  
@ Hugh pays for the electricity he consumed last month.  
  A shoplifter steals a candy bar from a local convenience store.  
  none of the above  

==============

7) It appeared to the middle school teacher that she could never obtain the art supplies, craft materials, and science    experiments she wanted to use with her class. Then she discovered a retail store called The Schoolbox, which allowed her    to use and enjoy all the school-related supplies she wanted. The Schoolbox created _____ utility.  

  form  
  location  
@ possession  
  marketing  
  fabrication  

8) It wasn't until three days before Christmas that Josh found the perfect gift for his girlfriend. He ordered the gift from    a catalog retailer, and the gift arrived safely at Josh's apartment on Christmas Eve. The catalog retailer created _____      utility.  

  form  
  marketing  
  possession  
@ place  
  fabrication 

================

9) Jocelyn and Will are moving tomorrow. They decided to rent a truck to haul their belongings from their current apartment      to their new house. They looked up a list of companies that rented trucks in their local Yellow Pages and are calling each    one to see if it has an available truck. They started by calling AAA-Rents. For Jocelyn and Will, a rental truck for        moving is an example of a(n):  

  heterogeneous product  
  industrial product  
  want  
  benefit  
@ need  

==============
 Many upscale supermarkets have added child-friendly carts that resemble racing cars because they theorize that if children are entertained, then the parents will shop longer and spend more money. The supermarkets' desires to extend the length of time a customer spends shopping along with their ability to pay for the carts (which costs about three times more than a regular shopping cart) create:  

  a production orientation  
@ demand for the specialized shopping carts  
 product benefits  
 homogeneous supplies  
 product features  
 


10) Marketing activities play a major role in the creation of _____, which means that a product provides benefits when it is     used.  

  optimization  
@ utility  
  synergy  
  functionality  
  entropy  

11) Mike likes to bicycle ride and believes in drinking plenty of water while he is riding. Unlike his friends he believes he     should not pay more than five dollars for a hydration system because he does not want to have to be concerned if it gets      broken or lost. Mike uses this rule of _______________ when buying his hydration system (a water bottle that attaches to      his bike).  

  synergistic criterion  
  core competency  
  buying plan  
@ heuristic  
  analogy 

===============

12) One of the problems associated with the creation of a value proposition is:  

@ the potential for overselling or underselling  
  how a need and a want are differentiated  
  the result of the price-value paradigm  
  the balancing of supply and demand  
  the size of the marketplace  
1) What do marketers mean when they speak of a "consideration set"  

  The group of people around them who might be affected by the purchase of a socially consumed good such as a luxury         automobile.  
  The group of features used to compare one product against another in the evaluate alternatives phase of the consumer buying   process  
  The "consideration set" is the marketing mix (product, price, promotion and distribution) as perceived by the consumer.  
@ The subset of the alternatives identified as possible solutions that the consumer actually evaluates or "considers" in        order to make the decision.

================

2) When a sign manufacturer takes plastic, metal, and neon tubing and creates a motel sign, it is an example of _____          utility.  

@  form  
   place  
   posession  
   time  
   creation  

3) When asked to discuss the _____ element of his marketing mix, the artisan said, "My average total costs for making a small    leaded glass window is $200, and I typically haggle with my customers. I feel like I have done well if I can earn $150 on    each window I sell."  

  production  
  distribution  
@ price  
  cost  
  promotion

4) When comparing itself to its competitors, Hidden Valley describes its Ranch dressing as the original one. This is the    _____ its manufacturer has selected for the product.  

  marketing mix 
  mass marketing strategy  
@ market position
  transactional aptitude  
  promotion mix  
  
5) When Tom wanted to buy a new gas grill, he went to dogpile.com and searched for gas grills. Through his search on the    Internet, he was able to gather information on gas grills and learn what features were available. Tom used the Internet    during the _______________ stage of his purchase decision.  

  product evaluation  
  situational analysis  
  need recognition  
  problem screening  
@ identify alternatives  

=======================

1) Which of the following is a cause of cognitive dissonance as discussed in class?  

  Cognitive dissonance occurs when two competing brands make opposite claims about a particular product.  
  Cognitive dissonance is caused be the deep sense of satisfaction a consumer feels when a problemn is finally solved.  
  Cognitive dissonance is caused when a consumer has to go through all five stages of the consumer buying process for a       
  habitual purchase  
@ When a consumer finally makes a decision, there is regret over all of the attractive features offered by the products       
  considered, but not chosen.

Which of the following is an example of a want?  

  Gerard has to buy some nails to finish roofing his house.  
@  Tomas decides a Big Mac would be nice for lunch.  
  Michelle is taking piano lessons and believes she should buy herself a piano.  
  Marie needs some margarine so she can bake cookies.  
  none of the above  
 
2) Which of the following is an example of an exchange?  

@ all of the above  

3) Which of the following is a fundamental principle of the marketing concept discussed in class?  

@ The coordination of all company activities  
  Balancing the needs of the stockholders against the needs of society as a whole.  
  Gaining a competitive advantage through the early adoption of technology.  
  Serving the customer by using mass-production techniques to provide inexpensive, quality products.  

3) Which of the following is a requirement of exchange discussed in class?  
  No more than one party  
  Making the product  
@ Voluntary participation  
  A target market

4) Which of the following relates most closely to the product element of the marketing mix?  

  a quantity discount  
  a store coupon  
  a newspaper advertisement  
@ the packaging  
  publicity releases 

5) Which of the following statements best describes a benefit rather than a feature of a product?  

  A benefit is an attribute of a product used to differentiate it from other products.  
  A benefit is determined by the producer when the product is designed.  
  A benefit is a feature of a product used in not-for-profit marketing.  
@ A benefit relates to the satisfaction the consumer receives from the product.

6) Which of the following statements represents a fundamental assumption of the consumer buying process discussed in class?  

  In order to satisfy the largest number of consumers, marketers must make the best products available.  
  Consumers enjoy the challenge of solving problems by making purchases.  
  Consumers are crazy about bargains.  
@ Consumer behavior is not "rational" as we use the term in economics.

7) Which of the following statements best describes a feature rather than a benefit of a product  

  A feature relates to the satisfaction a consumer receives from the product.  
@ A feature is an attribute of the product a marketer can use to position the product.  
  A feature relates to the satisfaction the producer receives from the product.  
  A feature is how benefits are described in business to business marketing. 

8) Which of the following statements about the need recognition stage of the consumer buying process is true  

  Only internal motivations can lead to recognition of a need.  
@ An advertisement can "trigger" a consumer to recognize an unmet need.  
  Once a need has been recognized, the consumer must satisfy it with a purchase.  
  Satisfaction with a product you are using can sometimes stimulate a need
    
9) Which of the following can be marketed?  

  an idea for a new television show based on the Robert Ludlum suspense series  
  the Methodist Church  
  a political candidate running for the U.S. Senate  
  the State of North Dakota  
@ all of the above

10) Which of the following is an example of a need?  

  Even though there are many types of flour on the market, Shameika will buy only the Gold Medal brand when she decides to      bake.  
  Although she claims to be thirsty, Elvania will not buy a soda from this vending machine because it does not contain any      Mountain Dew.  
@ Adam realizes he's hungry.  
  Marilyn decides she would like to have a Sonic milkshake.  
  all of the above  

11) Which of the following is an example of a not-for-profit organization that would use marketing principles?  

  the Rock and Roll Museum in Cleveland, Ohio  
  First Baptist Church  
  the American Red Cross  
  local governments  
@ all of the above

12) Which of the following is an example of a transaction that might occur in business-to-business marketing?  

  Maurice buys a new razor on his way home from work.  
  The amateur gardener buys a new wheelbarrow.  
@ The retailer buys athletic shoes to sell in its store.  
  The professional chef bakes a birthday cake for her son.  
  Robyn's mother hires a math tutor to help Robyn pass algebra.

Which of the following is one of the requirements of an exchange discussed in class?  
 
  A marketspace  
@ Two or more parties  
  Legal permission  
  Standard currency  

13) Which of the following is the best example of a consumer good?  

  the cookies purchased for the daycare center  
@ the disposable diapers purchased by the new mother for her baby

14)Which of these markets could best be served by an organization adopting a consumer orientation?  

  A population recovering from conflict, needing to rebuild basic infrastructure such as roads and communication systems.  
  A population in a less developed country (LDC) struggling to grow enough food to eat  
@ A population where consumers have many alternatives to satisfy their needs.  
  A population where the government subsidizes agricultural production.  
        
15) Which of these statements about mass marketing is true?  

  A mass marketing strategy is preferable to a target marketing strategy.  
  The success of any organization's marketing efforts depends on its ability to engage in mass marketing.  
@ Mass marketing can be cost-effective.  
  Mass marketing allows marketers to develop products to satisfy specific needs and wants of specific groups of customers.  
  Movie production and distribution companies typically use a mass marketing strategy. 

16) Which statement best characterizes the "Stupid Customer Syndrome" presented in class?  
  In advertising, you have to repeat your message over and over or else people will forget what you said.  
@ We have the best product on the market, but our customers are too stupid to realize it.  
  If you want to compete on price, you need to make the numbers on the tag really big so people can read them.  
  All of the above statements characterize "Stupid Customer Syndrome."

================

6) Why do marketers sometimes use intrusive ads, such as interrupting a television program in order to promote a product you've never heard of?  

  Because consumers are more likely to pay attention to a television advertisement for a product that they have never heard     of.  
@ In order to increase the chances that the product is in your "awareness" set if you recognize a need for the product in the   future.  
  Because consumers use the presence of intrusive ads on popular television shows as a "heuristic" in their decision making.  
  Because television advertising is much less expensive than other forms of advertising such as direct mail or print           advertising.

7) Why is it important for a marketer to understand the difference between a feature and a benefit?  

  Because you need both features and benefits in order to satisfy customers' needs.  
  Because In order to attract "Stupid Customers", you need features, not benefits.  
@ Because marketers succeed by satisfying needs, not by creating exciting products.  
  Because a transaction is an exchange of a benefit for a feature.

====================

8) _____ utility occurs when you reach the service station just as your fuel gauge drops below empty.  

  Creation  
@ Place  
  Possession  
  Marketing  
  Form  

9) _____ is the process of planning and executing the conception, pricing, promotion, and distribution of goods and services     to create exchanges that satisfy individual and organizational objectives.  

  Management  
  Planning  
  Selling  
  Promotion  
@ Marketing

10)  _____ marketing is the marketing of goods and services from one organization to another.  

  Distributive  
  Consumer  
  Customer  
@ Business-to-business  
  Target  
  Question 1    Multiple Choice  0 of 1 point(s)    
  Robert has taken up bicycle riding as a hobby and as a way to maintain his physical stamina. He understands he will need to make sure he gets adequate water when he is bike riding. He wants to buy a hydration system. Having gathered a great deal of information, he has decided to evaluate three systems - Waterbags for Roadies, Supertanker Hydropacks, and Fast Water. Robert is in the _______________ stage of his purchase decision.  
 Your Answer:  [None Given]  
Correct Answer:  evaluation of alternatives  
 


  Question 2    Multiple Choice  0 of 1 point(s)    
  When a sign manufacturer takes plastic, metal, and neon tubing and creates a motel sign, it is an example of _____ utility.  
 Your Answer:  [None Given]  
Correct Answer:  form  
 


  Question 3    Multiple Choice  0 of 1 point(s)    
  An advertisement featuring "no money down" and "free delivery" is probably targeted at consumers in the ____________ stage of the consumer buying process?  
 Your Answer:  [None Given]  
Correct Answer:  Decision  
 


  Question 4    Multiple Choice  0 of 1 point(s)    
  Many upscale supermarkets have added child-friendly carts that resemble racing cars because they theorize that if children are entertained, then the parents will shop longer and spend more money. The supermarkets' desires to extend the length of time a customer spends shopping along with their ability to pay for the carts (which costs about three times more than a regular shopping cart) create:  
 Your Answer:  [None Given]  
Correct Answer:  demand for the specialized shopping carts  
 


  Question 5    Multiple Choice  0 of 1 point(s)    
  The idea that a company has to convince customers to buy its products through the use of aggressive persuasion is descriptive of a _____ orientation.  
 Your Answer:  [None Given]  
Correct Answer:  selling  
 


  Question 6    Multiple Choice  0 of 1 point(s)    
  Donna wants to buy a new coat. During the _______________ stage of her purchase decision, she will ask her friends to recommend a store and/or a style of coat. She will search the newspapers for coat sales, and she will visit some nearby stores to see what is available in her price range.  
 Your Answer:  [None Given]  
Correct Answer:  identify alternatives  
 


  Question 7    Multiple Choice  0 of 1 point(s)    
  The first phase of marketing planning is:  
 Your Answer:  [None Given]  
Correct Answer:  analyzing the organization's current strengths and weaknesses  
 


  Question 8    Multiple Choice  0 of 1 point(s)    
  One of the problems associated with the creation of a value proposition is:  
 Your Answer:  [None Given]  
Correct Answer:  the potential for overselling or underselling  
 


  Question 9    Multiple Choice  0 of 1 point(s)    
  Which of the following is a requirement of exchange discussed in class?  
 Your Answer:  [None Given]  
Correct Answer:  Voluntary participation  
 


  Question 10    Multiple Choice  0 of 1 point(s)    
  A consumer who is considering purchasing a house would undoubtedly use:  
 Your Answer:  [None Given]  
Correct Answer:  extended problem solving  
 
Question 1    Multiple Choice  1 of 1 point(s)    
  Consumers often rely on decision guidelines when weighing the claims companies make. These rules are referred to as:  
 Your Answer:  heuristics  
Correct Answer:  heuristics  
 


  Question 2    Multiple Choice  1 of 1 point(s)    
  When comparing itself to its competitors, Hidden Valley describes its Ranch dressing as the original one. This is the _____ its manufacturer has selected for the product.  
 Your Answer:  market position  
Correct Answer:  market position  
 


  Question 3    Multiple Choice  0 of 1 point(s)    
  There are several different sports watches for cyclists. When compared to its competitors, the Bike Nashbar watch is the least expensive. The Acumen Basic is the only one designed for older cyclists who prefer a large display. Sports Instrument is the most comfortable of all the available watches. This describes the _____ of the three sports watches.  
 Your Answer:  [None Given]  
Correct Answer:  market position  
 


  Question 4    Multiple Choice  0 of 1 point(s)    
  How do fashion marketers stimulate the need-recognition phase of the consumer buying process?  
 Your Answer:  [None Given]  
Correct Answer:  By creating new "looks" every season, making fashion-conscious customers dissatisfied with old, outdated merchandise.  
 


  Question 5    Multiple Choice  0 of 1 point(s)    
  Before the 1950s, marketing was:  
 Your Answer:  [None Given]  
Correct Answer:  basically a means of making production more efficient  
 


  Question 6    Multiple Choice  0 of 1 point(s)    
  A market position refers to:  
 Your Answer:  [None Given]  
Correct Answer:  how the organization wants its products perceived in relation to the competition's products  
 


  Question 7    Multiple Choice  0 of 1 point(s)    
  The mother didn't want to have a climbing wall as a permanent feature in her backyard, but it was certainly convenient to rent one from the company that catered her twelve-year-old son's birthday party. The catering company created both _____ utility when it rented her the climbing wall.  
 Your Answer:  [None Given]  
Correct Answer:  place and possession  
 


  Question 8    Multiple Choice  0 of 1 point(s)    
  An advertisement featuring "no money down" and "free delivery" is probably targeted at consumers in the ____________ stage of the consumer buying process?  
 Your Answer:  [None Given]  
Correct Answer:  Decision  
 


  Question 9    Multiple Choice  0 of 1 point(s)    
  Why do marketers sometimes use intrusive ads, such as interrupting a television program in order to promote a product you've never heard of?  
 Your Answer:  [None Given]  
Correct Answer:  In order to increase the chances that the product is in your "awareness" set if you recognize a need for the product in the future.  
 


  Question 10    Multiple Choice  0 of 1 point(s)    
  Every morning at 10 a.m., Horatio buys a bottle of Diet Coke and a Three Musketeers candy bar from a vending machine that is located in a room near his office cubicle. Because there is no thought or effort put into this purchase decision, it would be an example of:  
 Your Answer:  [None Given]  
Correct Answer:  habitual decision making  
 
Question 1    Multiple Choice  1 of 1 point(s)    
  When Tom wanted to buy a new gas grill, he went to dogpile.com and searched for gas grills. Through his search on the Internet, he was able to gather information on gas grills and learn what features were available. Tom used the Internet during the _______________ stage of his purchase decision.  
 Your Answer:  identify alternatives  
Correct Answer:  identify alternatives  
 


  Question 2    Multiple Choice  1 of 1 point(s)    
  Which of the following can be marketed?  
 Your Answer:  all of the above  
Correct Answer:  all of the above  
 


  Question 3    Multiple Choice  1 of 1 point(s)    
  When a sign manufacturer takes plastic, metal, and neon tubing and creates a motel sign, it is an example of _____ utility.  
 Your Answer:  form  
Correct Answer:  form  
 


  Question 4    Multiple Choice  1 of 1 point(s)    
  Which of the following is an example of a transaction that might occur in business-to-business marketing?  
 Your Answer:  The retailer buys athletic shoes to sell in its store.  
Correct Answer:  The retailer buys athletic shoes to sell in its store.  
 


  Question 5    Multiple Choice  0 of 1 point(s)    
  The invitation to go skiing for the weekend forced Donna to look at her current wardrobe and decide she needed a much warmer coat. Donna was in which stage of the purchase decision?  
 Your Answer:  [None Given]  
Correct Answer:  need recognition  
 


  Question 6    Multiple Choice  0 of 1 point(s)    
  Which of the following is an example of a want?  
 Your Answer:  [None Given]  
Correct Answer:  Tomas decides a Big Mac would be nice for lunch.  
 


  Question 7    Multiple Choice  0 of 1 point(s)    
  Which of the following is an example of a not-for-profit organization that would use marketing principles?  
 Your Answer:  [None Given]  
Correct Answer:  all of the above  
 


  Question 8    Multiple Choice  0 of 1 point(s)    
  Donna wants to buy a new coat. During the _______________ stage of her purchase decision, she will ask her friends to recommend a store and/or a style of coat. She will search the newspapers for coat sales, and she will visit some nearby stores to see what is available in her price range.  
 Your Answer:  [None Given]  
Correct Answer:  identify alternatives  
 


  Question 9    Multiple Choice  0 of 1 point(s)    
  The challenge to the marketer is to create an attractive _____, a marketplace offering that fairly and accurately sums up the value that will be realized if the good or service is purchased.  
 Your Answer:  [None Given]  
Correct Answer:  value proposition  
 


  Question 10    Multiple Choice  0 of 1 point(s)    
  One of the problems associated with the creation of a value proposition is:  
 Your Answer:  [None Given]  
Correct Answer:  the potential for overselling or underselling  
 



Question 1    Multiple Choice  1 of 1 point(s)    
  Consumers often rely on decision guidelines when weighing the claims companies make. These rules are referred to as:  
 Your Answer:  heuristics  
Correct Answer:  heuristics  
 


  Question 2    Multiple Choice  1 of 1 point(s)    
  Which of the following is an example of a want?  
 Your Answer:  Tomas decides a Big Mac would be nice for lunch.  
Correct Answer:  Tomas decides a Big Mac would be nice for lunch.  
 


  Question 3    Multiple Choice  1 of 1 point(s)    
  One of the problems associated with the creation of a value proposition is:  
 Your Answer:  the potential for overselling or underselling  
Correct Answer:  the potential for overselling or underselling  
 


  Question 4    Multiple Choice  1 of 1 point(s)    
  Every morning at 10 a.m., Horatio buys a bottle of Diet Coke and a Three Musketeers candy bar from a vending machine that is located in a room near his office cubicle. Because there is no thought or effort put into this purchase decision, it would be an example of:  
 Your Answer:  habitual decision making  
Correct Answer:  habitual decision making  
 


  Question 5    Multiple Choice  1 of 1 point(s)    
  Donna wants to buy a new coat. During the _______________ stage of her purchase decision, she will ask her friends to recommend a store and/or a style of coat. She will search the newspapers for coat sales, and she will visit some nearby stores to see what is available in her price range.  
 Your Answer:  identify alternatives  
Correct Answer:  identify alternatives  
 


  Question 6    Multiple Choice  1 of 1 point(s)    
  According to your instructor, what is the difference between marketing and sales  
 Your Answer:  Sales focuses on creating transactions, marketing focuses on making it possible for transactions to happen.  
Correct Answer:  Sales focuses on creating transactions, marketing focuses on making it possible for transactions to happen.  
 


  Question 7    Multiple Choice  1 of 1 point(s)    
  The mother didn't want to have a climbing wall as a permanent feature in her backyard, but it was certainly convenient to rent one from the company that catered her twelve-year-old son's birthday party. The catering company created both _____ utility when it rented her the climbing wall.  
 Your Answer:  place and possession  
Correct Answer:  place and possession  
 


  Question 8    Multiple Choice  0 of 1 point(s)    
  A catalog retailer has identified African American professionals between the ages of 35 and 45 as the market segment toward which it will direct its marketing efforts. This group has become its:  
 Your Answer:  [None Given]  
Correct Answer:  target market  
 


  Question 9    Multiple Choice  0 of 1 point(s)    
  In class, we discussed the characteristics of a market. Based on that discussion, why would prisoners in a prison not be considered customers for prison uniforms?  
 Your Answer:  [None Given]  
Correct Answer:  They don't have the authority to make the exchange.  
 


  Question 10    Multiple Choice  0 of 1 point(s)    
  The _____ element of the marketing mix is used to communicate the value proposition.  
 Your Answer:  [None Given]  
Correct Answer:  promotion  
 

Question 1    Multiple Choice  1 of 1 point(s)    
  A _____ orientation is a management philosophy that focuses on ways to satisfy customers' needs and wants.  
 Your Answer:  consumer  
Correct Answer:  consumer  
 


  Question 2    Multiple Choice  0 of 1 point(s)    
  The consumer's _______________ determines whether he or she will engage in habitual decision making, extended problem solving, or something in-between the two.  
 Your Answer:  need for customization  
Correct Answer:  level of involvement  
 


  Question 3    Multiple Choice  0 of 1 point(s)    
  A bakery buys sugar, flour, and yeast to make doughnuts, and a retailer buys racks, a cash register, and display tables to sell merchandise. The sugar, flour, yeast, racks, cash register, and display tables are all examples of:  
 Your Answer:  [None Given]  
Correct Answer:  industrial goods  
 


  Question 4    Multiple Choice  0 of 1 point(s)    
  Robert has taken up bicycle riding as a hobby and as a way to maintain his physical stamina. He understands he will need to make sure he gets adequate water when he is bike riding. He wants to buy a hydration system. Having gathered a great deal of information, he has decided to evaluate three systems - Waterbags for Roadies, Supertanker Hydropacks, and Fast Water. Robert is in the _______________ stage of his purchase decision.  
 Your Answer:  [None Given]  
Correct Answer:  evaluation of alternatives  
 


  Question 5    Multiple Choice  0 of 1 point(s)    
  An organic farmer has identified three distinct groups for his products-vegetarians, people who are concerned about chemicals in their food, and people who consider themselves innovators and trendsetters. These three groups identified within a larger group are examples of:  
 Your Answer:  [None Given]  
Correct Answer:  market segments  
 


  Question 6    Multiple Choice  0 of 1 point(s)    
  The _____ element of the marketing mix is used to communicate the value proposition.  
 Your Answer:  [None Given]  
Correct Answer:  promotion  
 


  Question 7    Multiple Choice  0 of 1 point(s)    
  According to your instructor, which of the following is a major trend driving the evolution of marketing?  
 Your Answer:  [None Given]  
Correct Answer:  Advances in logistics and transportation techniques.  
 


  Question 8    Multiple Choice  0 of 1 point(s)    
  There are several different sports watches for cyclists. When compared to its competitors, the Bike Nashbar watch is the least expensive. The Acumen Basic is the only one designed for older cyclists who prefer a large display. Sports Instrument is the most comfortable of all the available watches. This describes the _____ of the three sports watches.  
 Your Answer:  [None Given]  
Correct Answer:  market position  
 


  Question 9    Multiple Choice  0 of 1 point(s)    
  Which of the following is an example of a need?  
 Your Answer:  [None Given]  
Correct Answer:  Adam realizes he's hungry.  
 


  Question 10    Multiple Choice  0 of 1 point(s)    
  According to your instructor, what is the difference between marketing and sales  
 Your Answer:  [None Given]  
Correct Answer:  Sales focuses on creating transactions, marketing focuses on making it possible for transactions to happen.  
 

Question 1    Multiple Choice  0 of 1 point(s)    
  Marketing activities play a major role in the creation of _____, which means that a product provides benefits when it is used.  
 Your Answer:  [None Given]  
Correct Answer:  utility  
 


  Question 2    Multiple Choice  0 of 1 point(s)    
  The impact of game shows like Who Wants to Be a Millionaire and The Weakest Link on society reflect how _____ can influence what we consume.  
 Your Answer:  [None Given]  
Correct Answer:  popular culture  
 


  Question 3    Multiple Choice  0 of 1 point(s)    
  The strategy that will help the organization meet its long-term marketing objectives is called its:  
 Your Answer:  [None Given]  
Correct Answer:  marketing plan  
 


  Question 4    Multiple Choice  0 of 1 point(s)    
  An organic farmer has identified three distinct groups for his products-vegetarians, people who are concerned about chemicals in their food, and people who consider themselves innovators and trendsetters. These three groups identified within a larger group are examples of:  
 Your Answer:  [None Given]  
Correct Answer:  market segments  
 


  Question 5    Multiple Choice  0 of 1 point(s)    
  Which of the following is an example of a transaction that might occur in business-to-business marketing?  
 Your Answer:  [None Given]  
Correct Answer:  The retailer buys athletic shoes to sell in its store.  
 


  Question 6    Multiple Choice  0 of 1 point(s)    
  Which of the following is one of the requirements of an exchange discussed in class?  
 Your Answer:  [None Given]  
Correct Answer:  Two or more parties  
 


  Question 7    Multiple Choice  0 of 1 point(s)    
  Which of these markets could best be served by an organization adopting a consumer orientation?  
 Your Answer:  [None Given]  
Correct Answer:  A population where consumers have many alternatives to satisfy their needs.  
 


  Question 8    Multiple Choice  0 of 1 point(s)    
  Every morning at 10 a.m., Horatio buys a bottle of Diet Coke and a Three Musketeers candy bar from a vending machine that is located in a room near his office cubicle. Because there is no thought or effort put into this purchase decision, it would be an example of:  
 Your Answer:  [None Given]  
Correct Answer:  habitual decision making  
 


  Question 9    Multiple Choice  0 of 1 point(s)    
  Employees at the Six Flags amusement parks are told to treat everyone who purchased a ticket to get into the parks as a guest. These guests are actually:  
 Your Answer:  [None Given]  
Correct Answer:  consumers  
 


  Question 10    Multiple Choice  0 of 1 point(s)    
  A(n) _____ occurs when Ronda Albers trades Max Lynch three hours of child care for his three-year-old daughter for fixing the hole in her porch roof.  
 Your Answer:  [None Given]  
Correct Answer:  exchange  
 

Question 7    Multiple Choice  0 of 1 point(s)    
  The marketer's strategic toolbox is called:  
 Your Answer:  [None Given]  
Correct Answer:  the marketing mix  
 


  Question 8    Multiple Choice  0 of 1 point(s)    
  In which kind of market would a product orientation be most successful?  
 Your Answer:  [None Given]  
Correct Answer:  in a seller's market when demand exceeds supply  
 


  Question 9    Multiple Choice  0 of 1 point(s)    
  _____ utility occurs when you reach the service station just as your fuel gauge drops below empty.  
 Your Answer:  [None Given]  
Correct Answer:  Place  
 


  Question 10    Multiple Choice  0 of 1 point(s)    
  There are several different sports watches for cyclists. When compared to its competitors, the Bike Nashbar watch is the least expensive. The Acumen Basic is the only one designed for older cyclists who prefer a large display. Sports Instrument is the most comfortable of all the available watches. This describes the _____ of the three sports watches.  
 Your Answer:  [None Given]  
Correct Answer:  market position  
 

Question 1    Multiple Choice  0 of 1 point(s)    
  The concept of customer relationship management (CRM):  
 Your Answer:  [None Given]  
Correct Answer:  sees marketing as a process of building long-term relationships with customers  
 


  Question 2    Multiple Choice  0 of 1 point(s)    
  Which of the following is an example of a want?  
 Your Answer:  [None Given]  
Correct Answer:  Tomas decides a Big Mac would be nice for lunch.  
 


  Question 3    Multiple Choice  0 of 1 point(s)    
  Which of the following is a cause of cognitive dissonance as discussed in class?  
 Your Answer:  [None Given]  
Correct Answer:  When a consumer finally makes a decision, there is regret over all of the attractive features offered by the products considered, but not chosen.  
 


  Question 4    Multiple Choice  0 of 1 point(s)    
  A market position refers to:  
 Your Answer:  [None Given]  
Correct Answer:  how the organization wants its products perceived in relation to the competition's products  
 


  Question 5    Multiple Choice  0 of 1 point(s)    
  Which of the following is one of the requirements of an exchange discussed in class?  
 Your Answer:  [None Given]  
Correct Answer:  Two or more parties  
 


  Question 6    Multiple Choice  0 of 1 point(s)    
  Why is it important to understand consumer behavior?  
 Your Answer:  [None Given]  
Correct Answer:  Because purchasing is problem solving and understanding how people solve problems will help us in our lives.  
 


  Question 7    Multiple Choice  0 of 1 point(s)    
  Many upscale supermarkets have added child-friendly carts that resemble racing cars because they theorize that if children are entertained, then the parents will shop longer and spend more money. The supermarkets' desires to extend the length of time a customer spends shopping along with their ability to pay for the carts (which costs about three times more than a regular shopping cart) create:  
 Your Answer:  [None Given]  
Correct Answer:  demand for the specialized shopping carts  
 


  Question 8    Multiple Choice  0 of 1 point(s)    
  A _____ orientation is a management philosophy that focuses on ways to satisfy customers' needs and wants.  
 Your Answer:  [None Given]  
Correct Answer:  consumer  
 


  Question 9    Multiple Choice  0 of 1 point(s)    
  The consumer's _______________ determines whether he or she will engage in habitual decision making, extended problem solving, or something in-between the two.  
 Your Answer:  [None Given]  
Correct Answer:  level of involvement  
 


  Question 10    Multiple Choice  0 of 1 point(s)    
  The idea that a company has to convince customers to buy its products through the use of aggressive persuasion is descriptive of a _____ orientation.  
 Your Answer:  [None Given]  
Correct Answer:  selling  
 


Question 1    Multiple Choice  1 of 1 point(s)    
  Every morning at 10 a.m., Horatio buys a bottle of Diet Coke and a Three Musketeers candy bar from a vending machine that is located in a room near his office cubicle. Because there is no thought or effort put into this purchase decision, it would be an example of:  
 Your Answer:  habitual decision making  
Correct Answer:  habitual decision making  
 


  Question 2    Multiple Choice  0 of 1 point(s)    
  Ruth loves to bake homemade breads, cakes, and pastries, but she will only use Pillsbury flour - no other flour will satisfy her even if it is superior in some ways to Pillsbury. Ruth's brand loyalty is a type of:  
 Your Answer:  [None Given]  
Correct Answer:  heuristic  
 


  Question 3    Multiple Choice  0 of 1 point(s)    
  The challenge to the marketer is to create an attractive _____, a marketplace offering that fairly and accurately sums up the value that will be realized if the good or service is purchased.  
 Your Answer:  [None Given]  
Correct Answer:  value proposition  
 


  Question 4    Multiple Choice  0 of 1 point(s)    
  The magazine ad for Alpo dog food is an example of the _____ element of the marketing mix.  
 Your Answer:  [None Given]  
Correct Answer:  promotion  
 


  Question 5    Multiple Choice  0 of 1 point(s)    
  Which of these markets could best be served by an organization adopting a consumer orientation?  
 Your Answer:  [None Given]  
Correct Answer:  A population where consumers have many alternatives to satisfy their needs.  
 


  Question 6    Multiple Choice  0 of 1 point(s)    
  _____ marketing is the marketing of goods and services from one organization to another.  
 Your Answer:  [None Given]  
Correct Answer:  Business-to-business  
 


  Question 7    Multiple Choice  0 of 1 point(s)    
  When Tom wanted to buy a new gas grill, he went to dogpile.com and searched for gas grills. Through his search on the Internet, he was able to gather information on gas grills and learn what features were available. Tom used the Internet during the _______________ stage of his purchase decision.  
 Your Answer:  [None Given]  
Correct Answer:  identify alternatives  
 


  Question 8    Multiple Choice  0 of 1 point(s)    
  A consumer who is considering purchasing a house would undoubtedly use:  
 Your Answer:  [None Given]  
Correct Answer:  extended problem solving  
 


  Question 9    Multiple Choice  0 of 1 point(s)    
  A(n) _____ consists of all the consumers who share a common need that can be satisfied by a specific good or service and who have the resources, willingness, and authority to make the exchange.  
 Your Answer:  [None Given]  
Correct Answer:  market  
 


  Question 10    Multiple Choice  0 of 1 point(s)    
  A catalog retailer has identified African Americans professionals between the ages of 35 and 45 as the group of customers within its larger market as a potential market for its products. The retailer has identified a:  
 Your Answer:  [None Given]  
Correct Answer:  market segment  
 


Question 1    Multiple Choice  1 of 1 point(s)    
  Which of the following statements represents a fundamental assumption of the consumer buying process discussed in class?  
 Your Answer:  Consumer behavior is not "rational" as we use the term in economics.  
Correct Answer:  Consumer behavior is not "rational" as we use the term in economics.  
 


  Question 2    Multiple Choice  1 of 1 point(s)    
  In which of the following examples does an exchange occur?  
 Your Answer:  Hugh pays for the electricity he consumed last month.  
Correct Answer:  Hugh pays for the electricity he consumed last month.  
 


  Question 3    Multiple Choice  0 of 1 point(s)    
  The availability of your favorite brand of jeans at a nearby retail store is an example of the _____ element of the marketing mix.  
 Your Answer:  [None Given]  
Correct Answer:  place  
 


  Question 4    Multiple Choice  0 of 1 point(s)    
  When comparing itself to its competitors, Hidden Valley describes its Ranch dressing as the original one. This is the _____ its manufacturer has selected for the product.  
 Your Answer:  [None Given]  
Correct Answer:  market position  
 


  Question 5    Multiple Choice  0 of 1 point(s)    
  Which of the following can be marketed?  
 Your Answer:  [None Given]  
Correct Answer:  all of the above  
 


  Question 6    Multiple Choice  0 of 1 point(s)    
  _____ utility occurs when you reach the service station just as your fuel gauge drops below empty.  
 Your Answer:  [None Given]  
Correct Answer:  Place  
 


  Question 7    Multiple Choice  0 of 1 point(s)    
  It appeared to the middle school teacher that she could never obtain the art supplies, craft materials, and science experiments she wanted to use with her class. Then she discovered a retail store called The Schoolbox, which allowed her to use and enjoy all the school-related supplies she wanted. The Schoolbox created _____ utility.  
 Your Answer:  [None Given]  
Correct Answer:  possession  
 


  Question 8    Multiple Choice  0 of 1 point(s)    
  The strategy that will help the organization meet its long-term marketing objectives is called its:  
 Your Answer:  [None Given]  
Correct Answer:  marketing plan  
 


  Question 9    Multiple Choice  0 of 1 point(s)    
  Mike likes to bicycle ride and believes in drinking plenty of water while he is riding. Unlike his friends he believes he should not pay more than five dollars for a hydration system because he does not want to have to be concerned if it gets broken or lost. Mike uses this rule of _______________ when buying his hydration system (a water bottle that attaches to his bike).  
 Your Answer:  [None Given]  
Correct Answer:  heuristic  
 


  Question 10    Multiple Choice  0 of 1 point(s)    
  Donna wants to buy a new coat. During the _______________ stage of her purchase decision, she will ask her friends to recommend a store and/or a style of coat. She will search the newspapers for coat sales, and she will visit some nearby stores to see what is available in her price range.  
 Your Answer:  [None Given]  
Correct Answer:  identify alternatives  
 

Question 1    Multiple Choice  1 of 1 point(s)    
  Why is it important for a marketer to understand the difference between a feature and a benefit?  
 Your Answer:  Because marketers succeed by satisfying needs, not by creating exciting products.  
Correct Answer:  Because marketers succeed by satisfying needs, not by creating exciting products.  
 


  Question 2    Multiple Choice  1 of 1 point(s)    
  _____ utility occurs when you reach the service station just as your fuel gauge drops below empty.  
 Your Answer:  Place  
Correct Answer:  Place  
 


  Question 3    Multiple Choice  1 of 1 point(s)    
  Which of the following relates most closely to the product element of the marketing mix?  
 Your Answer:  the packaging  
Correct Answer:  the packaging  
 


  Question 4    Multiple Choice  1 of 1 point(s)    
  Which of the following is one of the requirements of an exchange discussed in class?  
 Your Answer:  Two or more parties  
Correct Answer:  Two or more parties  
 


  Question 5    Multiple Choice  0 of 1 point(s)    
  It wasn't until three days before Christmas that Josh found the perfect gift for his girlfriend. He ordered the gift from a catalog retailer, and the gift arrived safely at Josh's apartment on Christmas Eve. The catalog retailer created _____ utility.  
 Your Answer:  [None Given]  
Correct Answer:  place  
 


  Question 6    Multiple Choice  0 of 1 point(s)    
  The first step in extended problem solving is:  
 Your Answer:  [None Given]  
Correct Answer:  problem recognition  
 


  Question 7    Multiple Choice  0 of 1 point(s)    
  Which of the following is an example of a transaction that might occur in business-to-business marketing?  
 Your Answer:  [None Given]  
Correct Answer:  The retailer buys athletic shoes to sell in its store.  
 


  Question 8    Multiple Choice  0 of 1 point(s)    
  There are several different sports watches for cyclists. When compared to its competitors, the Bike Nashbar watch is the least expensive. The Acumen Basic is the only one designed for older cyclists who prefer a large display. Sports Instrument is the most comfortable of all the available watches. This describes the _____ of the three sports watches.  
 Your Answer:  [None Given]  
Correct Answer:  market position  
 


  Question 9    Multiple Choice  0 of 1 point(s)    
  Jocelyn and Will are moving tomorrow. They decided to rent a truck to haul their belongings from their current apartment to their new house. They looked up a list of companies that rented trucks in their local Yellow Pages and are calling each one to see if it has an available truck. They started by calling AAA-Rents. For Jocelyn and Will, a rental truck for moving is an example of a(n):  
 Your Answer:  [None Given]  
Correct Answer:  need  
 


  Question 10    Multiple Choice  0 of 1 point(s)    
  Which of the following is a cause of cognitive dissonance as discussed in class?  
 Your Answer:  [None Given]  
Correct Answer:  When a consumer finally makes a decision, there is regret over all of the attractive features offered by the products considered, but not chosen.  
 

Question 1    Multiple Choice  1 of 1 point(s)    
  Before the 1950s, marketing was:  
 Your Answer:  basically a means of making production more efficient  
Correct Answer:  basically a means of making production more efficient  
 


  Question 2    Multiple Choice  1 of 1 point(s)    
  Which of the following can be marketed?  
 Your Answer:  all of the above  
Correct Answer:  all of the above  
 


  Question 3    Multiple Choice  0 of 1 point(s)    
  According to your instructor, which of the following is a barrier to implementing the Marketing concept?  
 Your Answer:  [None Given]  
Correct Answer:  Managers in other functional areas of the company may resent being told what to do by the marketing department.  
 


  Question 4    Multiple Choice  0 of 1 point(s)    
  When Tom wanted to buy a new gas grill, he went to dogpile.com and searched for gas grills. Through his search on the Internet, he was able to gather information on gas grills and learn what features were available. Tom used the Internet during the _______________ stage of his purchase decision.  
 Your Answer:  [None Given]  
Correct Answer:  identify alternatives  
 


  Question 5    Multiple Choice  0 of 1 point(s)    
  In which kind of market would a product orientation be most successful?  
 Your Answer:  [None Given]  
Correct Answer:  in a seller's market when demand exceeds supply  
 


  Question 6    Multiple Choice  0 of 1 point(s)    
  Jocelyn and Will are moving tomorrow. They decided to rent a truck to haul their belongings from their current apartment to their new house. They looked up a list of companies that rented trucks in their local Yellow Pages and are calling each one to see if it has an available truck. They started by calling AAA-Rents. For Jocelyn and Will, a rental truck for moving is an example of a(n):  
 Your Answer:  [None Given]  
Correct Answer:  need  
 


  Question 7    Multiple Choice  0 of 1 point(s)    
  The first step in the target marketing process is to:  
 Your Answer:  [None Given]  
Correct Answer:  divide the overall market into segments in which consumers have similar characteristics  
 


  Question 8    Multiple Choice  0 of 1 point(s)    
  A catalog retailer has identified African Americans professionals between the ages of 35 and 45 as the group of customers within its larger market as a potential market for its products. The retailer has identified a:  
 Your Answer:  [None Given]  
Correct Answer:  market segment  
 


  Question 9    Multiple Choice  0 of 1 point(s)    
  Robert has taken up bicycle riding as a hobby and as a way to maintain his physical stamina. He understands he will need to make sure he gets adequate water when he is bike riding. He wants to buy a hydration system. Having gathered a great deal of information, he has decided to evaluate three systems-Waterbags for Roadies, Supertanker Hydropacks, and Fast Water-based on the following _______________: size, ease with which water can be reached while riding, and insulation.  
 Your Answer:  [None Given]  
Correct Answer:  evaluative criteria  
 


  Question 10    Multiple Choice  0 of 1 point(s)    
  What do marketers mean when they speak of a "consideration set"  
 Your Answer:  [None Given]  
Correct Answer:  The subset of the alternatives identified as possible solutions that the consumer actually evaluates or "considers" in order to make the decision.  
 
 Question 1    Multiple Choice  1 of 1 point(s)    
  Consumers often rely on decision guidelines when weighing the claims companies make. These rules are referred to as:  
 Your Answer:  heuristics  
Correct Answer:  heuristics  
 


  Question 2    Multiple Choice  0 of 1 point(s)    
  Ruth Terry is a realtor. In her ads, Terry offers the free use of a moving truck to every customer she helps either buy or sell a house. This free use of a moving truck is an example of a:  
 Your Answer:  [None Given]  
Correct Answer:  feature  
 


  Question 3    Multiple Choice  0 of 1 point(s)    
  IBM manager Celia Moore coordinates IBM's long-standing "Reinventing Education" program, which involves intensive research into how educational institutions can use the fruits of new technologies to transform what they do. In the process, the program is actually helping to shape a market of significant interest to IBM. "We see the program very much as an investment, rather than handing out money," says Moore. The existence of this program indicates IBM adheres to:  
 Your Answer:  [None Given]  
Correct Answer:  the social marketing concept  
 


  Question 4    Multiple Choice  0 of 1 point(s)    
  When asked to discuss the _____ element of his marketing mix, the artisan said, "My average total costs for making a small leaded glass window is $200, and I typically haggle with my customers. I feel like I have done well if I can earn $150 on each window I sell."  
 Your Answer:  [None Given]  
Correct Answer:  price  
 


  Question 5    Multiple Choice  0 of 1 point(s)    
  According to the instructor, the "law of large numbers" means...  
 Your Answer:  [None Given]  
Correct Answer:  You can't say for sure what any one person will do, but you can predict the behavior of groups of people.  
 


  Question 6    Multiple Choice  0 of 1 point(s)    
  Which of the following is an example of a need?  
 Your Answer:  [None Given]  
Correct Answer:  Adam realizes he's hungry.  
 


  Question 7    Multiple Choice  0 of 1 point(s)    
  In which of the following examples does an exchange occur?  
 Your Answer:  [None Given]  
Correct Answer:  Hugh pays for the electricity he consumed last month.  
 


  Question 8    Multiple Choice  0 of 1 point(s)    
  Robert has taken up bicycle riding as a hobby and as a way to maintain his physical stamina. He understands he will need to make sure he gets adequate water when he is bike riding. He wants to buy a hydration system. Having gathered a great deal of information, he has decided to evaluate three systems - Waterbags for Roadies, Supertanker Hydropacks, and Fast Water. Robert is in the _______________ stage of his purchase decision.  
 Your Answer:  [None Given]  
Correct Answer:  evaluation of alternatives  
 


  Question 9    Multiple Choice  0 of 1 point(s)    
  An advertisement featuring Barry Bonds talking about life insurance would be most effective for baseball fans who are in the ___________ stage of the consumer buying process.  
 Your Answer:  [None Given]  
Correct Answer:  Evaluate alternatives  
 


  Question 10    Multiple Choice  0 of 1 point(s)    
  Two teachers purchased art supplies for their classes. One paid $103.45 for eight large boxes of crayons. The other teacher purchased ten identical boxes for $105.55. The second teacher got more for her money because she took advantage of a quantity discount-a reduced price for buying ten or more boxes. Which element of the marketing mix most directly relates to these purchases?  
 Your Answer:  [None Given]  
Correct Answer:  price  
 
1) A description of how business-to-business marketing differs from consumer marketing must include the fact business-to-   business marketing:  

  is composed of many individual and household customers  
  consists of frequent impulse purchases  
  emphasizes advertising more than any other element of the promotion mix  
@ uses competitive bidding and price negotiations  
  is accurately described by none of the above  


2) A European city fire brigade wanted to build a long-term relationship with a supplier of firefighter apparel. Lion Apparel    wanted to convince the fire brigade it should become the city's only supplier of firefighter apparel. Lion was trying to    convince the fire brigade to use:  

@ single sourcing  
  a sole proprietorship  
  transaction-based sourcing  
  cross docking  
  a unique sourcing platform  

3) A person's attitude has three components. They are:  
@ affect, cognition, and behavior  
  psychological, physiological, and situational  
  situation, emotion, and environment  
  cognitive, operant, and situation  
  strategic, tactic, and operationa 

4) A Miami restaurant has created atmosphere through its subtle use of blue and green lighting, ceiling fans over each table,    and frozen drinks. If its electrical energy supplier raises its prices, the restaurant will most likely:  

  continue its present operation because it has an elastic demand for electricity  
  ask other Miami restaurants to create a derived demand for cheaper electricity  
  decide to find other ways to use electricity  
@ continue its present operation because it has an inelastic demand for electricity  
  remove its lighting, turn off its fans, and economize on its use of electricity 

5) A trucking company is considering purchasing new trucks that are powered by ethanol instead of diesel fuel. After
   gathering information on the trucks, the company's management decided to buy one ethanol-powered truck as a test to see
   how well it met the company's needs. The purchasing agent who executed the purchase took which buying center role?  

  initiator  
  decider  
@ buyer  
  user  
  influencer 

6) A rifle manufacturer like Springfield needs muzzles, trigger mechanism, and wooden gunstocks to produce rifles. Without    these items, Springfield cannot produce its rifles. Springfield will reduce its demand for trigger mechanism and gunstocks
   if suppliers are unable to sell it the muzzles it needs. This is an example of _____ demand.  

  elastic  
  synergistic  
@ joint  
  reciprocal  
  inelastic 

===============

7) An attitude is:  

  an internal state that drives us to satisfy needs  
  the process by which people select, organize, and interpret information from the outside world  
  a change in behavior caused by information or experience  
@ a lasting evaluation of a person, object, or issue  
  the set of unique psychological characteristics that consistently influence the way a person responds to situations in the
  environment  

===============

8) According to the instructor, which of the following factors has the greatest influence on a consumer's buying habits?  

@ The patterns of consumption learned in childhood  
  Television advertising  
  The mood a person is in at the time of purchase  
  The amount of information we have about a specific product 

9) According to the instructor, why is the business market so much larger than the consumer market.  

  Because there are more businesses than there are individual consumers.  
  Because consumers can make purchases from international as well as domestic firms.  
@ Because each consumer purchase may represent several intermediary business to business transactions.  
  Because businesses have better access to credit than individual consumers do.

10) According to the discussion in class, which of the following pieces of information would have the strongest influence on
    your decision to buy a sports utility vehicle?  

  A newspaper advertisement offers a $3,000 rebate on the purchase of a SUV.  
  A television commercial shows how exciting it is to drive a SUV in the wilderness.  
  The sales manager at the auto dealership offers you a generous trade-in allowance on your mini-van.  
@ Your sister in law says that people who SUV's are compensating for sexual inadequacy 

11) According to the instructor, why are there so many job opportunities in business to business marketing?  

@ Because every company needs at least one salesperson  
  Because training programs in the business sector are better than in the consumer sector.  
  Because the Federal government subsidized the creation of marketing jobs in the business sector.  
  Because most marketing activities are wasted in the consumer market.  
 
12) According to your instructor, you are at a competitive disadvantage if your _____________ is not automated somehow.  

  New task buying  
  Modified rebuy purchasing  
@ Straight rebuy purchasing  
  New and improved purchasing  
   
=====================

13) After a patient of Dr. Brown told the dentist he needed to buy a machine that would sterilize his dental tools without     using any water, which tends to cause the tools to rust or corrode over time, the dentist, who runs a solo practice in a     small rural town, located some articles on the chemical sterilizer and read about how the machine works. After gathering
    more information and talking to salespeople, Dr. Brown placed his order for the machine. In terms of the buying center,
    Dr. Brown had the role of:  

  initiator  
@ decider  
  agent  
  user  
  influencer  

========================

14) Because Bonnie believed she had fat ankles, she always wore 3-inch high heels to make her ankles look less chunky. This
    belief about how her ankles look is an example of her:  

  selective perception  
  selective exposure  
  self-attitude  
@ self-concept  
  inward-directed motivation  

15) Behavioral learning theorists:  

  believe learning occurs when people watch the actions of others and note what happens to them as a result  
@ assume learning takes place as the results of connections that form between events that we perceive  
  believe the most important component of learning is behavior, which is directed by affect and cognition  
  view people as problem solvers who do more than passively react to associations between stimuli  
  stress the role of creativity and intuition during the learning process  

16) Business-to-business marketing is _____ because a business's demand for goods and services comes either directly or
    indirectly from consumers' demands.  

  price inelastic  
  value-added demand  
@ derived demand  
  micro-dependent  
  price elastic  

17) Business buyers often develop _____, written descriptions of the quality, size, weight, color, features, quantity, 
    training, warranty, service terms, and delivery requirements for the purchase.  

  FAB lists  
  value analyses  
  vendor analyses  
@ product specifications  
  product maps  
   
=======================
 
18) Caterpillar, the manufacturer of road building and earth moving equipment, maintains very low inventory levels of the     component parts it uses to build its bull dozers, road scrapers, etc. Its suppliers have guaranteed the company a
    constant inventory and have agreed to make deliveries just prior to when the parts are needed on the assembly line. Based
    on this information you can say with certainty, Caterpillar uses:  

  electronic data exchange (EDE)  
  computer-assisted manufacturing (CAM)  
  quick warehousing  
  direct distribution  
@ a just-in-time (JIT) system  
 
 
19) Celebrity-HealthWatch.com features diseases and ailments along with information on celebrities who have dealt with them.
    The site is designed to capitalize on relaxed FDA regulations that allow increased direct sales to consumers by
    pharmaceutical companies and the belief that learning how celebrities deal with their illnesses will influence how people
    deal with their own illnesses through the use of:  

  operant conditioning  
  stimulus generalization  
  stimulus discrimination  
  classical conditioning  
@ cognitive learning  
 
 
20) Chen woke up in the middle of the night feeling hungry. He walked to the refrigerator, grabbed the first thing he saw,
    ate it, and then went back to bed. This is an example of:  

  Consideration set  
@ Cognitive miserliness  
  Need recognition  
  Social influence 
 
21) Cognitive learning theorists:  

  believe learning occurs when people learn their actions result in rewards or punishments  
  assume learning takes place as the results of connections that form between events that we perceive  
  believe the most important component of learning is behavior, which is directed by affect and cognition  
@ view people as problem solvers who do more than passively react to associations between stimuli  
  stress the passive role of the individual during the learning process   

=======================

22) During the _____ stage of the business buying decisions process, the buying center assesses the proposals.  
    obtain written or verbal bids  

@ evaluate alternatives  
  engage in a value analysis  
  do an information search  
  problem recognition  

==========================

23) eBags is a Denver-based manufacturer of luggage, handbags, backpacks, and duffel bags. While much of the company's
    business is done over the Internet, it discovered it was missing many sales to customers who prefer to deal with an
    individual operator. Instead of hiring its own telephone agents to work weekends, off-hours, and peak-call periods, eBags
    hired Finali Corp, a teleservices company, to take all of its phone orders from customers and to answer commonly asked
    customer questions. eBags used:
  
@ outsourcing  
  external resources management  
  resource contracting  
  multiple sourcing  
  cross-functionalization  
 
=================

24) For years Roger Ebert and Gene Siskel used a "thumbs up" and "thumbs down" notation to rate movies. Many people would
    wait to see if these movie critics liked a movie before they would go to see it. If the critics gave the movie a "thumbs
    down," many people would not go to see the movie even though it had actors that they admired or a plot, which interested
    them. Ebert and Siskel were:  

  cultural icons  
  futurists  
@ opinion leaders  
  cultural dictators  
  status seekers  

1) Guinevere Laundry washes the towels, sheets, and uniforms for hospitals, nursing homes, and other similar facilities. A    recent drought in the area where its main plant is located led the company's CEO to ask its purchasing agent to gather    information about a water system that would allow the company to reuse its waste water. The purchasing agent examined    literature, talked to several salespeople, and scheduled three sales presentations by those companies that seemed most     likely to have the product at the price the laundry could afford. By not sharing every piece of information he had learned
   about water-saving systems with the CEO and other plant managers, the purchasing agent assumed the role of:  

  initiator  
  liaison  
@ gatekeeper  
  user  
  influencer
 
2) Gillette Shows of Pittsfield, Massachusetts, is a touring carnival company. In a season, it will do shows in New York and    most of the New England states. The show has 19 different rides. After looking at receipts from the 1999-2000 season, its    owner Betty Gleason decided the carnival needed to buy a new ride. Because of this decision, in terms of the buying
   center, Gleason's role was as the:  

@ initiator  
  liaison  
  agent  
  user  
  influencer  
 
=======================

3) How does the law of large numbers apply to personality types?  

  If a large number of people have the same personality type then you can't make statistical predictions about their   behavior.  
@ We can't predict how an individual will respond to an advertisement, but we can predict how groups of people with similar   personalities might respond.  
  You can't predict whan an individual personality type might do in a given situation, but you can make predictions about how   groups of personality types might behave.  
  Marketers need to restrict the number of personality types to four in order to have large numbers of each personality type.

=====================

4) If you want to reserve a campsite in one of the 50 state parks in Florida, you will find the reservation process is
   handled by a subsidiary of TicketMaster, not by any state government agency. An outside vendor handling campsite   
   reservations for the state is an example of _____ because it is a task the state could do in-house.  

@ outsourcing  
  external resources management  
  resource contracting  
  multiple sourcing  
  cross-functionalization  
   
5) In class, the instructor spoke of "pipeline" supply. What was he talking about?  

  A partnership between a producer and a reseller where inventory is automatically re-stocked after a product is sold.  
@ Inventory stored by different businesses in the supply chain for a given product that can act as buffer stock to minimize   volatility in demand.  
  An inventory management system where just in time delivery is used to increase the elasticity of demand.  
  Demand for commodity products such as oil or natural gas is derived using a pipeline.  
   
6) In class, the instructor suggested that although we are exposed to hundreds of advertising messages a day...  

  We are able to pay attention to all of them  
  We only notice them if we are at the identify alternatives stage of the consumer buying process  
  We are more likely to pay attention to the ones in print media such as newspapers or magazines.  
@ We only notice a small percentage of them and have a reaction to only a small proportion of the ads we notice.  

7) In Iran, the thirteenth day of spring April 2 is a holiday. Iranians consider it unlucky to stay indoors on that day, so
   they picnic in the countryside to celebrate the season. This is an example of how _______________ influences consumer
   decisions.  

  social class  
  demographic analysis  
  culture  
  an opinion leader  
@ the physical environment 

8) In terms of organizational markets, Ford Motor Company, Anheuser-Busch, and Kraft are all examples of:  

  resellers  
  resuppliers  
  not-for-profit markets  
@ producers  
  distributors 

9) In terms of organizational markets, JCPenney, Target, and www.amazon.com are all examples of:  

@ resellers  
  resuppliers  
  not-for-profit markets  
  producers  
  facilitating agents  

======================

10) Jason will not go shopping with his wife the day after Thanksgiving even though there are a lot of bargains to be found
    on that day. He finds the crowd of people negates all of the enjoyment he might get from finding a bargain. In this case,
   Jason is reacting negatively to which situational influence?  

@ the physical environment  
  opinion leaders  
  reference groups  
  time pressures  
  all of the above 

11) Juana looked at her September issue of "Rosie" magazine and did not see anything of interest. After her mother was
    diagnosed with bipolar disorder, she found the issue extremely interesting because it offered advice on how to help
    people who are suffering from this problem. The fact the boring issue became quite interesting is due to:  

  subliminal messaging  
  sociocultural influences  
@ selective perception  
  segment selectivity  
  the hierarchy of needs  

========================

12) Learning is:  

  an internal state that drives us to satisfy needs  
  the process by which people select, organize, and interpret information from the outside world  
@ a change in behavior caused by information or experience  
  a lasting evaluation of a person, object, or issue  
  the set of unique psychological characteristics that consistently influence the way a person responds to situations in the environment  
 
====================

13) Many businesses today are abandoning the traditional business attire and allowing their employees to come to work in     business casual, which typically does not include ties, cufflinks, or business suits. There has been a decrease in the
    sale of silk ties as a result of this cultural trend. A decrease in consumer demand for silk ties has led to a decrease
    in demand for silk because the demand for silk is:  

  elastic  
  micro-dependent  
@ derived  
  inelastic  
  fixed  

14) Maslow's hierarchy of needs:  

  examines six levels of motivation  
@ suggests a person cannot move to the next level until previous need levels are satisfied  
  begins with the most basic need for a positive self-image  
  culminates in the need for belonging  
  is accurately described by all of the abov 

15) Most newspapers such as the "New York Times" and the "Memphis Commercial-Appeal" rely on _____ and use several different     suppliers to provide the tons of paper that they use annually.  

  single sourcing  
  segmented suppliers  
@ multiple sourcing  
  cross docking  
  a composite source 

=================

16) Often when two people watch the same television programming, only one of them will notice an ad for a specific product,
    and the other will be oblivious to the ad. _______________ can be used to explain why this happens.  

  Subliminal advertising  
  A behavioral influence  
@ Selective attention  
  Segment selectivity  
  The hierarchy of ne 

17) Opinion leaders:  

  often are among the first to buy new products  
  are not sponsored by any company  
  impart both positive and negative information about product performance  
  influence others' attitudes or behaviors because they are perceived as possessing expertise about a product  
@ are accurately described by all of the above  

=====================

18) Personality is:  

  an internal state that drives us to satisfy needs  
  the process by which people select, organize, and interpret information from the outside world  
  a change in behavior caused by information or experience  
  a lasting evaluation of a person, object, or issue  
@ the set of unique psychological characteristics that consistently influence the way a person responds to situations in the environment  

19) Professional buyers are responsible for:  

  negotiating transportation and delivery charges  
  selecting quality products  
  ensuring products are delivered in a timely fashion  
  making sure all accessory goods and services are available at a reasonable cost  
@ all of the above


1) Robertson industries supplies ice cream cones to Walt Disney World. Walt Walt Disney World changed ice cream suppliers.    The new ice cream melted faster causing an increase in customer complaints. Even though there was nothing wrong with the    cones, Robertson industries took back all of the cones in inventory and replaced them with a leak-proof cone at no charge
   to Walt Disney World. Why did Robertson industries do this?  

  Because the Robertson industries should have known better than to sell leaky cones in the first place.  
@ In order to maintain a long-term relationship with Walt Disney World focused on creating satisfied ice cream cone customers.  
 Because it is important to lose money when your customers lose money.  
 In order to boost profitability in the short-term.  
 
======================

2) Sara Lee found that consumers who had negative _______________ about themselves preferred portioned snack items because
   they felt they lacked the will power to regulate how much they ate without help.  

  selective perceptions  
  selective exposures  
  self-attitudes  
@ self-concepts  
  inward-directed motivations  

3) Several years ago, when California raisin growers faced a surplus of raisins, they realized the problem resulted from    consumers' evaluation of raisins as boring. To change consumers' _______________ raisins, the California Raisin Advisory    Board created the dancing raisins.  

  motivation for  
  self-concept of  
  personality profiles of  
@ attitude toward  
  selective perception of  
 
4) Sometimes supplier selection is based on _____, which means a buyer and a seller agree to purchase each other's goods and    services even though the terms of the sales may not be the best available.  

  source synergy  
@ reciprocity  
  tying arrangements  
  sourcing agreements  
  reverse marketing  
 
=======================

5) The ad for the San Francisco State University MBA program said, "Reach your full potential in the business world with an    Executive MBA from SFSU." The program indicates that it satisfies which of Maslow's needs?  

  safety needs  
  psychological needs  
  physiological needs  
@ self-actualization needs  
  belongingness needs  
 
6) The group of people in the organization who participate in the purchase decision-making process is referred to as the:  

  buyer institution  
  resource center  
@ buying center  
  purchasing hierarchy  
  purchasing office  
 
7)The lowest level of needs on Maslow's hierarchy of needs is:  

  safety needs  
  ego needs  
@ physiological needs  
  self-actualization needs  
 belongingness needs  

8) The multiplier effect is another name for the:  

  hierarchy of needs  
@ acceleration principle  
  80/20 rule  
  iceberg principle  
  principle of price/quality  

9) The North American Industry Classification System (NAICS):  

  is primarily used to determine where specific industries are geographically concentrated  
  was the first classification of this type ever developed  
  includes information about industries in Brazil, Argentina, Mexico, Canada, and the U.S.  
@ uses a six-digit code that sorts industries into subsectors, industry groups, industries, and specific country industries  
  is accurately described by all of the above  
 

10) The North American Industry Classification System (NAICS):  

  is a numerical coding of industries developed by the United States, Canada, and Mexico  
  can be used by firms to assess potential markets  
  replaced the Standard Industrial Classification (SIC) system  
  can be used to find new customers  
@ is accurately described by all of the above 

11)The use of actresses Sarah Jessica Parker and Katie Holmes in ads for Garnier hair coloring products is an application of    which learning theory?  

  operant conditioning  
  stimulus generalization  
  stimulus discrimination  
  classical conditioning  
@ observational learning  
     
12) "There is no such thing as bad publicity" is an example of which of the principles of perception discussed in class?  
 
  Cognitive miserliness. People don't process negative information.  
  Selective attention. People don't notice negative information.  
  Selective interpretation. People don't tend to see things in a positive light.  
@ Selective retention. People dissociate the object and the evaluation over time.  

====================
   
13) Vera wants to buy her mother a jar of thyme honey for her birthday. Which of the following is an example of a situational     influence on this purchase decision?  

  The only store carrying the honey is fifty miles away from Vera's home.  
  The clerk was rude to Vera when she asked if there were any other types of honey that tasted like thyme honey.  
  Vera could not find a parking place anywhere near the store that carries specialized honeys.  
  Vera needs the present bought and wrapped by tonight at 7.  
@ All of the above are examples of situational influences on the purchase decision.  

===============

14) What did your instructor mean when he claimed that you can't have "attitude"?  
  An attitude is an intrinsic state.  
@ An attitude has to have an object.  
  An attitude is relatively stable over time.  
  An attitude has to be either positive or negative.  
   
15) What is an advantage of having multiple suppliers for the same product?  

  Increased purchasing power with each supplier.  
  Lower transaction costs.  
  Easier to arrange just in time (JIT) delivery.  
@ Readily available alternative supplier in case of a problem. 

16) What is meant by the term "confirmation bias"  

  People are more likely to believe positive claims in advertisements than they are to belive negative claims.  
  People are more likely to react to an advertisement the second time they have seen it because it confirms their perception. 
@ People tend to interpret advertisements in a way that agrees with their expectations.  
  Consumers are less likely to return a product if it comes with a money-back gaurantee. 

==============
    
17) When a dentist's office manager orders latex gloves, disposable masks, file folders, and complementary toothbrushes to     give to patients, he or she is most likely making a:  

@  straight rebuy  
  simple-task purchase  
  modified rebuy  
  value purchase  
  utility buy 

18) When a product has _____ demand, an increase or decrease in the price of the product has little or no effect on the
    demand for the product.  

  elastic  
  economic  
  derived  
@ inelastic  
  fixed  
 
19) When the city of Houston buys a John Deere golf turf mower for maintaining the fairways and greens at a golf course owned
    and operated by the municipality, the sale occurs in which market?  

  reseller  
  producer  
  consumer  
  facilitator  
@ government  

20) When the city of New Orleans decided to build a $110 million stadium to hold 19,000 people, it was a difficult decision     because New Orleans already had the Superdome. City planners convinced the people of New Orleans that the older 70,000 
    seat Superdome was too large for most touring shows and better suited for sporting events. In terms of the buy class
    framework, the decision to build an arena for touring shows featuring big name stars was a:  

  complex-task purchase  
@ new-task buy  
  simplified rebuy  
  utility buy  
  government purchase  

====================
    
21) Which of these factors that influence consumer behavior discussed in class has the least influence on consumer behavior?  

  Psychological factors  
  Family  
  Situational factors  
@ Commercial information  

22) Which of the following statements about social class as an influence on consumer decision making is true?  

  The people in a particular social class tend not to share religious or political beliefs.  
  The people in a particular social class do not have similar income levels.  
  Products designed to appeal to people in a specific social class typically fail.  
@ People in the same social class tend to work in similar occupations.  
  Products that are status symbols today will always be status symbols.  

23) Which of the following statements about the alternative evaluation stage of the business buying decision process is true?  
  Price is the only factor businesses consider before making a purchase decision.  
  Even companies that have adopted a total quality management approach consider price to be the most important determinant of   whom to buy from.  
@ Before making a purchase decision, many businesses consider quality, reliability, durability, after-the-sale service as   well as price.  
  Since after-the-sale service is a negotiation point, it is not considered at this stage of the business buying decision   process.  
  Price and warranty are the only two factors businesses consider before making a purchase decision. 

24) Which of the statements about social class in the United States is truest?  

  There are no class distinctions in the "Land of the free and the home of the brave."  
  Ancestry is an important determinant of social class in the United States.  
@ Wealth is an important determinant of social class in the United States.  
  Geographic location is an important determinant of social class in the United States.  
 
25) Which of the following will have the greatest influence on your decision to purchase a Starbuck's Dopio Mocha Latte
    Cappucino Grande DeCaf on the way to class tomorrow?  

  You are a Starbucks regular customer and this is your favorite coffee drink.  
  You are feeling tired and there is a test tomorrow so the decaf isn't sounding so good.  
  You hear an advertisement for Starbucks Coffee while waiting to board the MUNI.  
@ All seven of the Starbucks stores on the way to school are closed.  
 

26) Which of the following is an example of business-to-business marketing?  

  Scott Sign Systems sells interior signs to an Alabama resort.  
  A municipal government buys chemicals for its city swimming pools.  
  A Canadian software company buys tickets to send a group of salespeople to make a sales presentation to a heavy equipment   manufacturer in Japan.  
  Airmark sells a vinyl printing press to a manufacturer of plastic bags.  
@ All of the above are examples of business-to-business marketing. 
 
27) Which of the following statements best describes the principle of "cognitive miserliness?"  

  We perceive the price of a product to be higher immediately after making a purchase.  
  It is not unusual for people to think about several problems at the same time.  
  Consumers feel better about a product if they do extensive research after the purchase is made.  
@ We only do enough thinking to solve the immediate problem.  

28) Which of the following statements about perception is true?  

  Perception is a sociocultural influence on consumer purchase decisions.  
  Perception is unrelated to any sense other than sight.  
  If a marketer can reach a consumer with a message, then the marketer knows the consumer will interpret it correctly.  
@ There is no perception without exposure.  
  Research shows subliminal messages are just as likely to be perceived as any other type of message. 

===========

29) While much of the equipment used by dentists is thrown away after one use, many of the instruments used to clean teeth,
    fill cavities, and deal with impactions and other dental problems are made of steel and must be sterilized. A dentist is
    considering replacing his conventional steam sterilizer with a chemical sterilizer that cleans without rusting or
    corroding instruments. Since the dentist is already familiar with sterilizers, he is looking to find the best price
    possible on a chemical sterilizer. He is engaged in a:  

  straight rebuy  
  simple-task purchase  
@ modified rebuy  
  value purchase  
  utility buy  

================

30) Why does your instructor include the Myers-Briggs personality Typology in an introductory Marketing course?  

  Because there are four universal personality types used by marketers.  
  As an example of how marketers can be mislead by becoming overly scientific.  
@ Because it helps us understand how different personality types might respond differently to a similar situation.  
  In order to ddemonstrate the relationship between consumer behavior and attitude formation.  

=======================

31) Worthington Farm raises chickens. For years, it has used wooden coops for hauling its poultry to market. When Bob
    Worthington went to reuse some of his coops, he noticed many of them could not be sufficiently cleaned for reuse and
    ought to be replaced. Worthington was at which stage of the business buying process when he decided to replace his old
    coops?  

@  need recognition  
  a vendor analysis  
  identify alternatives  
  information evaluation  
  prepurchase evaluation 

32) Worthington Farm raises chickens. For years, it has used wooden coops for hauling its poultry to market. The owner of the 
    farm needs to buy some replacement coops and is considering buying plastic coops, which are slightly more expensive than     wooden ones but much easier to clean after use. This purchase of coops is an example of a:  

  straight rebuy  
  simple-task purchase  
@ modified rebuy  
  value purchase  
  utility buy  

=====================

33) _______________ is an internal state that drives us to satisfy needs.  

@  Motivation  
  Interpretation  
  Selective exposure  
  Selective perception  
  An environmental stimulus  

34) _______________ is the process by which people select, organize, and interpret information from the outside world.  

@  Perception  
  Learning  
  Operant conditioning  
  Motivation  
  Cognition  

35) _______________ is at work when a person changes as a reaction to real or imagined group pressure.  

  Synergy  
  Empathy  
  Entropy  
  Ambiguity  
@ Conformity  

 Question 1    Multiple Choice  0 of 1 point(s)    
  Jane's aunt wants a cashmere blanket to put over her legs when she watches television from her favorite chair. Jane can drive 50 miles and pay $90 for the blanket, or she can order one off the Internet for $168. Because Jane works 50 hours a week, cares for three children, and tries to help her aunt, she opted to buy the blanket at the Internet site. This is an example of how _______________ influence consumer decisions.  
 Your Answer:  [None Given]  
Correct Answer:  time pressures  
 


  Question 2    Multiple Choice  0 of 1 point(s)    
  A trucking company is considering purchasing new trucks that are powered by ethanol instead of diesel fuel. In terms of the buying center, the truck drivers who must make sure that the trucks' fuel tanks do not run out of fuel would be the:  
 Your Answer:  [None Given]  
Correct Answer:  users  
 


  Question 3    Multiple Choice  0 of 1 point(s)    
  Which of the following will have the greatest influence on your decision to purchase a Starbuck's Dopio Mocha Latte Cappucino Grande DeCaf on the way to class tomorrow?  
 Your Answer:  [None Given]  
Correct Answer:  All seven of the Starbucks stores on the way to school are closed.  
 


  Question 4    Multiple Choice  0 of 1 point(s)    
  According to the instructor, why is the business market so much larger than the consumer market.  
 Your Answer:  [None Given]  
Correct Answer:  Because each consumer purchase may represent several intermediary business to business transactions.  
 


  Question 5    Multiple Choice  0 of 1 point(s)    
  Sometimes supplier selection is based on _____, which means a buyer and a seller agree to purchase each other's goods and services even though the terms of the sales may not be the best available.  
 Your Answer:  [None Given]  
Correct Answer:  reciprocity  
 


  Question 6    Multiple Choice  0 of 1 point(s)    
  Robertson industries supplies ice cream cones to Walt Disney World. Walt Walt Disney World changed ice cream suppliers. The new ice cream melted faster causing an increase in customer complaints. Even though there was nothing wrong with the cones, Robertson industries took back all of the cones in inventory and replaced them with a leak-proof cone at no charge to Walt Disney World. Why did Robertson industries do this?  
 Your Answer:  [None Given]  
Correct Answer:  In order to maintain a long-term relationship with Walt Disney World focused on creating satisfied ice cream cone customers.  
 


  Question 7    Multiple Choice  0 of 1 point(s)    
  In Iran, the thirteenth day of spring April 2 is a holiday. Iranians consider it unlucky to stay indoors on that day, so they picnic in the countryside to celebrate the season. This is an example of how _______________ influences consumer decisions.  
 Your Answer:  [None Given]  
Correct Answer:  culture  
 


  Question 8    Multiple Choice  0 of 1 point(s)    
  Caterpillar, the manufacturer of road building and earth moving equipment, maintains very low inventory levels of the component parts it uses to build its bull dozers, road scrapers, etc. Its suppliers have guaranteed the company a constant inventory and have agreed to make deliveries just prior to when the parts are needed on the assembly line. Based on this information you can say with certainty, Caterpillar uses:  
 Your Answer:  [None Given]  
Correct Answer:  a just-in-time (JIT) system  
 


  Question 9    Multiple Choice  0 of 1 point(s)    
  When a product has _____ demand, an increase or decrease in the price of the product has little or no effect on the demand for the product.  
 Your Answer:  [None Given]  
Correct Answer:  inelastic  
 


  Question 10    Multiple Choice  0 of 1 point(s)    
  Easy Spirit shoes uses ads featuring country music singer Martina McBride with the hope that her fans will see her wearing Easy Spirit shoes and want to wear the same shoes as McBride wears. Easy Spirit shoes is hoping Martina McBride's fans view her as a:  
 Your Answer:  [None Given]  
Correct Answer:  reference group  
 

 Question 1    Multiple Choice  0 of 1 point(s)    
  A trucking company is considering purchasing new trucks that are powered by ethanol instead of diesel fuel. After gathering information on the trucks, the company's management decided to buy one ethanol-powered truck as a test to see how well it met the company's needs. The purchasing agent who executed the purchase took which buying center role?  
 Your Answer:  [None Given]  
Correct Answer:  buyer  
 


  Question 2    Multiple Choice  0 of 1 point(s)    
  The lowest level of needs on Maslow's hierarchy of needs is:  
 Your Answer:  [None Given]  
Correct Answer:  physiological needs  
 


  Question 3    Multiple Choice  0 of 1 point(s)    
  Sometimes supplier selection is based on _____, which means a buyer and a seller agree to purchase each other's goods and services even though the terms of the sales may not be the best available.  
 Your Answer:  [None Given]  
Correct Answer:  reciprocity  
 


  Question 4    Multiple Choice  0 of 1 point(s)    
  Which of these factors that influence consumer behavior discussed in class has the least influence on consumer behavior?  
 Your Answer:  [None Given]  
Correct Answer:  Commercial information  
 


  Question 5    Multiple Choice  0 of 1 point(s)    
  For years Roger Ebert and Gene Siskel used a "thumbs up" and "thumbs down" notation to rate movies. Many people would wait to see if these movie critics liked a movie before they would go to see it. If the critics gave the movie a "thumbs down," many people would not go to see the movie even though it had actors that they admired or a plot, which interested them. Ebert and Siskel were:  
 Your Answer:  [None Given]  
Correct Answer:  opinion leaders  
 


  Question 6    Multiple Choice  0 of 1 point(s)    
  Personality is:  
 Your Answer:  [None Given]  
Correct Answer:  the set of unique psychological characteristics that consistently influence the way a person responds to situations in the environment  
 


  Question 7    Multiple Choice  0 of 1 point(s)    
  After a patient of Dr. Brown told the dentist he needed to buy a machine that would sterilize his dental tools without using any water, which tends to cause the tools to rust or corrode over time, the dentist, who runs a solo practice in a small rural town, located some articles on the chemical sterilizer and read about how the machine works. After gathering more information and talking to salespeople, Dr. Brown placed his order for the machine. In terms of the buying center, Dr. Brown had the role of:  
 Your Answer:  [None Given]  
Correct Answer:  decider  
 


  Question 8    Multiple Choice  0 of 1 point(s)    
  In terms of organizational markets, JCPenney, Target, and www.amazon.com are all examples of:  
 Your Answer:  [None Given]  
Correct Answer:  resellers  
 


  Question 9    Multiple Choice  0 of 1 point(s)    
  Gillette Shows of Pittsfield, Massachusetts, is a touring carnival company. In a season, it will do shows in New York and most of the New England states. The show has 19 different rides. After looking at receipts from the 1999-2000 season, its owner Betty Gleason decided the carnival needed to buy a new ride. Because of this decision, in terms of the buying center, Gleason's role was as the:  
 Your Answer:  [None Given]  
Correct Answer:  initiator  
 


  Question 10    Multiple Choice  0 of 1 point(s)    
  Celebrity-HealthWatch.com features diseases and ailments along with information on celebrities who have dealt with them. The site is designed to capitalize on relaxed FDA regulations that allow increased direct sales to consumers by pharmaceutical companies and the belief that learning how celebrities deal with their illnesses will influence how people deal with their own illnesses through the use of:  
 Your Answer:  [None Given]  
Correct Answer:  cognitive learning  
 


Question 1    Multiple Choice  1 of 1 point(s)    
  For years Roger Ebert and Gene Siskel used a "thumbs up" and "thumbs down" notation to rate movies. Many people would wait to see if these movie critics liked a movie before they would go to see it. If the critics gave the movie a "thumbs down," many people would not go to see the movie even though it had actors that they admired or a plot, which interested them. Ebert and Siskel were:  
 Your Answer:  opinion leaders  
Correct Answer:  opinion leaders  
 


  Question 2    Multiple Choice  0 of 1 point(s)    
  In Iran, the thirteenth day of spring April 2 is a holiday. Iranians consider it unlucky to stay indoors on that day, so they picnic in the countryside to celebrate the season. This is an example of how _______________ influences consumer decisions.  
 Your Answer:  [None Given]  
Correct Answer:  culture  
 


  Question 3    Multiple Choice  0 of 1 point(s)    
  In class, the instructor spoke of "pipeline" supply. What was he talking about?  
 Your Answer:  [None Given]  
Correct Answer:  Inventory stored by different businesses in the supply chain for a given product that can act as buffer stock to minimize volatility in demand.  
 


  Question 4    Multiple Choice  0 of 1 point(s)    
  Maslow's hierarchy of needs:  
 Your Answer:  [None Given]  
Correct Answer:  suggests a person cannot move to the next level until previous need levels are satisfied  
 


  Question 5    Multiple Choice  0 of 1 point(s)    
  What is meant by the term "confirmation bias"  
 Your Answer:  [None Given]  
Correct Answer:  People tend to interpret advertisements in a way that agrees with their expectations.  
 


  Question 6    Multiple Choice  0 of 1 point(s)    
  While much of the equipment used by dentists is thrown away after one use, many of the instruments used to clean teeth, fill cavities, and deal with impactions and other dental problems are made of steel and must be sterilized. A dentist is considering replacing his conventional steam sterilizer with a chemical sterilizer that cleans without rusting or corroding instruments. Since the dentist is already familiar with sterilizers, he is looking to find the best price possible on a chemical sterilizer. He is engaged in a:  
 Your Answer:  [None Given]  
Correct Answer:  modified rebuy  
 


  Question 7    Multiple Choice  0 of 1 point(s)    
  A European city fire brigade wanted to build a long-term relationship with a supplier of firefighter apparel. Lion Apparel wanted to convince the fire brigade it should become the city's only supplier of firefighter apparel. Lion was trying to convince the fire brigade to use:  
 Your Answer:  [None Given]  
Correct Answer:  single sourcing  
 


  Question 8    Multiple Choice  0 of 1 point(s)    
  The group of people in the organization who participate in the purchase decision-making process is referred to as the:  
 Your Answer:  [None Given]  
Correct Answer:  buying center  
 


  Question 9    Multiple Choice  0 of 1 point(s)    
  Which of the following statements about social class as an influence on consumer decision making is true?  
 Your Answer:  [None Given]  
Correct Answer:  People in the same social class tend to work in similar occupations.  
 


  Question 10    Multiple Choice  0 of 1 point(s)    
  Professional buyers are responsible for:  
 Your Answer:  [None Given]  
Correct Answer:  all of the above  
 

Question 1    Multiple Choice  0 of 1 point(s)    
  Why does your instructor include the Myers-Briggs personality Typology in an introductory Marketing course?  
 Your Answer:  [None Given]  
Correct Answer:  Because it helps us understand how different personality types might respond differently to a similar situation.  
 


  Question 2    Multiple Choice  0 of 1 point(s)    
  In class, the instructor suggested that although we are exposed to hundreds of advertising messages a day...  
 Your Answer:  [None Given]  
Correct Answer:  We only notice a small percentage of them and have a reaction to only a small proportion of the ads we notice.  
 


  Question 3    Multiple Choice  0 of 1 point(s)    
  In terms of organizational markets, JCPenney, Target, and www.amazon.com are all examples of:  
 Your Answer:  [None Given]  
Correct Answer:  resellers  
 


  Question 4    Multiple Choice  0 of 1 point(s)    
  What is meant by the term "confirmation bias"  
 Your Answer:  [None Given]  
Correct Answer:  People tend to interpret advertisements in a way that agrees with their expectations.  
 


  Question 5    Multiple Choice  0 of 1 point(s)    
  In class, the instructor spoke of "pipeline" supply. What was he talking about?  
 Your Answer:  [None Given]  
Correct Answer:  Inventory stored by different businesses in the supply chain for a given product that can act as buffer stock to minimize volatility in demand.  
 


  Question 6    Multiple Choice  0 of 1 point(s)    
  How does the law of large numbers apply to personality types?  
 Your Answer:  [None Given]  
Correct Answer:  We can't predict how an individual will respond to an advertisement, but we can predict how groups of people with similar personalities might respond.  
 


  Question 7    Multiple Choice  0 of 1 point(s)    
  When a product has _____ demand, an increase or decrease in the price of the product has little or no effect on the demand for the product.  
 Your Answer:  [None Given]  
Correct Answer:  inelastic  
 


  Question 8    Multiple Choice  0 of 1 point(s)    
  A rifle manufacturer like Springfield needs muzzles, trigger mechanism, and wooden gunstocks to produce rifles. Without these items, Springfield cannot produce its rifles. Springfield will reduce its demand for trigger mechanism and gunstocks if suppliers are unable to sell it the muzzles it needs. This is an example of _____ demand.  
 Your Answer:  [None Given]  
Correct Answer:  joint  
 


  Question 9    Multiple Choice  0 of 1 point(s)    
  According to the instructor, which of the following factors has the greatest influence on a consumer's buying habits?  
 Your Answer:  [None Given]  
Correct Answer:  The patterns of consumption learned in childhood  
 


  Question 10    Multiple Choice  0 of 1 point(s)    
  After a patient of Dr. Brown told the dentist he needed to buy a machine that would sterilize his dental tools without using any water, which tends to cause the tools to rust or corrode over time, the dentist, who runs a solo practice in a small rural town, located some articles on the chemical sterilizer and read about how the machine works. After gathering more information and talking to salespeople, Dr. Brown placed his order for the machine. In terms of the buying center, Dr. Brown had the role of:  
 Your Answer:  [None Given]  
Correct Answer:  decider  
 

Question 1    Multiple Choice  0 of 1 point(s)    
  Shareen is an executive for Robertson industries, a manufacturer of food processing equipment. Each year, she is a sponsor of the food processors golf outing. As a sponsor, Shareen plays golf with purchasing agents from potential customers. She is always alert for news that one of her golf partners is going to add a new food processing line. Shareen's approach works best for which type of buying decision?  
 Your Answer:  [None Given]  
Correct Answer:  New task buying  
 


  Question 2    Multiple Choice  0 of 1 point(s)    
  Robertson industries supplies ice cream cones to Walt Disney World. Walt Walt Disney World changed ice cream suppliers. The new ice cream melted faster causing an increase in customer complaints. Even though there was nothing wrong with the cones, Robertson industries took back all of the cones in inventory and replaced them with a leak-proof cone at no charge to Walt Disney World. Why did Robertson industries do this?  
 Your Answer:  [None Given]  
Correct Answer:  In order to maintain a long-term relationship with Walt Disney World focused on creating satisfied ice cream cone customers.  
 


  Question 3    Multiple Choice  0 of 1 point(s)    
  A trucking company is considering purchasing new trucks that are powered by ethanol instead of diesel fuel. After gathering information on the trucks, the company's management decided to buy one ethanol-powered truck as a test to see how well it met the company's needs. The purchasing agent who executed the purchase took which buying center role?  
 Your Answer:  [None Given]  
Correct Answer:  buyer  
 


  Question 4    Multiple Choice  0 of 1 point(s)    
  Which of the following statements about perception is true?  
 Your Answer:  [None Given]  
Correct Answer:  There is no perception without exposure.  
 


  Question 5    Multiple Choice  0 of 1 point(s)    
  Worthington Farm raises chickens. For years, it has used wooden coops for hauling its poultry to market. When Bob Worthington went to reuse some of his coops, he noticed many of them could not be sufficiently cleaned for reuse and ought to be replaced. Worthington was at which stage of the business buying process when he decided to replace his old coops?  
 Your Answer:  [None Given]  
Correct Answer:  need recognition  
 


  Question 6    Multiple Choice  0 of 1 point(s)    
  The multiplier effect is another name for the:  
 Your Answer:  [None Given]  
Correct Answer:  acceleration principle  
 


  Question 7    Multiple Choice  0 of 1 point(s)    
  Business-to-business marketing is _____ because a business's demand for goods and services comes either directly or indirectly from consumers' demands.  
 Your Answer:  [None Given]  
Correct Answer:  derived demand  
 


  Question 8    Multiple Choice  0 of 1 point(s)    
  While much of the equipment used by dentists is thrown away after one use, many of the instruments used to clean teeth, fill cavities, and deal with impactions and other dental problems are made of steel and must be sterilized. A dentist is considering replacing his conventional steam sterilizer with a chemical sterilizer that cleans without rusting or corroding instruments. Since the dentist is already familiar with sterilizers, he is looking to find the best price possible on a chemical sterilizer. He is engaged in a:  
 Your Answer:  [None Given]  
Correct Answer:  modified rebuy  
 


  Question 9    Multiple Choice  0 of 1 point(s)    
  According to the instructor, why are there so many job opportunities in business to business marketing?  
 Your Answer:  [None Given]  
Correct Answer:  Because every company needs at least one salesperson  
 


  Question 10    Multiple Choice  0 of 1 point(s)    
  In terms of organizational markets, JCPenney, Target, and www.amazon.com are all examples of:  
 Your Answer:  [None Given]  
Correct Answer:  resellers  
 
Question 7    Multiple Choice  0 of 1 point(s)    
  It's Just Lunch is a dating service for busy professionals that brings two people together in a low-pressure lunch setting and gives them a chance to get to know each other. The service's ads ask, "Do you invest in your personal life?" The service provided by It's Just Lunch would most directly satisfy which of Maslow's needs?  
 Your Answer:  [None Given]  
Correct Answer:  belongingness needs  
 


  Question 8    Multiple Choice  0 of 1 point(s)    
  A person's attitude has three components. They are:  
 Your Answer:  [None Given]  
Correct Answer:  affect, cognition, and behavior  
 


  Question 9    Multiple Choice  0 of 1 point(s)    
  Which of the statements about social class in the United States is truest?  
 Your Answer:  [None Given]  
Correct Answer:  Wealth is an important determinant of social class in the United States.  
 


  Question 10    Multiple Choice  0 of 1 point(s)    
  The North American Industry Classification System (NAICS):  
 Your Answer:  [None Given]  
Correct Answer:  uses a six-digit code that sorts industries into subsectors, industry groups, industries, and specific country industries  
 

Question 1    Multiple Choice  1 of 1 point(s)    
  Which of the following statements about social class as an influence on consumer decision making is true?  
 Your Answer:  People in the same social class tend to work in similar occupations.  
Correct Answer:  People in the same social class tend to work in similar occupations.  
 


  Question 2    Multiple Choice  1 of 1 point(s)    
  Caterpillar, the manufacturer of road building and earth moving equipment, maintains very low inventory levels of the component parts it uses to build its bull dozers, road scrapers, etc. Its suppliers have guaranteed the company a constant inventory and have agreed to make deliveries just prior to when the parts are needed on the assembly line. Based on this information you can say with certainty, Caterpillar uses:  
 Your Answer:  a just-in-time (JIT) system  
Correct Answer:  a just-in-time (JIT) system  
 


  Question 3    Multiple Choice  0 of 1 point(s)    
  The ad for the San Francisco State University MBA program said, "Reach your full potential in the business world with an Executive MBA from SFSU." The program indicates that it satisfies which of Maslow's needs?  
 Your Answer:  [None Given]  
Correct Answer:  self-actualization needs  
 


  Question 4    Multiple Choice  0 of 1 point(s)    
  Personality is:  
 Your Answer:  [None Given]  
Correct Answer:  the set of unique psychological characteristics that consistently influence the way a person responds to situations in the environment  
 


  Question 5    Multiple Choice  0 of 1 point(s)    
  A description of how business-to-business marketing differs from consumer marketing must include the fact business-to-business marketing:  
 Your Answer:  [None Given]  
Correct Answer:  uses competitive bidding and price negotiations  
 


  Question 6    Multiple Choice  0 of 1 point(s)    
  Which of the statements about social class in the United States is truest?  
 Your Answer:  [None Given]  
Correct Answer:  Wealth is an important determinant of social class in the United States.  
 


  Question 7    Multiple Choice  0 of 1 point(s)    
  In terms of organizational markets, Ford Motor Company, Anheuser-Busch, and Kraft are all examples of:  
 Your Answer:  [None Given]  
Correct Answer:  producers  
 


  Question 8    Multiple Choice  0 of 1 point(s)    
  A trucking company is considering purchasing new trucks that are powered by ethanol instead of diesel fuel. After gathering information on the trucks, the company's management decided to buy one ethanol-powered truck as a test to see how well it met the company's needs. The purchasing agent who executed the purchase took which buying center role?  
 Your Answer:  [None Given]  
Correct Answer:  buyer  
 


  Question 9    Multiple Choice  0 of 1 point(s)    
  The use of actresses Sarah Jessica Parker and Katie Holmes in ads for Garnier hair coloring products is an application of which learning theory?  
 Your Answer:  [None Given]  
Correct Answer:  observational learning  
 


  Question 10    Multiple Choice  0 of 1 point(s)    
  A trucking company is considering purchasing new trucks that are powered by ethanol instead of diesel fuel. In terms of the buying center, the truck drivers who must make sure that the trucks' fuel tanks do not run out of fuel would be the:  
 Your Answer:  [None Given]  
Correct Answer:  users  
 

Question 5    Multiple Choice  0 of 1 point(s)    
  A patient of Dr. Brown's told the dentist he needed to buy a machine that would sterilize his dental tools without using any water, which tends to cause the tools to rust or corrode over time. In terms of the buying center, the patient had the role of:  
 Your Answer:  [None Given]  
Correct Answer:  influencer  
 


  Question 6    Multiple Choice  0 of 1 point(s)    
  Sara Lee found that consumers who had negative _______________ about themselves preferred portioned snack items because they felt they lacked the will power to regulate how much they ate without help.  
 Your Answer:  [None Given]  
Correct Answer:  self-concepts  
 


  Question 7    Multiple Choice  0 of 1 point(s)    
  Professional buyers are responsible for:  
 Your Answer:  [None Given]  
Correct Answer:  all of the above  
 


  Question 8    Multiple Choice  0 of 1 point(s)    
  Personality is:  
 Your Answer:  [None Given]  
Correct Answer:  the set of unique psychological characteristics that consistently influence the way a person responds to situations in the environment  
 


  Question 9    Multiple Choice  0 of 1 point(s)    
  Which of these factors that influence consumer behavior discussed in class has the least influence on consumer behavior?  
 Your Answer:  [None Given]  
Correct Answer:  Commercial information  
 


  Question 10    Multiple Choice  0 of 1 point(s)    
  The ad for the San Francisco State University MBA program said, "Reach your full potential in the business world with an Executive MBA from SFSU." The program indicates that it satisfies which of Maslow's needs?  
 Your Answer:  [None Given]  
Correct Answer:  self-actualization needs  
 
 A _____ underlying the demand for watch batteries is the number of people who own and wear watches.  

@ market factor  
  differential advantage  
  sales catalyst  
  profit consideration  
  product-market growth strategy

  A company that uses a(n) _____ targeting strategy is appealing to a broad spectrum of people.  

  fragmented  
@ undifferentiated  
  concentrated  
  differentiated  
  customized  

   A condition of _____ exists today as a result of people's diverse interests and backgrounds that divide them into numerous 
   groups with distinct wants and needs.  

  market aggregation  
  market similitude  
  economic aggregation  
  disintermediation  
@ market fragmentation  

   A former top manager at Mazda was quoted as saying, "Customers want to express their individuality with the products they buy." His statement reflects a belief that companies need to develop a _____ marketing strategy.  

  fragmented  
  heterogeneous  
  concentrated  
  differentiated  
@ customized 

A national retailer of uniforms targeted male nurses in a recent advertising campaign. It used _____ segmentation to define its target market.  

  lifestyle  
  behavioral  
  socioeconomic  
@ demographic  
  psychographic 

  A perceptual map is used to:  

  identify psychographic segments such as people who cheat on their income taxes or steal from their employers  
  create consumer cross-tabulations for marketing research  
  locate consumers within a primary metropolitan statistical area (PMSA)  
@ determine how consumers position products in relation to each other  
  identify at which stage of the targeting process the marketer needs to be  

A retailer that wanted to target prospective grooms between the ages of 21 and 40 who are within six months of their marriage is using _____ segmentation to define its target market.  

  lifestyle  
  behavioral  
  socioeconomic  
@ demographic  
  psychographic 

   A viable target market should:  

@ have needs that can adequately be served by the marketer  
  not be reachable by other organizations  
  have the same needs as similar sized groups  
  have enough growth potential to become profitable sometime in the future  
  be all of the above  

 According to your instructor, what is a major disadvantage of the trend analysis method of forecasting?  
  It is very easy to use.  

  It can only provide long-term (more than one year) forecasts.  
@ Relying on statistical analysis means you risk ignoring forseeable environmental factors.  
  It can only be used for new products.  
  It is most effective at the early stages of a new product introduction.  

   According to your instructor, what is the second step you need to take in order to develop a positioning strategy?  

  Figure out where you were a year ago  
  Figure out where you are now  
@ Figure out where you want to be  
  Develop a plan to get from where you are now to where you want to be  

   According to your instructor, which of Michael Porter's fundamental strategies is most commonly adopted by businesses?  

  Customization  
  Concentration  
  Low cost producer  
@ Differentiation  

  According to the instructor, which of the following sources of information is frequently ignored when determining a company's actual market position?  

  Information about how the product is selling.  
  Information provided by the company's advertising agency.  
@ Complaints from dissatisfied customers.  
  The judgment of top executives.  
  Information provided by retailers who sell your product.  


 According to your instructor, which of the following is the best source of information to use in determining where you are as you develop a positioning strategy?  

  Reports from sales people who call on you  
  The expertise of top management  
@ Complaints from people who are dissatisfied with your product  
  All of the above are equally good sources of information.  

   Age, ethnicity, geographical region, and lifestyle are all examples of:  

@ segmentation variables  
  positioning strategies  
  synergistic samples  
  homogenous populations  
  sampling heuristics  

  Airlines have developed frequent-flyer programs to encourage passengers to use the same airlines repeatedly. These programs and other similar ones use _____ segmentation to target their markets.  

  demographic  
@ behavioral  
  geodemographic  
  socioeconomic  
  psychographic

  Baby boomers:  

  were born between 1965 and 1976  
  tend to view their homes as an expression of their individuality rather than material success  
@ are important to marketers because of the size of the segment  
  are aged 65 or older  
  are accurately described by none of the above

Bruce has just completed the "figure out where you are" stage of developing a positioning strategy. Which of the following statements would lead you to believe he has done a good job.  
 
  We have listened to salespeople, talked to our best people and surveyed our suppliers.  
  We have silenced our critics once and for all.  
  I have personally reviewed the recommendations and have deleted the portions of the report that don't apply to our   situation.  
@ We now have an honest and balanced understanding of who our customers perceive us to be.  

   Bruce has the Coppertone Suntan Lotion Franchise for the Bay area. He has two stores - one in Pacifica (on the coast) and one in Walnut Creek (in the desert). He sells suntan lotion with a sun protection factor (SPF) of 5 in Pacifica and a SPF of 45 in Walnut Creek. Bruce is using __________ segmentation.  

@  Geographic  
  Demographic  
  Lifestyle  
  Behavioral

Companies that successfully practice customer relationship management (CRM) measure success by looking at:  

  share of market and net present value of customer over a period of years  
  business potential and industry potential  
  share of customer and lifetime value of the customer  
  a perceptual map and share of voice  
  current future value of the customer and share of market  

   Connors Bros., a Maritime seafood products manufacturer, hopes to attract Ontario consumers for its Brunswick sardines through an ad campaign pushing the small fish as a positive food choice. Its _____ strategy aims to nullify the notion that sardines are boring, by stressing their health benefits and their good taste.  

  customerization  
  disintermediation  
  undifferentiated marketing  
  diversification  
@ repositioning  
 
   Connors Bros., a Maritime seafood products manufacturer, hopes to attract Ontario consumers for its Brunswick sardines through an ad campaign pushing the small fish as a positive food choice. Its _____ strategy aims to nullify the notion that sardines are boring, by stressing their health benefits and their good taste.  

  customerization  
  disintermediation  
  undifferentiated marketing  
  diversification  
@ repositioning  

  Customer relationship management (CRM) programs are also called:  

  transactional marketing programs  
  market segmentation strategies  
@ one-to-one marketing  
  marketing with a product-orientation  
  undifferentiated marketing 


   Emmis Broadcasting owns several radio stations in New York. Its KISS-FM plays smooth rhythm and blues and classic soul music to appeal to older listeners. Its WQHT-FM ("Hot 97") plays hip-hop for listeners in the under-25 segment. Emmis Broadcasting uses a(n) _____ targeting strategy.  

  fragmented  
  undifferentiated  
  concentrated  
@ differentiated  
  customized

 Five Alive markets fruit-based drinks in Canada. Its two strongest competitors are Sunny Delight and Tropicana Tropics, which target teens and older consumers, respectively. Five Alive was targeting to everyone in between. This strategy was not producing the desired results. In 2001, Five Alive used a(n) _____ strategy and began focusing its marketing efforts on younger and hipper 24- to 34-year-old consumers.  

  customerization  
  disintermediation  
  undifferentiated marketing  
  diversification  
@ repositioning  

  If a company wanted to send e-mail to African American professional managers and business owners to promote its new employment service, it could buy a list of e-mail addresses of people who have visited Black Enterprise.com and who have given permission for the marketing of their names and e-mail addresses. This company would be using _____ segmentation variables.  

  psychographic  
@ demographic  
  geodemographic  
  ethnocentric  
  behavioral  

If Carter Carpet Mills sold $75 million worth of carpeting in an industry whose total sales last year were $2.5 billion, its _____ was 3 percent.  

  economic forecast  
  sales forecast  
  market potential  
  market index  
@ market share  

In class we talked about how America loves a winner. Which of these statements best illustrates how a marketer might develop a positioning strategy based on that concept?  

  People always root for the underdog.  
  It is important to associate your name with that of a winner.  
  The best way to stake out a position is to sponsor a sporting event.  
@ It is better to be first in a small category than to be second in a large category. 

   What is the hard way to become a category leader discussed in class?  

  Advertise your product on the same television shows as the category leader.  
  Use comparative advertising to show how your product is superior to that of the category leader.  
@ Carefully manage the four "p's" (product, price, promotion, place) in a planned move to overtake the category leader.  
  Reduce prices in order to gain market share at the expense of the category leader.  


   What did your instructor mean when he spoke of the "fit" between a potential target market and the organization?  

@ Is there a match between the benefits offered by the organization and the needs of the members of the segment?  
  Do the customers in the segment deserve the benefits the organization offers?  
  Is the market segment growing or shrinking?  
  Are there additional segments similar to the one being targeted?  

   What is a major disadvantage of the market factor analysis method of forecasting?  

  The factors associated with a market change each season.  
  It is a very difficult method to use effectively.  
  It only works in industrial markets.  
  You can only use it to generate short-term (1 year or less) forecasts.  
@ In order to use it, there has to be a known market factor that correlates with your product's sales.  

What is the easy way to become a category leader?  

  Introduce your product with a commercial during the Super Bowl.  
@ Be the first company to offer a product in a category.  
  Spend more money on advertising than the category leader.  
  Develop a better product than the category leader offers.  

 What is the hard way to become a category leader discussed in class?  

  Advertise your product on the same television shows as the category leader.  
  Use comparative advertising to show how your product is superior to that of the category leader.  
@ Carefully manage the four "p's" (product, price, promotion, place) in a planned move to overtake the category leader.  
  Reduce prices in order to gain market share at the expense of the category leader.  

What is the point your instructor was trying to make by using the portfolio approach to managing market segments?  

  By combining marketing programs for different segments, you can acheive economies of scale.  
  The Boston Consulting Group (BCG) matrix classifies segments as stars, dogs, question marks or cash cows.  
  The portfolio approach works best when a company chooses a concentration strategy.  
@ For long-term growth, you need a mix of segments that are currently profitable as well as segments that might become    profitable in the future 

   What is the "textbook" definition of a forecast?  

  The probability of achieving a given sales level based on the analysis of past trends.  
@ The estimated sales of a product for a defined future period  
  An estimate of the product, price, place and promotion needed to acheive a given level of sales in a future period.  
  The amount of a product you intend to sell in a specified future period.  

   When many men in the Baby boomer segment starting getting gray hair, they were reluctant to use hair dyes made for women even though men were just as eager as women to fight the aging process. As a result, hair care product manufacturers introduced hair dying systems for men to use on their head and their facial hair. The hair care product manufacturers used _____ segmentation to define its target markets.  

  lifestyle  
  behavioral  
  socioeconomic  
@ demographic  
  psychographic 

Which of the following is a factor to consider when deciding whether or not to target a specific market segment?  

  Whether or not there is a recognized name for the segment  
@ How targeting the segment might impact other marketing programs already in place  
  The degree of separation  
  Whether the motivation for the segment is internal or external  

 Which of the following is a false assumption marketers frequently make when marketing to ethnic groups discussed in class?  

  Ethnic groups are homogeneous  
@ Ethnic groups are a growing segment  
  Ethnicity can be combined with other segmentation variables to develop meaningful segments  
  Ethnic marketing is based on stereotyping  

  Which of the following statements about forecasting is true  

  Long-term forecasts are more accurate than short-term forecasts.  
  In order to develop an actionable forecast, you have to use three or more different methods.  
  When the stakes are low (not much at risk if you make a bad decision) you should make a greater effort in your sales    forecast.  
  The past sales method is more accurate in volatile (high uncertainty) markets.  
@ All forecasts are guesses.

   Which of the following is one of the requirements for a market segment discussed in class?  

  Geographic  
  Behavioral  
@ Measurable  
  Low cost producer 

   Which of these segmentation variables might be useful for the makers of Gerber Baby Food?  

  Differentiation  
  NAICS Data  
@ Family life cycle  
  Geographic 


   Which of the following statements about the steps in the target marketing process is true?  

  The first step in the target marketing process is to set marketing objectives.  
  There are four steps in the target marketing process.  
  The second step in the target marketing process is the identification of all segmentation variables.  
@ The final step in the target marketing process is positioning.  
  All of the above statements about the target marketing process are true.  

   Which of the following measures would most likely be used in segmenting business markets?  
 
  Psychographics  
@ NAICS Data  
  Census Data  
  Cumulative grade point average

   Which of the following is a disadvantage of Customer Relationship Management (CRM) discussed in class?  

  The relationship is more important to the customer than it is to the manufacturer.  
  CRM works best in a shrinking market segment.  
@ The relationship is more important to the producer than it is to the customer.  
  CRM only works when the market segment is growing.  

Which of the following is a factor to consider when deciding whether or not to target a specific market segment?  

  Whether or not there is a recognized name for the segment  
@ How targeting the segment might impact other marketing programs already in place  
  The degree of separation  
  Whether the motivation for the segment is internal or external  

   Which of the following is a major disadvantage of the test-market approach to forecasting.  

  It only works in markets where demand is relatively stable.  
  Test markets can only provide short-term (1 year or less) market forecasts..  
  Tests markets for new products tend to have relatively short-term (1 year or less) forecasts.  
@ The time it takes to conduct the test market may put you at a competitive disadvantage in the marketplace.  
  Test markets only work in markets where demand fluctuates wildly (high volatility).  

Which of the following is an example of an organizational demographic?  

  whether a business is an international or a domestic company  
  a firm's total sales  
  the number of employees a firm has  
  the number of manufacturing plants a company has  
@ all of the above 

   Which of the following is the best reason a mattress comapny may use geographic segmentation?  

  People living on the East coast sleep differently than people living on the West coast  
  It is less expensive to advertise in the Midwest than it is to advertise on the Coasts.  
@ Because mattresses are very bulky and is is more efficient to produce them near where the customers live than to produce    them at a central location and ship them to the customer.  
  Because older people live in the South and older people spend more time using mattresses.  

   Why does your instructor recommend the chain ratio method of sales forecasting?  

@ Because it integrates sales forecasting into the strategic management process.  
  Because is uses math to quantify subjective issues.  
  Because it is the easiest method of forecasting.  
  Because it is the best approach for very long-term (3 years or more) forecasts.  

   Why is it important to develop contingency plans when you adopt a sales forecast?  

  Because you need the best case and worst case scenarios.  
  Because you create a best case, a worst case, and a most likely scenario.  
@ Because once a plan is underway, there is a tendency to explain away unexpected results rather than try to adapt the plan    to the new circumstances.  
  A contingency plan is what you do if the forecast is correct.  
  A contingency plan is what you do if a forecast is incorrect.  

   With a _____ marketing strategy, products and the messages about them are tailored to the individual customer.  

  fragmented  
  heterogeneous  
  concentrated  
  differentiated  
@ customized  

   _____ data is useful in understanding why some people who are over 65 lead active lives, take regular vacations, and eat a healthy diet while others in the same age segment do not exercise, eat and drink to excess, and refuse to walk more than five feet to pick up their mail.  

  Demographic  
  Geoclustering  
  Ethnocentric  
@ Psychographic  
  Socioeconomic  

_____ is the process of dividing a larger market into smaller pieces based on one or more meaningful, shared characteristics.  
  Acculturation  
@ Segmentation  
  Customerization  
  Market aggregation  
  Positioning  

   _____ means developing a marketing strategy aimed at influencing how a particular market segment perceives a good or service in comparison to the competition.  

  Concentrated targeting  
@ Positioning  
  Benefit segmentation  
  Mass customization  
  Customerization 

 _____ refers to the proportion of total sales of a product during a stated period in a specific market that is captured by a single firm.  

  Market share  
  Sales potential  
  Sales index  
  Economic potential  
@ Market index 

   _____ segmentation variables classifies consumers on the basis of how they act toward, feel about, or use a product.  

  Demographic  
@ Behavioral  
  Geodemographic  
  Socioeconomic  
  Psychographic

_____ variables segments markets in terms of shared activities, interests, and opinions.  

  Demographic  
  Ethnographic  
  Geodemographic  
  Socioeconomic  
@ Psychographic 

Question 1    Multiple Choice  1 of 1 point(s)    
  With a _____ marketing strategy, products and the messages about them are tailored to the individual customer.  
 Your Answer:  customized  
Correct Answer:  customized  
 


  Question 2    Multiple Choice  1 of 1 point(s)    
  In class we talked about how America loves a winner. Which of these statements best illustrates how a marketer might develop a positioning strategy based on that concept?  
 Your Answer:  It is better to be first in a small category than to be second in a large category.  
Correct Answer:  It is better to be first in a small category than to be second in a large category.  
 


  Question 3    Multiple Choice  0 of 1 point(s)    
  The average collectors of Thomas Kinkade collectible miniatures will buy four new ones each year. The collectibles are available at a number of different retail locations and from several different catalog retailers. To ensure she gets a greater _____, the owner of The Pine Door Gift Shop sends her customers, who are Thomas Kinkade collectors, a postcard every time the store adds a new one to her store's inventory.  
 Your Answer:  [None Given]  
Correct Answer:  share of customer  
 


  Question 4    Multiple Choice  0 of 1 point(s)    
  A former top manager at Mazda was quoted as saying, "Customers want to express their individuality with the products they buy." His statement reflects a belief that companies need to develop a _____ marketing strategy.  
 Your Answer:  [None Given]  
Correct Answer:  customized  
 


  Question 5    Multiple Choice  0 of 1 point(s)    
  _____ variables segments markets in terms of shared activities, interests, and opinions.  
 Your Answer:  [None Given]  
Correct Answer:  Psychographic  
 


  Question 6    Multiple Choice  0 of 1 point(s)    
  Which of the following is a factor to consider when deciding whether or not to target a specific market segment?  
 Your Answer:  [None Given]  
Correct Answer:  How targeting the segment might impact other marketing programs already in place  
 


  Question 7    Multiple Choice  0 of 1 point(s)    
  Perceptual maps:  
 Your Answer:  [None Given]  
Correct Answer:  cannot be created without consumer research  
 


  Question 8    Multiple Choice  0 of 1 point(s)    
  Which of the following is the best reason a mattress comapny may use geographic segmentation?  
 Your Answer:  [None Given]  
Correct Answer:  Because mattresses are very bulky and is is more efficient to produce them near where the customers live than to produce them at a central location and ship them to the customer.  
 


  Question 9    Multiple Choice  0 of 1 point(s)    
  Which of the following measures would most likely be used in segmenting business markets?  
 Your Answer:  [None Given]  
Correct Answer:  NAICS Data  
 


  Question 10    Multiple Choice  0 of 1 point(s)    
  A perceptual map is used to:  
 Your Answer:  [None Given]  
Correct Answer:  determine how consumers position products in relation to each other 
 

Question 1    Multiple Choice  1 of 1 point(s)    
  What did your instructor mean when he spoke of the "fit" between a potential target market and the organization?  
 Your Answer:  Is there a match between the benefits offered by the organization and the needs of the members of the segment?  
Correct Answer:  Is there a match between the benefits offered by the organization and the needs of the members of the segment?  
 


  Question 2    Multiple Choice  0 of 1 point(s)    
  The average collectors of Thomas Kinkade collectible miniatures will buy four new ones each year. The collectibles are available at a number of different retail locations and from several different catalog retailers. As part of her _____ program, the owner of The Pine Door Gift Shop sends her customers, who are Thomas Kinkade collectors, a postcard every time the store adds a new one to her store's inventory.  
 Your Answer:  [None Given]  
Correct Answer:  customer equity  
 


  Question 3    Multiple Choice  0 of 1 point(s)    
  One of the reasons General Motors deleted the Oldsmobile brand from its product line is because it was unable to develop a marketing strategy that caused consumers to perceive the Oldsmobile as having significant differences from other automobile brands that were currently on the market. In other words, the deletion of the Oldsmobile brand resulted from the absence of an effective _____ strategy.  
 Your Answer:  [None Given]  
Correct Answer:  positioning  
 


  Question 4    Multiple Choice  0 of 1 point(s)    
  According to your instructor, what is the second step you need to take in order to develop a positioning strategy?  
 Your Answer:  [None Given]  
Correct Answer:  Figure out where you want to be  
 


  Question 5    Multiple Choice  0 of 1 point(s)    
  The most important aspect of customer relationship management (CRM) is that it:  
 Your Answer:  [None Given]  
Correct Answer:  requires organizations to look at customers as partners  
 


  Question 6    Multiple Choice  0 of 1 point(s)    
  Which of the following is a disadvantage of Customer Relationship Management (CRM) discussed in class?  
 Your Answer:  [None Given]  
Correct Answer:  The relationship is more important to the producer than it is to the customer.  
 


  Question 7    Multiple Choice  0 of 1 point(s)    
  Age, ethnicity, geographical region, and lifestyle are all examples of:  
 Your Answer:  [None Given]  
Correct Answer:  segmentation variables  
 


  Question 8    Multiple Choice  0 of 1 point(s)    
  Bruce has the Coppertone Suntan Lotion Franchise for the Bay area. He has two stores - one in Pacifica (on the coast) and one in Walnut Creek (in the desert). He sells suntan lotion with a sun protection factor (SPF) of 5 in Pacifica and a SPF of 45 in Walnut Creek. Bruce is using __________ segmentation.  
 Your Answer:  [None Given]  
Correct Answer:  Geographic  
 


  Question 9    Multiple Choice  0 of 1 point(s)    
  According to Ries and trout, what is the goal of positioning?  
 Your Answer:  [None Given]  
Correct Answer:  To become the first thing that comes to mind in the category.  
 


  Question 10    Multiple Choice  0 of 1 point(s)    
  Ramada Franchising Enterprises offers a variety of lodging alternatives. The Ramada Limited serves the economy-minded traveler. The Ramada Inn is designed for those travelers seeking a mid-priced, full-service hotel. Ramada Plaza offers three-star service in the upper-middle price range. Its Ramada Renaissance hotels offer four-star service at the higher end of the price continuum. Ramada uses a(n) _____ targeting strategy with its lodging offerings.  
 Your Answer:  [None Given]  
Correct Answer:  differentiated  
 

  Question 1    Multiple Choice  0 of 1 point(s)    
  What is meant by the term "confirmation bias"  
 Your Answer:  [None Given]  
Correct Answer:  People tend to interpret advertisements in a way that agrees with their expectations.  
 


  Question 2    Multiple Choice  0 of 1 point(s)    
  Shareen is an executive for Robertson industries, a manufacturer of food processing equipment. Each year, she is a sponsor of the food processors golf outing. As a sponsor, Shareen plays golf with purchasing agents from potential customers. She is always alert for news that one of her golf partners is going to add a new food processing line. Shareen's approach works best for which type of buying decision?  
 Your Answer:  [None Given]  
Correct Answer:  New task buying  
 


  Question 3    Multiple Choice  0 of 1 point(s)    
  Celebrity-HealthWatch.com features diseases and ailments along with information on celebrities who have dealt with them. The site is designed to capitalize on relaxed FDA regulations that allow increased direct sales to consumers by pharmaceutical companies and the belief that learning how celebrities deal with their illnesses will influence how people deal with their own illnesses through the use of:  
 Your Answer:  [None Given]  
Correct Answer:  cognitive learning  
 


  Question 4    Multiple Choice  0 of 1 point(s)    
  Sara Lee found that consumers who had negative _______________ about themselves preferred portioned snack items because they felt they lacked the will power to regulate how much they ate without help.  
 Your Answer:  [None Given]  
Correct Answer:  self-concepts  
 


  Question 5    Multiple Choice  0 of 1 point(s)    
  A reference group:  
 Your Answer:  [None Given]  
Correct Answer:  is a set of people a consumer wants to please or imitate  
 


  Question 6    Multiple Choice  0 of 1 point(s)    
  How does the law of large numbers apply to personality types?  
 Your Answer:  [None Given]  
Correct Answer:  We can't predict how an individual will respond to an advertisement, but we can predict how groups of people with similar personalities might respond.  
 


  Question 7    Multiple Choice  0 of 1 point(s)    
  eBags is a Denver-based manufacturer of luggage, handbags, backpacks, and duffel bags. While much of the company's business is done over the Internet, it discovered it was missing many sales to customers who prefer to deal with an individual operator. Instead of hiring its own telephone agents to work weekends, off-hours, and peak-call periods, eBags hired Finali Corp, a teleservices company, to take all of its phone orders from customers and to answer commonly asked customer questions. eBags used:  
 Your Answer:  [None Given]  
Correct Answer:  outsourcing  
 


  Question 8    Multiple Choice  0 of 1 point(s)    
  Chen woke up in the middle of the night feeling hungry. He walked to the refrigerator, grabbed the first thing he saw, ate it, and then went back to bed. This is an example of:  
 Your Answer:  [None Given]  
Correct Answer:  Cognitive miserliness  
 


  Question 9    Multiple Choice  0 of 1 point(s)    
  The ad for the San Francisco State University MBA program said, "Reach your full potential in the business world with an Executive MBA from SFSU." The program indicates that it satisfies which of Maslow's needs?  
 Your Answer:  [None Given]  
Correct Answer:  self-actualization needs  
 


  Question 10    Multiple Choice  0 of 1 point(s)    
  The North American Industry Classification System (NAICS):  
 Your Answer:  [None Given]  
Correct Answer:  uses a six-digit code that sorts industries into subsectors, industry groups, industries, and specific country industries  
 
 Question 1    Multiple Choice  1 of 1 point(s)    
  Five Alive markets fruit-based drinks in Canada. Its two strongest competitors are Sunny Delight and Tropicana Tropics, which target teens and older consumers, respectively. Five Alive was targeting to everyone in between. This strategy was not producing the desired results. In 2001, Five Alive used a(n) _____ strategy and began focusing its marketing efforts on younger and hipper 24- to 34-year-old consumers.  
 Your Answer:  repositioning  
Correct Answer:  repositioning  
 


  Question 2    Multiple Choice  0 of 1 point(s)    
  Which of these statements about Porter's fundamental strategies is true?  
 Your Answer:  [None Given]  
Correct Answer:  There is only room for one low cost producer in a market  
 


  Question 3    Multiple Choice  0 of 1 point(s)    
  Which of the following is a factor to consider when deciding whether or not to target a specific market segment?  
 Your Answer:  [None Given]  
Correct Answer:  Whether the segment is growing or shrinking  
 


  Question 4    Multiple Choice  0 of 1 point(s)    
  According to your instructor, what is the second step you need to take in order to develop a positioning strategy?  
 Your Answer:  [None Given]  
Correct Answer:  Figure out where you want to be  
 


  Question 5    Multiple Choice  0 of 1 point(s)    
  Which of the following is one of the requirements for a market segment discussed in class?  
 Your Answer:  [None Given]  
Correct Answer:  Reachable  
 


  Question 6    Multiple Choice  0 of 1 point(s)    
  A viable target market should:  
 Your Answer:  [None Given]  
Correct Answer:  be all of the above  
 


  Question 7    Multiple Choice  0 of 1 point(s)    
  Baby boomers:  
 Your Answer:  [None Given]  
Correct Answer:  are important to marketers because of the size of the segment  
 


  Question 8    Multiple Choice  0 of 1 point(s)    
  _____ data is useful in understanding why some people who are over 65 lead active lives, take regular vacations, and eat a healthy diet while others in the same age segment do not exercise, eat and drink to excess, and refuse to walk more than five feet to pick up their mail.  
 Your Answer:  [None Given]  
Correct Answer:  Psychographic  
 


  Question 9    Multiple Choice  0 of 1 point(s)    
  Which of the following is one of the requirements for a market segment discussed in class?  
 Your Answer:  [None Given]  
Correct Answer:  Measurable  
 


  Question 10    Multiple Choice  0 of 1 point(s)    
  Recognizing that some people eat cereal because of its nutritional value, that some people eat cereal because it is easy to prepare, and some eat cereal because it tastes good is part of the _____ process.  
 Your Answer:  [None Given]  
Correct Answer:  segmentation  
 




 Question 1    Multiple Choice  0 of 1 point(s)    
  The most important aspect of customer relationship management (CRM) is that it:  
 Your Answer:  requires organizations to view their customers as adversaries  
Correct Answer:  requires organizations to look at customers as partners  
 


  Question 2    Multiple Choice  0 of 1 point(s)    
  _____ refers to the proportion of total sales of a product during a stated period in a specific market that is captured by a single firm.  
 Your Answer:  Market index  
Correct Answer:  Market share  
 


  Question 3    Multiple Choice  1 of 1 point(s)    
  Bruce has the Coppertone Suntan Lotion Franchise for the Bay area. He has two stores - one in Pacifica (on the coast) and one in Walnut Creek (in the desert). He sells suntan lotion with a sun protection factor (SPF) of 5 in Pacifica and a SPF of 45 in Walnut Creek. Bruce is using __________ segmentation.  
 Your Answer:  Geographic  
Correct Answer:  Geographic  
 


  Question 4    Multiple Choice  0 of 1 point(s)    
  Magazines targeted to people who love to in-line skate, to people who enjoy cooking, and to people who would prefer a simpler lifestyle use _____ segmentation variables.  
 Your Answer:  [None Given]  
Correct Answer:  pychographic  
 


  Question 5    Multiple Choice  0 of 1 point(s)    
  A company that uses a(n) _____ targeting strategy is appealing to a broad spectrum of people.  
 Your Answer:  [None Given]  
Correct Answer:  undifferentiated  
 


  Question 6    Multiple Choice  0 of 1 point(s)    
  What is the easy way to become a category leader?  
 Your Answer:  [None Given]  
Correct Answer:  Be the first company to offer a product in a category.  
 


  Question 7    Multiple Choice  0 of 1 point(s)    
  A national retailer of uniforms targeted male nurses in a recent advertising campaign. It used _____ segmentation to define its target market.  
 Your Answer:  [None Given]  
Correct Answer:  demographic  
 


  Question 8    Multiple Choice  0 of 1 point(s)    
  Which of the following statements about the steps in the target marketing process is true?  
 Your Answer:  [None Given]  
Correct Answer:  The final step in the target marketing process is positioning.  
 


  Question 9    Multiple Choice  0 of 1 point(s)    
  Why is it important to develop contingency plans when you adopt a sales forecast?  
 Your Answer:  [None Given]  
Correct Answer:  Because once a plan is underway, there is a tendency to explain away unexpected results rather than try to adapt the plan to the new circumstances.  
 


  Question 10    Multiple Choice  0 of 1 point(s)    
  Tetra Food supplies 80 percent of the food for feeding tropical fish. Since Tetra Food only sells fish food and has no desire to sell other types of pet food, it can be said Tetra Food is using a(n) _____ targeting strategy.  
 Your Answer:  [None Given]  
Correct Answer:  concentrated  
 

Question 1    Multiple Choice  0 of 1 point(s)    
  Positioning strategies often try to create a _____ for a good or service-a distinct image that captures its character and benefits.  
 Your Answer:  [None Given]  
Correct Answer:  brand personality  
 


  Question 2    Multiple Choice  0 of 1 point(s)    
  What is the "textbook" definition of a forecast?  
 Your Answer:  [None Given]  
Correct Answer:  The estimated sales of a product for a defined future period  
 


  Question 3    Multiple Choice  0 of 1 point(s)    
  The first step in developing a positioning strategy is to:  
 Your Answer:  [None Given]  
Correct Answer:  analyze the competitors' positions in the marketplace  
 


  Question 4    Multiple Choice  0 of 1 point(s)    
  Progressive Corporation began as a small insurance provider that sold "nonstandard" auto insurance to risky drivers with a record of auto accidents or drunken driving. In those days, Progressive used a(n) _____ targeting strategy.  
 Your Answer:  [None Given]  
Correct Answer:  concentrated  
 


  Question 5    Multiple Choice  0 of 1 point(s)    
  Which of these segmentation variables might be useful for the makers of Gerber Baby Food?  
 Your Answer:  [None Given]  
Correct Answer:  Family life cycle  
 


  Question 6    Multiple Choice  0 of 1 point(s)    
  Tetra Food supplies 80 percent of the food for feeding tropical fish. Since Tetra Food only sells fish food and has no desire to sell other types of pet food, it can be said Tetra Food is using a(n) _____ targeting strategy.  
 Your Answer:  [None Given]  
Correct Answer:  concentrated  
 


  Question 7    Multiple Choice  0 of 1 point(s)    
  According to your instructor, what is a major disadvantage of the trend analysis method of forecasting?  
 Your Answer:  [None Given]  
Correct Answer:  Relying on statistical analysis means you risk ignoring forseeable environmental factors.  
 


  Question 8    Multiple Choice  0 of 1 point(s)    
  Age, ethnicity, geographical region, and lifestyle are all examples of:  
 Your Answer:  [None Given]  
Correct Answer:  segmentation variables  
 


  Question 9    Multiple Choice  0 of 1 point(s)    
  Which of the following is an example of an organizational demographic?  
 Your Answer:  [None Given]  
Correct Answer:  all of the above  
 


  Question 10    Multiple Choice  0 of 1 point(s)    
  In class we talked about how America loves a winner. Which of these statements best illustrates how a marketer might develop a positioning strategy based on that concept?  
 Your Answer:  [None Given]  
Correct Answer:  It is better to be first in a small category than to be second in a large category.  
 

Question 1    Multiple Choice  1 of 1 point(s)    
  _____ means developing a marketing strategy aimed at influencing how a particular market segment perceives a good or service in comparison to the competition.  
 Your Answer:  Positioning  
Correct Answer:  Positioning  
 


  Question 2    Multiple Choice  1 of 1 point(s)    
  A viable target market should:  
 Your Answer:  be all of the above  
Correct Answer:  be all of the above  
 


  Question 3    Multiple Choice  1 of 1 point(s)    
  Which of the following statements about the steps in the target marketing process is true?  
 Your Answer:  The final step in the target marketing process is positioning.  
Correct Answer:  The final step in the target marketing process is positioning.  
 


  Question 4    Multiple Choice  0 of 1 point(s)    
  Campbell's found its canned nacho cheese sauce was too hot for Americans in the East and not hot enough for those in the West and the Southwest. As a result, Campbell's plants in Texas and California produce a hotter nacho cheese sauce than that produced in its other plants. Campbell's is using _____ segmentation to satisfy these different markets.  
 Your Answer:  [None Given]  
Correct Answer:  geographic  
 


  Question 5    Multiple Choice  0 of 1 point(s)    
  A retailer that wanted to target prospective grooms between the ages of 21 and 40 who are within six months of their marriage is using _____ segmentation to define its target market.  
 Your Answer:  [None Given]  
Correct Answer:  demographic  
 


  Question 6    Multiple Choice  0 of 1 point(s)    
  Which of these segmentation variables might be useful for the makers of Gerber Baby Food?  
 Your Answer:  [None Given]  
Correct Answer:  Family life cycle  
 


  Question 7    Multiple Choice  0 of 1 point(s)    
  A company that uses a(n) _____ targeting strategy is appealing to a broad spectrum of people.  
 Your Answer:  [None Given]  
Correct Answer:  undifferentiated  
 


  Question 8    Multiple Choice  0 of 1 point(s)    
  Some beer drinkers choose their beer because of its great taste. Others like to drink beers to relax but don't like the bloated feeling beer sometimes gives them. An ad campaign for a beer that showed the beer had both a great taste and was less filling would be one way to overcome problems associated with:  
 Your Answer:  [None Given]  
Correct Answer:  market fragmentation  
 


  Question 9    Multiple Choice  0 of 1 point(s)    
  Market potential is the:  
 Your Answer:  [None Given]  
Correct Answer:  total sales volume that all organizations selling a product during a stated time period in a specific market can expect to achieve under ideal conditions.  
 


  Question 10    Multiple Choice  0 of 1 point(s)    
  Baby boomers:  
 Your Answer:  [None Given]  
Correct Answer:  are important to marketers because of the size of the segment  
 


Question 1    Multiple Choice  1 of 1 point(s)    
  In class we talked about how America loves a winner. Which of these statements best illustrates how a marketer might develop a positioning strategy based on that concept?  
 Your Answer:  It is better to be first in a small category than to be second in a large category.  
Correct Answer:  It is better to be first in a small category than to be second in a large category.  
 


  Question 2    Multiple Choice  0 of 1 point(s)    
  The group of American consumers born between 1977 and 1994 are often called:  
 Your Answer:  [None Given]  
Correct Answer:  Generation Y  
 


  Question 3    Multiple Choice  0 of 1 point(s)    
  Tetra Food supplies 80 percent of the food for feeding tropical fish. Since Tetra Food only sells fish food and has no desire to sell other types of pet food, it can be said Tetra Food is using a(n) _____ targeting strategy.  
 Your Answer:  [None Given]  
Correct Answer:  concentrated  
 


  Question 4    Multiple Choice  0 of 1 point(s)    
  Which of these segmentation variables might be useful for the makers of Gerber Baby Food?  
 Your Answer:  [None Given]  
Correct Answer:  Family life cycle  
 


  Question 5    Multiple Choice  0 of 1 point(s)    
  What is the easy way to become a category leader?  
 Your Answer:  [None Given]  
Correct Answer:  Be the first company to offer a product in a category.  
 


  Question 6    Multiple Choice  0 of 1 point(s)    
  The average collectors of Thomas Kinkade collectible miniatures will buy four new ones each year. The collectibles are available at a number of different retail locations and from several different catalog retailers. To ensure she gets a greater _____, the owner of The Pine Door Gift Shop sends her customers, who are Thomas Kinkade collectors, a postcard every time the store adds a new one to her store's inventory.  
 Your Answer:  [None Given]  
Correct Answer:  share of customer  
 


  Question 7    Multiple Choice  0 of 1 point(s)    
  The only time many people entertain is during November and December when their homes are decorated for the holidays. The increase in ads for prepared trays of food to be served at these functions is directly related to _____ segmentation.  
 Your Answer:  [None Given]  
Correct Answer:  behavioral  
 


  Question 8    Multiple Choice  0 of 1 point(s)    
  A retailer that wanted to target prospective grooms between the ages of 21 and 40 who are within six months of their marriage is using _____ segmentation to define its target market.  
 Your Answer:  [None Given]  
Correct Answer:  demographic  
 


  Question 9    Multiple Choice  0 of 1 point(s)    
  _____ means developing a marketing strategy aimed at influencing how a particular market segment perceives a good or service in comparison to the competition.  
 Your Answer:  [None Given]  
Correct Answer:  Positioning  
 


  Question 10    Multiple Choice  0 of 1 point(s)    
  What is the hard way to become a category leader discussed in class?  
 Your Answer:  [None Given]  
Correct Answer:  Carefully manage the four "p's" (product, price, promotion, place) in a planned move to overtake the category leader.  
 



Question 1    Multiple Choice  0 of 1 point(s)    
  Some people want healthy cereal that will help them lose weight or possibly prevent certain kinds of cancer. Some want presweetened cereal. Some want fruits and nuts in their cereal. Some people want whole-wheat cereal while others want a rice cereal. There are some who like hot cereal and others who like to eat their cereal right of the box. The diversity of consumers for cereal is an example of:  
 Your Answer:  [None Given]  
Correct Answer:  market fragmentation  
 


  Question 2    Multiple Choice  0 of 1 point(s)    
  The average collectors of Thomas Kinkade collectible miniatures will buy four new ones each year. The collectibles are available at a number of different retail locations and from several different catalog retailers. As part of her _____ program, the owner of The Pine Door Gift Shop sends her customers, who are Thomas Kinkade collectors, a postcard every time the store adds a new one to her store's inventory.  
 Your Answer:  [None Given]  
Correct Answer:  customer equity  
 


  Question 3    Multiple Choice  0 of 1 point(s)    
  Which of the following statements about the steps in the target marketing process is true?  
 Your Answer:  [None Given]  
Correct Answer:  The final step in the target marketing process is positioning.  
 


  Question 4    Multiple Choice  0 of 1 point(s)    
  According to your instructor, what is a major disadvantage of the trend analysis method of forecasting?  
 Your Answer:  [None Given]  
Correct Answer:  Relying on statistical analysis means you risk ignoring forseeable environmental factors.  
 


  Question 5    Multiple Choice  0 of 1 point(s)    
  The Gillette corporation invests heavily in capital equipment in order to mass produce high quality razor blades very efficiently. This gives Gillette an advantage over its competitors. Gillette has adopted Michael Porter's _________ strategy.  
 Your Answer:  [None Given]  
Correct Answer:  Low cost producer  
 


  Question 6    Multiple Choice  0 of 1 point(s)    
  What is a major disadvantage of the market factor analysis method of forecasting?  
 Your Answer:  [None Given]  
Correct Answer:  In order to use it, there has to be a known market factor that correlates with your product's sales.  
 


  Question 7    Multiple Choice  0 of 1 point(s)    
  Companies that successfully practice customer relationship management (CRM) measure success by looking at:  
 Your Answer:  [None Given]  
Correct Answer:  share of customer and lifetime value of the customer  
 


  Question 8    Multiple Choice  0 of 1 point(s)    
  Bruce has the Coppertone Suntan Lotion Franchise for the Bay area. He has two stores - one in Pacifica (on the coast) and one in Walnut Creek (in the desert). He sells suntan lotion with a sun protection factor (SPF) of 5 in Pacifica and a SPF of 45 in Walnut Creek. Bruce is using __________ segmentation.  
 Your Answer:  [None Given]  
Correct Answer:  Geographic  
 


  Question 9    Multiple Choice  0 of 1 point(s)    
  Weekly Reader is a newspaper designed to keep school children informed of what is going on in their country and their world. It has 16 different formats geared to the reading capabilities of children from pre-kindergarten through high school. It uses _____ segmentation to define its target markets.  
 Your Answer:  [None Given]  
Correct Answer:  demographic  
 


  Question 10    Multiple Choice  0 of 1 point(s)    
  Recognizing that some people eat cereal because of its nutritional value, that some people eat cereal because it is easy to prepare, and some eat cereal because it tastes good is part of the _____ process.  
 Your Answer:  [None Given]  
Correct Answer:  segmentation  
 

People who are regular visitors to scifi.com (a Web site maintained by the science fiction cable network) and those who are regular visitors to BHGTV.com (a Web site maintained by the producers of BHG-TV and the publishers of Better Homes and Gardens magazine) belong to different _____ segments.  
 Your Answer:  [None Given]  
Correct Answer:  psychographic  

Question 1    Multiple Choice  1 of 1 point(s)    
  If a company wanted to send e-mail to African American professional managers and business owners to promote its new employment service, it could buy a list of e-mail addresses of people who have visited Black Enterprise.com and who have given permission for the marketing of their names and e-mail addresses. This company would be using _____ segmentation variables.  
 Your Answer:  demographic  
Correct Answer:  demographic  
 


  Question 2    Multiple Choice  0 of 1 point(s)    
  One of the goals of customer relationship management (CRM) programs is to:  
 Your Answer:  [None Given]  
Correct Answer:  increase the total profit generated by a customer over his or her lifetime  
 


  Question 3    Multiple Choice  0 of 1 point(s)    
  Baby boomers:  
 Your Answer:  [None Given]  
Correct Answer:  are important to marketers because of the size of the segment  
 


  Question 4    Multiple Choice  0 of 1 point(s)    
  The average collectors of Thomas Kinkade collectible miniatures will buy four new ones each year. The collectibles are available at a number of different retail locations and from several different catalog retailers. As part of her _____ program, the owner of The Pine Door Gift Shop sends her customers, who are Thomas Kinkade collectors, a postcard every time the store adds a new one to her store's inventory.  
 Your Answer:  [None Given]  
Correct Answer:  customer equity  
 


  Question 5    Multiple Choice  0 of 1 point(s)    
  Which of the following is a major disadvantage of the test-market approach to forecasting.  
 Your Answer:  [None Given]  
Correct Answer:  The time it takes to conduct the test market may put you at a competitive disadvantage in the marketplace.  
 


  Question 6    Multiple Choice  0 of 1 point(s)    
  Some beer drinkers choose their beer because of its great taste. Others like to drink beers to relax but don't like the bloated feeling beer sometimes gives them. An ad campaign for a beer that showed the beer had both a great taste and was less filling would be one way to overcome problems associated with:  
 Your Answer:  [None Given]  
Correct Answer:  market fragmentation  
 


  Question 7    Multiple Choice  0 of 1 point(s)    
  Progressive Corporation began as a small insurance provider that sold "nonstandard" auto insurance to risky drivers with a record of auto accidents or drunken driving. In those days, Progressive used a(n) _____ targeting strategy.  
 Your Answer:  [None Given]  
Correct Answer:  concentrated  
 


  Question 8    Multiple Choice  0 of 1 point(s)    
  Emmis Broadcasting owns several radio stations in New York. Its KISS-FM plays smooth rhythm and blues and classic soul music to appeal to older listeners. Its WQHT-FM ("Hot 97") plays hip-hop for listeners in the under-25 segment. Emmis Broadcasting uses a(n) _____ targeting strategy.  
 Your Answer:  [None Given]  
Correct Answer:  differentiated  
 


  Question 9    Multiple Choice  0 of 1 point(s)    
  Five Alive markets fruit-based drinks in Canada. Its two strongest competitors are Sunny Delight and Tropicana Tropics, which target teens and older consumers, respectively. Five Alive was targeting to everyone in between. This strategy was not producing the desired results. In 2001, Five Alive used a(n) _____ strategy and began focusing its marketing efforts on younger and hipper 24- to 34-year-old consumers.  
 Your Answer:  [None Given]  
Correct Answer:  repositioning  
 


  Question 10    Multiple Choice  0 of 1 point(s)    
  The first step in developing a positioning strategy is to:  
 Your Answer:  [None Given]  
Correct Answer:  analyze the competitors' positions in the marketplace  
 


Question 6    Multiple Choice  0 of 1 point(s)    
  The Hispanic American segment is important to marketers because:  
 Your Answer:  [None Given]  
Correct Answer:  of all of the above  
 


  Question 7    Multiple Choice  0 of 1 point(s)    
  _____ refers to the proportion of total sales of a product during a stated period in a specific market that is captured by a single firm.  
 Your Answer:  [None Given]  
Correct Answer:  Market share  
 


  Question 8    Multiple Choice  0 of 1 point(s)    
  Emmis Broadcasting owns several radio stations in New York. Its KISS-FM plays smooth rhythm and blues and classic soul music to appeal to older listeners. Its WQHT-FM ("Hot 97") plays hip-hop for listeners in the under-25 segment. Emmis Broadcasting uses a(n) _____ targeting strategy.  
 Your Answer:  [None Given]  
Correct Answer:  differentiated  
 


  Question 9    Multiple Choice  0 of 1 point(s)    
  The group of American consumers born between 1965 and 1976 are called:  
 Your Answer:  [None Given]  
Correct Answer:  Generation X  
 


  Question 10    Multiple Choice  0 of 1 point(s)    
  Why is it important to develop contingency plans when you adopt a sales forecast?  
 Your Answer:  [None Given]  
Correct Answer:  Because once a plan is underway, there is a tendency to explain away unexpected results rather than try to adapt the plan to the new circumstances.  
 

Question 4    Multiple Choice  0 of 1 point(s)    
  Which of the following is the best example of the "fit" between a producer and a potential target market?  
 Your Answer:  [None Given]  
Correct Answer:  The features provided by the organization are perceived as benefits by the target market.  
 


  Question 5    Multiple Choice  0 of 1 point(s)    
  What did your instructor mean when he spoke of the "fit" between a potential target market and the organization?  
 Your Answer:  [None Given]  
Correct Answer:  Is there a match between the benefits offered by the organization and the needs of the members of the segment?  
 


  Question 6    Multiple Choice  0 of 1 point(s)    
  Which of the following statements about the steps in the target marketing process is true?  
 Your Answer:  [None Given]  
Correct Answer:  The final step in the target marketing process is positioning.  
 


  Question 7    Multiple Choice  0 of 1 point(s)    
  The sales force composite as a method of forecasting demand is most effective when:  
 Your Answer:  [None Given]  
Correct Answer:  the market is composed of a relatively few, but large, customers.  
 


  Question 8    Multiple Choice  0 of 1 point(s)    
  _____ is the process of dividing a larger market into smaller pieces based on one or more meaningful, shared characteristics.  
 Your Answer:  [None Given]  
Correct Answer:  Segmentation  
 


  Question 9    Multiple Choice  0 of 1 point(s)    
  Weekly Reader is a newspaper designed to keep school children informed of what is going on in their country and their world. It has 16 different formats geared to the reading capabilities of children from pre-kindergarten through high school. It uses _____ segmentation to define its target markets.  
 Your Answer:  [None Given]  
Correct Answer:  demographic  
 


  Question 10    Multiple Choice  0 of 1 point(s)    
  Progressive Corporation began as a small insurance provider that sold "nonstandard" auto insurance to risky drivers with a record of auto accidents or drunken driving. In those days, Progressive used a(n) _____ targeting strategy.  
 Your Answer:  [None Given]  
Correct Answer:  concentrated  
 
Question 1    Multiple Choice  1 of 1 point(s)    
  What is a major disadvantage of the market factor analysis method of forecasting?  
 Your Answer:  In order to use it, there has to be a known market factor that correlates with your product's sales.  
Correct Answer:  In order to use it, there has to be a known market factor that correlates with your product's sales.  
 


  Question 2    Multiple Choice  0 of 1 point(s)    
  Starting in February and ending sometime in late April, department stores promote the fact they stock a wide range of formal attire to wear to high school proms. The dresses and tuxedoes are available year-round, but because most proms are held in the spring that is when the heaviest promotion of these products occur. This spring promotion to high school students is based on usage occasion and uses both ______ segmentation.  
 Your Answer:  [None Given]  
Correct Answer:  demographic and behavioral  
 


  Question 3    Multiple Choice  0 of 1 point(s)    
  A national retailer of uniforms targeted male nurses in a recent advertising campaign. It used _____ segmentation to define its target market.  
 Your Answer:  [None Given]  
Correct Answer:  demographic  
 


  Question 4    Multiple Choice  0 of 1 point(s)    
  People who are regular visitors to scifi.com (a Web site maintained by the science fiction cable network) and those who are regular visitors to BHGTV.com (a Web site maintained by the producers of BHG-TV and the publishers of Better Homes and Gardens magazine) belong to different _____ segments.  
 Your Answer:  [None Given]  
Correct Answer:  psychographic  
 


  Question 5    Multiple Choice  0 of 1 point(s)    
  Airlines have developed frequent-flyer programs to encourage passengers to use the same airlines repeatedly. These programs and other similar ones use _____ segmentation to target their markets.  
 Your Answer:  [None Given]  
Correct Answer:  behavioral  
 


  Question 6    Multiple Choice  0 of 1 point(s)    
  According to your instructor, what is the third step you need to take in order to develop a positioning strategy?  
 Your Answer:  [None Given]  
Correct Answer:  Develop a plan to get from where you are now to where you want to be  
 


  Question 7    Multiple Choice  0 of 1 point(s)    
  Which of the following measures would most likely be used in segmenting business markets?  
 Your Answer:  [None Given]  
Correct Answer:  NAICS Data  
 


  Question 8    Multiple Choice  0 of 1 point(s)    
  Which of these segmentation variables might be useful for the makers of Gerber Baby Food?  
 Your Answer:  [None Given]  
Correct Answer:  Family life cycle  
 


  Question 9    Multiple Choice  0 of 1 point(s)    
  Progressive Corporation began as a small insurance provider that sold "nonstandard" auto insurance to risky drivers with a record of auto accidents or drunken driving. In those days, Progressive used a(n) _____ targeting strategy.  
 Your Answer:  [None Given]  
Correct Answer:  concentrated  
 


  Question 10    Multiple Choice  0 of 1 point(s)    
  Which of the following is one of the requirements for a market segment discussed in class?  
 Your Answer:  [None Given]  
Correct Answer:  Reachable  
 


 Question 1    Multiple Choice  0 of 1 point(s)    
  The Wright Bros. Collection is a catalog retailer that sells clothing and collectibles to aviation enthusiasts. Aviation enthusiasts are the company's:  
 Your Answer:  [None Given]  
Correct Answer:  target market  
 


  Question 2    Multiple Choice  0 of 1 point(s)    
  Which of the following is an example of an organizational demographic?  
 Your Answer:  [None Given]  
Correct Answer:  all of the above  
 


  Question 3    Multiple Choice  0 of 1 point(s)    
  Weekly Reader is a newspaper designed to keep school children informed of what is going on in their country and their world. It has 16 different formats geared to the reading capabilities of children from pre-kindergarten through high school. It uses _____ segmentation to define its target markets.  
 Your Answer:  [None Given]  
Correct Answer:  demographic  
 


  Question 4    Multiple Choice  0 of 1 point(s)    
  According to your instructor, which of Michael Porter's fundamental strategies is most commonly adopted by businesses?  
 Your Answer:  [None Given]  
Correct Answer:  Differentiation  
 


  Question 5    Multiple Choice  0 of 1 point(s)    
  _____ data is useful in understanding why some people who are over 65 lead active lives, take regular vacations, and eat a healthy diet while others in the same age segment do not exercise, eat and drink to excess, and refuse to walk more than five feet to pick up their mail.  
 Your Answer:  [None Given]  
Correct Answer:  Psychographic  
 


  Question 6    Multiple Choice  0 of 1 point(s)    
  The first step in developing a positioning strategy is to:  
 Your Answer:  [None Given]  
Correct Answer:  analyze the competitors' positions in the marketplace  
 


  Question 7    Multiple Choice  0 of 1 point(s)    
  Which of the following is a factor to consider when deciding whether or not to target a specific market segment?  
 Your Answer:  [None Given]  
Correct Answer:  How targeting the segment might impact other marketing programs already in place  
 


  Question 8    Multiple Choice  0 of 1 point(s)    
  According to your instructor, what is the third step you need to take in order to develop a positioning strategy?  
 Your Answer:  [None Given]  
Correct Answer:  Develop a plan to get from where you are now to where you want to be  
 


  Question 9    Multiple Choice  0 of 1 point(s)    
  The average collectors of Thomas Kinkade collectible miniatures will buy four new ones each year. The collectibles are available at a number of different retail locations and from several different catalog retailers. To ensure she gets a greater _____, the owner of The Pine Door Gift Shop sends her customers, who are Thomas Kinkade collectors, a postcard every time the store adds a new one to her store's inventory.  
 Your Answer:  [None Given]  
Correct Answer:  share of customer  
 


  Question 10    Multiple Choice  0 of 1 point(s)    
  Customer relationship management (CRM) programs are also called:  
 Your Answer:  [None Given]  
Correct Answer:  one-to-one marketing  
 

 Question 1    Multiple Choice  0 of 1 point(s)    
  The Asian American segment of the population:  
 Your Answer:  [None Given]  
Correct Answer:  is the fastest growing minority group in the U.S.  
 


  Question 2    Multiple Choice  0 of 1 point(s)    
  _____ refers to the proportion of total sales of a product during a stated period in a specific market that is captured by a single firm.  
 Your Answer:  [None Given]  
Correct Answer:  Market share  
 


  Question 3    Multiple Choice  0 of 1 point(s)    
  A company that uses a(n) _____ targeting strategy is appealing to a broad spectrum of people.  
 Your Answer:  [None Given]  
Correct Answer:  undifferentiated  
 


  Question 4    Multiple Choice  0 of 1 point(s)    
  _____ segmentation variables classifies consumers on the basis of how they act toward, feel about, or use a product.  
 Your Answer:  [None Given]  
Correct Answer:  Behavioral  
 


  Question 5    Multiple Choice  0 of 1 point(s)    
  According to your instructor, what is the second step you need to take in order to develop a positioning strategy?  
 Your Answer:  [None Given]  
Correct Answer:  Figure out where you want to be  
 


  Question 6    Multiple Choice  0 of 1 point(s)    
  The Gillette corporation invests heavily in capital equipment in order to mass produce high quality razor blades very efficiently. This gives Gillette an advantage over its competitors. Gillette has adopted Michael Porter's _________ strategy.  
 Your Answer:  [None Given]  
Correct Answer:  Low cost producer  
 


  Question 7    Multiple Choice  0 of 1 point(s)    
  What is the easy way to become a category leader?  
 Your Answer:  [None Given]  
Correct Answer:  Be the first company to offer a product in a category.  
 


  Question 8    Multiple Choice  0 of 1 point(s)    
  According to your instructor, what is a major disadvantage of the trend analysis method of forecasting?  
 Your Answer:  [None Given]  
Correct Answer:  Relying on statistical analysis means you risk ignoring forseeable environmental factors.  
 


  Question 9    Multiple Choice  0 of 1 point(s)    
  _____ data is useful in understanding why some people who are over 65 lead active lives, take regular vacations, and eat a healthy diet while others in the same age segment do not exercise, eat and drink to excess, and refuse to walk more than five feet to pick up their mail.  
 Your Answer:  [None Given]  
Correct Answer:  Psychographic  
 


  Question 10    Multiple Choice  0 of 1 point(s)    
  When many men in the Baby boomer segment starting getting gray hair, they were reluctant to use hair dyes made for women even though men were just as eager as women to fight the aging process. As a result, hair care product manufacturers introduced hair dying systems for men to use on their head and their facial hair. The hair care product manufacturers used _____ segmentation to define its target markets.  
 Your Answer:  [None Given]  
Correct Answer:  demographic  
 

Question 1    Multiple Choice  1 of 1 point(s)    
  Which of the following is a factor to consider when deciding whether or not to target a specific market segment?  
 Your Answer:  Whether the segment is growing or shrinking  
Correct Answer:  Whether the segment is growing or shrinking  
 


  Question 2    Multiple Choice  1 of 1 point(s)    
  The group of American consumers born between 1965 and 1976 are called:  
 Your Answer:  Generation X  
Correct Answer:  Generation X  
 


  Question 3    Multiple Choice  0 of 1 point(s)    
  Sales potential is the:  
 Your Answer:  total sales volume that all organizations selling a product during a stated time period in a specific market can expect to sell under ideal conditions.  
Correct Answer:  portion of market potential a specific company could expect to achieve under ideal conditions.  
 


  Question 4    Multiple Choice  1 of 1 point(s)    
  Airlines have developed frequent-flyer programs to encourage passengers to use the same airlines repeatedly. These programs and other similar ones use _____ segmentation to target their markets.  
 Your Answer:  behavioral  
Correct Answer:  behavioral  
 


  Question 5    Multiple Choice  1 of 1 point(s)    
  Customer relationship management (CRM) programs are also called:  
 Your Answer:  one-to-one marketing  
Correct Answer:  one-to-one marketing  
 


  Question 6    Multiple Choice  1 of 1 point(s)    
  Connors Bros., a Maritime seafood products manufacturer, hopes to attract Ontario consumers for its Brunswick sardines through an ad campaign pushing the small fish as a positive food choice. Its _____ strategy aims to nullify the notion that sardines are boring, by stressing their health benefits and their good taste.  
 Your Answer:  repositioning  
Correct Answer:  repositioning  
 


  Question 7    Multiple Choice  1 of 1 point(s)    
  _____ variables segments markets in terms of shared activities, interests, and opinions.  
 Your Answer:  Psychographic  
Correct Answer:  Psychographic  
 


  Question 8    Multiple Choice  1 of 1 point(s)    
  According to Ries and trout, what is the goal of positioning?  
 Your Answer:  To become the first thing that comes to mind in the category.  
Correct Answer:  To become the first thing that comes to mind in the category.  
 


  Question 9    Multiple Choice  0 of 1 point(s)    
  A viable target market should:  
 Your Answer:  be all of the above  
Correct Answer:  have needs that can adequately be served by the marketer  
 


  Question 10    Multiple Choice  1 of 1 point(s)    
  _____ means developing a marketing strategy aimed at influencing how a particular market segment perceives a good or service in comparison to the competition.  
 Your Answer:  Positioning  
Correct Answer:  Positioning  
 

 A company wanted to know if there were enough demand for videoconferencing and other forms of video networking through laptop computers to make it profitable to offer this feature. Its marketing research department engaged in _____ and discovered that 48 percent of laptop owners use their machine while undressed or in their underwear.  

  projective data  
  experiential research  
@ exploratory research  
  ethnography  
  hierarchical research  

A group of investors wants to bring a minor league baseball team to a community of 100,000 as a way to create hometown spirit and offer a new form of family entertainment. It surveyed people in the community to see if they would attend the games, how frequently they would attend, and whether the location of the proposed baseball stadium was acceptable. This is an example of the use of:  

  data mining  
  a secondary internal source  
  exponential data  
@ a primary data source  
  a secondary external source    

   A marketing information system:  

  is designed to meet the information needs of marketing managers  
  begins by learning what information marketing managers need  
  includes data, computer hardware and software as well as MIS experts  
  gathers information from internal data, the external business environment, and marketing research  
@ is accurately described by all of the above  

  A research report typically contains:  

  an understandable description of the research methodology  
  limitations of the study  
  an executive summary of the report  
  recommendations for managerial action  
@ all of the above  

   A restaurant chain is considering giving its customers single-serving portions of salad dressing as a way to cut waste. It now allows the customers to take as much dressing as they want from containers provided by the servers. Before it introduces the single-serving portions, the restaurant chain hired researchers to record how much dressing each customer took and then to examine the bowl to see how much was left after the customer finished the salad. The restaurant used _____ research.  

@ observation  
  causal  
  inferential  
  projective  
  exploratory  

   A sign manufacturer that wanted to know which companies were developing proposals for creating and producing the signs at a new Australian sports complex could use:  

  service digitalization  
@ a marketing intelligence system  
  intelligence customerization  
  an intranet  
  trend extrapolation  

A survey questioned men and women to determine how each group felt about condom advertising on television. The results from male and female respondents were examined separately to see if their responses were different. The research results were analyzed using:  

  ethnographic techniques  
  single-source data methods  
@ cross-tabulation  
  standard correlation calculations  
  projective techniques  

A survey about pet ownership used a sample derived from a local telephone book. This sample would lack _____ because not everyone has a listed phone number.  

  flexibility  
  validity  
  responsiveness  
  reliability  
@ representativeness  

   A trained researcher was paid to visit a home with three pre-schoolers every day for two weeks to observe the children's bath time. From this information, you can infer that the organization paying the observer was using:  

  a focus group  
  experiential research  
  a case study  
  a mall intercept  
@ ethnography  
 
   An independent grocer that wanted to know if cocktail and tartar sauce would sell better if the two products were moved from the salad dressing section of the store to a display next to the seafood selection could use _____ research.  

  longitudinal  
  exploratory  
  discriminating  
  descriptive  
@  causal  

   A(n) _____ is an internal corporate communication network that uses Internet technology to link company departments, employees, and databases.  

  network structure  
  affiliate program  
  horizontal user network  
@ intranet  
  traditional communication system 

   A(n) _____ is a form of experiment that is conducted in the real world when researchers can control the independent variables.  

  concept test  
@ field study  
  stimulated laboratory  
  OJT setting  
  regulated scenario  

As part of his summer job, Earl was instructed to collect data from ten people to determine how they felt about a number of political issues including socialized medicine, food stamps, and educational vouchers. He decided to sample five Republicans and five Democrats. Earl's sample is an example of a _____ sample.  

  probability  
  convenience  
  stratified  
@ quota  
  simple random 

 As part of her summer job, Martina was instructed to collect data from ten people to determine how they felt about a number of political issues including socialized medicine, food stamps, and educational vouchers. She asked her father, grandfather, mother, brother, two uncles, and four sorority sisters to answer the survey questions. Martina used a _____ sample.  

  probability  
@ convenience  
  stratified  
  quota  
  simple random  

   According to the text, the basic marketing information system includes:  

  competitive intelligence, internal records, all sales personnel, and all managers  
  existing data, market researchers to gather more data, and the technology to convert those data into information  
  managers at all levels, corporate culture, and external and internal data  
@ data, computer hardware and software as well as MIS experts  
  information resources, technical experts, and marketing managers  

  According to your instructor, what makes information different from data?  

  Information comes from external sources, data comes from internal sources.  
  Data is quantitative, information is qualitative.  
@ Information is data used to make a decision.  
  Information is facts, data is numbers. 

   According to your instructor, what is the purpose of an "exceptions" report - that is a routine report that focuses on results that are different than expected.  

  To assist in assigning the blame when a plan fails.  
  To monitor progress toward pre-approved goals.  
  To determine whether or not goals have been met.  
  To track expenditures and relate them to the progress of the marketing plan.  
@ To identify potential problem areas in time to take remedial action.  

According to your instructor, why is the environmental scan so important to marketing managers?  

  A complete environmental scan includes monitoring the competition.  
  Because the environmental scan can be conducted with secondary research.  
  Because, as a manager, you have no control over the environmental scan.  
@ Because quality of the environmental scan affects the quality of the strategies a manager develops.  
  Because the environmental scan is the first step of the marketing research process. 

   According to your instructor, why is it important to understand the assumptions that went into the research.  

  Different techniques require different assumptions.  
@ So that you don't read more into the data than there really is.  
  Because secondary research is less expensive.  
  Primary research can involve questionnaires or syndicated research. 

   Bratwurst is not a food typically eaten in the southeastern U.S. To determine how to encourage Southerners to eat more bratwurst, a manufacturer conducted a survey in which it asked, "If bratwurst were going to a picnic, what kind of clothes would it wear?" and "Of the foods in your refrigerator now, what would a bratwurst most want to socialize with?" This survey used:  

@  projective techniques  
  ethnographic methods  
  hypothetical testing  
  concept testing  
  biofeedback techniques 

   Barry was sitting on a bench near the center of the shopping center waiting for his wife to finish her shopping when he was approached by a woman holding a clipboard with a dress shirt laid over her arm. She asked if Barry had the time to answer a few questions about the shirt. After Barry examined the shirt, the woman asked him to assess the quality of the shirt and to state how much he would be willing to pay for the shirt. Barry participated in a(n):  

  focus group  
  on-line interview  
  chat room  
  traditional aggregation of information research program  
@ mall intercept  

Every six months, Doug is asked to fill out a survey in which he is asked when he expects to buy a new vehicle and what type and model he is planning on buying. Doug along with a group of consumers with similar buying habits has been participating in this survey for the last eight years. The gathering of this quantitative data would be an example of descriptive research using:  

  a hierarchical format  
  a cross-sectional design  
  latitudinal tracking  
@ a longitudinal design  
  ethnographic research 

   Fingerhut catalog retailer studies about 3,500 variables over the lifetime of a customer's relationship. It has found that customers who change residences are three times more likely as regular customers to buy tables, fax machines, and decorative products but are no more likely to buy jewelry or footwear. By using data analysis to determine which promotions to target to customers who have recently moved, Fingerhut has engaged in:  

  customer retention  
  customer acquisition  
  customer abandonment  
  customer accrual  
@ market basket analysis 

   Fingerhut catalog retailer studies about 3,500 variables over the lifetime of a customer's relationship. Through _____, it has found that customers who change residences are three times more likely as regular customers to buy tables, fax machines, and decorative products but are no more likely to buy jewelry or footwear.  

@  data mining  
  prospecting  
  cold calling  
  data qualification  
  data tallying  

 Fingerhut catalog retailer studies about 3,500 variables over the lifetime of a customer's relationship. It has twenty-five million customers. Fingerhut will more than likely use a(n) _____ to store and process its data.  

  distribution center  
@ data warehouse  
  information downloading area  
  information network  
  marketing research department  

   Focus groups:  

  are typically conducted in mall settings  
  are a type of descriptive research  
  are one-on-one interviews  
@ are used to gather exploratory data  
  do not use a discussion leader 


    What is "psuedo research"  

@ Research that is done to support a decision that has already been made.  
  Exploratory research using projective techniques.  
  Descriptive research to determine which scenario to adopt.  
  Using exploratory research techniques in causal research.  

   Which method of data collection was most affected by the 2001 anthrax scares?  

@ mail questionnaire  
  on-line questionnaire  
  one-way interviewing  
  telephone interviewing  
  face-to-face interviewing 

   Which of the following is a source of information for a marketing intelligence system?  

  trade publications  
  salespeople  
  observational research  
  current customers  
@ all of the above  

   Which of the following is NOT one of the ways marketers typically use data mining?  
 
  customer retention  
  customer acquisition  
  customer abandonment  
@ customer accrual  
  market basket analysis  

   Which of the following statements about how market research data are gathered in foreign countries is true?  

  The same amount of secondary data is available for every country.  
  Telephones facilitate data collection in countries with poor road systems.  
  All countries can be described as computer-literate.  
@ Cultural differences affect how people respond to survey questions.  
  All of the above statements about how market research data are gathered in foreign countries are true. 

 Which of the following statements about the steps in the marketing research process is true?  

  The research design should be determined before the problem is defined.  
  The last step is to analyze and interpret the data.  
  The sample must be designed after the data is collected.  
@ The first step is problem definition.  
  The data collection method is selected after the researcher has determined the research design.  

Which of the following would be an example of a secondary data source for a company that wants to open a casino near New Orleans, Louisiana?  

  U.S. Census reports on New Orleans growth patterns  
  report on tourism industry in Louisiana  
  study of the growth of online gambling activities  
  stockholders' report from Harrah's, a casino that operates in downtown New Orleans  
@ all of the above  

   Which of these statements best describes bernard Litaud's definition of intelligence?  

  Information used to make decisions that turn out to be correct.  
  A means of organizing information into "silos"  
  Focusing information on the future needs of the company.  
@ A property of the organization  

Which type of data collection method is most likely to have the lowest response rate?  

@ mail questionnaire  
  on-line questionnaire  
  one-way interviewing  
  telephone interviewing  
  face-to-face interviewing 

  Will has eight puppies. He is concerned his puppies may be infected with parvo, a disease that is deadly in young dogs unless diagnosed and treated early. The cost of the test to determine if the dog has parvo is $59. Will cannot afford to have all of his puppies tested. The veterinarian has suggested that Will reach into the pen where the dogs stay and pick up the first one that walks into his hand. (If one dog tests positive, then Will can assume that all the puppies are infected. If the puppy is not infected, then Will can assume that his puppies are healthy.) What kind of a sampling technique is the vet suggesting Will use?  

@ simple random sampling  
  cluster sampling  
  stratified sampling  
  systematic sampling  
  none  

  With a non-probability sample,:  

  the hierarchical sampling method is used to select people who appear most willing to participate  
  representativeness is the key determinant of who participates  
@ a researcher uses personal judgment to select participants  
  every member of the population is included in the sample  
  each member of the population has some chance of being included in the sample 

   With a probability sample,:  

  the observation method is used to select people who appear most willing to participate  
  convenience is the key determinant of who participates  
  a researcher uses personal judgment to select participants  
  every member of the population is included in the sample  
@ each member of the population has a chance of being included in the sample


  _____ could be used to determine which athlete consumers perceive as the most reliable spokesperson for a new long-acting pain reliever.  

  Syndicated research reports  
@ Custom research reports  
  Customerization reports  
  Perceptual tracking  
  Perceptual-attitudinal testing 

   _____ is the extent to which the research actually measures what it was intended to measure.  

  Flexibility  
@ Validity  
  Responsiveness  
  Reliability  
  Representativeness 

   _____ is the extent to which the research measurement techniques are free of errors.  

  Flexibility  
  Validity  
  Responsiveness  
@ Reliability  
  Representativeness 

  _____ is the process of collecting, analyzing, and interpreting data about customers, competitors, and the business environment to improve marketing effectiveness.  

@ Marketing research  
  Marketing management  
  Competitive departmentalization  
  Data synergy  
  Internal marketing  

   _____ is the process of selecting respondents for a survey.  
@ Sampling  
  Prospecting  
  Qualifying  
  Surveying  
  Sample linking  

Question 7    Multiple Choice  0 of 1 point(s)    
  Several retailers might use information from _____ to determine which fashion trends appeal to teenagers or how older Americans feel about senior discounts.  
 Your Answer:  [None Given]  
Correct Answer:  syndicated research reports  
 


  Question 8    Multiple Choice  0 of 1 point(s)    
  The management at an amusement park was concerned that annual attendance figures were gradually declining even though the park had added several new attractions. ______ showed that 78 percent of all Americans desire more spontaneity in their lives and that amusement parks did not satisfy this need for spontaneity. As a result the park is studying the feasibility of "good weather" discounts.  
 Your Answer:  [None Given]  
Correct Answer:  Exploratory research  
 


  Question 9    Multiple Choice  0 of 1 point(s)    
  Which method of data collection was most affected by the 2001 anthrax scares?  
 Your Answer:  [None Given]  
Correct Answer:  mail questionnaire  
 


  Question 10    Multiple Choice  0 of 1 point(s)    
  Which of these statements best describes bernard Litaud's definition of intelligence?  
 Your Answer:  [None Given]  
Correct Answer:  A property of the organization  
 
Question 1    Multiple Choice  0 of 1 point(s)    
  Marketers use _____ to generate topics for future, more rigorous studies, ideas for new strategies and opportunities, and to better understand the source of problems with which they are currently dealing.  
 Your Answer:  [None Given]  
Correct Answer:  exploratory research  
 


  Question 2    Multiple Choice  0 of 1 point(s)    
  _____ could be used to determine which athlete consumers perceive as the most reliable spokesperson for a new long-acting pain reliever.  
 Your Answer:  [None Given]  
Correct Answer:  Custom research reports  
 


  Question 3    Multiple Choice  0 of 1 point(s)    
  The first step in the marketing research process is to define the problem. This step has three components-specifying the research objectives, identifying the consumer population of interest, and:  
 Your Answer:  [None Given]  
Correct Answer:  placing the problem in an environmental context  
 


  Question 4    Multiple Choice  0 of 1 point(s)    
  A survey questioned men and women to determine how each group felt about condom advertising on television. The results from male and female respondents were examined separately to see if their responses were different. The research results were analyzed using:  
 Your Answer:  [None Given]  
Correct Answer:  cross-tabulation  
 


  Question 5    Multiple Choice  0 of 1 point(s)    
  The second step of the marketing research process is to:  
 Your Answer:  [None Given]  
Correct Answer:  determine the research design  
 


  Question 6    Multiple Choice  0 of 1 point(s)    
  A trained researcher was paid to visit a home with three pre-schoolers every day for two weeks to observe the children's bath time. From this information, you can infer that the organization paying the observer was using:  
 Your Answer:  [None Given]  
Correct Answer:  ethnography  
 


  Question 7    Multiple Choice  0 of 1 point(s)    
  According to your instructor, why is it important to do a cost-benefit analysis before proceeding with a market research plan?  
 Your Answer:  [None Given]  
Correct Answer:  In order to make sure the money you spend to do the research is appropriate, given how the information will be used.  
 


  Question 8    Multiple Choice  0 of 1 point(s)    
  One of the primary sources of _____ is checkout scanners in stores.  
 Your Answer:  [None Given]  
Correct Answer:  single-source data  
 


  Question 9    Multiple Choice  0 of 1 point(s)    
  A company wanted to know if there were enough demand for videoconferencing and other forms of video networking through laptop computers to make it profitable to offer this feature. Its marketing research department engaged in _____ and discovered that 48 percent of laptop owners use their machine while undressed or in their underwear.  
 Your Answer:  [None Given]  
Correct Answer:  exploratory research  
 


  Question 10    Multiple Choice  0 of 1 point(s)    
  Telemarketing has had the greatest impact on which of the following data collection methods?  
 Your Answer:  [None Given]  
Correct Answer:  telephone interviewing  
 



  Question 1    Multiple Choice  0 of 1 point(s)    
  The text refers to the two main types of samples used by researchers as:  
 Your Answer:  [None Given]  
Correct Answer:  probability and non-probability  
 


  Question 2    Multiple Choice  0 of 1 point(s)    
  Six women were invited to participate in a discussion on motels, hotels, and resorts. A discussion leader first asked them what differentiated motels, hotels, and resort accommodations. Then she asked them to identify all the amenities they expect to find in overnight accommodations they might stay at. These six women and their group leader participated in a:  
 Your Answer:  [None Given]  
Correct Answer:  focus group  
 


  Question 3    Multiple Choice  0 of 1 point(s)    
  _____ could be used to determine which athlete consumers perceive as the most reliable spokesperson for a new long-acting pain reliever.  
 Your Answer:  [None Given]  
Correct Answer:  Custom research reports  
 


  Question 4    Multiple Choice  0 of 1 point(s)    
  Which of the following is a source of information for a marketing intelligence system?  
 Your Answer:  [None Given]  
Correct Answer:  all of the above  
 


  Question 5    Multiple Choice  0 of 1 point(s)    
  According to your instructor, why is the environmental scan so important to marketing managers?  
 Your Answer:  [None Given]  
Correct Answer:  Because quality of the environmental scan affects the quality of the strategies a manager develops.  
 


  Question 6    Multiple Choice  0 of 1 point(s)    
  As part of his summer job, Earl was instructed to collect data from ten people to determine how they felt about a number of political issues including socialized medicine, food stamps, and educational vouchers. He decided to sample five Republicans and five Democrats. Earl's sample is an example of a _____ sample.  
 Your Answer:  [None Given]  
Correct Answer:  quota  
 


  Question 7    Multiple Choice  0 of 1 point(s)    
  Fingerhut catalog retailer studies about 3,500 variables over the lifetime of a customer's relationship. It has found that customers who change residences are three times more likely as regular customers to buy tables, fax machines, and decorative products but are no more likely to buy jewelry or footwear. By using data analysis to determine which promotions to target to customers who have recently moved, Fingerhut has engaged in:  
 Your Answer:  [None Given]  
Correct Answer:  market basket analysis  
 


  Question 8    Multiple Choice  0 of 1 point(s)    
  With a probability sample,:  
 Your Answer:  [None Given]  
Correct Answer:  each member of the population has a chance of being included in the sample  
 


  Question 9    Multiple Choice  0 of 1 point(s)    
  _____ is the extent to which the research measurement techniques are free of errors.  
 Your Answer:  [None Given]  
Correct Answer:  Reliability  
 


  Question 10    Multiple Choice  0 of 1 point(s)    
  Nielsen/NetRatings uses an electronic meter to record Internet user behavior. These data are gathered by tracking the actual mouse clicks made by users from over 225,000 individuals in over 26 countries as they surf the Net. This system reveals the Web sites that have the most number of unique visitors. It is a form of _____ research.  
 Your Answer:  [None Given]  
Correct Answer:  observation  
 


Question 3    Multiple Choice  0 of 1 point(s)    
  A company did a comprehensive examination of how pit crews in an auto race work to get a racecar into the pit area, gas the car, perform other maintenance, and get the car back on the track so quickly. The company wanted to see if it could adapt some of the teamwork techniques used by pit crews to speed up its order processing. The company conducted a(n):  
 Your Answer:  [None Given]  
Correct Answer:  case study  
 


  Question 4    Multiple Choice  0 of 1 point(s)    
  Which of the following is the BEST example of internal data for a company that produces and sells customized signs?  
 Your Answer:  [None Given]  
Correct Answer:  record of international sales for the last five years  
 


  Question 5    Multiple Choice  0 of 1 point(s)    
  Researchers are conducting a survey on how well parents encourage their children to brush and floss their teeth on a regular schedule. Some information came from surveys filled out by parents while their children saw a dentist who specializes in preteens. The rest of the survey results were gathered from anonymous surveys parents received in the mail. If you knew this about the survey results, you could legitimately doubt the _____ of the survey.  
 Your Answer:  [None Given]  
Correct Answer:  reliability  
 


  Question 6    Multiple Choice  0 of 1 point(s)    
  Researchers design _____ to test specified relationships among variables in a controlled environment.  
 Your Answer:  [None Given]  
Correct Answer:  experiments  
 


  Question 7    Multiple Choice  0 of 1 point(s)    
  Typically, three factors influence the quality of research results. They are:  
 Your Answer:  [None Given]  
Correct Answer:  validity, reliability, and representativeness  
 


  Question 8    Multiple Choice  0 of 1 point(s)    
  Every six months, Doug is asked to fill out a survey in which he is asked when he expects to buy a new vehicle and what type and model he is planning on buying. Doug along with a group of consumers with similar buying habits has been participating in this survey for the last eight years. The gathering of this quantitative data would be an example of descriptive research using:  
 Your Answer:  [None Given]  
Correct Answer:  a longitudinal design  
 


  Question 9    Multiple Choice  0 of 1 point(s)    
  An independent grocer that wanted to know if cocktail and tartar sauce would sell better if the two products were moved from the salad dressing section of the store to a display next to the seafood selection could use _____ research.  
 Your Answer:  [None Given]  
Correct Answer:  causal  
 


  Question 10    Multiple Choice  0 of 1 point(s)    
  Telemarketing has had the greatest impact on which of the following data collection methods?  
 Your Answer:  [None Given]  
Correct Answer:  telephone interviewing  
 


 Question 1    Multiple Choice  0 of 1 point(s)    
  The most popular game at a carnival arcade is the water-gun contests. The arcade section of the carnival is typically in a section by itself. A carnival owner who wanted to know how revenue would be affected if the water-gun contests should be the first one customers come to or if it were the last one could use _____ research.  
 Your Answer:  [None Given]  
Correct Answer:  causal  
 


  Question 2    Multiple Choice  0 of 1 point(s)    
  Which of the following is NOT one of the ways marketers typically use data mining?  
 Your Answer:  [None Given]  
Correct Answer:  customer accrual  
 


  Question 3    Multiple Choice  0 of 1 point(s)    
  _____ is the extent to which the research actually measures what it was intended to measure.  
 Your Answer:  [None Given]  
Correct Answer:  Validity  
 


  Question 4    Multiple Choice  0 of 1 point(s)    
  _____ could be used to determine which athlete consumers perceive as the most reliable spokesperson for a new long-acting pain reliever.  
 Your Answer:  [None Given]  
Correct Answer:  Custom research reports  
 


  Question 5    Multiple Choice  0 of 1 point(s)    
  _____ is the process of selecting respondents for a survey.  
 Your Answer:  [None Given]  
Correct Answer:  Sampling  
 


  Question 6    Multiple Choice  0 of 1 point(s)    
  Which of the following would be an example of a secondary data source for a company that wants to open a casino near New Orleans, Louisiana?  
 Your Answer:  [None Given]  
Correct Answer:  all of the above  
 


  Question 7    Multiple Choice  0 of 1 point(s)    
  A survey about pet ownership used a sample derived from a local telephone book. This sample would lack _____ because not everyone has a listed phone number.  
 Your Answer:  [None Given]  
Correct Answer:  representativeness  
 


  Question 8    Multiple Choice  0 of 1 point(s)    
  As part of his summer job, Earl was instructed to collect data from ten people to determine how they felt about a number of political issues including socialized medicine, food stamps, and educational vouchers. He decided to sample five Republicans and five Democrats. Earl's sample is an example of a _____ sample.  
 Your Answer:  [None Given]  
Correct Answer:  quota  
 


  Question 9    Multiple Choice  0 of 1 point(s)    
  Every six months, Doug is asked to fill out a survey in which he is asked when he expects to buy a new vehicle and what type and model he is planning on buying. Doug along with a group of consumers with similar buying habits has been participating in this survey for the last eight years. The gathering of this quantitative data would be an example of descriptive research using:  
 Your Answer:  [None Given]  
Correct Answer:  a longitudinal design  
 


  Question 10    Multiple Choice  0 of 1 point(s)    
  Many researchers use _____ to get at people's underlying feelings, especially when they think people will be unwilling to express their true reactions.  
 Your Answer:  [None Given]  
Correct Answer:  projective techniques  
 
 Question 1    Multiple Choice  1 of 1 point(s)    
  Will has eight puppies. He is concerned his puppies may be infected with parvo, a disease that is deadly in young dogs unless diagnosed and treated early. The cost of the test to determine if the dog has parvo is $59. Will cannot afford to have all of his puppies tested. The veterinarian has suggested that Will reach into the pen where the dogs stay and pick up the first one that walks into his hand. (If one dog tests positive, then Will can assume that all the puppies are infected. If the puppy is not infected, then Will can assume that his puppies are healthy.) What kind of a sampling technique is the vet suggesting Will use?  
 Your Answer:  simple random sampling  
Correct Answer:  simple random sampling  
 


  Question 2    Multiple Choice  0 of 1 point(s)    
  Which of the following statements about the steps in the marketing research process is true?  
 Your Answer:  The last step is to analyze and interpret the data.  
Correct Answer:  The first step is problem definition.  
 


  Question 3    Multiple Choice  0 of 1 point(s)    
  A marketing information system:  
 Your Answer:  [None Given]  
Correct Answer:  is accurately described by all of the above  
 


  Question 4    Multiple Choice  0 of 1 point(s)    
  Which of the following would be an example of a secondary data source for a company that wants to open a casino near New Orleans, Louisiana?  
 Your Answer:  [None Given]  
Correct Answer:  all of the above  
 


  Question 5    Multiple Choice  0 of 1 point(s)    
  Focus groups:  
 Your Answer:  [None Given]  
Correct Answer:  are used to gather exploratory data  
 


  Question 6    Multiple Choice  0 of 1 point(s)    
  The most popular game at a carnival arcade is the water-gun contests. The arcade section of the carnival is typically in a section by itself. A carnival owner who wanted to know how revenue would be affected if the water-gun contests should be the first one customers come to or if it were the last one could use _____ research.  
 Your Answer:  [None Given]  
Correct Answer:  causal  
 


  Question 7    Multiple Choice  0 of 1 point(s)    
  The text describes the two main primary data collection methods as:  
 Your Answer:  [None Given]  
Correct Answer:  communication and observation  
 


  Question 8    Multiple Choice  0 of 1 point(s)    
  A company did a comprehensive examination of how pit crews in an auto race work to get a racecar into the pit area, gas the car, perform other maintenance, and get the car back on the track so quickly. The company wanted to see if it could adapt some of the teamwork techniques used by pit crews to speed up its order processing. The company conducted a(n):  
 Your Answer:  [None Given]  
Correct Answer:  case study  
 


  Question 9    Multiple Choice  0 of 1 point(s)    
  A trained researcher was paid to visit a home with three pre-schoolers every day for two weeks to observe the children's bath time. From this information, you can infer that the organization paying the observer was using:  
 Your Answer:  [None Given]  
Correct Answer:  ethnography  
 


  Question 10    Multiple Choice  0 of 1 point(s)    
  A company wanted to know if there were enough demand for videoconferencing and other forms of video networking through laptop computers to make it profitable to offer this feature. Its marketing research department engaged in _____ and discovered that 48 percent of laptop owners use their machine while undressed or in their underwear.  
 Your Answer:  [None Given]  
Correct Answer:  exploratory research  
 


Question 7    Multiple Choice  0 of 1 point(s)    
  To determine what type of information to add to its Web site in order to attract more visitors, an Internet company could look at the 2002 survey by American Online/Roper Starch Adult Cyberstudy, which showed the primary activities that Americans are most likely to engage in when surfing the Internet. This cyberstudy would be an example of:  
 Your Answer:  [None Given]  
Correct Answer:  a secondary data source  
 


  Question 8    Multiple Choice  0 of 1 point(s)    
  _____ is the extent to which the research actually measures what it was intended to measure.  
 Your Answer:  [None Given]  
Correct Answer:  Validity  
 


  Question 9    Multiple Choice  0 of 1 point(s)    
  The text describes the two main primary data collection methods as:  
 Your Answer:  [None Given]  
Correct Answer:  communication and observation  
 


  Question 10    Multiple Choice  0 of 1 point(s)    
  Barry was sitting on a bench near the center of the shopping center waiting for his wife to finish her shopping when he was approached by a woman holding a clipboard with a dress shirt laid over her arm. She asked if Barry had the time to answer a few questions about the shirt. After Barry examined the shirt, the woman asked him to assess the quality of the shirt and to state how much he would be willing to pay for the shirt. Barry participated in a(n):  
 Your Answer:  [None Given]  
Correct Answer:  mall intercept  
 


 Question 7    Multiple Choice  0 of 1 point(s)    
  Panama City, Florida, hired a research company to survey spring-breakers to learn about this market. One of the questions asked, "If you were able to shower with any celebrity, who would it be?" (Brad Pitt and Britney Spears were selected as most showerable.) This type of question is a means for gathering:  
 Your Answer:  [None Given]  
Correct Answer:  qualitative data  
 


  Question 8    Multiple Choice  0 of 1 point(s)    
  The first step in the marketing research process is to:  
 Your Answer:  [None Given]  
Correct Answer:  clearly understand the information needed by managers that the research is to provide  
 


  Question 9    Multiple Choice  0 of 1 point(s)    
  With a probability sample,:  
 Your Answer:  [None Given]  
Correct Answer:  each member of the population has a chance of being included in the sample  
 


  Question 10    Multiple Choice  0 of 1 point(s)    
  As part of her summer job, Martina was instructed to collect data from ten people to determine how they felt about a number of political issues including socialized medicine, food stamps, and educational vouchers. She asked her father, grandfather, mother, brother, two uncles, and four sorority sisters to answer the survey questions. Martina used a _____ sample.  
 Your Answer:  [None Given]  
Correct Answer:  convenience  
 


A consumer products company like Procter & Gamble that makes a variety of different cleaners, detergents, and polishers to appeal to a variety of target markets is using a(n):  
@ full line strategy  

  undifferentiated strategy  
  limited-line strategy  
  marketing mix orientation  
  production mix strategy  

A few years ago, Quaker introduced Fruit Oatmeal Bites, a nugget version of its cereal bars. The product was very successful because it is not messy. As a result Kellogg's has introduced a similar product under its Nutri-Grain brand. Mini-cereal bars are most likely in the _______________ stage of their product life cycle.  

  maturity  
  introduction  
  expansion  
@ growth  
  extension 

A few years ago, Quaker introduced Fruit Oatmeal Bites, a nugget version of its cereal bars. The product was very successful because it is not messy. As a result Kellogg's has introduced a similar product under its Nutri-Grain brand. Mini-cereal bars are most likely in the _______________ stage of their product life cycle.  

  maturity  
@ introduction  
  expansion  
  growth  
  extension

A firm's _______________ is its entire range of products the company offers for sale.  

  branding strategy  
  promotional mix  
  positioning strategy  
@ product mix  
  product line  

A group of business entrepreneurs who worried about their teenage children drinking and driving decided there must be some way to approach this problem proactively. They came up with an idea for a pair of blurry goggles. They believed looking through the goggles would simulate what the world looks like when someone is seriously intoxicated. The first crude pair of blurry goggles developed by the entrepreneurs to show to prospective customers was:  

@ a prototype  
  a heuristic  
  a synergistic model  
  raw material  
  processed material 

A group of business entrepreneurs who worried about their teenage children drinking and driving decided there must be some way to approach this problem proactively. They tried to develop products that would show young drivers how drinking dulls the senses. They came up with an idea for a pair of blurry goggles. They believed looking through the goggles would simulate what the world looks like when someone is seriously intoxicated. In which stage of the new-product development process was the group of business entrepreneurs?  

  problem recognition  
  idea screening  
  idea generation  
  concept testing  
@ product concept development 

A group of business entrepreneurs who worried about their teenage children drinking and driving decided there must be some way to approach this problem proactively. They tried to develop products that would show young drivers how drinking dulls the senses. In which stage of the new-product development process was the group of business entrepreneurs?  

  problem recognition  
  idea screening  
@ idea generation  
  concept testing  
  a business analysis 

=======================

According to the definition in the text, which of the following was an example of an innovation?  

  the first Survivors television show  
  phone service with no long distance charges  
  fat-free ice cream  
  flavored bottled water  
@ all of the above

=======================

At a retail store, the displays racks on which clothes are hung, the shelving, and the cash registers are all examples of:  
@ accessory equipment  
  specialty products  
  capital equipment  
  building staples  
  MRO products  

==========================

A _______________ is a firm's total product offering designed to satisfy a single need or desire of a group of target customers.  

  branding strategy  
  market segmentation strategy  
  positioning strategy  
  product mix  
@ product line 

A _____ is a good or service for which consumers will spend time and effort gathering information on price, product attributes, and product quality.  

  specialty product  
  homogeneous product  
@ shopping product  
  convenience product  
  generic product  

A _____ is a modification to an existing product to differentiate one brand from another. The consumer doesn't have to learn anything to use the new product.  

  dynamically continuous innovation  
  nonradical innovation  
  competitive innovation  
@ continuous innovation  
  discontinuous innovation

===========================

Cheese crackers have been around for quite awhile. Pepperidge Farm created a cracker variation by making its crackers in the shape of goldfish. To attract new users and to maintain market share, Pepperidge Farms has enlarged its Goldfish crackers and added cheese and peanut butter fillings. This strategy indicates that Goldfish cheese crackers are in the _______________ stage of their product life cycle.  

  expansion  
@ maturity  
  decline  
  leveling  
  extension 

==========================

  Clarice and Beth both want to buy Christmas lights to decorate their new homes. Clarice wants lights that are guaranteed safe for outdoor use. Having found several different sets, she now knows she wants white lights that blink sequentially, and she wants each strand of lights to contain 200 bulbs. Beth wants something to make her house look more festive. At various stores, Beth found lights priced from $6.99 a strand to $25.99 a strand. Beth bought almost $200 worth of the cheapest lights so that she could have as many lights on her house as possible. For Beth, Christmas lights are a(n):  

  homogeneous product  
  attribute-based shopping product  
  specialized shopping product  
@ price-based shopping product  
 commodity  

  Clarice and Beth both want to buy Christmas lights to decorate their new homes. Clarice wants lights that are guaranteed safe for outdoor use. Having found several different sets, she now knows she wants white lights that blink sequentially, and she wants each strand of lights to contain 200 bulbs. Beth wants something to make her house look more festive. At various stores, Beth found lights priced from $6.99 a strand to $25.99 a strand. Beth bought almost $200 worth of the cheapest lights so that she could have as many lights on her house as possible. For Clarice, Christmas lights are a(n):  

  homogeneous product  
@ attribute-based shopping product  
  specialized shopping product  
  price-based shopping product  
  commodity  

==========================

Consumers must engage in a great amount of learning to use a _____ because no similar product has ever been on the market.  
  dynamically continuous innovation  
  nonradical innovation  
  competitive innovation  
  continuous innovation  
@ discontinuous innovation 

Customers usually know a good deal about _____, are loyal to specific brands, and spend little, if any, time comparing alternatives.  

@ specialty products  
  homogeneous products  
  shopping products  
  convenience products  
  commodities  

============================

Diffusion refers to:  

  how promotion is used to support products throughout their product life cycle  
  the second stage of the market segmentation process  
  the methodology used for evaluating new product ideas  
@ how the use of a product spreads throughout a population  
  the selection of opinion leaders to encourage product acceptance  

Disposable diapers for infants and toddlers are in the maturity stage of their product life cycle. This means:  

@ manufacturers of disposable diapers are experiencing narrowing profit margins  
  marketers are relying heavily on informative advertising  
  manufacturers of disposable diapers are reducing the number of product variations  
  manufacturers of disposable diapers are decreasing their use of marketing communications to maintain profitability  
  none of the above is happening  

==============================

   Even though most people in the original target market for electric toothbrushes already knew how to brush their teeth, consumers still had some learning to do in order to make good use of the new brushing technique. The electric toothbrush is an example of a:  

@ dynamically continuous innovation  
  nonradical innovation  
  competitive innovation  
  continuous innovation  
  discontinuous innovation 

==============================

   For 85 years, Jamesway has made egg incubators for all kinds and sizes of eggs for poultry raisers. Egg incubators are Jamesway's total product offering. These egg incubators are an example of a:  

  branding strategy  
  market segmentation strategy  
  positioning strategy  
  product mix  
@ product line 

   For the business market, a multi-purpose absorbent used to clean up any type of spills would be an example of a(n):  

  repair product  
  operating supply  
@ maintenance product  
  restoration product  
  organizational product  

   For an amusement park, concession food trailers, the parking lot, the first-aid building, and the building where the park's daily receipts are tallied are all examples of:  

  MRO products  
  specialty products  
  processed materials  
@ equipment  
  convenience products

==============================

   How do marketers use the product life cycle?  

@ to explain how product features change over the life of a product  
  to prevent cannibalization  
  to more efficiently compete in international markets  
  to support a product orientation  
  to implement demographic segmentation 

==============================

 In addition to regular Alka-Seltzer, there is Alka-Seltzer for cold sufferers, people with upset stomachs, people who can't sleep, people with allergies, and people who have a worse than normal headache. The manufacturer of Alka-Seltzer is using a(n) _______________ to reach as many different target markets as possible.  

@ full line strategy  
  undifferentiated strategy  
  limited-line strategy  
  marketing mix orientation  
  production mix strategy 

  In the business market, steel tubing, natural cloth fabric, and lumber are all examples of:  

  component parts  
  accessory equipment  
@ processed materials  
  MRO products  
  raw materials 

  In the business market, wheat, coal, and bauxite (used to make aluminum) are all examples of:  

  component parts  
  accessory equipment  
  processed materials  
  MRO products  
@ raw materials 

==============================

Kitchen dishcloths are often difficult to rinse clean and typically dry slowly. 3M has developed a disposable ScotchBrite Microfiber Kitchen Cloth that rinses easily and dries quickly. What kind of marketing communications should 3M use with this new product?  

  reminder advertising  
  heavy advertising to counter new competition  
@ informative advertising  
  personal selling to customers  
  advocacy advertising 

Kitchen dishcloths are often difficult to rinse clean and typically dry slowly. 3M has developed a disposable ScotchBrite Microfiber Kitchen Cloth that rinses easily and dries quickly. The marketing goal for the product manager of this product is to:  

@ get first-time buyers to try the product  
  encourage continued brand loyalty  
  remain profitable  
  price to maintain market share  
  add new features as needed to maintain market share 

Kitchen dishcloths are often difficult to rinse clean and typically dry slowly. 3M has developed a disposable ScotchBrite Microfiber Kitchen Cloth that rinses easily and dries quickly. Because this is a brand-new product to the market, it can be said to be in the _______________ stage of its product life cycle.  

  maturity  
@ introduction  
  expansion  
  growth  
  extension

==========================

Kodak has developed the Kodak picture kit, which contains "everything you need to share and enjoy your pictures." The kit has five rolls of Kodak Max film, a decorative photo album, ready-to-mail photo frames, photo software CD, and photo storage case. This is a new product designed to attract people who take traditional photos as well as digital pictures. Which of the following statements about this new product are true?  

  Kodak should use reminder advertising.  
  Kodak prices the product to maintain its market share.  
  Sales of the product are in a decline.  
@ The purpose of its marketing communications should be to inform customers about the benefits of the new product.  
  All of the above statements about the new Kodak picture kit are true. 

Kodak introduced the Kodak Funtime film to compete with lower-priced brands, but it found many of its regular customers buying Funtime instead of their usual Kodak film. Kodak experienced:  

  disintermediation  
@ cannibalization  
  cross-docking  
  a piggyback exchange  
  pyramiding

   Underwear-like disposable training pants for toddlers are in the _______________ stage of their product life cycle because new competitors are entering the market as well as variations and improvements of the original product. The goal of manufacturers of this product is to encourage brand loyalty through heavy advertising to counter new competitors.  

  maturity  
  introduction  
  expansion  
@ growth  
  extension 

   Unilever, the manufacturer of many recognizable brands of consumer products, has announced that it plans to cut its number of product lines from a high of 1,600 different products to 400 products so that it can concentrate on increasing its efficiency. This is an example of a(n):  

  downward line stretch  
  value stretch  
  upward line stretch  
@ product line contraction  
  filling-out strategy 

=============================

 What did your instructor mean when he talked about synergy in product mix expansion?  

  Synergy describes the amount of management effort needed to introduce a new product to the mix  
@ Synergy describes the marginal cost of adding a new product or line given the competencies a company already has  
  Synergy describes the legal costs associated with defending a profitable, but unethical product  
  Synergy is the difference between the width and length of the product mix.  

 What is the assumption behind product strategy discussed in class?  

  Products are intended to bring profits to the company  
  A product is a bundle of attributes and services  
  Product is the least important component of the marketing mix.  
@ No company has only one product  

 What strategic options does a manufacturer of a product in the decline stage of its product life cycle have?  

  It can continue producing the product for loyal users.  
  It can reduce or eliminate its advertising budget for the product.  
  It can phase out production of the product.  
  It can drop the product immediately.  
@ It can do any of the above.

============================

  When Bank One, a traditional banking company, launched WingSpanBank.com as a freestanding all-Internet bank, many regular customers of Bank One became WingSpanBank.com customers. Bank One experienced:  

  disintermediation  
@ cannibalization  
  cross-docking  
  a piggyback exchange  
  pyramiding

  When Mae's mother died, Mae was glad her mother had prepaid for a funeral. Then Mae was told new state regulations require each coffin placed in the ground had to be in a burial vault-an additional $800 expense. For Mae, the burial vault was a(n):  

  specialty product  
  homogeneous product  
  shopping product  
  convenience product  
@ unsought product

========================

   Whenever a product line or a product family is extended, there is a risk of _______________, which occurs when sales of an existing brand decline as the firm's current customers switch to the new product.  

  disintermediation  
@ cannibalization  
  cross-docking  
  a piggyback exchange  
  pyramiding  

=========================

 Which of the following adjectives best describes a product that would be classified as a good?  

  non-perishable  
  convenient  
  disposable  
@ tangible  
  heterogeneous

Which of the following is an example of a good?  

  a massage  
@ a text book  
  a medical examination by a doctor  
  an idea for a new television show  
  all of the above

   Which of the following is an example of a product?  

  the formula for a cellulose-based plastic  
  a garbage can  
  a hair cut  
  a musical performance by Dixie Chicks  
@ all of the above 

   Which of the following is a definition of fashion discussed in class?  

@ A style that has been adopted by a significant number of people for a significant length of time.  
  A style that has trickled down from the higher classes  
  A product that serves no purpose other than to demonstrate the taste of the purchaser  
  A product adopted by members of a group as a means of identifying each other

Which of the following is an example of a product dimension consumers use to determine product quality?  

  ease of use  
  degree of pleasure  
  reliability  
  versatility  
@ all of the above

==================================

Which of the following is the best definition of style (as discussed in class)  

  Anything Professor Robertson wears in class  
@ A distinctive manner of presentation or construction  
  Style is in the eyes of the beholder  
  The manner in which benefits are engineered into a product. 

Which of the following is the best example of a convenience product?  

  a silk scarf  
  a subscription to Martha Stewart Living magazine  
@ chewing gum  
  a lawn mower  
  a CD player

**  Which of the following is the best example of a durable good?  

  bag of cat food  
  computer printer  
@ bottle of nail polish  
  package of light bulbs  
  detergent

  Which of the following is the best example of a nondurable good?  

  dishwasher  
  book shelf  
  freezer  
@ ream of paper  
  file cabinet

========================

   Which of the following products would most consumers classify as a shopping product?  

  a subscription to The Wall Street Journal  
@ a doctor to check your children's vision  
  a box of adhesive bandages  
  a can of spinach  
  a flea collar

=======================

Which of the following statements about fashion inventory is closest to being true?  

  It is better to have too much inventory than it is to have too little  
  It is better to have too little inventory than it is to have too much  
@ Having too little or too much inventory is bad  
  Inventory levels are less important than catching the right trend in your merchandise
 
 Which of the following statements about innovation is true?  

  A product may legally be described as new and innovative for three years.  
@ The Federal Trade Commission has created a legal definition of the term innovation.  
  A product may be called an innovation even though it is not significantly changed from its predecessors.  
  Innovation is anything that the marketer decides to call new.  
  Customer perception has nothing to do with the definition of innovation.  

   Which of the following statements about the adopter categories is true?  

  Over 50 percent of the population is classified as late majority adopters.  
  Early adopters are risk-aversive.  
@ Early majority adopters are typically middle class and like to buy well-established products.  
  Innovators are a much larger group than laggards.  
  Late majority adopters are typically very concerned about social acceptance. 

=========================

Why is successful new-product introduction becoming increasingly more difficult?  

  Many international companies are engaging in product innovation.  
  New-product development costs are so low that any company with a good idea can create a new product.  
  Government regulations have become so stringent that new products are usually years in the testing stage while waiting   approvals.  
@ Products become outdated at a much faster rate than ever before.  
  All of the above are reasons why successful new-product introduction is becoming increasingly more difficult. 

Why is the portfolio concept a useful tool to use in managing the product mix?  

  In order to maximize synergy  
  Question marks and stars are in growing markets  
  In order to be sure you have both cash cows and dogs in your product mix  
@ Because markets are dynamic  

===========================

   _____ are manufactured goods or subassemblies of finished items that organizations need to complete their own products.  

@ Component parts  
  Accessory equipment  
  Processed materials  
  MRO products  
  Raw materials  

   _____ are the last in the population to adopt a new product.  

@ Laggards  
  Late majority adopters  
  Slackers  
  Shirkers  
  Dawdlers 

   _____ is the process by which a consumer or business customer begins to buy and use a new good, service, or an idea.  

  Commercialization  
  Product testing  
@ Product adoption  
  Market modification  
  Positioning  

Question 1    Multiple Choice  0 of 1 point(s)    
  Procter & Gamble introduced its Duncan Hines ready-to-spread frosting in a small geographic area. When General Foods became aware of the product, it rushed to market its own Betty Crocker ready-to-spread frosting, which eclipsed the Duncan Hines product introduction. This act illustrates:  
 Your Answer:  [None Given]  
Correct Answer:  one of the problems associated with test marketing  
 


  Question 2    Multiple Choice  0 of 1 point(s)    
  A _____ is a modification to an existing product to differentiate one brand from another. The consumer doesn't have to learn anything to use the new product.  
 Your Answer:  [None Given]  
Correct Answer:  continuous innovation  
 


  Question 3    Multiple Choice  0 of 1 point(s)    
  Practically every brand of soft drink ever made is available at Pop's Soda Shop-regional beverages from around the world can be purchased at the retailer's Web site. The retailer targets people who have developed a fondness for a particular soft drink and can no longer find it. The retailer's _______________ is all types of soft drinks.  
 Your Answer:  [None Given]  
Correct Answer:  product mix  
 


  Question 4    Multiple Choice  0 of 1 point(s)    
  How do marketers use the product life cycle?  
 Your Answer:  [None Given]  
Correct Answer:  to explain how product features change over the life of a product  
 


  Question 5    Multiple Choice  0 of 1 point(s)    
  Which of the following statements about the adopter categories is true?  
 Your Answer:  [None Given]  
Correct Answer:  Early majority adopters are typically middle class and like to buy well-established products.  
 


  Question 6    Multiple Choice  0 of 1 point(s)    
  For 85 years, Jamesway has made egg incubators for all kinds and sizes of eggs for poultry raisers. Egg incubators are Jamesway's total product offering. These egg incubators are an example of a:  
 Your Answer:  [None Given]  
Correct Answer:  product line  
 


  Question 7    Multiple Choice  0 of 1 point(s)    
  A group of business entrepreneurs who worried about their teenage children drinking and driving decided there must be some way to approach this problem proactively. They came up with an idea for a pair of blurry goggles. They believed looking through the goggles would simulate what the world looks like when someone is seriously intoxicated. The first crude pair of blurry goggles developed by the entrepreneurs to show to prospective customers was:  
 Your Answer:  [None Given]  
Correct Answer:  a prototype  
 


  Question 8    Multiple Choice  0 of 1 point(s)    
  Why is the portfolio concept a useful tool to use in managing the product mix?  
 Your Answer:  [None Given]  
Correct Answer:  Because markets are dynamic  
 


  Question 9    Multiple Choice  0 of 1 point(s)    
  _____ is the process by which a consumer or business customer begins to buy and use a new good, service, or an idea.  
 Your Answer:  [None Given]  
Correct Answer:  Product adoption  
 


  Question 10    Multiple Choice  0 of 1 point(s)    
  What did your instructor mean when he talked about synergy in product mix expansion?  
 Your Answer:  [None Given]  
Correct Answer:  Synergy describes the marginal cost of adding a new product or line given the competencies a company already has  
 




Question 1    Multiple Choice  0 of 1 point(s)    
  SoBe targets the same people who watch the Gravity Games and enjoy skateboarding, inline skating, mountain biking, and other extreme sports. It makes high-energy drinks for this target market. It does not attempt to make any drinks that are not targeted to this market. Therefore, SoBe has a(n):  
 Your Answer:  [None Given]  
Correct Answer:  limited-line strategy  
 


  Question 2    Multiple Choice  0 of 1 point(s)    
  _____ are the last in the population to adopt a new product.  
 Your Answer:  [None Given]  
Correct Answer:  Laggards  
 


  Question 3    Multiple Choice  0 of 1 point(s)    
  For the business market, a multi-purpose absorbent used to clean up any type of spills would be an example of a(n):  
 Your Answer:  [None Given]  
Correct Answer:  maintenance product  
 


  Question 4    Multiple Choice  0 of 1 point(s)    
  The built-in, fold-down child seats such as are found in Chrysler minivans were an example of a _____ because they required consumers to learn how to properly use the seat as well as the advantages of the seat.  
 Your Answer:  [None Given]  
Correct Answer:  dynamically continuous innovation  
 


  Question 5    Multiple Choice  0 of 1 point(s)    
  When Bank One, a traditional banking company, launched WingSpanBank.com as a freestanding all-Internet bank, many regular customers of Bank One became WingSpanBank.com customers. Bank One experienced:  
 Your Answer:  [None Given]  
Correct Answer:  cannibalization  
 


  Question 6    Multiple Choice  0 of 1 point(s)    
  _____ describes how the use of a product spreads throughout a population.  
 Your Answer:  [None Given]  
Correct Answer:  Diffusion  
 


  Question 7    Multiple Choice  0 of 1 point(s)    
  Unlike innovators, early adopters:  
 Your Answer:  [None Given]  
Correct Answer:  have greater concern for social acceptance  
 


  Question 8    Multiple Choice  0 of 1 point(s)    
  There are numerous kinds of shampoo-vitamin-enriched, fruit-enhanced, color-sensitive, therapeutic, etc. But, shampoo that simply promises to clean hair has been a product with declining sales, little or no advertising, and phased out by most manufacturers. Shampoo that makes no other promise than to clean hair is in the _______________ stage of its product life cycle.  
 Your Answer:  [None Given]  
Correct Answer:  decline  
 


  Question 9    Multiple Choice  0 of 1 point(s)    
  A consumer products company like Procter & Gamble that makes a variety of different cleaners, detergents, and polishers to appeal to a variety of target markets is using a(n):  
 Your Answer:  [None Given]  
Correct Answer:  full line strategy  
 


  Question 10    Multiple Choice  0 of 1 point(s)    
  Which of the following is an example of a good?  
 Your Answer:  [None Given]  
Correct Answer:  a text book  
 

Question 1    Multiple Choice  1 of 1 point(s)    
  In addition to regular Alka-Seltzer, there is Alka-Seltzer for cold sufferers, people with upset stomachs, people who can't sleep, people with allergies, and people who have a worse than normal headache. The manufacturer of Alka-Seltzer is using a(n) _______________ to reach as many different target markets as possible.  
 Your Answer:  full line strategy  
Correct Answer:  full line strategy  
 


  Question 2    Multiple Choice  0 of 1 point(s)    
  Why would a marketer set the initial price of a product at a high level during the introductory stage of the product life cycle?  
 Your Answer:  [None Given]  
Correct Answer:  To earn back the money invested in developing the new product as quickly as possible  
 


  Question 3    Multiple Choice  0 of 1 point(s)    
  Which of the following is an example of a product?  
 Your Answer:  [None Given]  
Correct Answer:  all of the above  
 


  Question 4    Multiple Choice  0 of 1 point(s)    
  A few years ago, Quaker introduced Fruit Oatmeal Bites, a nugget version of its cereal bars. The product was very successful because it is not messy. As a result Kellogg's has introduced a similar product under its Nutri-Grain brand. Mini-cereal bars are most likely in the _______________ stage of their product life cycle.  
 Your Answer:  [None Given]  
Correct Answer:  growth  
 


  Question 5    Multiple Choice  0 of 1 point(s)    
  Which of the following is an example of a product dimension consumers use to determine product quality?  
 Your Answer:  [None Given]  
Correct Answer:  all of the above  
 


  Question 6    Multiple Choice  0 of 1 point(s)    
  What is the assumption behind product strategy discussed in class?  
 Your Answer:  [None Given]  
Correct Answer:  No company has only one product  
 


  Question 7    Multiple Choice  0 of 1 point(s)    
  Kitchen dishcloths are often difficult to rinse clean and typically dry slowly. 3M has developed a disposable ScotchBrite Microfiber Kitchen Cloth that rinses easily and dries quickly. What kind of marketing communications should 3M use with this new product?  
 Your Answer:  [None Given]  
Correct Answer:  informative advertising  
 


  Question 8    Multiple Choice  0 of 1 point(s)    
  The _____ for a guest paying for a night's stay at a hotel would include fresh flowers in the room, remote control television, express check-out, and superb room service.  
 Your Answer:  [None Given]  
Correct Answer:  augmented product  
 


  Question 9    Multiple Choice  0 of 1 point(s)    
  Even though most people in the original target market for electric toothbrushes already knew how to brush their teeth, consumers still had some learning to do in order to make good use of the new brushing technique. The electric toothbrush is an example of a:  
 Your Answer:  [None Given]  
Correct Answer:  dynamically continuous innovation  
 


  Question 10    Multiple Choice  0 of 1 point(s)    
  Which of the following is the best example of a convenience product?  
 Your Answer:  [None Given]  
Correct Answer:  chewing gum  
 



Question 1    Multiple Choice  0 of 1 point(s)    
  The business analysis for a new product begins with:  
 Your Answer:  [None Given]  
Correct Answer:  assessing how the new product will fit into a firm's total product mix  
 


  Question 2    Multiple Choice  0 of 1 point(s)    
**  Which of the following is the best example of a durable good?  
 Your Answer:  [None Given]  
Correct Answer:  computer printer  
 


  Question 3    Multiple Choice  0 of 1 point(s)    
  A few years ago, Quaker introduced Fruit Oatmeal Bites, a nugget version of its cereal bars. The product was very successful because it is not messy. As a result Kellogg's has introduced a similar product under its Nutri-Grain brand. Mini-cereal bars are most likely in the _______________ stage of their product life cycle.  
 Your Answer:  [None Given]  
Correct Answer:  growth  
 


  Question 4    Multiple Choice  0 of 1 point(s)    
  Procter & Gamble introduced its Duncan Hines ready-to-spread frosting in a small geographic area. When General Foods became aware of the product, it rushed to market its own Betty Crocker ready-to-spread frosting, which eclipsed the Duncan Hines product introduction. This act illustrates:  
 Your Answer:  [None Given]  
Correct Answer:  one of the problems associated with test marketing  
 


  Question 5    Multiple Choice  0 of 1 point(s)    
  Which of the following is the best definition of style (as discussed in class)  
 Your Answer:  [None Given]  
Correct Answer:  A distinctive manner of presentation or construction  
 


  Question 6    Multiple Choice  0 of 1 point(s)    
  Kodak has developed the Kodak picture kit, which contains "everything you need to share and enjoy your pictures." The kit has five rolls of Kodak Max film, a decorative photo album, ready-to-mail photo frames, photo software CD, and photo storage case. This is a new product designed to attract people who take traditional photos as well as digital pictures. Which of the following statements about this new product are true?  
 Your Answer:  [None Given]  
Correct Answer:  The purpose of its marketing communications should be to inform customers about the benefits of the new product.  
 


  Question 7    Multiple Choice  0 of 1 point(s)    
  _____ describes how the use of a product spreads throughout a population.  
 Your Answer:  [None Given]  
Correct Answer:  Diffusion  
 


  Question 8    Multiple Choice  0 of 1 point(s)    
  Prior to the development of the cardiac stent, surgeons used balloons to clear obstructed heart vessels, but there was no way to ensure the vessels would stay open. The stent is a tiny metal scaffold that props open the formerly obstructed blood vessel and keeps it from becoming obstructed again. Surgeons who use the stent had to engage in a great deal of learning to understand its use and how valuable a tool it was in patient care. There was nothing like the stent on the market when it first appeared. The cardiac stent was an example of a:  
 Your Answer:  [None Given]  
Correct Answer:  discontinuous innovation  
 


  Question 9    Multiple Choice  0 of 1 point(s)    
  Underwear-like disposable training pants for toddlers are in the _______________ stage of their product life cycle because new competitors are entering the market as well as variations and improvements of the original product. The goal of manufacturers of this product is to encourage brand loyalty through heavy advertising to counter new competitors.  
 Your Answer:  [None Given]  
Correct Answer:  growth  
 


  Question 10    Multiple Choice  0 of 1 point(s)    
  What strategic options does a manufacturer of a product in the decline stage of its product life cycle have?  
 Your Answer:  [None Given]  
Correct Answer:  It can do any of the above.  


Question 5    Multiple Choice  0 of 1 point(s)    
  A knockoff is a:  
 Your Answer:  [None Given]  
Correct Answer:  new product that copies, with slight modifications, the design of an original product  
 


  Question 6    Multiple Choice  0 of 1 point(s)    
  A few years ago, Quaker introduced Fruit Oatmeal Bites, a nugget version of its cereal bars. The product was very successful because it is not messy. As a result Kellogg's has introduced a similar product under its Nutri-Grain brand. Mini-cereal bars are most likely in the _______________ stage of their product life cycle.  
 Your Answer:  [None Given]  
Correct Answer:  growth  
 


  Question 7    Multiple Choice  0 of 1 point(s)    
  Kitchen dishcloths are often difficult to rinse clean and typically dry slowly. 3M has developed a disposable ScotchBrite Microfiber Kitchen Cloth that rinses easily and dries quickly. The marketing goal for the product manager of this product is to:  
 Your Answer:  [None Given]  
Correct Answer:  get first-time buyers to try the product  
 


  Question 8    Multiple Choice  0 of 1 point(s)    
  A firm's _______________ is its entire range of products the company offers for sale.  
 Your Answer:  [None Given]  
Correct Answer:  product mix  
 


  Question 9    Multiple Choice  0 of 1 point(s)    
  Which of the following is an example of a product dimension consumers use to determine product quality?  
 Your Answer:  [None Given]  
Correct Answer:  all of the above  
 


  Question 10    Multiple Choice  0 of 1 point(s)    
  At a retail store, the displays racks on which clothes are hung, the shelving, and the cash registers are all examples of:  
 Your Answer:  [None Given]  
Correct Answer:  accessory equipment  
 


 Question 1    Multiple Choice  0 of 1 point(s)    
  Marketers classify convenience products as:  
 Your Answer:  [None Given]  
Correct Answer:  staples, impulse products, and emergency products  
 


  Question 2    Multiple Choice  0 of 1 point(s)    
  A knockoff is a:  
 Your Answer:  [None Given]  
Correct Answer:  new product that copies, with slight modifications, the design of an original product  
 


  Question 3    Multiple Choice  0 of 1 point(s)    
  _____ describes how the use of a product spreads throughout a population.  
 Your Answer:  [None Given]  
Correct Answer:  Diffusion  
 


  Question 4    Multiple Choice  0 of 1 point(s)    
  Kitchen dishcloths are often difficult to rinse clean and typically dry slowly. 3M has developed a disposable ScotchBrite Microfiber Kitchen Cloth that rinses easily and dries quickly. The marketing goal for the product manager of this product is to:  
 Your Answer:  [None Given]  
Correct Answer:  get first-time buyers to try the product  
 


  Question 5    Multiple Choice  0 of 1 point(s)    
  Consumers must engage in a great amount of learning to use a _____ because no similar product has ever been on the market.  
 Your Answer:  [None Given]  
Correct Answer:  discontinuous innovation  
 


  Question 6    Multiple Choice  0 of 1 point(s)    
  Which of the following is the best example of a nondurable good?  
 Your Answer:  [None Given]  
Correct Answer:  ream of paper  
 


  Question 7    Multiple Choice  0 of 1 point(s)    
  Unlike innovators, early adopters:  
 Your Answer:  [None Given]  
Correct Answer:  have greater concern for social acceptance  
 


  Question 8    Multiple Choice  0 of 1 point(s)    
  Which of the following statements about the adopter categories is true?  
 Your Answer:  [None Given]  
Correct Answer:  Early majority adopters are typically middle class and like to buy well-established products.  
 


  Question 9    Multiple Choice  0 of 1 point(s)    
  What did your instructor mean when he talked about synergy in product mix expansion?  
 Your Answer:  [None Given]  
Correct Answer:  Synergy describes the marginal cost of adding a new product or line given the competencies a company already has  
 


  Question 10    Multiple Choice  0 of 1 point(s)    
  How do marketers use the product life cycle?  
 Your Answer:  [None Given]  
Correct Answer:  to explain how product features change over the life of a product  
 


Question 8    Multiple Choice  0 of 1 point(s)    
  Innovators are:  
 Your Answer:  [None Given]  
Correct Answer:  accurately described by all of the above  
 


  Question 9    Multiple Choice  0 of 1 point(s)    
  Kodak introduced the Kodak Funtime film to compete with lower-priced brands, but it found many of its regular customers buying Funtime instead of their usual Kodak film. Kodak experienced:  
 Your Answer:  [None Given]  
Correct Answer:  cannibalization  
 


  Question 10    Multiple Choice  0 of 1 point(s)    
  Marketers typically view all products as:  
 Your Answer:  [None Given]  
Correct Answer:  bundles of attributes  
 
Question 1    Multiple Choice  0 of 1 point(s)    
  Prior to the development of the cardiac stent, surgeons used balloons to clear obstructed heart vessels, but there was no way to ensure the vessels would stay open. The stent is a tiny metal scaffold that props open the formerly obstructed blood vessel and keeps it from becoming obstructed again. In terms of the five characteristics of innovations that affect the rate of adoption, the cardiac stent would have a low degree of:  
 Your Answer:  [None Given]  
Correct Answer:  trialability  
 


  Question 2    Multiple Choice  0 of 1 point(s)    
  The last stage in the new-product development process is:  
 Your Answer:  [None Given]  
Correct Answer:  commercialization  
 


  Question 3    Multiple Choice  0 of 1 point(s)    
  The Cray Inc. has only one product line-all the company manufactures is supercomputers. This one product line is also the company's:  
 Your Answer:  [None Given]  
Correct Answer:  product mix  
 


  Question 4    Multiple Choice  0 of 1 point(s)    
  Underwear-like disposable training pants for toddlers are in the _______________ stage of their product life cycle because new competitors are entering the market as well as variations and improvements of the original product. The goal of manufacturers of this product is to encourage brand loyalty through heavy advertising to counter new competitors.  
 Your Answer:  [None Given]  
Correct Answer:  growth  
 


  Question 5    Multiple Choice  0 of 1 point(s)    
  At a retail store, the displays racks on which clothes are hung, the shelving, and the cash registers are all examples of:  
 Your Answer:  [None Given]  
Correct Answer:  accessory equipment  
 


  Question 6    Multiple Choice  0 of 1 point(s)    
  Which of the following is an example of a good?  
 Your Answer:  [None Given]  
Correct Answer:  a text book  
 


  Question 7    Multiple Choice  0 of 1 point(s)    
  Why is successful new-product introduction becoming increasingly more difficult?  
 Your Answer:  [None Given]  
Correct Answer:  Products become outdated at a much faster rate than ever before.  
 


  Question 8    Multiple Choice  0 of 1 point(s)    
  A firm's _______________ is its entire range of products the company offers for sale.  
 Your Answer:  [None Given]  
Correct Answer:  product mix  
 


  Question 9    Multiple Choice  0 of 1 point(s)    
  What is the assumption behind product strategy discussed in class?  
 Your Answer:  [None Given]  
Correct Answer:  No company has only one product  
 


  Question 10    Multiple Choice  0 of 1 point(s)    
  Procter & Gamble introduced its Duncan Hines ready-to-spread frosting in a small geographic area. When General Foods became aware of the product, it rushed to market its own Betty Crocker ready-to-spread frosting, which eclipsed the Duncan Hines product introduction. General Foods was able to enter the _____ stage of the new-product development process before Procter & Gamble could.  
 Your Answer:  [None Given]  
Correct Answer:  commercialization  
 

Question 1    Multiple Choice  1 of 1 point(s)    
  _____ are the last in the population to adopt a new product.  
 Your Answer:  Laggards  
Correct Answer:  Laggards  
 


  Question 2    Multiple Choice  1 of 1 point(s)    
**  Which of the following is the best example of a durable good?  
 Your Answer:  computer printer  
Correct Answer:  computer printer  
 


  Question 3    Multiple Choice  0 of 1 point(s)    
  Pepsi-Cola's development of Wild Cherry Pepsi, a cherry-flavored soda, is an example of a:  
 Your Answer:  [None Given]  
Correct Answer:  continuous innovation  
 


  Question 4    Multiple Choice  0 of 1 point(s)    
  For the business market, a multi-purpose absorbent used to clean up any type of spills would be an example of a(n):  
 Your Answer:  [None Given]  
Correct Answer:  maintenance product  
 


  Question 5    Multiple Choice  0 of 1 point(s)    
  The _____ for a guest paying for a night's stay at a hotel would include fresh flowers in the room, remote control television, express check-out, and superb room service.  
 Your Answer:  [None Given]  
Correct Answer:  augmented product  
 


  Question 6    Multiple Choice  0 of 1 point(s)    
  Marketers classify innovations based on their:  
 Your Answer:  [None Given]  
Correct Answer:  degree of newness  
 


  Question 7    Multiple Choice  0 of 1 point(s)    
  Cheese crackers have been around for quite awhile. Pepperidge Farm created a cracker variation by making its crackers in the shape of goldfish. To attract new users and to maintain market share, Pepperidge Farms has enlarged its Goldfish crackers and added cheese and peanut butter fillings. This strategy indicates that Goldfish cheese crackers are in the _______________ stage of their product life cycle.  
 Your Answer:  [None Given]  
Correct Answer:  maturity  
 


  Question 8    Multiple Choice  0 of 1 point(s)    
  Which of the following is an example of a product dimension consumers use to determine product quality?  
 Your Answer:  [None Given]  
Correct Answer:  all of the above  
 


  Question 9    Multiple Choice  0 of 1 point(s)    
  Multivitamins are in the _______________ stage of their product life cycle. Sales are primarily to current users who are restocking. New variations of the product such as special formulas for women and men are employed to attract new users. Reminder advertising is commonly used.  
 Your Answer:  [None Given]  
Correct Answer:  maturity  
 


  Question 10    Multiple Choice  0 of 1 point(s)    
  Which of the following is an example of a good?  
 Your Answer:  [None Given]  
Correct Answer:  a text book  
 


Question 1    Multiple Choice  1 of 1 point(s)    
  _____ are the last in the population to adopt a new product.  
 Your Answer:  Laggards  
Correct Answer:  Laggards  
 


  Question 2    Multiple Choice  1 of 1 point(s)    
  A _______________ is a firm's total product offering designed to satisfy a single need or desire of a group of target customers.  
 Your Answer:  product line  
Correct Answer:  product line  
 


  Question 3    Multiple Choice  1 of 1 point(s)    
  When Mae's mother died, Mae was glad her mother had prepaid for a funeral. Then Mae was told new state regulations require each coffin placed in the ground had to be in a burial vault-an additional $800 expense. For Mae, the burial vault was a(n):  
 Your Answer:  unsought product  
Correct Answer:  unsought product  
 


  Question 4    Multiple Choice  1 of 1 point(s)    
  Which of the following is an example of a product dimension consumers use to determine product quality?  
 Your Answer:  all of the above  
Correct Answer:  all of the above  
 


  Question 5    Multiple Choice  0 of 1 point(s)    
  The Disney Web site is full of games, activities, music, and stories to appeal to children of all ages. Before a parent commits to paying the annual membership fee to gain access to the site, he or she can register for one free month of use. By accepting the free month's usage, potential adopters move into the _____ stage.  
 Your Answer:  [None Given]  
Correct Answer:  trial  
 


  Question 6    Multiple Choice  0 of 1 point(s)    
  When Bank One, a traditional banking company, launched WingSpanBank.com as a freestanding all-Internet bank, many regular customers of Bank One became WingSpanBank.com customers. Bank One experienced:  
 Your Answer:  [None Given]  
Correct Answer:  cannibalization  
 


  Question 7    Multiple Choice  0 of 1 point(s)    
  Why is successful new-product introduction becoming increasingly more difficult?  
 Your Answer:  [None Given]  
Correct Answer:  Products become outdated at a much faster rate than ever before.  
 


  Question 8    Multiple Choice  0 of 1 point(s)    
  Disposable diapers for infants and toddlers are in the maturity stage of their product life cycle. This means:  
 Your Answer:  [None Given]  
Correct Answer:  manufacturers of disposable diapers are experiencing narrowing profit margins  
 


  Question 9    Multiple Choice  0 of 1 point(s)    
  In addition to regular Alka-Seltzer, there is Alka-Seltzer for cold sufferers, people with upset stomachs, people who can't sleep, people with allergies, and people who have a worse than normal headache. The manufacturer of Alka-Seltzer is using a(n) _______________ to reach as many different target markets as possible.  
 Your Answer:  [None Given]  
Correct Answer:  full line strategy  
 


  Question 10    Multiple Choice  0 of 1 point(s)    
  The _____ for a guest paying for a night's stay at a hotel is rest and sleep.  
 Your Answer:  [None Given]  
Correct Answer:  core product  
 

 Question 1    Multiple Choice  1 of 1 point(s)    
  Why is the portfolio concept a useful tool to use in managing the product mix?  
 Your Answer:  Because markets are dynamic  
Correct Answer:  Because markets are dynamic  
 


  Question 2    Multiple Choice  0 of 1 point(s)    
  The _____ for a guest paying for a night's stay at a hotel is rest and sleep.  
 Your Answer:  augmented product  
Correct Answer:  core product  
 


  Question 3    Multiple Choice  0 of 1 point(s)    
  Mercedes was able to successfully introduce its C-Class car at $30,000 without injuring its ability to sell other Mercedes cars for $100,000 or more. Mercedes implemented a(n):  
 Your Answer:  [None Given]  
Correct Answer:  downward line stretch  
 


  Question 4    Multiple Choice  0 of 1 point(s)    
  How do marketers use the product life cycle?  
 Your Answer:  [None Given]  
Correct Answer:  to explain how product features change over the life of a product  
 


  Question 5    Multiple Choice  0 of 1 point(s)    
  In the business market, steel tubing, natural cloth fabric, and lumber are all examples of:  
 Your Answer:  [None Given]  
Correct Answer:  processed materials  
 


  Question 6    Multiple Choice  0 of 1 point(s)    
  Tony wanted to buy a pair of binoculars but not just any brand. He wanted to buy a pair of Steiner Nighthunter just like the pair he had learned about in his newest copy of Petersen's Hunting magazine. He had to visit five different sporting goods store before he found this brand. For Tony, the Steiner Nighthunter is an example of a:  
 Your Answer:  [None Given]  
Correct Answer:  specialty product  
 


  Question 7    Multiple Choice  0 of 1 point(s)    
  Prior to the development of the cardiac stent, surgeons used balloons to clear obstructed heart vessels, but there was no way to ensure the vessels would stay open. The stent is a tiny metal scaffold that props open the formerly obstructed blood vessel and keeps it from becoming obstructed again. Surgeons who use the stent had to engage in a great deal of learning to understand its use and how valuable a tool it was in patient care. There was nothing like the stent on the market when it first appeared. The cardiac stent was an example of a:  
 Your Answer:  [None Given]  
Correct Answer:  discontinuous innovation  
 


  Question 8    Multiple Choice  0 of 1 point(s)    
**  Which of the following is the best example of a durable good?  
 Your Answer:  [None Given]  
Correct Answer:  computer printer  
 


  Question 9    Multiple Choice  0 of 1 point(s)    
  The _____ for a guest paying for a night's stay at a hotel would include fresh flowers in the room, remote control television, express check-out, and superb room service.  
 Your Answer:  [None Given]  
Correct Answer:  augmented product  
 


  Question 10    Multiple Choice  0 of 1 point(s)    
  _____ are manufactured goods or subassemblies of finished items that organizations need to complete their own products.  
 Your Answer:  [None Given]  
Correct Answer:  Component parts  
 


Question 1    Multiple Choice  0 of 1 point(s)    
  One of the advantages of test marketing is:  
 Your Answer:  [None Given]  
Correct Answer:  the ability to evaluate a product and its marketing program before commercialization  
 


  Question 2    Multiple Choice  0 of 1 point(s)    
  Marketers classify convenience products as:  
 Your Answer:  [None Given]  
Correct Answer:  staples, impulse products, and emergency products  
 


  Question 3    Multiple Choice  0 of 1 point(s)    
  Marketers typically view all products as:  
 Your Answer:  [None Given]  
Correct Answer:  bundles of attributes  
 


  Question 4    Multiple Choice  0 of 1 point(s)    
  A knockoff is a:  
 Your Answer:  [None Given]  
Correct Answer:  new product that copies, with slight modifications, the design of an original product  
 


  Question 5    Multiple Choice  0 of 1 point(s)    
  Cheese crackers have been around for quite awhile. Pepperidge Farm created a cracker variation by making its crackers in the shape of goldfish. To attract new users and to maintain market share, Pepperidge Farms has enlarged its Goldfish crackers and added cheese and peanut butter fillings. This strategy indicates that Goldfish cheese crackers are in the _______________ stage of their product life cycle.  
 Your Answer:  [None Given]  
Correct Answer:  maturity  
 


  Question 6    Multiple Choice  0 of 1 point(s)    
  In the business market, wheat, coal, and bauxite (used to make aluminum) are all examples of:  
 Your Answer:  [None Given]  
Correct Answer:  raw materials  
 


  Question 7    Multiple Choice  0 of 1 point(s)    
  In terms of the five characteristics of innovations that affect the rate of adoption, the digital camera would most clearly have a higher degree of _____ than flavored bottled water.  
 Your Answer:  [None Given]  
Correct Answer:  complexity  
 


  Question 8    Multiple Choice  0 of 1 point(s)    
  A _____ is a pronounced modification to an existing product that requires a modest amount of learning or change in behavior to use it.  
 Your Answer:  [None Given]  
Correct Answer:  dynamically continuous innovation  
 


  Question 9    Multiple Choice  0 of 1 point(s)    
  What strategic options does a manufacturer of a product in the decline stage of its product life cycle have?  
 Your Answer:  [None Given]  
Correct Answer:  It can do any of the above.  
 


  Question 10    Multiple Choice  0 of 1 point(s)    
  According to the definition in the text, which of the following was an example of an innovation?  
 Your Answer:  [None Given]  
Correct Answer:  all of the above  
 

 Question 1    Multiple Choice  0 of 1 point(s)    
  One of the reasons given for the success of the reality television shows that appeared on networks in the 1990s was the fact the shows appealed to a cultural bias that real life drama is preferable to fiction. At this point in U.S. history, Americans desired adventure and escapism that offered them the opportunity for participation but did not require it. In terms of the five characteristics of innovations that affect the rate of adoption, reality television shows have a high degree of _____ with current cultural values.  
 Your Answer:  [None Given]  
Correct Answer:  compatability  
 


  Question 2    Multiple Choice  0 of 1 point(s)    
  A consumer products company like Procter & Gamble that makes a variety of different cleaners, detergents, and polishers to appeal to a variety of target markets is using a(n):  
 Your Answer:  [None Given]  
Correct Answer:  full line strategy  
 


  Question 3    Multiple Choice  0 of 1 point(s)    
  Kodak has developed the Kodak picture kit, which contains "everything you need to share and enjoy your pictures." The kit has five rolls of Kodak Max film, a decorative photo album, ready-to-mail photo frames, photo software CD, and photo storage case. This is a new product designed to attract people who take traditional photos as well as digital pictures. Which of the following statements about this new product are true?  
 Your Answer:  [None Given]  
Correct Answer:  The purpose of its marketing communications should be to inform customers about the benefits of the new product.  
 


  Question 4    Multiple Choice  0 of 1 point(s)    
  In the business market, wheat, coal, and bauxite (used to make aluminum) are all examples of:  
 Your Answer:  [None Given]  
Correct Answer:  raw materials  
 


  Question 5    Multiple Choice  0 of 1 point(s)    
  For an amusement park, concession food trailers, the parking lot, the first-aid building, and the building where the park's daily receipts are tallied are all examples of:  
 Your Answer:  [None Given]  
Correct Answer:  equipment  
 


  Question 6    Multiple Choice  0 of 1 point(s)    
  What is the assumption behind product strategy discussed in class?  
 Your Answer:  [None Given]  
Correct Answer:  No company has only one product  
 


  Question 7    Multiple Choice  0 of 1 point(s)    
  The first stage of the new-product development process is:  
 Your Answer:  [None Given]  
Correct Answer:  idea generation  
 


  Question 8    Multiple Choice  0 of 1 point(s)    
  Customers usually know a good deal about _____, are loyal to specific brands, and spend little, if any, time comparing alternatives.  
 Your Answer:  [None Given]  
Correct Answer:  specialty products  
 


  Question 9    Multiple Choice  0 of 1 point(s)    
  How do marketers use the product life cycle?  
 Your Answer:  [None Given]  
Correct Answer:  to explain how product features change over the life of a product  
 


  Question 10    Multiple Choice  0 of 1 point(s)    
  Innovators are:  
 Your Answer:  [None Given]  
Correct Answer:  accurately described by all of the above  
 


   A _______________ is probably the most used and most recognized form of branding.  

@  brand name  
  copyright  
  patent  
  logo  
  trade character 

 A _______________ is a name, a term, a symbol, or other unique element of a product that identifies one firm's products and sets it apart from the competition.  

  credence quality  
  customer expectation  
  promotion strategy  
@ brand  
  positioning strategy

 A company that promises to respond to e-mail requests for information within 24 hours is positioning its service using the ________ dimension.  

  Tangibles  
  Reliability  
@ Responsiveness  
  Empathy  
  Cognition 

   A service provider would use _______________ in an effort to sell the firm's employees on the idea they work for a superior company of which they can be proud.  

  interactive communication  
  transactional exchange  
@ internal marketing  
  disintermediation  
  employee empowerment

An advertisement for the Blue Ridge Parkway (a highway) that runs from the Shenandoah Valley to the Smokey Mountains describes the highway as a "trip back in time." This is an example of:  

  idea marketing  
  services wholesaling  
  services distribution  
  product management  
@ place marketing 

   According to the goods/services continuum, which of the following is the best example of a pure service provider?  

  a furniture retailer  
  a magazine publisher  
  a building contractor  
  a paper manufacturer  
@ a massage therapist 

 According to the goods/services continuum, which of the following is the best example of a pure good?  

@ a text book  
  a class lecture  
  a haircut  
  a muffler repair  
  a massage  

  According to the text, which of the following is an example of an intangible service?  

  auto repair  
  manicure  
@ a religious service  
  plastic surgery  
  all of the above 

 Alfredo would have used _______________ qualities to determine that his English tutor had successfully taught him how to conjugate a verb.  

  operational  
  credence  
@ experience  
  locational  
  search  

  Another name for a store brand is a:  

  manufacturer brand  
  national brand  
  licensed brand  
  generic brand  
@ private-label brand

Arm & Hammer began as a producer of baking soda. Now the company manufactures detergents, deodorant, and fabric softeners under the Arm & Hammer brand name. Arm & Hammer uses a _______________ strategy.  

  market positioning  
  customer perception  
  divestment  
@ brand extension  
  product contraction

  As Martin checked into the motel, the clerk at the registration desk was rude and laughed at Martin's request for an early morning wake-up call. In terms of the service encounter dimensions, Martin experienced trouble with the _______________ dimension.  

  production  
  equipment-based  
  fifth  
@ social contact  
  economic 

Capacity management is one of the ways to meet potential problems related to the _______________ characteristic of services.  

  intangibility  
  variability  
  responsiveness  
  inseparability  
@ perishability 

Dan's annual medical examination gives him peace of mind and removes all anxieties about undiscovered health problems. These feelings are the benefits of the doctor's examination and are called:  

@  core services  
  differentiated advantages  
  augmented benefits  
  finite benefits  
  differentiated feature 

   Dell Computer Corp. uses the same name that has been used by Dell comics for more than fifty years. This action:  

  is illegal when done intentionally  
@ is completely legal because the products do not compete  
  is in direct violation of the Lanham Act  
  was addressed in a memorandum by the World Trade Organization  
  is prohibited in international markets  

Disintermediation is one of the ways to meet potential problems related to the _______________ characteristic of services.  

  intangibility  
  variability  
  responsiveness  
@ inseparability  
  perishability  

 Disney allowed a paint manufacturer to produce paint using the Disney brand name for a specific period of time. Disney paint is an example of:  
 
  market franchising  
@ licensing  
  trademarking  
  copyright infringement  
  piggybacking 

  Employee empowerment is one of the ways to meet potential problems related to the _______________ characteristic of services.  

  intangibility  
@ variability  
  responsiveness  
  inseparability  
  perishability 
 
   Every time Bonnie gets her weekly massage, she enjoys the aromatherapy candles her masseur lights and the New Age music that is playing in the background. This is the only spa in her area that offers candles and music to create an atmosphere in which she can more easily relax to enjoy her massage. The candles and soft music are additional services provided to enhance the value of her massage. They are examples of:  
 
  core services  
  differentiated advantages  
@ augmented services  
  finite benefits  
  differentiated feature

 Every time Bonnie gets her weekly massage, she leaves the spa feeling completely stress-free. This feeling is the benefit the massage provides and is called a(n):  

@  core service  
  differentiated advantage  
  augmented benefit  
  finite benefit  
  differentiated feature

For many years the Grand Ole Opry was a fixture at the Ryman Auditorium in downtown Nashville, Tennessee. The auditorium had superior acoustics but a structure that was not readily updated to meet new safety requirements. The Ryman was also located in a part of Nashville that many did not consider safe. Thus, the Grand Ole Opry built a new facility for the show. The need to make the structure in which the Grand Ole Opry was held more attractive is an example of a(n) _______________ factor.  
 
  operational  
  cross-docking  
  educational  
  functional  
@ environmental 

 Generic brands:  

  are more popular today than ever before  
  were first developed during the last period of economic prosperity in the United States  
@ were designed to appeal to the price conscious consumer  
  attract approximately 25 percent of all dollars spent in the U.S. supermarkets  
  are accurately described by all of the above 
 
   Have you ever gotten a great haircut, gone back to the same person to get a second haircut, and received one of the worst haircuts of your life? The differences between the two haircuts are due to the _______________ characteristic of services.  

  empathy  
@ variability  
  responsiveness  
  reliability  
  intangibility
  
   Hershey Foods and General Mills worked together to create Reese's Peanut Butter Puffs cereal. This is an example of:  

@ ingredient branding  
  contract manufacturing  
  generic branding  
  family branding  
  aggregated branding 

   Henry stopped at a motel that had a vacancy. Upon entering the lobby, he saw two chairs patched with duct tape, a dusty plastic plant, an overflowing trashcan, and an employee reading Soap Opera Digest. Which service characteristic could have benefited if the motel had a more inviting appearance?  

@ intangibility  
  homogeneity  
  responsiveness  
  empathy  
  reliability

Hillary wants to be an author. She has sent a manuscript she wrote to an agent who will critique the manuscript and tell Hillary if her work is marketable. Because Hillary does not know if her work is marketable, she has decided to accept whatever the agent says as true. She will use _______________ qualities to evaluate the agent's critical opinion.  

  operational  
@ credence  
  experience  
  locational  
  search  

In many communities, you can find as many as thirty different fast-food restaurant franchises within a couple of miles of each other. The nearness of these restaurants to each other illustrates the importance of the _______________ factor.  

  operational  
  cross-docking  
@ locational  
  functional  
  educational

In 2001, the United Methodist Church in the United States budgeted $4 million for ads to encourage former church members to return to church through the use of gentle persuasion. This ad campaign is an example of:  

@  idea marketing  
  services wholesaling  
  services distribution  
  product management  
  place marketing  
 
 In the fall of 2002, Sheraton Hotels & Resorts launched a new service guarantee that includes a training program for its employees. This training program will lessen the problems related to the _______________ characteristic of services.  

  intangibility  
  viscosity  
  responsiveness  
@ inseparability  
  perishability 

 In the fall of 2002, Sheraton Hotels & Resorts launched a training program for its employees to convince them that management and employees working together can make each service encounter a competitive advantage. Sheraton is using:  

  interactive communication  
  transactional exchange  
@ internal marketing  
  disintermediation  
  employee empowerment

In order to make an evaluative judgment of the quality of a service, customers can use search quality, experience quality, and _______________ quality.  
 
  multitasking  
  empowerment  
  functional  
@ credence  
  operational 

 In _______________, branded materials become component parts of other branded products.  
 
  aggregated branding  
  brand franchising  
@ ingredient branding  
  compound branding  
  umbrella branding

   Jack was so enthusiastic about the band he saw performing at a local club last weekend that he persuaded a large group of friends to go to the club to hear this band on Thursday. Unfortunately, the band's performance was uninspired and at times off key. The performance differences are due to the _______________ characteristic of service.  
 
  empathy  
@ variability  
  responsiveness  
  reliability  
  intangibility 

 Keisha particularly enjoys taking her dog to Pampered Paws for its monthly bath and trim because the people who work on the dogs use a pleasant smelling soap, have full-sized kennels in which to dry the dog, and give the dog a treat when its treatment is finished. These additional services provided by Pampered Paws enhance the value of having a clean dog. Pampered Paws use _______________ to differentiate itself from its competitors.  

  core services  
  differentiated advantages  
@ augmented services  
  finite benefits  
  differentiated feature 


   Many visitors to Six Flags amusement park have complained about the long and slow-moving lines, and the fact that park employees are letting their friends break in line. This equipment-based service provider is having trouble with _______________ factors.  

@ operational  
  cross-docking  
  locational  
  functional  
  educational

   Mattel teamed with Coca-Cola to market Soda Fountain Sweetheart Barbie. This is an example of:  

@ co-branding  
  contract manufacturing  
  generic branding  
  family branding  
  aggregated branding  

   My bank has a little sticker in the window that says deposits are insured by the FDIC. The pank is trying to position its service using the ________ dimension.  

  Physical evidence  
  Intangibles  
  Empathy  
  Relaibility  
@ Assurance 

   Naomi is afraid to go to a dentist, but she has a really severe toothache. Her sister has recommended she go to Dr. Tompkins because he guarantees pain-free dentistry. Assuming Naomi visits Dr. Tompkins, the only way she can evaluate his dental techniques is through _______________ qualities.  

  operational  
  credence  
@ experience  
  locational  
  search  

 One of the advantages of brand equity is...  

  The ability to limit distribution.  
  The ability to spend less on advertising.  
  The ability to save money on quality assurance programs.  
@ The ability to charge higher prices.  
  None of the above are advantages of brand equity.  

 One of the ways Megan determined which day care center to send her children was by visiting neighboring ones and observing the indoor and outdoor play facilities, the cleanliness of the employees' uniforms, and how well infants were segregated from older children. She was evaluating the day care using _______________ qualities before she let the day care keep her children.  

  operational  
  credence  
  experience  
  locational  
@ search  

One of the ways physicians deal with potential problems associated with the _______________ characteristic of services is to hang their medical diplomas on the walls of the examining rooms and to wear white coats.  

@ intangibility  
  variability  
  responsiveness  
  inseparability  
  perishability 

People most commonly buy the services of a tattoo artist on the weekends. To encourage people to use their services on the weekdays, many tattoo artists adjust prices to influence demand. In other words, they use differing prices to lessen the problems related to the _______________ characteristic of services.  

  intangibility  
  variability  
  responsiveness  
  inseparability  
@ perishability 

 Pinnacle Foods Corp. markets Vlasic pickles. According to brand designers, Vlasic would:  

  be a perfect brand name for any product  
  be a perfect brand name because it is memorable  
@ not be a good brand name for a pickle because it is hard to spell, to say, to read, and to remember  
  not be a good brand name because it is multisyllabic  
  be a good brand name because it implies the product's benef 
 
 Procter & Gamble makes Tide, Cheer, Ivory Snow, and Bold detergents as well as PertPlus, Rejoice, and Vidal Sassoon shampoos. Through its use of a separate and unique brand name for each of these products, Procter & Gamble is using a(n) _______________ strategy.  
 
  family brand  
  private-label brand  
@ individual brand  
  compound branding  
  umbrella brand  

  Product packaging:  
 
  protects the product  
  makes the product easy for consumers to store  
  communicates brand personality  
  can create a competitive advantage  
@ is accurately described by all of the above 

  Randy Stuart rents storage space to college students who want to go home for the summer, but who do not want to haul all of their property home and then back. The business is profitable during the summer months, but when the storage space is unoccupied, she is losing money. What characteristic of service is most likely the source of Stuart's problem?  

  intangibility  
  variability  
  empathy  
@ perishability  
  responsiveness

 Search qualities would be most useful to a consumer buying:  

  an operation to neuter her dog  
  Spanish tutoring  
  the expertise of a marketing consultant  
  the services of an advertising agency  
@ a new refrigerator 

Service _______________ means that a firm cannot store its services.  

  intangibility  
  variability  
  empathy  
  inseparability  
@ perishability 

  Service _______________ means that customers cannot store services.  

  intangibility  
  variability  
  empathy  
  inseparability  
@ perishability

   Service _______________ means that a service can only take place at the time the service provider performs it on either the customer or the customer's possessions.  

bert@ inseparability  
  variability  
  responsiveness  
  reliability  
  intangibility ?

Service _______________ refers to the inevitable differences in a service provider's performances from one day to the next.  

  intangibility  
@ variability  
  empathy  
  perishability  
  responsiveness 

 The bank management decided its customers would not mind if it only opened the drive-thru window on Saturdays and did not require other bank employees to work on weekends. Customers who work during the week could not apply for a loan on Saturday because that was not a service available at the drive-thru window. Gap analysis would show this is an example of a gap between:  

@ customer expectations and management perceptions  
  management perception and quality standards set by the firm  
  established quality standards and service delivery  
  service quality standards and consumer expectations  
  expected service and perceived service

   The billboard announced that all the rooms in the Courtland Inn had been recently redecorated and that its room rate was still only $39 per night. This offer sounded like a good deal to Brenda, but when she went to her motel room, she found the blinds would not go down, the toilet seat was cracked, the rug was stained, and there was a new bedspread on the lumpy mattress. Brenda felt as if she had been deceived because in her mind the purchase of new bedspreads does not mean redecorated. A gap analysis would reveal a gap between:  

  customer expectations and management predictions  
  management perception and quality standards set by the firm  
  established quality standards and service delivery  
@ service quality standards and consumer expectations  
  expected service and service encounters 

The Christian music industry is the only facet of the music industry that has shown significant growth during the last decade. A recording company executive believes the American people are ready for gospel singers to enter the mainstream market and record some pop music. The executive has found a group of five who have a small following in the Christian music market and has the group recording new songs that are uplifting without being preachy. He hopes that a nationwide tour will expand the market for other Christian music performers. Which of the following strategies is the executive using to market this new group?  

  diversification  
@ market fulfillment  
  customerization  
  pure selling  
  brand extension 

The Federal Fair Packaging and Labeling Act, the Nutrition Labeling and Education Act, and the U.S. Food and Drug Administration work together to:  

  generate new product ideas  
  ensure new product success  
@ ensure consumer safety  
  prevent competitive disadvantages  
  do all of the above  

   The J. M. Smuckers Company makes jams, jellies, fruit spreads, and ice cream toppings. All of these products are sold using the Smucker's brand name. This company uses a(n) _______________ strategy.  

  aggregated branding  
  store brand  
@ family brand  
  generic brand  
  compound branding   

The National Rifle Association (NRA) promotes the safe use of firearms in its advertising as well as encouraging people to support the right of all Americans to bear arms. The NRA is engaged in:  

@  idea marketing  
  services wholesaling  
  services distribution  
  product management  
  place marketing 

 The red devil figure that appears on all Underwood meat spreads is an example of a:  

  packaging messenger  
  copyrighted tactic  
@ brand  
  synergistic promotion  
  a nonregistered trademark  

 The three strategies marketers use to "sell" people are:  

@  pure selling, product improvement, and market fulfillment  
  product development, market development, and diversification  
  product diversification, target marketing, and market development  
  market harvesting, market facilitation, and disintermediation  
  disintermediation, pure selling, and multidistribution 

There are five dimensions that are commonly used to position a service. A service provider might use physical evidence to position a company on the basis of its:  

  reliability  
  responsiveness  
  empathy  
@ tangibles  
  assurance  

Tide detergent, Smucker's toppings, Purina Dog Chow dog food, and Starkist tuna are all examples of:  
 
  family brands  
  licensed brands  
  umbrella brands  
@ national or manufacturer brands  
  store brands 

To minimize the potentially negative effects of bad service encounters and to save on labor costs, some service businesses are using _______________, which eliminates the need for customers to interact with people.  
 
  capacity management  
  service facilitation  
@ disintermediation  
  supply management  
  transactional marketing 

 To reduce problems associated with the intangibility of the services offered by a restaurant, the restaurant owner can:  

  train employees on the polite way to serve food  
  give employees the authority to handle a customer's complaint  
@ provide clean tablecloths for each new customer  
  charge different prices for food purchased between 4 p.m. and 6 p.m.  
  offer a service guarantee to ensure customers that food quality will be consistent  

   Toronto-based Loblaw supermarket chain developed President's Choice brand products to sell exclusively in its stores. President's Choice is an example of a:  

  manufacturer brand  
  national brand  
  licensed brand  
  generic brand  
@ private-label brand 

   Toronto-based Loblaw supermarket chain developed President's Choice brand products to sell exclusively in its stores. The products were so popular the supermarket chain has agreed to allow noncompetitive retailers in other countries to use the President's Choice brand in exchange for paying a fee to Loblaw. By selling its legally protected brand name to another retailer to use for a period of time, Loblaw has engaged in:  

  market franchising  
  piggybacing  
  trademarking  
  copyright infringement  
@ licensing


 Vlasic pickles use a stork with a blue billed hat and a red bow tie on its packaging and its marketing communications. The company uses the stork to:  

  position the product to nature lovers  
@ set itself apart from its competitors  
  avoid using traditional spokespeople  
  insure profitability  
  implement a trading down branding strategy  

 Well-organized desks, professionally attired employees, and travel posters on the wall are all examples of _______________ that can be used to position a travel agency.  

  critical incidents  
@ physical evidence  
  empathy  
  credence qualities  
  experiential evidence 

What do we mean when we speak of negative brand equity?  

  In order to build brand equity, you have to discount prices.  
  Small decreases in price lead to large increases in brand awareness.  
  Positioning a brand as superior to an inferior competitor.  
@ Consumers have unfavorable associations with the brand name.  
  Consumers preference for the brand leads to overconsumption. 

What is the most common explanation for why retailers carry private-label brands?  

  to eliminate the need for competitive advantages  
  to support trade promotions  
  to encourage more transaction marketing  
@ to earn more profit  
  to prevent sales cannibalization  

   When Ria hired an interior designer to redecorate her living room, she was not sure she had made the right decision until she saw how gorgeous her newly decorated living room looked. Ria used _______________ qualities to evaluate the designer's services.  

  operational  
  credence  
@ experience  
  locational  
  search  

Which method of measuring service quality concentrates on customer complaints?  

  the breakdown method  
  the buildup method  
@ the critical incident technique  
  the quality audit  
  the focus group

Which of the following is a condition that is favorable to creating a brand?  

  There is limited availability of the product.  
  The product is a one-of-a-kind.  
  Consumers are unwilling to pay premium prices for similar products.  
  There is no way to distinguish the product from its compteitors other than by the brand name.  
@ Quality and value are easy to maintain.  

Which of the following is an example of a service encounter?  

  when a nurse listens to your heart rate  
  when you check in to a motel  
  when you get a massage  
  when a mechanic tells you what's wrong with your car  
@ all of the above 

 Which of the following is an example of a brand?  

  the picture of the strong arm holding a mallet up in the air that appears on all Arm & Hammer products  
  the Keebler elf  
  the red letter "K" that appears on all Kellogg's brand cereals  
  the red flag with the word French's that appears on all French's brand mustard yellow containers  
@ all of the above

 Which of the following is an example of an equipment-based service?  

  math tutoring  
@ an amusement park  
  a hand carwash  
  a massage  
  a religious service 

Which of the following is the best example of a people-based service?  

  an automatic carwash  
  a museum  
  an x-ray lab  
@ a dating service  
  a movie theater  

   Which of the following levels of familiarity would indicate the largest brand equity?  

@ Brand preference  
  Brand recognition  
  Brand non-recognition  
  Brand rejection  
  Family brand recognition 

Which of the following product offerings would be most likely described as intangible?  

  fish for an aquarium  
  refillable ink cartridges for a computer printer  
  a mink coat  
  a meal from a fast-food restaurant  
@ a physical examination by a doctor

   Which of the following statements about brand equity is true?  

  Brand equity means a brand has customer loyalty.  
  Brand equity refers to the brand's value to an organization.  
  Brand equity can provide a competitive advantage.  
  Brand equity gives a firm the power to capture and hold onto a larger share of the market and to sell at prices with higher profit margins.  
@ All of the above statements about brand equity are true. 

   Which of the following statements best represents your instructor's concept of the essence of branding?  

  A brand is a liability created through advertising  
  A brand is created through operant conditioning  
@ A brand is a promise you make to your customers  
  A brand is the most important component of an intangible product  

  Which of the following product offerings would be most likely described as intangible?  

  fish for an aquarium  
  refillable ink cartridges for a computer printer  
  a mink coat  
  a meal from a fast-food restaurant  
@ a physical examination by a doctor 

   Which of the following products would be a desirable brand name because of how it fits the product benefits?  

  French's mustard  
  Hill's coffee  
  Tombstone pizza  
  Green Giant frozen vegetables  
@ StrideGuard insoles for shoes 

   Which of the following services will most likely be evaluated using credence qualities?  

@  surgery to repair a leaking heart valve  
  math tutoring  
  a cooking class  
  a haircut  
  a stay in a hotel resort  

 Which if the following statements about creating a brand is true?  

  Creating a brand is very different from positioning.  
@ Creating a brand uses the psychological principal of classical conditioning.  
  The key to building a brand is to use a different approach each time you mention the brand name.  
  It is important not to let the customer confuse product benefits with benefit with the brand name.  
  It is relatively inexpensive to create a national brand when the product sells for less than $1  

 Which of the following would be useful if a service provider wanted to reduce the problems associated with the intangibility characteristic of services?  

  capacity management  
  gap analysis  
@ an eye-catching logo  
  employee training  
  a service guarantee  

   Which of the service positioning dimensions suggests that it is important to have consistency in you service delivery?  

  Physical evidence  
  Tangibles  
  Viscosity  
@ Reliability  
  Empathy  

_______________ are acts, efforts, or performances exchanged from producer to user without ownership rights.  

  Expropriation  
@ Services  
  Nationalization  
  Customized retailing  
  Tangibles  

_______________ is a measurement tool that gauges the difference between a customer's expectations of service quality and what actually occurred.  

  Break-even analysis  
  The 80/20 rule  
  A market audit  
@ Gap analysis  
  A SWOT (Services with Optimal Tools) assessment 

   _______________ is the process by which organizations adjust the supply of their services in an attempt to match demand.  

  Disintermediation  
@ Capacity management  
  Warehouse facilitation  
  Inventory control  
  Just-in-time (JIT) inventory management 

   _______________ strategies regard a city, state, country, or other locale as a brand and attempt to position the location so that consumers choose to visit.  
 
  Services retailing  
  Services wholesaling  
  Services distribution  
  Product management  
@ Place marketing  


Question 4    Multiple Choice  0 of 1 point(s)    
  Barry was having stomach trouble and was sent to have an ultrasound performed on his abdomen. The technician performing the ultrasound told Barry he had gallstones. Because Barry could not recognize gallstones even as the technician pointed them out, Barry was left to evaluate the ultrasound on the basis of _______________ qualities.  
 Your Answer:  [None Given]  
Correct Answer:  credence  
 


  Question 5    Multiple Choice  0 of 1 point(s)    
  Have you ever gotten a great haircut, gone back to the same person to get a second haircut, and received one of the worst haircuts of your life? The differences between the two haircuts are due to the _______________ characteristic of services.  
 Your Answer:  [None Given]  
Correct Answer:  variability  
 


  Question 6    Multiple Choice  0 of 1 point(s)    
  Another name for a store brand is a:  
 Your Answer:  [None Given]  
Correct Answer:  private-label brand  
 


  Question 7    Multiple Choice  0 of 1 point(s)    
  For many years the Grand Ole Opry was a fixture at the Ryman Auditorium in downtown Nashville, Tennessee. The auditorium had superior acoustics but a structure that was not readily updated to meet new safety requirements. The Ryman was also located in a part of Nashville that many did not consider safe. Thus, the Grand Ole Opry built a new facility for the show. The need to make the structure in which the Grand Ole Opry was held more attractive is an example of a(n) _______________ factor.  
 Your Answer:  [None Given]  
Correct Answer:  environmental  
 


  Question 8    Multiple Choice  0 of 1 point(s)    
  Which of the following is an example of a brand?  
 Your Answer:  [None Given]  
Correct Answer:  all of the above  
 


  Question 9    Multiple Choice  0 of 1 point(s)    
  Toronto-based Loblaw supermarket chain developed President's Choice brand products to sell exclusively in its stores. The products were so popular the supermarket chain has agreed to allow noncompetitive retailers in other countries to use the President's Choice brand in exchange for paying a fee to Loblaw. By selling its legally protected brand name to another retailer to use for a period of time, Loblaw has engaged in:  
 Your Answer:  [None Given]  
Correct Answer:  licensing  
 


  Question 10    Multiple Choice  0 of 1 point(s)    
  In the fall of 2002, Sheraton Hotels & Resorts launched a training program for its employees to convince them that management and employees working together can make each service encounter a competitive advantage. Sheraton is using:  
 Your Answer:  [None Given]  
Correct Answer:  internal marketing  
 
Question 4    Multiple Choice  0 of 1 point(s)    
  Kellogg's has marked Special K cereal for a number of years to a loyal customer base. Its introduction of a new cereal called Special K with red berries is an example of a _______________ strategy.  
 Your Answer:  [None Given]  
Correct Answer:  brand extension  
 


  Question 5    Multiple Choice  0 of 1 point(s)    
  The Federal Fair Packaging and Labeling Act, the Nutrition Labeling and Education Act, and the U.S. Food and Drug Administration work together to:  
 Your Answer:  [None Given]  
Correct Answer:  ensure consumer safety  
 


  Question 6    Multiple Choice  0 of 1 point(s)    
  Randy Stuart rents storage space to college students who want to go home for the summer, but who do not want to haul all of their property home and then back. The business is profitable during the summer months, but when the storage space is unoccupied, she is losing money. What characteristic of service is most likely the source of Stuart's problem?  
 Your Answer:  [None Given]  
Correct Answer:  perishability  
 


  Question 7    Multiple Choice  0 of 1 point(s)    
  The _______________ is the interaction between the customer and the service provider.  
 Your Answer:  [None Given]  
Correct Answer:  service encounter  
 


  Question 8    Multiple Choice  0 of 1 point(s)    
  Product packaging:  
 Your Answer:  [None Given]  
Correct Answer:  is accurately described by all of the above  
 


  Question 9    Multiple Choice  0 of 1 point(s)    
  According to your instructor, what is the big issue faced by not-for-profit marketers?  
 Your Answer:  [None Given]  
Correct Answer:  They have multiple sets of customers to deal with  
 


  Question 10    Multiple Choice  0 of 1 point(s)    
  _______________ is an agreement between two brands to work together in marketing a new product.  
 Your Answer:  [None Given]  
Correct Answer:  Co-branding  
 


Question 1    Multiple Choice  1 of 1 point(s)    
  One of the ways physicians deal with potential problems associated with the _______________ characteristic of services is to hang their medical diplomas on the walls of the examining rooms and to wear white coats.  
 Your Answer:  intangibility  
Correct Answer:  intangibility  
 


  Question 2    Multiple Choice  1 of 1 point(s)    
  The _______________ is the interaction between the customer and the service provider.  
 Your Answer:  service encounter  
Correct Answer:  service encounter  
 


  Question 3    Multiple Choice  0 of 1 point(s)    
  The act of including a service with the purchase of a good is called:  
 Your Answer:  [None Given]  
Correct Answer:  embodying  
 


  Question 4    Multiple Choice  0 of 1 point(s)    
  One of the advantages of brand equity is...  
 Your Answer:  [None Given]  
Correct Answer:  The ability to charge higher prices.  
 


  Question 5    Multiple Choice  0 of 1 point(s)    
  Disney allowed a paint manufacturer to produce paint using the Disney brand name for a specific period of time. Disney paint is an example of:  
 Your Answer:  [None Given]  
Correct Answer:  licensing  
 


  Question 6    Multiple Choice  0 of 1 point(s)    
  One of the ways Megan determined which day care center to send her children was by visiting neighboring ones and observing the indoor and outdoor play facilities, the cleanliness of the employees' uniforms, and how well infants were segregated from older children. In other words, Megan was examining _______________ of the day care service.  
 Your Answer:  [None Given]  
Correct Answer:  physical evidence  
 


  Question 7    Multiple Choice  0 of 1 point(s)    
  For many years the Grand Ole Opry was a fixture at the Ryman Auditorium in downtown Nashville, Tennessee. The auditorium had superior acoustics but a structure that was not readily updated to meet new safety requirements. The Ryman was also located in a part of Nashville that many did not consider safe. Thus, the Grand Ole Opry built a new facility for the show. The need to make the structure in which the Grand Ole Opry was held more attractive is an example of a(n) _______________ factor.  
 Your Answer:  [None Given]  
Correct Answer:  environmental  
 


  Question 8    Multiple Choice  0 of 1 point(s)    
  General Electric worked with Culligan to develop its Water by Culligan Profile Performance refrigerator with a built in Culligan water filtration system. This is an example of:  
 Your Answer:  [None Given]  
Correct Answer:  co-branding  
 


  Question 9    Multiple Choice  0 of 1 point(s)    
  As Martin checked into the motel, the clerk at the registration desk was rude and laughed at Martin's request for an early morning wake-up call. In terms of the service encounter dimensions, Martin experienced trouble with the _______________ dimension.  
 Your Answer:  [None Given]  
Correct Answer:  social contact  
 


  Question 10    Multiple Choice  0 of 1 point(s)    
  In 2001, the United Methodist Church in the United States budgeted $4 million for ads to encourage former church members to return to church through the use of gentle persuasion. This ad campaign is an example of:  
 Your Answer:  [None Given]  
Correct Answer:  idea marketing  
 
Question 1    Multiple Choice  0 of 1 point(s)    
  One of the ways Megan determined which day care center to send her children was by visiting neighboring ones and observing the indoor and outdoor play facilities, the cleanliness of the employees' uniforms, and how well infants were segregated from older children. She was evaluating the day care using _______________ qualities before she let the day care keep her children.  
 Your Answer:  experience  
Correct Answer:  search  
 


  Question 2    Multiple Choice  0 of 1 point(s)    
  Vlasic pickles use a stork with a blue billed hat and a red bow tie on its packaging and its marketing communications. The company uses the stork to:  
 Your Answer:  avoid using traditional spokespeople  
Correct Answer:  set itself apart from its competitors  
 


  Question 3    Multiple Choice  1 of 1 point(s)    
  _______________ are acts, efforts, or performances exchanged from producer to user without ownership rights.  
 Your Answer:  Services  
Correct Answer:  Services  
 


  Question 4    Multiple Choice  1 of 1 point(s)    
  The billboard announced that all the rooms in the Courtland Inn had been recently redecorated and that its room rate was still only $39 per night. This offer sounded like a good deal to Brenda, but when she went to her motel room, she found the blinds would not go down, the toilet seat was cracked, the rug was stained, and there was a new bedspread on the lumpy mattress. Brenda felt as if she had been deceived because in her mind the purchase of new bedspreads does not mean redecorated. A gap analysis would reveal a gap between:  
 Your Answer:  service quality standards and consumer expectations  
Correct Answer:  service quality standards and consumer expectations  
 


  Question 5    Multiple Choice  0 of 1 point(s)    
  Which of the following is a criticism of packaging discussed in class?  
 Your Answer:  Packaging encourages consumers to purchase products they don't need.  
Correct Answer:  Excessive packaging depletes natural resources.  
 


  Question 6    Multiple Choice  0 of 1 point(s)    
  Which of the following statements best represents your instructor's concept of the essence of branding?  
 Your Answer:  A brand is created through operant conditioning  
Correct Answer:  A brand is a promise you make to your customers  
 


  Question 7    Multiple Choice  0 of 1 point(s)    
  The act of including a service with the purchase of a good is called:  
 Your Answer:  supply management  
Correct Answer:  embodying  
 


  Question 8    Multiple Choice  1 of 1 point(s)    
  Alfredo would have used _______________ qualities to determine that his English tutor had successfully taught him how to conjugate a verb.  
 Your Answer:  experience  
Correct Answer:  experience  
 


  Question 9    Multiple Choice  1 of 1 point(s)    
  The bank management decided its customers would not mind if it only opened the drive-thru window on Saturdays and did not require other bank employees to work on weekends. Customers who work during the week could not apply for a loan on Saturday because that was not a service available at the drive-thru window. Gap analysis would show this is an example of a gap between:  
 Your Answer:  customer expectations and management perceptions  
Correct Answer:  customer expectations and management perceptions  
 


  Question 10    Multiple Choice  0 of 1 point(s)    
  Many visitors to Six Flags amusement park have complained about the long and slow-moving lines, and the fact that park employees are letting their friends break in line. This equipment-based service provider is having trouble with _______________ factors.  
 Your Answer:  cross-docking  
Correct Answer:  operational  
 


Question 1    Multiple Choice  1 of 1 point(s)    
  A _______________ is a name, a term, a symbol, or other unique element of a product that identifies one firm's products and sets it apart from the competition.  
 Your Answer:  brand  
Correct Answer:  brand  
 


  Question 2    Multiple Choice  0 of 1 point(s)    
  Generic brands:  
 Your Answer:  are more popular today than ever before  
Correct Answer:  were designed to appeal to the price conscious consumer  
 


  Question 3    Multiple Choice  0 of 1 point(s)    
  As Martin checked into the motel, the clerk at the registration desk was rude and laughed at Martin's request for an early morning wake-up call. In terms of the service encounter dimensions, Martin experienced trouble with the _______________ dimension.  
 Your Answer:  fifth  
Correct Answer:  social contact  
 


  Question 4    Multiple Choice  0 of 1 point(s)    
  Service _______________ means that customers cannot store services.  
 Your Answer:  intangibility  
Correct Answer:  perishability  
 


  Question 5    Multiple Choice  0 of 1 point(s)    
  Which of the following is a condition that is favorable to creating a brand?  
 Your Answer:  There is limited availability of the product.  
Correct Answer:  Quality and value are easy to maintain.  
 


  Question 6    Multiple Choice  0 of 1 point(s)    
  In _______________, branded materials become component parts of other branded products.  
 Your Answer:  umbrella branding  
Correct Answer:  ingredient branding  
 


  Question 7    Multiple Choice  0 of 1 point(s)    
  My bank has a little sticker in the window that says deposits are insured by the FDIC. The pank is trying to position its service using the ________ dimension.  
 Your Answer:  Relaibility  
Correct Answer:  Assurance  
 


  Question 8    Multiple Choice  1 of 1 point(s)    
  To reduce problems associated with the intangibility of the services offered by a restaurant, the restaurant owner can:  
 Your Answer:  provide clean tablecloths for each new customer  
Correct Answer:  provide clean tablecloths for each new customer  
 


  Question 9    Multiple Choice  0 of 1 point(s)    
  General Electric worked with Culligan to develop its Water by Culligan Profile Performance refrigerator with a built in Culligan water filtration system. This is an example of:  
 Your Answer:  contract manufacturing  
Correct Answer:  co-branding  
 


  Question 10    Multiple Choice  0 of 1 point(s)    
  One of the ways Megan determined which day care center to send her children was by visiting neighboring ones and observing the indoor and outdoor play facilities, the cleanliness of the employees' uniforms, and how well infants were segregated from older children. She was evaluating the day care using _______________ qualities before she let the day care keep her children.  
 Your Answer:  operational  
Correct Answer:  search  
 


Question 1    Multiple Choice  1 of 1 point(s)    
  A company that promises to respond to e-mail requests for information within 24 hours is positioning its service using the ________ dimension.  
 Your Answer:  Responsiveness  
Correct Answer:  Responsiveness  
 


  Question 2    Multiple Choice  0 of 1 point(s)    
  The National Rifle Association (NRA) promotes the safe use of firearms in its advertising as well as encouraging people to support the right of all Americans to bear arms. The NRA is engaged in:  
 Your Answer:  services wholesaling  
Correct Answer:  idea marketing  
 


  Question 3    Multiple Choice  0 of 1 point(s)    
  Which of the following statements about brand equity is true?  
 Your Answer:  Brand equity means a brand has customer loyalty.  
Correct Answer:  All of the above statements about brand equity are true.  
 


  Question 4    Multiple Choice  1 of 1 point(s)    
  Service _______________ means that customers cannot store services.  
 Your Answer:  perishability  
Correct Answer:  perishability  
 


  Question 5    Multiple Choice  1 of 1 point(s)    
  According to the goods/services continuum, which of the following is the best example of a pure good?  
 Your Answer:  a text book  
Correct Answer:  a text book  
 


  Question 6    Multiple Choice  0 of 1 point(s)    
  To minimize the potentially negative effects of bad service encounters and to save on labor costs, some service businesses are using _______________, which eliminates the need for customers to interact with people.  
 Your Answer:  service facilitation  
Correct Answer:  disintermediation  
 


  Question 7    Multiple Choice  0 of 1 point(s)    
  Dell Computer Corp. uses the same name that has been used by Dell comics for more than fifty years. This action:  
 Your Answer:  is prohibited in international markets  
Correct Answer:  is completely legal because the products do not compete  
 


  Question 8    Multiple Choice  1 of 1 point(s)    
  A _______________ is a name, a term, a symbol, or other unique element of a product that identifies one firm's products and sets it apart from the competition.  
 Your Answer:  brand  
Correct Answer:  brand  
 


  Question 9    Multiple Choice  0 of 1 point(s)    
  In the fall of 2002, Sheraton Hotels & Resorts launched a new service guarantee that includes a training program for its employees. This training program will lessen the problems related to the _______________ characteristic of services.  
 Your Answer:  intangibility  
Correct Answer:  inseparability  
 


  Question 10    Multiple Choice  0 of 1 point(s)    
  Pinnacle Foods Corp. markets Vlasic pickles. According to brand designers, Vlasic would:  
 Your Answer:  not be a good brand name because it is multisyllabic  
Correct Answer:  not be a good brand name for a pickle because it is hard to spell, to say, to read, and to remember  
 


Question 1    Multiple Choice  1 of 1 point(s)    
  The three strategies marketers use to "sell" people are:  
 Your Answer:  pure selling, product improvement, and market fulfillment  
Correct Answer:  pure selling, product improvement, and market fulfillment  
 


  Question 2    Multiple Choice  1 of 1 point(s)    
  The act of including a service with the purchase of a good is called:  
 Your Answer:  embodying  
Correct Answer:  embodying  
 


  Question 3    Multiple Choice  0 of 1 point(s)    
  Kellogg's has marked Special K cereal for a number of years to a loyal customer base. Its introduction of a new cereal called Special K with red berries is an example of a _______________ strategy.  
 Your Answer:  divestment  
Correct Answer:  brand extension  
 


  Question 4    Multiple Choice  0 of 1 point(s)    
  In order to make an evaluative judgment of the quality of a service, customers can use search quality, experience quality, and _______________ quality.  
 Your Answer:  multitasking  
Correct Answer:  credence  
 


  Question 5    Multiple Choice  1 of 1 point(s)    
  The J. M. Smuckers Company makes jams, jellies, fruit spreads, and ice cream toppings. All of these products are sold using the Smucker's brand name. This company uses a(n) _______________ strategy.  
 Your Answer:  family brand  
Correct Answer:  family brand  
 


  Question 6    Multiple Choice  1 of 1 point(s)    
  General Electric worked with Culligan to develop its Water by Culligan Profile Performance refrigerator with a built in Culligan water filtration system. This is an example of:  
 Your Answer:  co-branding  
Correct Answer:  co-branding  
 


  Question 7    Multiple Choice  1 of 1 point(s)    
  Hershey Foods and General Mills worked together to create Reese's Peanut Butter Puffs cereal. This is an example of:  
 Your Answer:  ingredient branding  
Correct Answer:  ingredient branding  
 


  Question 8    Multiple Choice  0 of 1 point(s)    
  When Ria hired an interior designer to redecorate her living room, she was not sure she had made the right decision until she saw how gorgeous her newly decorated living room looked. Ria used _______________ qualities to evaluate the designer's services.  
 Your Answer:  operational  
Correct Answer:  experience  
 


  Question 9    Multiple Choice  0 of 1 point(s)    
  The Christian music industry is the only facet of the music industry that has shown significant growth during the last decade. A recording company executive believes the American people are ready for gospel singers to enter the mainstream market and record some pop music. The executive has found a group of five who have a small following in the Christian music market and has the group recording new songs that are uplifting without being preachy. He hopes that a nationwide tour will expand the market for other Christian music performers. Which of the following strategies is the executive using to market this new group?  
 Your Answer:  pure selling  
Correct Answer:  market fulfillment  
 


  Question 10    Multiple Choice  1 of 1 point(s)    
  According to your instructor, what is the big issue faced by not-for-profit marketers?  
 Your Answer:  They have multiple sets of customers to deal with  
Correct Answer:  They have multiple sets of customers to deal with  
 


Question 1    Multiple Choice  1 of 1 point(s)    
  Product packaging:  
 Your Answer:  is accurately described by all of the above  
Correct Answer:  is accurately described by all of the above  
 


  Question 2    Multiple Choice  0 of 1 point(s)    
  A _______________ is probably the most used and most recognized form of branding.  
 Your Answer:  copyright  
Correct Answer:  brand name  
 


  Question 3    Multiple Choice  1 of 1 point(s)    
  A _______________ is a name, a term, a symbol, or other unique element of a product that identifies one firm's products and sets it apart from the competition.  
 Your Answer:  brand  
Correct Answer:  brand  
 


  Question 4    Multiple Choice  0 of 1 point(s)    
  Generic brands:  
 Your Answer:  are accurately described by all of the above  
Correct Answer:  were designed to appeal to the price conscious consumer  
 


  Question 5    Multiple Choice  0 of 1 point(s)    
  Which method of measuring service quality concentrates on customer complaints?  
 Your Answer:  the breakdown method  
Correct Answer:  the critical incident technique  
 


  Question 6    Multiple Choice  1 of 1 point(s)    
  My bank has a little sticker in the window that says deposits are insured by the FDIC. The pank is trying to position its service using the ________ dimension.  
 Your Answer:  Assurance  
Correct Answer:  Assurance  
 


  Question 7    Multiple Choice  1 of 1 point(s)    
  The J. M. Smuckers Company makes jams, jellies, fruit spreads, and ice cream toppings. All of these products are sold using the Smucker's brand name. This company uses a(n) _______________ strategy.  
 Your Answer:  family brand  
Correct Answer:  family brand  
 


  Question 8    Multiple Choice  1 of 1 point(s)    
  Hershey Foods and General Mills worked together to create Reese's Peanut Butter Puffs cereal. This is an example of:  
 Your Answer:  ingredient branding  
Correct Answer:  ingredient branding  
 


  Question 9    Multiple Choice  1 of 1 point(s)    
  To minimize the potentially negative effects of bad service encounters and to save on labor costs, some service businesses are using _______________, which eliminates the need for customers to interact with people.  
 Your Answer:  disintermediation  
Correct Answer:  disintermediation  
 


  Question 10    Multiple Choice  0 of 1 point(s)    
  Which of the following product offerings would be most likely described as intangible?  
 Your Answer:  a mink coat  
Correct Answer:  a physical examination by a doctor  
 


Question 1    Multiple Choice  1 of 1 point(s)    
  Dell Computer Corp. uses the same name that has been used by Dell comics for more than fifty years. This action:  
 Your Answer:  is completely legal because the products do not compete  
Correct Answer:  is completely legal because the products do not compete  
 


  Question 2    Multiple Choice  0 of 1 point(s)    
  In 2001, the United Methodist Church in the United States budgeted $4 million for ads to encourage former church members to return to church through the use of gentle persuasion. This ad campaign is an example of:  
 Your Answer:  services wholesaling  
Correct Answer:  idea marketing  
 


  Question 3    Multiple Choice  1 of 1 point(s)    
  Pinnacle Foods Corp. markets Vlasic pickles. According to brand designers, Vlasic would:  
 Your Answer:  not be a good brand name for a pickle because it is hard to spell, to say, to read, and to remember  
Correct Answer:  not be a good brand name for a pickle because it is hard to spell, to say, to read, and to remember  
 


  Question 4    Multiple Choice  1 of 1 point(s)    
  One of the ways Megan determined which day care center to send her children was by visiting neighboring ones and observing the indoor and outdoor play facilities, the cleanliness of the employees' uniforms, and how well infants were segregated from older children. She was evaluating the day care using _______________ qualities before she let the day care keep her children.  
 Your Answer:  search  
Correct Answer:  search  
 


  Question 5    Multiple Choice  0 of 1 point(s)    
  Dan's annual medical examination gives him peace of mind and removes all anxieties about undiscovered health problems. These feelings are the benefits of the doctor's examination and are called:  
 Your Answer:  differentiated advantages  
Correct Answer:  core services  
 


  Question 6    Multiple Choice  1 of 1 point(s)    
  In order to make an evaluative judgment of the quality of a service, customers can use search quality, experience quality, and _______________ quality.  
 Your Answer:  credence  
Correct Answer:  credence  
 


  Question 7    Multiple Choice  1 of 1 point(s)    
  Service _______________ means that customers cannot store services.  
 Your Answer:  perishability  
Correct Answer:  perishability  
 


  Question 8    Multiple Choice  0 of 1 point(s)    
  Another name for a store brand is a:  
 Your Answer:  licensed brand  
Correct Answer:  private-label brand  
 


  Question 9    Multiple Choice  1 of 1 point(s)    
  Generic brands:  
 Your Answer:  were designed to appeal to the price conscious consumer  
Correct Answer:  were designed to appeal to the price conscious consumer  
 


  Question 10    Multiple Choice  1 of 1 point(s)    
  The act of including a service with the purchase of a good is called:  
 Your Answer:  embodying  
Correct Answer:  embodying  
 


Question 1    Multiple Choice  1 of 1 point(s)    
  Which of the following is an example of an equipment-based service?  
 Your Answer:  an amusement park  
Correct Answer:  an amusement park  
 


  Question 2    Multiple Choice  1 of 1 point(s)    
  Well-organized desks, professionally attired employees, and travel posters on the wall are all examples of _______________ that can be used to position a travel agency.  
 Your Answer:  physical evidence  
Correct Answer:  physical evidence  
 


  Question 3    Multiple Choice  1 of 1 point(s)    
  Service _______________ means that customers cannot store services.  
 Your Answer:  perishability  
Correct Answer:  perishability  
 


  Question 4    Multiple Choice  0 of 1 point(s)    
  Which of the following is a component of brand equity according to David Aaker?  
 Your Answer:  Co-branding  
Correct Answer:  Perceived quality.  
 


  Question 5    Multiple Choice  1 of 1 point(s)    
  A _______________ is a name, a term, a symbol, or other unique element of a product that identifies one firm's products and sets it apart from the competition.  
 Your Answer:  brand  
Correct Answer:  brand  
 


  Question 6    Multiple Choice  1 of 1 point(s)    
  Vlasic pickles use a stork with a blue billed hat and a red bow tie on its packaging and its marketing communications. The company uses the stork to:  
 Your Answer:  set itself apart from its competitors  
Correct Answer:  set itself apart from its competitors  
 


  Question 7    Multiple Choice  0 of 1 point(s)    
  According to the text, telemarketers who call to encourage you to vote for a particular candidate are engaged in:  
 Your Answer:  wholesaling  
Correct Answer:  intangibles marketing  
 


  Question 8    Multiple Choice  1 of 1 point(s)    
  Which of the following product offerings would be most likely described as intangible?  
 Your Answer:  a physical examination by a doctor  
Correct Answer:  a physical examination by a doctor  
 


  Question 9    Multiple Choice  1 of 1 point(s)    
  Every time Bonnie gets her weekly massage, she leaves the spa feeling completely stress-free. This feeling is the benefit the massage provides and is called a(n):  
 Your Answer:  core service  
Correct Answer:  core service  
 


  Question 10    Multiple Choice  1 of 1 point(s)    
  According to the goods/services continuum, which of the following is the best example of a pure good?  
 Your Answer:  a text book  
Correct Answer:  a text book  
 


 Question 1    Multiple Choice  0 of 1 point(s)    
  _______________ is a measurement tool that gauges the difference between a customer's expectations of service quality and what actually occurred.  
 Your Answer:  Break-even analysis  
Correct Answer:  Gap analysis  
 


  Question 2    Multiple Choice  1 of 1 point(s)    
  To minimize the potentially negative effects of bad service encounters and to save on labor costs, some service businesses are using _______________, which eliminates the need for customers to interact with people.  
 Your Answer:  disintermediation  
Correct Answer:  disintermediation  
 


  Question 3    Multiple Choice  0 of 1 point(s)    
  What is the most common explanation for why retailers carry private-label brands?  
 Your Answer:  to prevent sales cannibalization  
Correct Answer:  to earn more profit  
 


  Question 4    Multiple Choice  1 of 1 point(s)    
  Service _______________ means that customers cannot store services.  
 Your Answer:  perishability  
Correct Answer:  perishability  
 


  Question 5    Multiple Choice  1 of 1 point(s)    
  Every time Bonnie gets her weekly massage, she leaves the spa feeling completely stress-free. This feeling is the benefit the massage provides and is called a(n):  
 Your Answer:  core service  
Correct Answer:  core service  
 


  Question 6    Multiple Choice  1 of 1 point(s)    
  My bank has a little sticker in the window that says deposits are insured by the FDIC. The pank is trying to position its service using the ________ dimension.  
 Your Answer:  Assurance  
Correct Answer:  Assurance  
 


  Question 7    Multiple Choice  1 of 1 point(s)    
  The Christian music industry is the only facet of the music industry that has shown significant growth during the last decade. A recording company executive believes the American people are ready for gospel singers to enter the mainstream market and record some pop music. The executive has found a group of five who have a small following in the Christian music market and has the group recording new songs that are uplifting without being preachy. He hopes that a nationwide tour will expand the market for other Christian music performers. Which of the following strategies is the executive using to market this new group?  
 Your Answer:  market fulfillment  
Correct Answer:  market fulfillment  
 


  Question 8    Multiple Choice  0 of 1 point(s)    
  What do we mean when we speak of negative brand equity?  
 Your Answer:  Small decreases in price lead to large increases in brand awareness.  
Correct Answer:  Consumers have unfavorable associations with the brand name.  
 


  Question 9    Multiple Choice  1 of 1 point(s)    
  Dell Computer Corp. uses the same name that has been used by Dell comics for more than fifty years. This action:  
 Your Answer:  is completely legal because the products do not compete  
Correct Answer:  is completely legal because the products do not compete  
 


  Question 10    Multiple Choice  1 of 1 point(s)    
  In order to make an evaluative judgment of the quality of a service, customers can use search quality, experience quality, and _______________ quality.  
 Your Answer:  credence  
Correct Answer:  credence  
 


Question 1    Multiple Choice  0 of 1 point(s)    
  Which of the following levels of familiarity would indicate the largest brand equity?  
 Your Answer:  Family brand recognition  
Correct Answer:  Brand preference  
 


  Question 2    Multiple Choice  0 of 1 point(s)    
  Search qualities would be most useful to a consumer buying:  
 Your Answer:  the expertise of a marketing consultant  
Correct Answer:  a new refrigerator  
 


  Question 3    Multiple Choice  0 of 1 point(s)    
  Capacity management is one of the ways to meet potential problems related to the _______________ characteristic of services.  
 Your Answer:  intangibility  
Correct Answer:  perishability  
 


  Question 4    Multiple Choice  0 of 1 point(s)    
  A _______________ is the legal term for a brand name, brand mark, or trade character.  
 Your Answer:  [None Given]  
Correct Answer:  trademark  
 


  Question 5    Multiple Choice  0 of 1 point(s)    
  Which of the service positioning dimensions suggests that it is important to have consistency in you service delivery?  
 Your Answer:  Tangibles  
Correct Answer:  Reliability  
 


  Question 6    Multiple Choice  1 of 1 point(s)    
  _______________ is an agreement between two brands to work together in marketing a new product.  
 Your Answer:  Co-branding  
Correct Answer:  Co-branding  
 


  Question 7    Multiple Choice  1 of 1 point(s)    
  _______________ are acts, efforts, or performances exchanged from producer to user without ownership rights.  
 Your Answer:  Services  
Correct Answer:  Services  
 


  Question 8    Multiple Choice  1 of 1 point(s)    
  Which of the following product offerings would be most likely described as intangible?  
 Your Answer:  a physical examination by a doctor  
Correct Answer:  a physical examination by a doctor  
 


  Question 9    Multiple Choice  1 of 1 point(s)    
  In many communities, you can find as many as thirty different fast-food restaurant franchises within a couple of miles of each other. The nearness of these restaurants to each other illustrates the importance of the _______________ factor.  
 Your Answer:  locational  
Correct Answer:  locational  
 


  Question 10    Multiple Choice  1 of 1 point(s)    
  As Martin checked into the motel, the clerk at the registration desk was rude and laughed at Martin's request for an early morning wake-up call. In terms of the service encounter dimensions, Martin experienced trouble with the _______________ dimension.  
 Your Answer:  social contact  
Correct Answer:  social contact  
 


 Question 1    Multiple Choice  0 of 1 point(s)    
  According to the text, which of the following is an example of an intangible service?  
 Your Answer:  all of the above  
Correct Answer:  a religious service  
 


  Question 2    Multiple Choice  0 of 1 point(s)    
  General Electric worked with Culligan to develop its Water by Culligan Profile Performance refrigerator with a built in Culligan water filtration system. This is an example of:  
 Your Answer:  generic branding  
Correct Answer:  co-branding  
 


  Question 3    Multiple Choice  0 of 1 point(s)    
  One of the advantages of brand equity is...  
 Your Answer:  None of the above are advantages of brand equity.  
Correct Answer:  The ability to charge higher prices.  
 


  Question 4    Multiple Choice  1 of 1 point(s)    
  In the fall of 2002, Sheraton Hotels & Resorts launched a new service guarantee that includes a training program for its employees. This training program will lessen the problems related to the _______________ characteristic of services.  
 Your Answer:  inseparability  
Correct Answer:  inseparability  
 


  Question 5    Multiple Choice  1 of 1 point(s)    
  As Martin checked into the motel, the clerk at the registration desk was rude and laughed at Martin's request for an early morning wake-up call. In terms of the service encounter dimensions, Martin experienced trouble with the _______________ dimension.  
 Your Answer:  social contact  
Correct Answer:  social contact  
 


  Question 6    Multiple Choice  1 of 1 point(s)    
  The billboard announced that all the rooms in the Courtland Inn had been recently redecorated and that its room rate was still only $39 per night. This offer sounded like a good deal to Brenda, but when she went to her motel room, she found the blinds would not go down, the toilet seat was cracked, the rug was stained, and there was a new bedspread on the lumpy mattress. Brenda felt as if she had been deceived because in her mind the purchase of new bedspreads does not mean redecorated. A gap analysis would reveal a gap between:  
 Your Answer:  service quality standards and consumer expectations  
Correct Answer:  service quality standards and consumer expectations  
 


  Question 7    Multiple Choice  1 of 1 point(s)    
  To minimize the potentially negative effects of bad service encounters and to save on labor costs, some service businesses are using _______________, which eliminates the need for customers to interact with people.  
 Your Answer:  disintermediation  
Correct Answer:  disintermediation  
 


  Question 8    Multiple Choice  0 of 1 point(s)    
  Which of the following is a criticism of packaging discussed in class?  
 Your Answer:  Packaging encourages consumers to purchase products they don't need.  
Correct Answer:  Excessive packaging depletes natural resources.  
 


  Question 9    Multiple Choice  1 of 1 point(s)    
  Service _______________ means that customers cannot store services.  
 Your Answer:  perishability  
Correct Answer:  perishability  
 


  Question 10    Multiple Choice  0 of 1 point(s)    
  Which of the following services will most likely be evaluated using credence qualities?  
 Your Answer:  a stay in a hotel resort  
Correct Answer:  surgery to repair a leaking heart valve  
 


Question 1    Multiple Choice  1 of 1 point(s)    
  Which of the following would be useful if a service provider wanted to reduce the problems associated with the intangibility characteristic of services?  
 Your Answer:  an eye-catching logo  
Correct Answer:  an eye-catching logo  
 


  Question 2    Multiple Choice  0 of 1 point(s)    
  Service _______________ means that a service can only take place at the time the service provider performs it on either the customer or the customer's possessions.  
 Your Answer:  reliability  
Correct Answer:  inseparability  
 


  Question 3    Multiple Choice  0 of 1 point(s)    
  Tide detergent, Smucker's toppings, Purina Dog Chow dog food, and Starkist tuna are all examples of:  
 Your Answer:  licensed brands  
Correct Answer:  national or manufacturer brands  
 


  Question 4    Multiple Choice  1 of 1 point(s)    
  In 2001, the United Methodist Church in the United States budgeted $4 million for ads to encourage former church members to return to church through the use of gentle persuasion. This ad campaign is an example of:  
 Your Answer:  idea marketing  
Correct Answer:  idea marketing  
 


  Question 5    Multiple Choice  0 of 1 point(s)    
  The Federal Fair Packaging and Labeling Act, the Nutrition Labeling and Education Act, and the U.S. Food and Drug Administration work together to:  
 Your Answer:  generate new product ideas  
Correct Answer:  ensure consumer safety  
 


  Question 6    Multiple Choice  1 of 1 point(s)    
  Hershey Foods and General Mills worked together to create Reese's Peanut Butter Puffs cereal. This is an example of:  
 Your Answer:  ingredient branding  
Correct Answer:  ingredient branding  
 


  Question 7    Multiple Choice  1 of 1 point(s)    
  Disney allowed a paint manufacturer to produce paint using the Disney brand name for a specific period of time. Disney paint is an example of:  
 Your Answer:  licensing  
Correct Answer:  licensing  
 


  Question 8    Multiple Choice  0 of 1 point(s)    
  Toronto-based Loblaw supermarket chain developed President's Choice brand products to sell exclusively in its stores. The products were so popular the supermarket chain has agreed to allow noncompetitive retailers in other countries to use the President's Choice brand in exchange for paying a fee to Loblaw. By selling its legally protected brand name to another retailer to use for a period of time, Loblaw has engaged in:  
 Your Answer:  piggybacing  
Correct Answer:  licensing  
 


  Question 9    Multiple Choice  0 of 1 point(s)    
  _______________ strategies regard a city, state, country, or other locale as a brand and attempt to position the location so that consumers choose to visit.  
 Your Answer:  Services wholesaling  
Correct Answer:  Place marketing  
 


  Question 10    Multiple Choice  1 of 1 point(s)    
  Dell Computer Corp. uses the same name that has been used by Dell comics for more than fifty years. This action:  
 Your Answer:  is completely legal because the products do not compete  
Correct Answer:  is completely legal because the products do not compete  
 


Question 1    Multiple Choice  0 of 1 point(s)    
  Product packaging:  
 Your Answer:  can create a competitive advantage  
Correct Answer:  is accurately described by all of the above  
 


  Question 2    Multiple Choice  0 of 1 point(s)    
  The red devil figure that appears on all Underwood meat spreads is an example of a:  
 Your Answer:  a nonregistered trademark  
Correct Answer:  brand  
 


  Question 3    Multiple Choice  0 of 1 point(s)    
  Randy Stuart rents storage space to college students who want to go home for the summer, but who do not want to haul all of their property home and then back. The business is profitable during the summer months, but when the storage space is unoccupied, she is losing money. What characteristic of service is most likely the source of Stuart's problem?  
 Your Answer:  intangibility  
Correct Answer:  perishability  
 


  Question 4    Multiple Choice  0 of 1 point(s)    
  Which of the service positioning dimensions suggests that it is important to have consistency in you service delivery?  
 Your Answer:  Physical evidence  
Correct Answer:  Reliability  
 


  Question 5    Multiple Choice  0 of 1 point(s)    
  _______________ is the process by which organizations adjust the supply of their services in an attempt to match demand.  
 Your Answer:  Disintermediation  
Correct Answer:  Capacity management  
 


  Question 6    Multiple Choice  1 of 1 point(s)    
  Every time Bonnie gets her weekly massage, she leaves the spa feeling completely stress-free. This feeling is the benefit the massage provides and is called a(n):  
 Your Answer:  core service  
Correct Answer:  core service  
 


  Question 7    Multiple Choice  1 of 1 point(s)    
  One of the ways physicians deal with potential problems associated with the _______________ characteristic of services is to hang their medical diplomas on the walls of the examining rooms and to wear white coats.  
 Your Answer:  intangibility  
Correct Answer:  intangibility  
 


  Question 8    Multiple Choice  1 of 1 point(s)    
  The three strategies marketers use to "sell" people are:  
 Your Answer:  pure selling, product improvement, and market fulfillment  
Correct Answer:  pure selling, product improvement, and market fulfillment  
 


  Question 9    Multiple Choice  0 of 1 point(s)    
  Employee empowerment is one of the ways to meet potential problems related to the _______________ characteristic of services.  
 Your Answer:  intangibility  
Correct Answer:  variability  
 


  Question 10    Multiple Choice  0 of 1 point(s)    
  Political candidates often hire image consultants who tell them what clothes to wear, how to wear their hair, and how to act in front of a crowd of people. Political candidates who use image consultants to help them market themselves to the voters are using which of the following strategies?  
 Your Answer:  brand extension  
Correct Answer:  product improvement  
 


Question 1    Multiple Choice  0 of 1 point(s)    
  Which of the following is a component of brand equity according to David Aaker?  
 Your Answer:  Co-branding  
Correct Answer:  Perceived quality.  
 


  Question 2    Multiple Choice  0 of 1 point(s)    
  Naomi is afraid to go to a dentist, but she has a really severe toothache. Her sister has recommended she go to Dr. Tompkins because he guarantees pain-free dentistry. Assuming Naomi visits Dr. Tompkins, the only way she can evaluate his dental techniques is through _______________ qualities.  
 Your Answer:  credence  
Correct Answer:  experience  
 


  Question 3    Multiple Choice  1 of 1 point(s)    
  The act of including a service with the purchase of a good is called:  
 Your Answer:  embodying  
Correct Answer:  embodying  
 


  Question 4    Multiple Choice  1 of 1 point(s)    
  Which of the following is the best example of a people-based service?  
 Your Answer:  a dating service  
Correct Answer:  a dating service  
 


  Question 5    Multiple Choice  0 of 1 point(s)    
  Have you ever gotten a great haircut, gone back to the same person to get a second haircut, and received one of the worst haircuts of your life? The differences between the two haircuts are due to the _______________ characteristic of services.  
 Your Answer:  empathy  
Correct Answer:  variability  
 


  Question 6    Multiple Choice  0 of 1 point(s)    
  Which of the following products would be a desirable brand name because of how it fits the product benefits?  
 Your Answer:  Tombstone pizza  
Correct Answer:  StrideGuard insoles for shoes  
 


  Question 7    Multiple Choice  1 of 1 point(s)    
  Another name for a store brand is a:  
 Your Answer:  private-label brand  
Correct Answer:  private-label brand  
 


  Question 8    Multiple Choice  1 of 1 point(s)    
  According to the goods/services continuum, which of the following is the best example of a pure good?  
 Your Answer:  a text book  
Correct Answer:  a text book  
 


  Question 9    Multiple Choice  1 of 1 point(s)    
  Service _______________ means that customers cannot store services.  
 Your Answer:  perishability  
Correct Answer:  perishability  
 


  Question 10    Multiple Choice  0 of 1 point(s)    
  One of the ways Megan determined which day care center to send her children was by visiting neighboring ones and observing the indoor and outdoor play facilities, the cleanliness of the employees' uniforms, and how well infants were segregated from older children. She was evaluating the day care using _______________ qualities before she let the day care keep her children.  
 Your Answer:  operational  
Correct Answer:  search  
 


Question 1    Multiple Choice  1 of 1 point(s)    
  Which of the following would be useful if a service provider wanted to reduce the problems associated with the intangibility characteristic of services?  
 Your Answer:  an eye-catching logo  
Correct Answer:  an eye-catching logo  
 


  Question 2    Multiple Choice  0 of 1 point(s)    
  An advertisement for the Blue Ridge Parkway (a highway) that runs from the Shenandoah Valley to the Smokey Mountains describes the highway as a "trip back in time." This is an example of:  
 Your Answer:  services distribution  
Correct Answer:  place marketing  
 


  Question 3    Multiple Choice  1 of 1 point(s)    
  Service _______________ means that a firm cannot store its services.  
 Your Answer:  perishability  
Correct Answer:  perishability  
 


  Question 4    Multiple Choice  0 of 1 point(s)    
  Keisha particularly enjoys taking her dog to Pampered Paws for its monthly bath and trim because the people who work on the dogs use a pleasant smelling soap, have full-sized kennels in which to dry the dog, and give the dog a treat when its treatment is finished. These additional services provided by Pampered Paws enhance the value of having a clean dog. Pampered Paws use _______________ to differentiate itself from its competitors.  
 Your Answer:  differentiated advantages  
Correct Answer:  augmented services  
 


  Question 5    Multiple Choice  1 of 1 point(s)    
  _______________ is a measurement tool that gauges the difference between a customer's expectations of service quality and what actually occurred.  
 Your Answer:  Gap analysis  
Correct Answer:  Gap analysis  
 


  Question 6    Multiple Choice  1 of 1 point(s)    
  General Electric worked with Culligan to develop its Water by Culligan Profile Performance refrigerator with a built in Culligan water filtration system. This is an example of:  
 Your Answer:  co-branding  
Correct Answer:  co-branding  
 


  Question 7    Multiple Choice  0 of 1 point(s)    
  For many years the Grand Ole Opry was a fixture at the Ryman Auditorium in downtown Nashville, Tennessee. The auditorium had superior acoustics but a structure that was not readily updated to meet new safety requirements. The Ryman was also located in a part of Nashville that many did not consider safe. Thus, the Grand Ole Opry built a new facility for the show. The need to make the structure in which the Grand Ole Opry was held more attractive is an example of a(n) _______________ factor.  
 Your Answer:  operational  
Correct Answer:  environmental  
 


  Question 8    Multiple Choice  1 of 1 point(s)    
  Dan's annual medical examination gives him peace of mind and removes all anxieties about undiscovered health problems. These feelings are the benefits of the doctor's examination and are called:  
 Your Answer:  core services  
Correct Answer:  core services  
 


  Question 9    Multiple Choice  0 of 1 point(s)    
  Management of a chain of fast-food restaurants have determined one of its corporate goals is to have no customer wait longer than five minutes to receive his or her food once the order is placed. A manager of a local restaurant from this chain has interpreted the goal to apply only to the drive-thru window. This restaurant has earned the award for the fastest drive-thru window in the chain, but customers inside often are kept waiting for their orders ten or more minutes. A gap analysis would reveal a gap between:  
 Your Answer:  expected service and perceived service  
Correct Answer:  management perception and quality standards set by the firm  
 


  Question 10    Multiple Choice  1 of 1 point(s)    
  In order to make an evaluative judgment of the quality of a service, customers can use search quality, experience quality, and _______________ quality.  
 Your Answer:  credence  
Correct Answer:  credence  
 


 Question 1    Multiple Choice  1 of 1 point(s)    
  The three strategies marketers use to "sell" people are:  
 Your Answer:  pure selling, product improvement, and market fulfillment  
Correct Answer:  pure selling, product improvement, and market fulfillment  
 


  Question 2    Multiple Choice  0 of 1 point(s)    
  Employee empowerment is one of the ways to meet potential problems related to the _______________ characteristic of services.  
 Your Answer:  inseparability  
Correct Answer:  variability  
 


  Question 3    Multiple Choice  1 of 1 point(s)    
  Which of the following is an example of an equipment-based service?  
 Your Answer:  an amusement park  
Correct Answer:  an amusement park  
 


  Question 4    Multiple Choice  1 of 1 point(s)    
  My bank has a little sticker in the window that says deposits are insured by the FDIC. The pank is trying to position its service using the ________ dimension.  
 Your Answer:  Assurance  
Correct Answer:  Assurance  
 


  Question 5    Multiple Choice  1 of 1 point(s)    
  Service _______________ means that a firm cannot store its services.  
 Your Answer:  perishability  
Correct Answer:  perishability  
 


  Question 6    Multiple Choice  1 of 1 point(s)    
  An advertisement for the Blue Ridge Parkway (a highway) that runs from the Shenandoah Valley to the Smokey Mountains describes the highway as a "trip back in time." This is an example of:  
 Your Answer:  place marketing  
Correct Answer:  place marketing  
 


  Question 7    Multiple Choice  0 of 1 point(s)    
  Barry was having stomach trouble and was sent to have an ultrasound performed on his abdomen. The technician performing the ultrasound told Barry he had gallstones. Because Barry could not recognize gallstones even as the technician pointed them out, Barry was left to evaluate the ultrasound on the basis of _______________ qualities.  
 Your Answer:  experience  
Correct Answer:  credence  
 


  Question 8    Multiple Choice  1 of 1 point(s)    
  Which of the following is a component of brand equity according to David Aaker?  
 Your Answer:  Perceived quality.  
Correct Answer:  Perceived quality.  
 


  Question 9    Multiple Choice  1 of 1 point(s)    
  Hershey Foods and General Mills worked together to create Reese's Peanut Butter Puffs cereal. This is an example of:  
 Your Answer:  ingredient branding  
Correct Answer:  ingredient branding  
 


  Question 10    Multiple Choice  1 of 1 point(s)    
  Toronto-based Loblaw supermarket chain developed President's Choice brand products to sell exclusively in its stores. President's Choice is an example of a:  
 Your Answer:  private-label brand  
Correct Answer:  private-label brand  
 


 Question 1    Multiple Choice  0 of 1 point(s)    
  Search qualities would be most useful to a consumer buying:  
 Your Answer:  the services of an advertising agency  
Correct Answer:  a new refrigerator  
 


  Question 2    Multiple Choice  1 of 1 point(s)    
  Tide detergent, Smucker's toppings, Purina Dog Chow dog food, and Starkist tuna are all examples of:  
 Your Answer:  national or manufacturer brands  
Correct Answer:  national or manufacturer brands  
 


  Question 3    Multiple Choice  0 of 1 point(s)    
  Every time Bonnie gets her weekly massage, she enjoys the aromatherapy candles her masseur lights and the New Age music that is playing in the background. This is the only spa in her area that offers candles and music to create an atmosphere in which she can more easily relax to enjoy her massage. The candles and soft music are additional services provided to enhance the value of her massage. They are examples of:  
 Your Answer:  core services  
Correct Answer:  augmented services  
 


  Question 4    Multiple Choice  1 of 1 point(s)    
  Dan's annual medical examination gives him peace of mind and removes all anxieties about undiscovered health problems. These feelings are the benefits of the doctor's examination and are called:  
 Your Answer:  core services  
Correct Answer:  core services  
 


  Question 5    Multiple Choice  1 of 1 point(s)    
  A _______________ is the legal term for a brand name, brand mark, or trade character.  
 Your Answer:  trademark  
Correct Answer:  trademark  
 


  Question 6    Multiple Choice  1 of 1 point(s)    
  To minimize the potentially negative effects of bad service encounters and to save on labor costs, some service businesses are using _______________, which eliminates the need for customers to interact with people.  
 Your Answer:  disintermediation  
Correct Answer:  disintermediation  
 


  Question 7    Multiple Choice  1 of 1 point(s)    
  In many communities, you can find as many as thirty different fast-food restaurant franchises within a couple of miles of each other. The nearness of these restaurants to each other illustrates the importance of the _______________ factor.  
 Your Answer:  locational  
Correct Answer:  locational  
 


  Question 8    Multiple Choice  1 of 1 point(s)    
  Product packaging:  
 Your Answer:  is accurately described by all of the above  
Correct Answer:  is accurately described by all of the above  
 


  Question 9    Multiple Choice  1 of 1 point(s)    
  Generic brands:  
 Your Answer:  were designed to appeal to the price conscious consumer  
Correct Answer:  were designed to appeal to the price conscious consumer  
 


  Question 10    Multiple Choice  0 of 1 point(s)    
  Which of the following levels of familiarity would indicate the largest brand equity?  
 Your Answer:  Family brand recognition  
Correct Answer:  Brand preference  
 


Question 1    Multiple Choice  1 of 1 point(s)    
  In many communities, you can find as many as thirty different fast-food restaurant franchises within a couple of miles of each other. The nearness of these restaurants to each other illustrates the importance of the _______________ factor.  
 Your Answer:  locational  
Correct Answer:  locational  
 


  Question 2    Multiple Choice  1 of 1 point(s)    
  Employee empowerment is one of the ways to meet potential problems related to the _______________ characteristic of services.  
 Your Answer:  variability  
Correct Answer:  variability  
 


  Question 3    Multiple Choice  1 of 1 point(s)    
  A company that promises to respond to e-mail requests for information within 24 hours is positioning its service using the ________ dimension.  
 Your Answer:  Responsiveness  
Correct Answer:  Responsiveness  
 


  Question 4    Multiple Choice  0 of 1 point(s)    
  Many visitors to Six Flags amusement park have complained about the long and slow-moving lines, and the fact that park employees are letting their friends break in line. This equipment-based service provider is having trouble with _______________ factors.  
 Your Answer:  cross-docking  
Correct Answer:  operational  
 


  Question 5    Multiple Choice  1 of 1 point(s)    
  Service _______________ means that customers cannot store services.  
 Your Answer:  perishability  
Correct Answer:  perishability  
 


  Question 6    Multiple Choice  1 of 1 point(s)    
  Toronto-based Loblaw supermarket chain developed President's Choice brand products to sell exclusively in its stores. The products were so popular the supermarket chain has agreed to allow noncompetitive retailers in other countries to use the President's Choice brand in exchange for paying a fee to Loblaw. By selling its legally protected brand name to another retailer to use for a period of time, Loblaw has engaged in:  
 Your Answer:  licensing  
Correct Answer:  licensing  
 


  Question 7    Multiple Choice  0 of 1 point(s)    
  One of the ways Megan determined which day care center to send her children was by visiting neighboring ones and observing the indoor and outdoor play facilities, the cleanliness of the employees' uniforms, and how well infants were segregated from older children. She was evaluating the day care using _______________ qualities before she let the day care keep her children.  
 Your Answer:  experience  
Correct Answer:  search  
 


  Question 8    Multiple Choice  1 of 1 point(s)    
  The act of including a service with the purchase of a good is called:  
 Your Answer:  embodying  
Correct Answer:  embodying  
 


  Question 9    Multiple Choice  1 of 1 point(s)    
  _______________ is the process by which organizations adjust the supply of their services in an attempt to match demand.  
 Your Answer:  Capacity management  
Correct Answer:  Capacity management  
 


  Question 10    Multiple Choice  1 of 1 point(s)    
  The billboard announced that all the rooms in the Courtland Inn had been recently redecorated and that its room rate was still only $39 per night. This offer sounded like a good deal to Brenda, but when she went to her motel room, she found the blinds would not go down, the toilet seat was cracked, the rug was stained, and there was a new bedspread on the lumpy mattress. Brenda felt as if she had been deceived because in her mind the purchase of new bedspreads does not mean redecorated. A gap analysis would reveal a gap between:  
 Your Answer:  service quality standards and consumer expectations  
Correct Answer:  service quality standards and consumer expectations  
 


Question 1    Multiple Choice  1 of 1 point(s)    
  There are five dimensions that are commonly used to position a service. A service provider might use physical evidence to position a company on the basis of its:  
 Your Answer:  tangibles  
Correct Answer:  tangibles  
 


  Question 2    Multiple Choice  1 of 1 point(s)    
  The three strategies marketers use to "sell" people are:  
 Your Answer:  pure selling, product improvement, and market fulfillment  
Correct Answer:  pure selling, product improvement, and market fulfillment  
 


  Question 3    Multiple Choice  1 of 1 point(s)    
  The act of including a service with the purchase of a good is called:  
 Your Answer:  embodying  
Correct Answer:  embodying  
 


  Question 4    Multiple Choice  0 of 1 point(s)    
  In _______________, branded materials become component parts of other branded products.  
 Your Answer:  compound branding  
Correct Answer:  ingredient branding  
 


  Question 5    Multiple Choice  1 of 1 point(s)    
  My bank has a little sticker in the window that says deposits are insured by the FDIC. The pank is trying to position its service using the ________ dimension.  
 Your Answer:  Assurance  
Correct Answer:  Assurance  
 


  Question 6    Multiple Choice  1 of 1 point(s)    
  Which of the following statements about brand equity is true?  
 Your Answer:  All of the above statements about brand equity are true.  
Correct Answer:  All of the above statements about brand equity are true.  
 


  Question 7    Multiple Choice  1 of 1 point(s)    
  Dan's annual medical examination gives him peace of mind and removes all anxieties about undiscovered health problems. These feelings are the benefits of the doctor's examination and are called:  
 Your Answer:  core services  
Correct Answer:  core services  
 


  Question 8    Multiple Choice  1 of 1 point(s)    
  A _______________ is a name, a term, a symbol, or other unique element of a product that identifies one firm's products and sets it apart from the competition.  
 Your Answer:  brand  
Correct Answer:  brand  
 


  Question 9    Multiple Choice  0 of 1 point(s)    
  In the fall of 2002, Sheraton Hotels & Resorts launched a training program for its employees to convince them that management and employees working together can make each service encounter a competitive advantage. Sheraton is using:  
 Your Answer:  disintermediation  
Correct Answer:  internal marketing  
 


  Question 10    Multiple Choice  0 of 1 point(s)    
  Every time Bonnie gets her weekly massage, she leaves the spa feeling completely stress-free. This feeling is the benefit the massage provides and is called a(n):  
 Your Answer:  augmented benefit  
Correct Answer:  core service  
 


  Question 1    Multiple Choice  1 of 1 point(s)    
  As Martin checked into the motel, the clerk at the registration desk was rude and laughed at Martin's request for an early morning wake-up call. In terms of the service encounter dimensions, Martin experienced trouble with the _______________ dimension.  
 Your Answer:  social contact  
Correct Answer:  social contact  
 


  Question 2    Multiple Choice  1 of 1 point(s)    
  _______________ are acts, efforts, or performances exchanged from producer to user without ownership rights.  
 Your Answer:  Services  
Correct Answer:  Services  
 


  Question 3    Multiple Choice  1 of 1 point(s)    
  Toronto-based Loblaw supermarket chain developed President's Choice brand products to sell exclusively in its stores. President's Choice is an example of a:  
 Your Answer:  private-label brand  
Correct Answer:  private-label brand  
 


  Question 4    Multiple Choice  1 of 1 point(s)    
  Pinnacle Foods Corp. markets Vlasic pickles. According to brand designers, Vlasic would:  
 Your Answer:  not be a good brand name for a pickle because it is hard to spell, to say, to read, and to remember  
Correct Answer:  not be a good brand name for a pickle because it is hard to spell, to say, to read, and to remember  
 


  Question 5    Multiple Choice  1 of 1 point(s)    
  Keisha particularly enjoys taking her dog to Pampered Paws for its monthly bath and trim because the people who work on the dogs use a pleasant smelling soap, have full-sized kennels in which to dry the dog, and give the dog a treat when its treatment is finished. These additional services provided by Pampered Paws enhance the value of having a clean dog. Pampered Paws use _______________ to differentiate itself from its competitors.  
 Your Answer:  augmented services  
Correct Answer:  augmented services  
 


  Question 6    Multiple Choice  1 of 1 point(s)    
  Alfredo would have used _______________ qualities to determine that his English tutor had successfully taught him how to conjugate a verb.  
 Your Answer:  experience  
Correct Answer:  experience  
 


  Question 7    Multiple Choice  1 of 1 point(s)    
  Search qualities would be most useful to a consumer buying:  
 Your Answer:  a new refrigerator  
Correct Answer:  a new refrigerator  
 


  Question 8    Multiple Choice  1 of 1 point(s)    
  Which of the following is a condition that is favorable to creating a brand?  
 Your Answer:  Quality and value are easy to maintain.  
Correct Answer:  Quality and value are easy to maintain.  
 


  Question 9    Multiple Choice  0 of 1 point(s)    
  Which if the following statements about creating a brand is true?  
 Your Answer:  It is important not to let the customer confuse product benefits with benefit with the brand name.  
Correct Answer:  Creating a brand uses the psychological principal of classical conditioning.  
 


  Question 10    Multiple Choice  1 of 1 point(s)    
  Have you ever gotten a great haircut, gone back to the same person to get a second haircut, and received one of the worst haircuts of your life? The differences between the two haircuts are due to the _______________ characteristic of services.  
 Your Answer:  variability  
Correct Answer:  variability  
 


Question 1    Multiple Choice  0 of 1 point(s)    
  According to the text, telemarketers who call to encourage you to vote for a particular candidate are engaged in:  
 Your Answer:  wholesaling  
Correct Answer:  intangibles marketing  
 


  Question 2    Multiple Choice  1 of 1 point(s)    
  An advertisement for the Blue Ridge Parkway (a highway) that runs from the Shenandoah Valley to the Smokey Mountains describes the highway as a "trip back in time." This is an example of:  
 Your Answer:  place marketing  
Correct Answer:  place marketing  
 


  Question 3    Multiple Choice  1 of 1 point(s)    
  In 2001, the United Methodist Church in the United States budgeted $4 million for ads to encourage former church members to return to church through the use of gentle persuasion. This ad campaign is an example of:  
 Your Answer:  idea marketing  
Correct Answer:  idea marketing  
 


  Question 4    Multiple Choice  1 of 1 point(s)    
  Disney allowed a paint manufacturer to produce paint using the Disney brand name for a specific period of time. Disney paint is an example of:  
 Your Answer:  licensing  
Correct Answer:  licensing  
 


  Question 5    Multiple Choice  1 of 1 point(s)    
  A _______________ is a name, a term, a symbol, or other unique element of a product that identifies one firm's products and sets it apart from the competition.  
 Your Answer:  brand  
Correct Answer:  brand  
 


  Question 6    Multiple Choice  1 of 1 point(s)    
  The _______________ is the interaction between the customer and the service provider.  
 Your Answer:  service encounter  
Correct Answer:  service encounter  
 


  Question 7    Multiple Choice  1 of 1 point(s)    
  Which of the following is a criticism of packaging discussed in class?  
 Your Answer:  Excessive packaging depletes natural resources.  
Correct Answer:  Excessive packaging depletes natural resources.  
 


  Question 8    Multiple Choice  0 of 1 point(s)    
  Jack was so enthusiastic about the band he saw performing at a local club last weekend that he persuaded a large group of friends to go to the club to hear this band on Thursday. Unfortunately, the band's performance was uninspired and at times off key. The performance differences are due to the _______________ characteristic of service.  
 Your Answer:  empathy  
Correct Answer:  variability  
 


  Question 9    Multiple Choice  0 of 1 point(s)    
  The bank management decided its customers would not mind if it only opened the drive-thru window on Saturdays and did not require other bank employees to work on weekends. Customers who work during the week could not apply for a loan on Saturday because that was not a service available at the drive-thru window. Gap analysis would show this is an example of a gap between:  
 Your Answer:  management perception and quality standards set by the firm  
Correct Answer:  customer expectations and management perceptions  
 


  Question 10    Multiple Choice  1 of 1 point(s)    
  What do we mean when we speak of negative brand equity?  
 Your Answer:  Consumers have unfavorable associations with the brand name.  
Correct Answer:  Consumers have unfavorable associations with the brand name.  
 


Question 1    Multiple Choice  1 of 1 point(s)    
  In _______________, branded materials become component parts of other branded products.  
 Your Answer:  ingredient branding  
Correct Answer:  ingredient branding  
 


  Question 2    Multiple Choice  1 of 1 point(s)    
  _______________ is a measurement tool that gauges the difference between a customer's expectations of service quality and what actually occurred.  
 Your Answer:  Gap analysis  
Correct Answer:  Gap analysis  
 


  Question 3    Multiple Choice  1 of 1 point(s)    
  What do we mean when we speak of negative brand equity?  
 Your Answer:  Consumers have unfavorable associations with the brand name.  
Correct Answer:  Consumers have unfavorable associations with the brand name.  
 


  Question 4    Multiple Choice  1 of 1 point(s)    
  One of the ways physicians deal with potential problems associated with the _______________ characteristic of services is to hang their medical diplomas on the walls of the examining rooms and to wear white coats.  
 Your Answer:  intangibility  
Correct Answer:  intangibility  
 


  Question 5    Multiple Choice  1 of 1 point(s)    
  The Christian music industry is the only facet of the music industry that has shown significant growth during the last decade. A recording company executive believes the American people are ready for gospel singers to enter the mainstream market and record some pop music. The executive has found a group of five who have a small following in the Christian music market and has the group recording new songs that are uplifting without being preachy. He hopes that a nationwide tour will expand the market for other Christian music performers. Which of the following strategies is the executive using to market this new group?  
 Your Answer:  market fulfillment  
Correct Answer:  market fulfillment  
 


  Question 6    Multiple Choice  1 of 1 point(s)    
  _______________ is an agreement between two brands to work together in marketing a new product.  
 Your Answer:  Co-branding  
Correct Answer:  Co-branding  
 


  Question 7    Multiple Choice  0 of 1 point(s)    
  People most commonly buy the services of a tattoo artist on the weekends. To encourage people to use their services on the weekdays, many tattoo artists adjust prices to influence demand. In other words, they use differing prices to lessen the problems related to the _______________ characteristic of services.  
 Your Answer:  intangibility  
Correct Answer:  perishability  
 


  Question 8    Multiple Choice  1 of 1 point(s)    
  Which of the following services will most likely be evaluated using credence qualities?  
 Your Answer:  surgery to repair a leaking heart valve  
Correct Answer:  surgery to repair a leaking heart valve  
 


  Question 9    Multiple Choice  1 of 1 point(s)    
  Management of a chain of fast-food restaurants have determined one of its corporate goals is to have no customer wait longer than five minutes to receive his or her food once the order is placed. A manager of a local restaurant from this chain has interpreted the goal to apply only to the drive-thru window. This restaurant has earned the award for the fastest drive-thru window in the chain, but customers inside often are kept waiting for their orders ten or more minutes. A gap analysis would reveal a gap between:  
 Your Answer:  management perception and quality standards set by the firm  
Correct Answer:  management perception and quality standards set by the firm  
 


  Question 10    Multiple Choice  1 of 1 point(s)    
  Service _______________ means that a firm cannot store its services.  
 Your Answer:  perishability  
Correct Answer:  perishability  
 


Question 1    Multiple Choice  1 of 1 point(s)    
  Which of the following is an example of a service encounter?  
 Your Answer:  all of the above  
Correct Answer:  all of the above  
 


  Question 2    Multiple Choice  1 of 1 point(s)    
  Management of a chain of fast-food restaurants have determined one of its corporate goals is to have no customer wait longer than five minutes to receive his or her food once the order is placed. A manager of a local restaurant from this chain has interpreted the goal to apply only to the drive-thru window. This restaurant has earned the award for the fastest drive-thru window in the chain, but customers inside often are kept waiting for their orders ten or more minutes. A gap analysis would reveal a gap between:  
 Your Answer:  management perception and quality standards set by the firm  
Correct Answer:  management perception and quality standards set by the firm  
 


  Question 3    Multiple Choice  1 of 1 point(s)    
  The Christian music industry is the only facet of the music industry that has shown significant growth during the last decade. A recording company executive believes the American people are ready for gospel singers to enter the mainstream market and record some pop music. The executive has found a group of five who have a small following in the Christian music market and has the group recording new songs that are uplifting without being preachy. He hopes that a nationwide tour will expand the market for other Christian music performers. Which of the following strategies is the executive using to market this new group?  
 Your Answer:  market fulfillment  
Correct Answer:  market fulfillment  
 


  Question 4    Multiple Choice  0 of 1 point(s)    
  Which of the service positioning dimensions suggests that it is important to have consistency in you service delivery?  
 Your Answer:  Tangibles  
Correct Answer:  Reliability  
 


  Question 5    Multiple Choice  1 of 1 point(s)    
  Every time Bonnie gets her weekly massage, she leaves the spa feeling completely stress-free. This feeling is the benefit the massage provides and is called a(n):  
 Your Answer:  core service  
Correct Answer:  core service  
 


  Question 6    Multiple Choice  1 of 1 point(s)    
  Pinnacle Foods Corp. markets Vlasic pickles. According to brand designers, Vlasic would:  
 Your Answer:  not be a good brand name for a pickle because it is hard to spell, to say, to read, and to remember  
Correct Answer:  not be a good brand name for a pickle because it is hard to spell, to say, to read, and to remember  
 


  Question 7    Multiple Choice  1 of 1 point(s)    
  Vlasic pickles use a stork with a blue billed hat and a red bow tie on its packaging and its marketing communications. The company uses the stork to:  
 Your Answer:  set itself apart from its competitors  
Correct Answer:  set itself apart from its competitors  
 


  Question 8    Multiple Choice  0 of 1 point(s)    
  Capacity management is one of the ways to meet potential problems related to the _______________ characteristic of services.  
 Your Answer:  intangibility  
Correct Answer:  perishability  
 


  Question 9    Multiple Choice  1 of 1 point(s)    
  Which of the following product offerings would be most likely described as intangible?  
 Your Answer:  a physical examination by a doctor  
Correct Answer:  a physical examination by a doctor  
 


  Question 10    Multiple Choice  1 of 1 point(s)    
  The act of including a service with the purchase of a good is called:  
 Your Answer:  embodying  
Correct Answer:  embodying  
 


 Question 1    Multiple Choice  1 of 1 point(s)    
  The _______________ is the interaction between the customer and the service provider.  
 Your Answer:  service encounter  
Correct Answer:  service encounter  
 


  Question 2    Multiple Choice  1 of 1 point(s)    
  For many years the Grand Ole Opry was a fixture at the Ryman Auditorium in downtown Nashville, Tennessee. The auditorium had superior acoustics but a structure that was not readily updated to meet new safety requirements. The Ryman was also located in a part of Nashville that many did not consider safe. Thus, the Grand Ole Opry built a new facility for the show. The need to make the structure in which the Grand Ole Opry was held more attractive is an example of a(n) _______________ factor.  
 Your Answer:  environmental  
Correct Answer:  environmental  
 


  Question 3    Multiple Choice  1 of 1 point(s)    
  Capacity management is one of the ways to meet potential problems related to the _______________ characteristic of services.  
 Your Answer:  perishability  
Correct Answer:  perishability  
 


  Question 4    Multiple Choice  1 of 1 point(s)    
  The Federal Fair Packaging and Labeling Act, the Nutrition Labeling and Education Act, and the U.S. Food and Drug Administration work together to:  
 Your Answer:  ensure consumer safety  
Correct Answer:  ensure consumer safety  
 


  Question 5    Multiple Choice  1 of 1 point(s)    
  A _______________ is the legal term for a brand name, brand mark, or trade character.  
 Your Answer:  trademark  
Correct Answer:  trademark  
 


  Question 6    Multiple Choice  1 of 1 point(s)    
  Toronto-based Loblaw supermarket chain developed President's Choice brand products to sell exclusively in its stores. The products were so popular the supermarket chain has agreed to allow noncompetitive retailers in other countries to use the President's Choice brand in exchange for paying a fee to Loblaw. By selling its legally protected brand name to another retailer to use for a period of time, Loblaw has engaged in:  
 Your Answer:  licensing  
Correct Answer:  licensing  
 


  Question 7    Multiple Choice  0 of 1 point(s)    
  There are five dimensions that are commonly used to position a service. A service provider might use physical evidence to position a company on the basis of its:  
 Your Answer:  reliability  
Correct Answer:  tangibles  
 


  Question 8    Multiple Choice  1 of 1 point(s)    
  _______________ strategies regard a city, state, country, or other locale as a brand and attempt to position the location so that consumers choose to visit.  
 Your Answer:  Place marketing  
Correct Answer:  Place marketing  
 


  Question 9    Multiple Choice  1 of 1 point(s)    
  According to your instructor, what is the big issue faced by not-for-profit marketers?  
 Your Answer:  They have multiple sets of customers to deal with  
Correct Answer:  They have multiple sets of customers to deal with  
 


  Question 10    Multiple Choice  1 of 1 point(s)    
  Dell Computer Corp. uses the same name that has been used by Dell comics for more than fifty years. This action:  
 Your Answer:  is completely legal because the products do not compete  
Correct Answer:  is completely legal because the products do not compete  
 


Question 1    Multiple Choice  1 of 1 point(s)    
  Which of the following is an example of a service encounter?  
 Your Answer:  all of the above  
Correct Answer:  all of the above  
 


  Question 2    Multiple Choice  1 of 1 point(s)    
  Have you ever gotten a great haircut, gone back to the same person to get a second haircut, and received one of the worst haircuts of your life? The differences between the two haircuts are due to the _______________ characteristic of services.  
 Your Answer:  variability  
Correct Answer:  variability  
 


  Question 3    Multiple Choice  1 of 1 point(s)    
  Political candidates often hire image consultants who tell them what clothes to wear, how to wear their hair, and how to act in front of a crowd of people. Political candidates who use image consultants to help them market themselves to the voters are using which of the following strategies?  
 Your Answer:  product improvement  
Correct Answer:  product improvement  
 


  Question 4    Multiple Choice  1 of 1 point(s)    
  The Federal Fair Packaging and Labeling Act, the Nutrition Labeling and Education Act, and the U.S. Food and Drug Administration work together to:  
 Your Answer:  ensure consumer safety  
Correct Answer:  ensure consumer safety  
 


  Question 5    Multiple Choice  1 of 1 point(s)    
  My bank has a little sticker in the window that says deposits are insured by the FDIC. The pank is trying to position its service using the ________ dimension.  
 Your Answer:  Assurance  
Correct Answer:  Assurance  
 


  Question 6    Multiple Choice  1 of 1 point(s)    
  The billboard announced that all the rooms in the Courtland Inn had been recently redecorated and that its room rate was still only $39 per night. This offer sounded like a good deal to Brenda, but when she went to her motel room, she found the blinds would not go down, the toilet seat was cracked, the rug was stained, and there was a new bedspread on the lumpy mattress. Brenda felt as if she had been deceived because in her mind the purchase of new bedspreads does not mean redecorated. A gap analysis would reveal a gap between:  
 Your Answer:  service quality standards and consumer expectations  
Correct Answer:  service quality standards and consumer expectations  
 


  Question 7    Multiple Choice  1 of 1 point(s)    
  Which of the service positioning dimensions suggests that it is important to have consistency in you service delivery?  
 Your Answer:  Reliability  
Correct Answer:  Reliability  
 


  Question 8    Multiple Choice  1 of 1 point(s)    
  A _______________ is the legal term for a brand name, brand mark, or trade character.  
 Your Answer:  trademark  
Correct Answer:  trademark  
 


  Question 9    Multiple Choice  0 of 1 point(s)    
  Procter & Gamble makes Tide, Cheer, Ivory Snow, and Bold detergents as well as PertPlus, Rejoice, and Vidal Sassoon shampoos. Through its use of a separate and unique brand name for each of these products, Procter & Gamble is using a(n) _______________ strategy.  
 Your Answer:  family brand  
Correct Answer:  individual brand  
 


  Question 10    Multiple Choice  0 of 1 point(s)    
  One of the ways physicians deal with potential problems associated with the _______________ characteristic of services is to hang their medical diplomas on the walls of the examining rooms and to wear white coats.  
 Your Answer:  perishability  
Correct Answer:  intangibility  
 


  Question 1    Multiple Choice  1 of 1 point(s)    
  Alfredo would have used _______________ qualities to determine that his English tutor had successfully taught him how to conjugate a verb.  
 Your Answer:  experience  
Correct Answer:  experience  
 


  Question 2    Multiple Choice  1 of 1 point(s)    
  To reduce problems associated with the intangibility of the services offered by a restaurant, the restaurant owner can:  
 Your Answer:  provide clean tablecloths for each new customer  
Correct Answer:  provide clean tablecloths for each new customer  
 


  Question 3    Multiple Choice  1 of 1 point(s)    
  The National Rifle Association (NRA) promotes the safe use of firearms in its advertising as well as encouraging people to support the right of all Americans to bear arms. The NRA is engaged in:  
 Your Answer:  idea marketing  
Correct Answer:  idea marketing  
 


  Question 4    Multiple Choice  1 of 1 point(s)    
  In many communities, you can find as many as thirty different fast-food restaurant franchises within a couple of miles of each other. The nearness of these restaurants to each other illustrates the importance of the _______________ factor.  
 Your Answer:  locational  
Correct Answer:  locational  
 


  Question 5    Multiple Choice  1 of 1 point(s)    
  The billboard announced that all the rooms in the Courtland Inn had been recently redecorated and that its room rate was still only $39 per night. This offer sounded like a good deal to Brenda, but when she went to her motel room, she found the blinds would not go down, the toilet seat was cracked, the rug was stained, and there was a new bedspread on the lumpy mattress. Brenda felt as if she had been deceived because in her mind the purchase of new bedspreads does not mean redecorated. A gap analysis would reveal a gap between:  
 Your Answer:  service quality standards and consumer expectations  
Correct Answer:  service quality standards and consumer expectations  
 


  Question 6    Multiple Choice  1 of 1 point(s)    
  Barry was having stomach trouble and was sent to have an ultrasound performed on his abdomen. The technician performing the ultrasound told Barry he had gallstones. Because Barry could not recognize gallstones even as the technician pointed them out, Barry was left to evaluate the ultrasound on the basis of _______________ qualities.  
 Your Answer:  credence  
Correct Answer:  credence  
 


  Question 7    Multiple Choice  1 of 1 point(s)    
  Which of the following services will most likely be evaluated using credence qualities?  
 Your Answer:  surgery to repair a leaking heart valve  
Correct Answer:  surgery to repair a leaking heart valve  
 


  Question 8    Multiple Choice  1 of 1 point(s)    
  Service _______________ refers to the inevitable differences in a service provider's performances from one day to the next.  
 Your Answer:  variability  
Correct Answer:  variability  
 


  Question 9    Multiple Choice  1 of 1 point(s)    
  The _______________ is the interaction between the customer and the service provider.  
 Your Answer:  service encounter  
Correct Answer:  service encounter  
 


  Question 10    Multiple Choice  1 of 1 point(s)    
  Which of the following is a component of brand equity according to David Aaker?  
 Your Answer:  Perceived quality.  
Correct Answer:  Perceived quality.  
 

 A business that uses price lining is:  
  trying to avoid the use of psychological pricing that may be adversely received by customers  
 trying to recoup its research and development costs for a new product  
 attempting to attain a large market share before any competitors can enter the marketplace  
@ using price points to determine its pricing strategy  
 potentially in legal trouble unless it can justify the practice  

A consumer product list price is also referred to as a(n):  
  unbundled price  
 captive price  
 skimming price  
@ suggested retail price  
 channel price 

A distributor of nutritional supplements wants to set prices that will allow the company to achieve a five percent profit margin on all supplements sold to retailers. What type of pricing objective does the distributor have?  
  break-even  
 image enhancement  
@ profit  
 customer satisfaction  
 sales or market share

  A firm is using a _____ strategy when it introduces a product at a very low price to encourage rapid product adoption.  
  skimming pricing  
 comparative pricing  
 price-quality inference pricing  
@ penetration pricing  
 corrective pricing  

A firm is using _____ when it charges a high, premium price for a new product with the intention of reducing the price in the future.  
@  a skimming price  
 trial pricing  
 price-quality inference pricing  
 penetration pricing  
 prestige pricing  

   A kinked demand occurs when:  
  the selling company endures several quarters without reaching its break-even point.  
 the seller must operate in a pure competition-like environment.  
@ total revenue declines when a product's price is increased or decreased in relation to the prevailing market level.  
 the seller has complete control over the selling price.  
 a new product enters a competitive monopolistic market. 

  A local restaurant sells lunch entrees for $7.95, $9.95, and $11.95. From this information, you can infer the restaurant uses:  
 Your Answer:  odd pricing and price lining  
Correct Answer:  odd pricing and price lining 

A major disadvantage of using demand-based pricing strategies is:  
@  how difficult it is estimate demand accurately  
 how to base price on demand rather than the seller's costs  
 the ease with which a competitors can figure out the pricing strategy used  
 the inability to change other elements of the marketing mix when this strategy is used  
 its inability to respond to changes in the environment  

 A person selling maple racks for cooling cakes and cookies knows that it costs $15.00 to make one rack. The person who crafted the rack prices each rack so that he earns a 25 percent profit from the money he spent to make one rack. What will be the selling price for one maple cooling rack?  
@  $18.75  
 $11.25  
 $21.00  
 $20.00  
 cannot be determined from the information given  
 

A person selling maple racks for cooling cakes and cookies knows that it costs $15.00 to make one rack. If the person who crafted the rack prices each rack so that he earns a 25 percent profit from the money he spent to make one rack, he is using which approach to pricing?  
  demand-based pricing  
 psychological pricing  
 skimming  
 price discrimination  
@ cost-plus pricing  
 
According to the law of demand,:  
@  if prices decrease, customers will buy more  
 customers are oblivious to price changes  
 it cannot be predicted how changes in price will affect demand  
 demand equals supply  
 if prices increase, customers will buy more

  According to the textbook, which of the elements of the marketing mix is probably the least understood and least appreciated?  
@  pricing  
 promotion  
 distribution  
 production  
 product  

 According to your instructor, what is a disadvantage of high-low pricing?  
  It creates the perception that prices are lower than they actually are.  
 It creates economies of scale because you re-price every product regularly.  
@ It teaches your customers to only buy things when they are on sale.  
 It creates the perception that prices are higher than they actually are. 

According to your instructor, which of these price tactics is typical for American consumer products?  
  "Haggle" pricing  
@ "No-Haggle" pricing  
 Haggis Pricing  
 Negotiated pricing  

After looking at the ads in her Sunday paper, Ruby decided to visit a local nursery and buy potting soil at $1.99 a bag. The regular price is $4.99 a bag. While Ruby was there she also purchased three bushes, six flowering plants, and a bird bath. You can infer from this information that the nursery used the bags of potting soil as:  
  generalizable stimuli  
@ loss leaders  
 assimilators  
 fixed costs  
 store bait  

Amerfit Nutrition is a leading manufacturer of nutritional supplements. When setting prices, the company decided to institute a pricing strategy that would support a five percent increase in sales over the next three years. What type of pricing objective does Amerfit Nutrition have?  
 Your Answer:  customer satisfaction  
Correct Answer:  sales or market share


  Anderson Industries makes weather-proof covers for barbecue grills. Each cover costs $7.00 to produce and is sold to distributors for $15. The company's fixed costs are $170,000. How many grill covers must it sell to reach a target profit of $30,000?  
  13,333 covers  
 20,000 covers  
@ 25,000 covers  
 28,572 covers  
 40,000 covers  
 
Anderson Industries makes weather-proof covers for barbecue grills. Each cover costs $7.00 to produce and is sold to distributors for $15. The company's total fixed costs are $170,000. Calculate Anderson's break-even point in units.  
  24,285 covers  
@ 21,250 covers  
 11,333 covers  
 17,272 covers  
 1,728 covers  

  Ann Harris Enterprises designs one-of-a-kind greeting cards. No two are exactly alike. Each card costs $.75 to create, and she sells her cards to a retail gift shop for $1.50. Her total fixed costs are $5,100. Calculate her break-even point.  
  3,400 greeting cards  
 4,533 greeting cards  
 3,825 greeting cards  
@ 6,800 greeting cards  
 5,100 greeting cards

   A&D Industries makes dog beds stuffed with cedar shavings. Each bed costs $3 to make and will be sold to a distributor for $5 each. Its total fixed cost is $12,500. Calculate A&D's break-even point in dollars.  
  $41,667  
 $20,834  
@ $31,250  
 $12,500  
 $6,250  



   A _____ allows marketers to identify how many units of a product will have to sell at a given price to be profitable.  
  profitability analysis  
 market audit  
 situational analysis  
@ break-even analysis  
 marketability analysis  

   Bayer AG agreed to pay $46 million as part of a law settlement for conspiring with Archer-Daniels-Midland (ADM) to keep prices of citric acid at a certain level. These two international companies were guilty of:  
@  price fixing  
 skimming pricing  
 price lining  
 dynamic pricing  
 price discrimination 

Bayer AG agreed to pay $46 million as part of a law settlement for conspiring with Archer-Daniels-Midland (ADM) to keep prices of citric acid at a certain level. These two international companies were guilty of:  
  price lining  
@ horizontal price fixing  
 price discrimination  
 skimming pricing  
 vertical price fixing 

 Bonnie has enough money to either buy a used car or to pay for an international marketing class that will spend three weeks visiting companies in Europe. She needs the car to get to work and to class, but she is also planning on a career in international marketing, and the trip would allow her to start networking. She will incur _____ whether she buys the car or takes the class.  
  stockholders' equity  
 retained earnings  
@ opportunity costs  
 liquidity  
 transformation costs  
 
Break-even analysis:  
  is used to determine how many more units need to be sold to increase market share by a specific amount  
 is a technique used to calculate contribution margin  
 determines the amount of retained earnings a company will have during an accounting period  
 is a technique marketers use to examine the relationship between supply and demand  
@ is calculated using per unit variable costs, the per unit price, and fixed costs 
 

 Bud K is a retail company that specializes in knives, swords, and related items. It sells the Accusharp blade sharpener for $10.95. If you buy two at the same time, you pay only $18. The reduced price is due to a:  
  functional discount  
 promotional allowance  
 trade discount  
@ quantity discount  
 cash discount

   Concerns about how the genetics of vegetables are being manipulated by growers has led to increased demand for organic, naturally-grown vegetables and an accompanying price increase for these products. Which external influence on pricing is most evident in this strategy?  
  the competition  
 the economic environment  
 currency exchange rates  
 government subsidies  
@ consumer trends  

   Dixie Pride Manufacturing began because several couples used to compete on who could provide the best snacks for their weekly football games. Four of the wives joined together in 1988 to form a company that sells serving-size packages of snacks. Ever since its beginnings Dixie Pride has followed a low cost strategy. The company is about to surprise the market with a line of seasoned peanut snacks. The owners believe the product will encounter elastic demand and that competitors will be able to quickly copy it. Which type of pricing strategy would you recommend it use?  
  skimming pricing  
 comparative pricing  
 price-quality inference pricing  
@ penetration pricing  
 corrective pricing  

Everyone has seen commercials in which consumers are told that a particular retailer is determined to offer the lowest price and if a lower price can be found, the retailer will match it. Which external influence on pricing is most evident in this type of advertising?  
@ the competition  
 the economic environment  
 currency exchange rates  
 government subsidies  
 consumer trends 

Flour, sugar, yeast, salt, and plastic bags are all examples of _____ for a company that bakes and sells loaves of bread.  
  liquidity expenses  
 amortization costs  
@ variable costs  
 retained earnings  
 fixed costs  

For a company that manufactures plastic signs, the printing press used to make the signs, its manager's salary, and its utilities are all examples of:  
  liquidity expenses  
 amortization costs  
 variable costs  
 retained earnings  
@ fixed costs  

  Go Green Power Company provides "green power," an alternative electrical power source made from biomass (clearing debris, stumps, leaves, and grass clippings). Each of its plants is designed to produce enough electricity to power 2,500 homes. Which environmental condition will influence the pricing of this electricity?  
  economic growth  
 consumer confidence  
 consumer feelings toward the environment  
 the business cycle  
@ all of the above  
 
 Government _____ to domestic industries, either in the form of an outright payment or as tax relief, allow some firms to sell their products at prices often below production prices.  
@  price subsidies  
 exchange rates  
 countertrades  
 tariffs  
 import quotas 


   Have you ever stayed in a resort hotel and tried to buy a soda from one of its vending machines? If you have, you were probably surprised when you had to deposit $1.50 or more to buy one can of soda since you are probably used to paying less. The lower price at which you expect to buy soda is called your:  
  discriminatory price  
 price generalization  
 motivational price concept  
@ internal reference price  
 price stimulus

 In an oligopoly situation, where the demand curve for the product is kinked, what is the best strategy to increase profits?  
  Increase price to increase revenues.  
 Decrease price to increase revenues.  
 Increase price to decrease revenues.  
 Decrease price to decrease revenues.  
@ Maintain price and reduce costs.  
 
 In some states, unfair trade practices acts:  
  regulate what markups can be used by various industries  
 control the sale of agricultural products and raw materials  
 regulate all forms of psychological pricing  
@ ban the use of loss leader pricing  
 prohibit price lining  

King David is a kosher chicken-processing plant in Georgia. Chicken processed at the King David facility is priced about 50 percent more than other chicken. One of the reasons King David sells its chicken, which is prepared according to Hebrew dietary law, at prices higher than chicken processed the ordinary way is to:  
@  send a signal to consumers about the quality of the product  
 eliminate opportunity costs  
 create sales without having to advertise  
 avoid having to use a long distribution channel  
 reduce its fixed costs  

  King David is a kosher chicken-processing plant in Georgia. It costs $.30 per pound to prepare the chicken. King David sells its chicken, which is prepared according to Hebrew dietary law, to distributors at $.46 per pound. The distributor sells the chicken to supermarkets for $.65 per pound, and the retail price of the chicken is on average $1.25 per pound. King David's margin per pound is:  
@  $.16  
 $.30  
 $.46  
 $.60  
 $1.25 

 Leah was excited that she could afford to bring her children to spend the day at the amusement park. Entrance to the park was only $5. Then Leah learned that each ride in the park costs between $1.50 and $3, but since she was already in the park and did not want to disappoint her children, she felt like she had no choice but to buy the ride tickets. This amusement park uses:  
  price skimming  
 comparative pricing  
@ captive pricing  
 price bundling  
 penetration pricing 

  When a company charges the same rate to ship a product anywhere in the United States, it is using which form of geographic pricing?  
  captive pricing  
 F.O.B. destination  
 F.O.B. delivered  
@ uniform delivered pricing  
 F.O.B. origin 

When demand is inelastic,:  
@  price and revenue change in the same direction  
 revenues decrease when price increases  
 revenue is unaffected by price changes  
 quantity demanded increases when price increases  
 the demand curve is more horizontal  

  When demand is ______, increases in price result in increases in total revenues while decreases in price result in decreases in total revenue.  
  flexible  
 supply-driven  
@ inelastic  
 elastic  
 synergistic

 When George ordered a DVD player and a set of surround-sound speakers on the Internet, he really felt like he had got a bargain because the seller did not charge him any shipping costs. The Web site where George made his purchase most likely used _____ and included the shipping costs in the base price.  
  captive pricing  
 F.O.B. delivered  
@ freight absorption pricing  
 uniform delivered pricing  
 zone pricing  

 When Joe's Coffee Nook raised the price of latte, Joe noticed a substantial change in how many cups of latte he sold daily. When he reduced the price of a cup of latte, his sales of latte increased. From this information, you can assume the demand for cups of latte is:  
  static  
 supply-driven  
 inelastic  
@ elastic  
 synergistic  

When operated at capacity, a company that markets silk-screened t-shirts can produce 75,000 shirts a year. Its variable and fixed costs are covered if each shirt is sold for $10. One year, the company found it could only sell 60,000 shirts at $10. To remain at full-production capacity, it decided to sell an additional 15,000 shirts for $8. What pricing method did the silk-screener use?  
  capacity management  
@ price-floor pricing  
 resale maintenance  
 penetration pricing  
 marketing mix-based pricing 

  When Radius Corporation introduced its unique, oval-headed toothbrush made of black neoprene that looks like a scuba-diving accessory, Radius priced the toothbrush at $9.95 at a time when regular toothbrushes were selling for $2. Radius wanted to recoup its investment in developing this toothbrush by using:  
@  a skimming price  
 trial pricing  
 price-quality inference pricing  
 penetration pricing  
 corrective pricing

  Why do marketers consider prestige products to be an exception to the law of demand?  
  because the demand curve for prestige products slopes downward and to the right  
@ because increasing the price of prestige products can make them seem more desirable  
 because demand for prestige products often is greater than supply  
 because prestige products like diamonds, sapphires, and emeralds are nonrenewable resources  
 because customers are not oblivious to the price of prestige products like they are to the price of other products

  Which of the following is a type of pricing objective?  
  break-even  
 contribution margin  
 environmental effect  
 transactional motivation  
@ sales or market share 

Which of the following is an example of a price?  
  the registration fee for summer camp  
 dues to a service organization  
 compensation for a hair cut  
 college tuition  
@ all of the above

 Which of the following is NOT a type of pricing objective?  
@ break-even  
 image enhancement  
 profit  
 customer satisfaction  
 competitive effect  

  Which of the following statements about pricing is true?  
  Pricing is the least important marketing mix element during a recession.  
@ Both consumers and purchasing agents focus on price when making a purchase decision.  
 Pricing is unaffected by changes in the business cycle.  
 Most consumers believe price has little influence on their purchase decisions.  
 All of the above statements about pricing are true.  

Which of the following would be an example of a variable cost for an amusement park?  
  salary of park manager  
@ rest room supplies  
 liability insurance  
 interest on loan  
 all of the above 

  Which of the following would be an example of a fixed cost for an amusement park?  
  prizes given away at the park arcade  
 hourly wages of employees operating the rides  
 balloons  
@ liability insurance  
 rest room supplies 

   Will Rogers, the famous American humorist was quoted as saying, "What this country needs is a good five cent cigar." This suggests an opportunity for _________.  
  Yield management pricing  
@ Target costing  
 Profit optimization costing  
 Cost plus pricing

With _____, a company deliberately sets a low price with the express idea of driving its competition out of business.  
@  predatory pricing  
 psychological pricing  
 a price hierarchy  
 price lining  
 dynamic pricing  

With _____, a new product carries a low price for a limited period of time to attract customers.  
@ trial pricing  
 comparative pricing  
 specialty pricing  
 price skimming  
 commodity pricing

With _____, the marketers figure; all costs for the product and then add the desired profit per unit.  
  supply-based pricing  
@ cost-plus pricing  
 marketing-oriented pricing  
 physiological pricing  
 demand-based pricing 

   Yield management pricing...  
  Is used by manufacturers of new products  
 Requires extensive advertising to succeed  
 Takes advantage of the price/value heuristic  
@ Uses price discrimination to maximize profits

 Zanger Waller makes ceramic tile, stone tile, mosaics, and glass tile. It is the most expensive tile on the market, and as a result, its ads are visually rich with luxurious imagery. The homes featured in Zanger Waller ads are mansions that have used the most expensive materials available. Zanger Waller uses its advertising to:  
  avoid having to manipulate the production element of its marketing mix  
 eliminate opportunity costs  
@ create a strong relationship between its promotion and its pricing  
 show that its fixed costs are high  
 show the relationship between its price and its distribution  


_____ are costs that do not vary with the number of units produced.  
  Liquidity expenses  
 Amortization costs  
 Variable costs  
 Retained earnings  
@ Fixed costs 

  _____ is a measure of the sensitivity of customers to changes in price.  
  A liquidity ratio  
 Demand sensitivity  
@ The price elasticity of demand  
 Customer apathy  
 A break-even analysis  



 _____ is the value customers give up or exchange to obtain a desired product.  
  Profit  
 Revenue  
 Contribution margin  
 Exchange value  
@ Price  

_____ means the selling price is based on an estimate of volume or quantity a firm can sell in different markets at different prices.  
  Capacity management  
 Price-floor planning  
 Resale maintenance  
 Penetration pricing  
@ Demand-based pricing 

_____ occurs when manufacturers or wholesalers attempt to force retailers to charge a certain price for their products.  
  Price lining  
 Horizontal price fixing  
 Price discrimination  
 Skimming pricing  
@ Vertical price fixing  

   _____ provides a way for marketers to look at cost and demand at the same time and identifies the output and the price that will generate the maximum profit.  
  Break-even analysis  
 A situational analysis  
 A competitive analysis  
@ Marginal analysis  
 A market audit 

   _____ refers to the sale of two or more goods and services as a single package for one price.  
  Price skimming  
 Comparative pricing  
 Captive pricing  
@ Price bundling  
 Penetration pricing

Question 1    Multiple Choice  1 of 1 point(s)    
  When George ordered a DVD player and a set of surround-sound speakers on the Internet, he really felt like he had got a bargain because the seller did not charge him any shipping costs. The Web site where George made his purchase most likely used _____ and included the shipping costs in the base price.  
 Your Answer:  freight absorption pricing  
Correct Answer:  freight absorption pricing  
 


  Question 2    Multiple Choice  1 of 1 point(s)    
  Concerns about how the genetics of vegetables are being manipulated by growers has led to increased demand for organic, naturally-grown vegetables and an accompanying price increase for these products. Which external influence on pricing is most evident in this strategy?  
 Your Answer:  consumer trends  
Correct Answer:  consumer trends  
 


  Question 3    Multiple Choice  0 of 1 point(s)    
  Market planners often choose cost-based pricing strategies because:  
 Your Answer:  [None Given]  
Correct Answer:  these strategies are simple to use and relatively risk-free  
 


  Question 4    Multiple Choice  0 of 1 point(s)    
  Bayer AG agreed to pay $46 million as part of a law settlement for conspiring with Archer-Daniels-Midland (ADM) to keep prices of citric acid at a certain level. These two international companies were guilty of:  
 Your Answer:  [None Given]  
Correct Answer:  horizontal price fixing  
 


  Question 5    Multiple Choice  0 of 1 point(s)    
  Which of the following statements about pricing is true?  
 Your Answer:  [None Given]  
Correct Answer:  Both consumers and purchasing agents focus on price when making a purchase decision.  
 


  Question 6    Multiple Choice  0 of 1 point(s)    
  A&D Industries makes dog beds stuffed with cedar shavings. Each bed costs $3 to make and will be sold to a distributor for $5 each. Its total fixed cost is $12,500. Calculate A&D's break-even point in dollars.  
 Your Answer:  [None Given]  
Correct Answer:  $31,250  
 


  Question 7    Multiple Choice  0 of 1 point(s)    
  A person selling maple racks for cooling cakes and cookies knows that it costs $15.00 to make one rack. The person who crafted the rack prices each rack so that he earns a 25 percent profit from the money he spent to make one rack. What will be the selling price for one maple cooling rack?  
 Your Answer:  [None Given]  
Correct Answer:  $18.75  
 


  Question 8    Multiple Choice  0 of 1 point(s)    
  To estimate what demand is likely to be at different prices for new or existing products, marketers can use:  
 Your Answer:  [None Given]  
Correct Answer:  laboratory studies  
 


  Question 9    Multiple Choice  0 of 1 point(s)    
  A distributor of nutritional supplements wants to set prices that will allow the company to achieve a five percent profit margin on all supplements sold to retailers. What type of pricing objective does the distributor have?  
 Your Answer:  [None Given]  
Correct Answer:  profit  
 


  Question 10    Multiple Choice  0 of 1 point(s)    
  The demand curve:  
 Your Answer:  [None Given]  
Correct Answer:  shows the relationship between product demand and product price  
 

 Question 1    Multiple Choice  1 of 1 point(s)    
  Flour, sugar, yeast, salt, and plastic bags are all examples of _____ for a company that bakes and sells loaves of bread.  
 Your Answer:  variable costs  
Correct Answer:  variable costs  
 


  Question 2    Multiple Choice  0 of 1 point(s)    
  There are many different manufacturers of riflescopes used by hunters. Leopold & Thomas is a German manufacturer that adopted a(n) _____ pricing objective for its premium scope. The firm improved its product's magnification and durability and then priced its top-of-the-line riflescope at five times the average price.  
 Your Answer:  [None Given]  
Correct Answer:  image enhancement  
 


  Question 3    Multiple Choice  0 of 1 point(s)    
  A _____ allows marketers to identify how many units of a product will have to sell at a given price to be profitable.  
 Your Answer:  [None Given]  
Correct Answer:  break-even analysis  
 


  Question 4    Multiple Choice  0 of 1 point(s)    
  Break-even analysis:  
 Your Answer:  [None Given]  
Correct Answer:  is calculated using per unit variable costs, the per unit price, and fixed costs  
 


  Question 5    Multiple Choice  0 of 1 point(s)    
  Everyone has seen commercials in which consumers are told that a particular retailer is determined to offer the lowest price and if a lower price can be found, the retailer will match it. Which external influence on pricing is most evident in this type of advertising?  
 Your Answer:  [None Given]  
Correct Answer:  the competition  
 


  Question 6    Multiple Choice  0 of 1 point(s)    
  Ann Harris Enterprises designs one-of-a-kind greeting cards. No two are exactly alike. Each card costs $.75 to create, and she sells her cards to a retail gift shop for $1.50. Her total fixed costs are $5,100. Calculate her break-even point.  
 Your Answer:  [None Given]  
Correct Answer:  6,800 greeting cards  
 


  Question 7    Multiple Choice  0 of 1 point(s)    
  According to your instructor, what is a disadvantage of high-low pricing?  
 Your Answer:  [None Given]  
Correct Answer:  It teaches your customers to only buy things when they are on sale.  
 


  Question 8    Multiple Choice  0 of 1 point(s)    
  The average price Xerox charged when it introduced the first stand-alone fax machine was $12,700. This premium price was a way for Xerox to recoup some of the research and development costs that went into producing this machine. Xerox used:  
 Your Answer:  [None Given]  
Correct Answer:  a skimming price  
 


  Question 9    Multiple Choice  0 of 1 point(s)    
  A firm is using _____ when it charges a high, premium price for a new product with the intention of reducing the price in the future.  
 Your Answer:  [None Given]  
Correct Answer:  a skimming price  
 


  Question 10    Multiple Choice  0 of 1 point(s)    
  A person selling maple racks for cooling cakes and cookies knows that it costs $15.00 to make one rack. If the person who crafted the rack prices each rack so that he earns a 25 percent profit from the money he spent to make one rack, he is using which approach to pricing?  
 Your Answer:  [None Given]  
Correct Answer:  cost-plus pricing  
 

  Question 1    Multiple Choice  1 of 1 point(s)    
  _____ means the selling price is based on an estimate of volume or quantity a firm can sell in different markets at different prices.  
 Your Answer:  Demand-based pricing  
Correct Answer:  Demand-based pricing  
 


  Question 2    Multiple Choice  0 of 1 point(s)    
  The changes in prices of other products affect the demand for an item. This is a phenomenon called:  
 Your Answer:  [None Given]  
Correct Answer:  cross-elasticity of demand  
 


  Question 3    Multiple Choice  0 of 1 point(s)    
  Rick is looking for a congratulatory present for his significant other who loves chocolate candy. Rick decided the candy priced at $2.50 per pound could not be nearly as good as the candy priced at $9.50 per pound. Rick's decision to buy the more expensive chocolate resulted directly from a(n):  
 Your Answer:  [None Given]  
Correct Answer:  price-quality inference  
 


  Question 4    Multiple Choice  0 of 1 point(s)    
  Monica makes seasoning mixes that she sells at craft fairs. Which of the following statements accurately describes her costs?  
 Your Answer:  [None Given]  
Correct Answer:  Her average fixed cost is based on how many mixes she makes.  
 


  Question 5    Multiple Choice  0 of 1 point(s)    
  Zanger Waller makes ceramic tile, stone tile, mosaics, and glass tile. It is the most expensive tile on the market, and as a result, its ads are visually rich with luxurious imagery. The homes featured in Zanger Waller ads are mansions that have used the most expensive materials available. Zanger Waller uses its advertising to:  
 Your Answer:  [None Given]  
Correct Answer:  create a strong relationship between its promotion and its pricing  
 


  Question 6    Multiple Choice  0 of 1 point(s)    
  _____ refers to the sale of two or more goods and services as a single package for one price.  
 Your Answer:  [None Given]  
Correct Answer:  Price bundling  
 


  Question 7    Multiple Choice  0 of 1 point(s)    
  Break-even analysis:  
 Your Answer:  [None Given]  
Correct Answer:  is calculated using per unit variable costs, the per unit price, and fixed costs  
 


  Question 8    Multiple Choice  0 of 1 point(s)    
  Leah was excited that she could afford to bring her children to spend the day at the amusement park. Entrance to the park was only $5. Then Leah learned that each ride in the park costs between $1.50 and $3, but since she was already in the park and did not want to disappoint her children, she felt like she had no choice but to buy the ride tickets. This amusement park uses:  
 Your Answer:  [None Given]  
Correct Answer:  captive pricing  
 


  Question 9    Multiple Choice  0 of 1 point(s)    
  Telma was preparing to do some holiday baking. She needed to buy 3 pounds of margarine. She normally uses Land o' Lakes margarine, but its price was about $.35 higher per pound. Because she believes all margarine is basically the same, she readily purchased the cheaper Blue Bonnet margarine. The availability of substitutes for margarine indicates that demand for the product is:  
 Your Answer:  [None Given]  
Correct Answer:  elastic  
 


  Question 10    Multiple Choice  0 of 1 point(s)    
  After looking at the ads in her Sunday paper, Ruby decided to visit a local nursery and buy potting soil at $1.99 a bag. The regular price is $4.99 a bag. While Ruby was there she also purchased three bushes, six flowering plants, and a bird bath. You can infer from this information that the nursery used the bags of potting soil as:  
 Your Answer:  [None Given]  
Correct Answer:  loss leaders  
 



Question 1    Multiple Choice  0 of 1 point(s)    
  _____ is a technique marketers use to examine the relationship between cost and price and to determine what sales volume must be reached at a given price before the company will completely cover its total costs and past which it will begin making a profit.  
 Your Answer:  [None Given]  
Correct Answer:  Break-even analysis  
 


  Question 2    Multiple Choice  0 of 1 point(s)    
  The price elasticity of demand is calculated by:  
 Your Answer:  [None Given]  
Correct Answer:  dividing percentage change in quantity demanded by percentage change in price  
 


  Question 3    Multiple Choice  0 of 1 point(s)    
  Rick is looking for a congratulatory present for his significant other who loves chocolate candy. Rick decided the candy priced at $2.50 per pound could not be nearly as good as the candy priced at $9.50 per pound. Rick's decision to buy the more expensive chocolate resulted directly from a(n):  
 Your Answer:  [None Given]  
Correct Answer:  price-quality inference  
 


  Question 4    Multiple Choice  0 of 1 point(s)    
  According to your instructor, which of these price tactics is typical for American consumer products?  
 Your Answer:  [None Given]  
Correct Answer:  "No-Haggle" pricing  
 


  Question 5    Multiple Choice  0 of 1 point(s)    
  In some states, unfair trade practices acts:  
 Your Answer:  [None Given]  
Correct Answer:  ban the use of loss leader pricing  
 


  Question 6    Multiple Choice  0 of 1 point(s)    
  Go Green Power Company provides "green power," an alternative electrical power source made from biomass (clearing debris, stumps, leaves, and grass clippings). Each of its plants is designed to produce enough electricity to power 2,500 homes. Which environmental condition will influence the pricing of this electricity?  
 Your Answer:  [None Given]  
Correct Answer:  all of the above  
 


  Question 7    Multiple Choice  0 of 1 point(s)    
  Bayer AG agreed to pay $46 million as part of a law settlement for conspiring with Archer-Daniels-Midland (ADM) to keep prices of citric acid at a certain level. These two international companies were guilty of:  
 Your Answer:  [None Given]  
Correct Answer:  price fixing  
 


  Question 8    Multiple Choice  0 of 1 point(s)    
  Which of the following is a type of pricing objective?  
 Your Answer:  [None Given]  
Correct Answer:  sales or market share  
 


  Question 9    Multiple Choice  0 of 1 point(s)    
  Which of the following is NOT a type of pricing objective?  
 Your Answer:  [None Given]  
Correct Answer:  break-even  
 


  Question 10    Multiple Choice  0 of 1 point(s)    
  _____ are the per-unit costs of production that will fluctuate depending on how many units or individual products a firm produces.  
 Your Answer:  [None Given]  
Correct Answer:  Variable costs  
 


Question 1    Multiple Choice  1 of 1 point(s)    
  A&D Industries makes dog beds stuffed with cedar shavings. Each bed costs $3 to make and will be sold to a distributor for $5 each. Its total fixed cost is $12,500. Calculate A&D's break-even point in dollars.  
 Your Answer:  $31,250  
Correct Answer:  $31,250  
 


  Question 2    Multiple Choice  1 of 1 point(s)    
  _____ occurs when manufacturers or wholesalers attempt to force retailers to charge a certain price for their products.  
 Your Answer:  Vertical price fixing  
Correct Answer:  Vertical price fixing  
 


  Question 3    Multiple Choice  1 of 1 point(s)    
  When George ordered a DVD player and a set of surround-sound speakers on the Internet, he really felt like he had got a bargain because the seller did not charge him any shipping costs. The Web site where George made his purchase most likely used _____ and included the shipping costs in the base price.  
 Your Answer:  freight absorption pricing  
Correct Answer:  freight absorption pricing  
 


  Question 4    Multiple Choice  0 of 1 point(s)    
  The break-even point is the point at which:  
 Your Answer:  [None Given]  
Correct Answer:  the total revenue and total costs lines intersect  
 


  Question 5    Multiple Choice  0 of 1 point(s)    
  King David is a kosher chicken-processing plant in Georgia. Chicken processed at the King David facility is priced about 50 percent more than other chicken. One of the reasons King David sells its chicken, which is prepared according to Hebrew dietary law, at prices higher than chicken processed the ordinary way is to:  
 Your Answer:  [None Given]  
Correct Answer:  send a signal to consumers about the quality of the product  
 


  Question 6    Multiple Choice  0 of 1 point(s)    
  Many people rely on the prescription drug Lipitor to control their cholesterol. An increase in the price of the drug would have little effect on the quantity demanded because there are no substitutes for the drug, and because people who are taking it have no choice but to continue taking it if they wish to stay healthy. The demand for Lipitor is:  
 Your Answer:  [None Given]  
Correct Answer:  inelastic  
 


  Question 7    Multiple Choice  0 of 1 point(s)    
  Zanger Waller makes ceramic tile, stone tile, mosaics, and glass tile. It is the most expensive tile on the market, and as a result, its ads are visually rich with luxurious imagery. The homes featured in Zanger Waller ads are mansions that have used the most expensive materials available. Zanger Waller uses its advertising to:  
 Your Answer:  [None Given]  
Correct Answer:  create a strong relationship between its promotion and its pricing  
 


  Question 8    Multiple Choice  0 of 1 point(s)    
  _____ is a technique marketers use to examine the relationship between cost and price and to determine what sales volume must be reached at a given price before the company will completely cover its total costs and past which it will begin making a profit.  
 Your Answer:  [None Given]  
Correct Answer:  Break-even analysis  
 


  Question 9    Multiple Choice  0 of 1 point(s)    
  Which of the following is a type of pricing objective?  
 Your Answer:  [None Given]  
Correct Answer:  sales or market share  
 


  Question 10    Multiple Choice  0 of 1 point(s)    
  In some states, unfair trade practices acts:  
 Your Answer:  [None Given]  
Correct Answer:  ban the use of loss leader pricing  
 


Question 7    Multiple Choice  0 of 1 point(s)    
  Members of the lumber industry in the Pacific Northwest were angered when the U.S. government increased the amount of Canadian lumber that can be imported into the U.S. The lumber industry lobbyists claim Canadian lumber producers receive government subsidies and can, therefore, sell their lumber cheaper. That means:  
 Your Answer:  [None Given]  
Correct Answer:  the government subsidies created a condition of unfair competition  
 


  Question 8    Multiple Choice  0 of 1 point(s)    
  Zanger Waller makes ceramic tile, stone tile, mosaics, and glass tile. It is the most expensive tile on the market, and as a result, its ads are visually rich with luxurious imagery. The homes featured in Zanger Waller ads are mansions that have used the most expensive materials available. Zanger Waller uses its advertising to:  
 Your Answer:  [None Given]  
Correct Answer:  create a strong relationship between its promotion and its pricing  
 


  Question 9    Multiple Choice  0 of 1 point(s)    
  _____ are the per-unit costs of production that will fluctuate depending on how many units or individual products a firm produces.  
 Your Answer:  [None Given]  
Correct Answer:  Variable costs  
 


  Question 10    Multiple Choice  0 of 1 point(s)    
  A distributor of nutritional supplements wants to set prices that will allow the company to achieve a five percent profit margin on all supplements sold to retailers. What type of pricing objective does the distributor have?  
 Your Answer:  [None Given]  
Correct Answer:  profit  
 

Question 1    Multiple Choice  1 of 1 point(s)    
  Have you ever stayed in a resort hotel and tried to buy a soda from one of its vending machines? If you have, you were probably surprised when you had to deposit $1.50 or more to buy one can of soda since you are probably used to paying less. The lower price at which you expect to buy soda is called your:  
 Your Answer:  internal reference price  
Correct Answer:  internal reference price  
 


  Question 2    Multiple Choice  1 of 1 point(s)    
  Bayer AG agreed to pay $46 million as part of a law settlement for conspiring with Archer-Daniels-Midland (ADM) to keep prices of citric acid at a certain level. These two international companies were guilty of:  
 Your Answer:  price fixing  
Correct Answer:  price fixing  
 


  Question 3    Multiple Choice  0 of 1 point(s)    
  To determine the break-even point, a firm needs first to:  
 Your Answer:  [None Given]  
Correct Answer:  calculate the contribution per unit to fixed costs  
 


  Question 4    Multiple Choice  0 of 1 point(s)    
  Why do marketers consider prestige products to be an exception to the law of demand?  
 Your Answer:  [None Given]  
Correct Answer:  because increasing the price of prestige products can make them seem more desirable  
 


  Question 5    Multiple Choice  0 of 1 point(s)    
  Which of the following is a type of pricing objective?  
 Your Answer:  [None Given]  
Correct Answer:  sales or market share  
 


  Question 6    Multiple Choice  0 of 1 point(s)    
  Which of the following would be an example of a variable cost for an amusement park?  
 Your Answer:  [None Given]  
Correct Answer:  rest room supplies  
 


  Question 7    Multiple Choice  0 of 1 point(s)    
  _____ occurs when manufacturers or wholesalers attempt to force retailers to charge a certain price for their products.  
 Your Answer:  [None Given]  
Correct Answer:  Vertical price fixing  
 


  Question 8    Multiple Choice  0 of 1 point(s)    
  Anderson Industries makes weather-proof covers for barbecue grills. Each cover costs $7.00 to produce and is sold to distributors for $15. The company's total fixed costs are $170,000. Calculate Anderson's break-even point in units.  
 Your Answer:  [None Given]  
Correct Answer:  21,250 covers  
 


  Question 9    Multiple Choice  0 of 1 point(s)    
  Which of the following is an example of a price?  
 Your Answer:  [None Given]  
Correct Answer:  all of the above  
 


  Question 10    Multiple Choice  0 of 1 point(s)    
  A _____ allows marketers to identify how many units of a product will have to sell at a given price to be profitable.  
 Your Answer:  [None Given]  
Correct Answer:  break-even analysis  
 

 Question 7    Multiple Choice  0 of 1 point(s)    
  Profit is maximized at the point at which:  
 Your Answer:  [None Given]  
Correct Answer:  marginal cost equals marginal revenue  
 


  Question 8    Multiple Choice  0 of 1 point(s)    
  To determine the break-even point, a firm needs first to:  
 Your Answer:  [None Given]  
Correct Answer:  calculate the contribution per unit to fixed costs  
 


  Question 9    Multiple Choice  0 of 1 point(s)    
  A major disadvantage of using demand-based pricing strategies is:  
 Your Answer:  [None Given]  
Correct Answer:  how difficult it is estimate demand accurately  
 


  Question 10    Multiple Choice  0 of 1 point(s)    
  Concerns about how the genetics of vegetables are being manipulated by growers has led to increased demand for organic, naturally-grown vegetables and an accompanying price increase for these products. Which external influence on pricing is most evident in this strategy?  
 Your Answer:  [None Given]  
Correct Answer:  consumer trends  

Question 1    Multiple Choice  0 of 1 point(s)    
  The developer of the Bent Creek subdivision charges homeowners a $500 annual fee to be eligible to use its pool, tennis courts, golf course, and other amenities. In addition, the homeowners are charged fees ranging from $1 to $10 every time they make use of one of these amenities. The developer is using:  
 Your Answer:  [None Given]  
Correct Answer:  two-part pricing  
 


  Question 2    Multiple Choice  0 of 1 point(s)    
  When Radius Corporation introduced its unique, oval-headed toothbrush made of black neoprene that looks like a scuba-diving accessory, Radius priced the toothbrush at $9.95 at a time when regular toothbrushes were selling for $2. Radius wanted to recoup its investment in developing this toothbrush by using:  
 Your Answer:  [None Given]  
Correct Answer:  a skimming price  
 


  Question 3    Multiple Choice  0 of 1 point(s)    
  For a skimming price to be successful,:  
 Your Answer:  [None Given]  
Correct Answer:  there should be little chance that competitors can get into the market quickly  
 


  Question 4    Multiple Choice  0 of 1 point(s)    
  Concerns about how the genetics of vegetables are being manipulated by growers has led to increased demand for organic, naturally-grown vegetables and an accompanying price increase for these products. Which external influence on pricing is most evident in this strategy?  
 Your Answer:  [None Given]  
Correct Answer:  consumer trends  
 


  Question 5    Multiple Choice  0 of 1 point(s)    
  Break-even analysis:  
 Your Answer:  [None Given]  
Correct Answer:  is calculated using per unit variable costs, the per unit price, and fixed costs  
 


  Question 6    Multiple Choice  0 of 1 point(s)    
  King David is a kosher chicken-processing plant in Georgia. It costs $.30 per pound to prepare the chicken. King David sells its chicken, which is prepared according to Hebrew dietary law, to distributors at $.46 per pound. The distributor sells the chicken to supermarkets for $.65 per pound, and the retail price of the chicken is on average $1.25 per pound. King David's margin per pound is:  
 Your Answer:  [None Given]  
Correct Answer:  $.16  
 


  Question 7    Multiple Choice  0 of 1 point(s)    
  Manufacturers of _____ would be most likely to use freight absorption pricing?  
 Your Answer:  [None Given]  
Correct Answer:  laptop computers  
 


  Question 8    Multiple Choice  0 of 1 point(s)    
  Have you ever noticed that most brands of toothpaste in similar sizes are priced within about a dollar of each other? What does this tell you about the toothpaste industry and its pricing strategy?  
 Your Answer:  [None Given]  
Correct Answer:  It is an oligopoly and has no price leadership.  
 


  Question 9    Multiple Choice  0 of 1 point(s)    
  A kinked demand occurs when:  
 Your Answer:  [None Given]  
Correct Answer:  total revenue declines when a product's price is increased or decreased in relation to the prevailing market level.  
 


  Question 10    Multiple Choice  0 of 1 point(s)    
  A distributor of nutritional supplements wants to set prices that will allow the company to achieve a five percent profit margin on all supplements sold to retailers. What type of pricing objective does the distributor have?  
 Your Answer:  [None Given]  
Correct Answer:  profit  
 
Question 1    Multiple Choice  1 of 1 point(s)    
  Which of the following is a type of pricing objective?  
 Your Answer:  sales or market share  
Correct Answer:  sales or market share  
 


  Question 2    Multiple Choice  1 of 1 point(s)    
  _____ means the selling price is based on an estimate of volume or quantity a firm can sell in different markets at different prices.  
 Your Answer:  Demand-based pricing  
Correct Answer:  Demand-based pricing  
 


  Question 3    Multiple Choice  0 of 1 point(s)    
  The _____ is the difference between the variable cost of the product and the selling price of the product.  
 Your Answer:  [None Given]  
Correct Answer:  contribution margin  
 


  Question 4    Multiple Choice  0 of 1 point(s)    
  Marvin wants to buy a new rifle. He saw just the one he wanted advertised in the newspaper at a very low price. When he went to buy the gun, the store salesperson told Marvin the sale gun was inferior and would not sell it to him. Then the salesperson proceeded to convince Marvin to buy a rifle that cost much more money than he had intended to spend. Marvin has been victimized by:  
 Your Answer:  [None Given]  
Correct Answer:  a bait-and-switch scheme  
 


  Question 5    Multiple Choice  0 of 1 point(s)    
  A distributor of nutritional supplements wants to set prices that will allow the company to achieve a five percent profit margin on all supplements sold to retailers. What type of pricing objective does the distributor have?  
 Your Answer:  [None Given]  
Correct Answer:  profit  
 


  Question 6    Multiple Choice  0 of 1 point(s)    
  Go Green Power Company provides "green power," an alternative electrical power source made from biomass (clearing debris, stumps, leaves, and grass clippings). Each of its plants is designed to produce enough electricity to power 2,500 homes. Which environmental condition will influence the pricing of this electricity?  
 Your Answer:  [None Given]  
Correct Answer:  all of the above  
 


  Question 7    Multiple Choice  0 of 1 point(s)    
  _____ provides a way for marketers to look at cost and demand at the same time and identifies the output and the price that will generate the maximum profit.  
 Your Answer:  [None Given]  
Correct Answer:  Marginal analysis  
 


  Question 8    Multiple Choice  0 of 1 point(s)    
  The package for new Ultra Strength Ben Gay pain relieving cream promoted a limited time price reduction. The manufacturer of Ben Gay used:  
 Your Answer:  [None Given]  
Correct Answer:  trial pricing  
 


  Question 9    Multiple Choice  0 of 1 point(s)    
  The Chef's Fork is a just like all other barbecue grilling forks except it has a meat thermometer in its handle, which allows a cook to make sure the meat being grilled is done to perfection. The Chef's Fork is in the introductory stage of its product life cycle. Most people are unaware of the product, but those who are aware are eager buyers. The fork's manufacturer is selling the fork for $45 while an ordinary meat fork and thermometer can be purchased for less than $20. From this information, it is obvious the manufacturer of the Chef's Fork is using:  
 Your Answer:  [None Given]  
Correct Answer:  a skimming price  
 


  Question 10    Multiple Choice  0 of 1 point(s)    
  Which of the following would be an example of a variable cost for an amusement park?  
 Your Answer:  [None Given]  
Correct Answer:  rest room supplies  

Question 1    Multiple Choice  0 of 1 point(s)    
  _____ is a pricing tactic a firm uses when it has two products that work only when used together. The firm sells one item at a very low price and then makes its profit on the second high-margin item.  
 Your Answer:  [None Given]  
Correct Answer:  Captive pricing  
 


  Question 2    Multiple Choice  0 of 1 point(s)    
  Dixie Pride Manufacturing began because several couples used to compete on who could provide the best snacks for their weekly football games. Four of the wives joined together in 1988 to form a company that sells serving-size packages of snacks. Ever since its beginnings Dixie Pride has followed a low cost strategy. The company is about to surprise the market with a line of seasoned peanut snacks. The owners believe the product will encounter elastic demand and that competitors will be able to quickly copy it. Which type of pricing strategy would you recommend it use?  
 Your Answer:  [None Given]  
Correct Answer:  penetration pricing  
 


  Question 3    Multiple Choice  0 of 1 point(s)    
  King David is a kosher chicken-processing plant in Georgia. Chicken processed at the King David facility is priced about 50 percent more than other chicken. One of the reasons King David sells its chicken, which is prepared according to Hebrew dietary law, at prices higher than chicken processed the ordinary way is to:  
 Your Answer:  [None Given]  
Correct Answer:  send a signal to consumers about the quality of the product  
 


  Question 4    Multiple Choice  0 of 1 point(s)    
  Manufacturers of _____ would be most likely to use freight absorption pricing?  
 Your Answer:  [None Given]  
Correct Answer:  laptop computers  
 


  Question 5    Multiple Choice  0 of 1 point(s)    
  A major disadvantage of using demand-based pricing strategies is:  
 Your Answer:  [None Given]  
Correct Answer:  how difficult it is estimate demand accurately  
 


  Question 6    Multiple Choice  0 of 1 point(s)    
  A consumer product list price is also referred to as a(n):  
 Your Answer:  [None Given]  
Correct Answer:  suggested retail price  
 


  Question 7    Multiple Choice  0 of 1 point(s)    
  _____ are the per-unit costs of production that will fluctuate depending on how many units or individual products a firm produces.  
 Your Answer:  [None Given]  
Correct Answer:  Variable costs  
 


  Question 8    Multiple Choice  0 of 1 point(s)    
  Ann Harris Enterprises designs one-of-a-kind greeting cards. No two are exactly alike. Each card costs $.75 to create, and she sells her cards to a retail gift shop for $1.50. Her total fixed costs are $5,100. Calculate her break-even point.  
 Your Answer:  [None Given]  
Correct Answer:  6,800 greeting cards  
 


  Question 9    Multiple Choice  0 of 1 point(s)    
  A person selling maple racks for cooling cakes and cookies knows that it costs $15.00 to make one rack. The person who crafted the rack prices each rack so that he earns a 25 percent profit from the money he spent to make one rack. What will be the selling price for one maple cooling rack?  
 Your Answer:  [None Given]  
Correct Answer:  $18.75  
 


  Question 10    Multiple Choice  0 of 1 point(s)    
  The price elasticity of demand is calculated by:  
 Your Answer:  [None Given]  
Correct Answer:  dividing percentage change in quantity demanded by percentage change in price  
 

Question 3    Multiple Choice  0 of 1 point(s)    
  The rate you pay to rent a motel room, the extra charge for the movie rental, and the money paid for room service are all examples of:  
 Your Answer:  [None Given]  
Correct Answer:  prices  
 


  Question 4    Multiple Choice  0 of 1 point(s)    
  A major disadvantage of using demand-based pricing strategies is:  
 Your Answer:  [None Given]  
Correct Answer:  how difficult it is estimate demand accurately  
 


  Question 5    Multiple Choice  0 of 1 point(s)    
  A consumer product list price is also referred to as a(n):  
 Your Answer:  [None Given]  
Correct Answer:  suggested retail price  
 


  Question 6    Multiple Choice  0 of 1 point(s)    
  _____ occurs when manufacturers or wholesalers attempt to force retailers to charge a certain price for their products.  
 Your Answer:  [None Given]  
Correct Answer:  Vertical price fixing  
 


  Question 7    Multiple Choice  0 of 1 point(s)    
  When operated at capacity, a company that markets silk-screened t-shirts can produce 75,000 shirts a year. Its variable and fixed costs are covered if each shirt is sold for $10. One year, the company found it could only sell 60,000 shirts at $10. To remain at full-production capacity, it decided to sell an additional 15,000 shirts for $8. What pricing method did the silk-screener use?  
 Your Answer:  [None Given]  
Correct Answer:  price-floor pricing  
 


  Question 8    Multiple Choice  0 of 1 point(s)    
  The _____ is the difference between the variable cost of the product and the selling price of the product.  
 Your Answer:  [None Given]  
Correct Answer:  contribution margin  
 


  Question 9    Multiple Choice  0 of 1 point(s)    
  With _____, a company deliberately sets a low price with the express idea of driving its competition out of business.  
 Your Answer:  [None Given]  
Correct Answer:  predatory pricing  
 


  Question 10    Multiple Choice  0 of 1 point(s)    
  Telma was preparing to do some holiday baking. She needed to buy 3 pounds of margarine. She normally uses Land o' Lakes margarine, but its price was about $.35 higher per pound. Because she believes all margarine is basically the same, she readily purchased the cheaper Blue Bonnet margarine. The availability of substitutes for margarine indicates that demand for the product is:  
 Your Answer:  [None Given]  
Correct Answer:  elastic  
 

Question 6    Multiple Choice  0 of 1 point(s)    
  Amerfit Nutrition is a leading manufacturer of nutritional supplements. When setting prices, the company decided to institute a pricing strategy that would support a five percent increase in sales over the next three years. What type of pricing objective does Amerfit Nutrition have?  
 Your Answer:  customer satisfaction  
Correct Answer:  sales or market share  
 

   A company using a push strategy is:  
  relying on sales promotions aimed at getting consumers to buy larger quantities of their products  
 allocating most of its promotional budget to advertising in newspaper, magazines, and television  
@ seeking to move its products through the channel by convincing channel members to offer the products and entice their customers to select these items  
 promoting a product in the maturity stage of its product life cycle  
 counting on consumers to learn about and express desire for a product, thus convincing retailers to stock the items 

   A company using a pull strategy is:  
  relying on purchase discounts aimed at getting retailers to stock larger quantities of their products  
 allocating most of its promotional budget to personal selling  
 seeking to move its products through the channel by convincing channel members to offer the products and entice their customers to select these items  
 promoting a product that is typically perceived as a commodity  
@ counting on consumers to learn about and express desire for a product, thus convincing retailers to stock the items 

 A company using a push strategy is:  
  relying on purchase discounts aimed at getting retailers to stock larger quantities of their products  
 allocating most of its promotional budget to personal selling  
@ seeking to move its products through the channel by convincing channel members to offer the products and entice their customers to select these items  
 promoting a product that is typically perceived as a commodity  
 counting on consumers to learn about and express desire for a product, thus convincing retailers to stock the items

 A florist who decided to budget 5 percent of last year's sales for promotional activities would be using which of the following budgeting techniques?  
  objective-task method  
 competitive-parity method  
 resource-allocation method  
 the bottom-up method  
@ percentage-of-sales method 

   A marcom manager would:  
  locate market niches for products in the maturity and decline stage of their product life cycle  
@ be expected to oversee integrated marketing communications (IMC) strategies  
 act as liaison between internal and external stakeholders  
 serve as a change agent for market functionality  
 do all of the above

 A newspaper article announcing that VoiceStream Wireless, the nation's sixth-largest wireless carrier, was changing its name to T-Mobile and that to begin the makeover process, it had replaced spokesperson Jamie Lee Curtis with Catherine Zeta-Jones is an example of the use of which element of the promotion mix?  
  sales promotion  
 advertising  
@ public relations  
 personal selling  
 product

  An ad for Post Selects brand cereal is an example of:  
@  product advertising  
 a sales promotion  
 one-to-one communication  
 institutional advertising  
 direct selling 

   An ad in a professional journal targeted to dentists asked them to recommend Crest toothpaste to their patients. It offered toothpaste samples that dentists could buy at cost to give to their patients to encourage patients to take better care of their teeth. The manufacturer of Crest toothpaste was using:  
  viral marketing  
 guerilla marketing  
@ a push strategy  
 public relations advertising  
 a pull strategy

According to the information presented in class, which of the following advertising media attracted the most money in 2003?  
  Broadcast television  
@ Newspapers  
 Yellow pages  
 Direct mail

According to the instructor, what do we mean when we speak of the richness of an information channel?  
  The relative cost of the information a channel can carry.  
 The ability of a channel of information to eliminate unnecessary noise.  
@ The number of different information cues the cannel can carry.  
 Whether or not the channel has a print component.  
 The cost per information cue in an information channel.  
 
   According to your instructor, when is the best time to decide how a promotional objective should be measured?  
  As the last step of the promotion planning process  
@ At the same time the promotion objectives are established  
 After the budget has been allocated to the promotional mix  
 The correct time will become apparent to you if you are open to waht the plan is telling you  
  According to the communication model, _____ is the process of translating an idea into a form of communication that will convey the desired meaning.  
  receiving  
@ encoding  
 signing  
 decoding  
 sending 

   According to the instructor, which of the following media is the richest information channel?  
  Newspaper  
 Outdoor  
 Radio  
 Television  
@ Face to face conversations

   Ads for Allegra allergy relief medication show people looking miserable and asks is their misery due to the event they are participating in or to pain and congestion from allergies. After a dose of Allegra, the same person is shown able to enjoy participating in life due to the relief provided by Allegra. What attention-getting device is being used in the Allegra ads?  
@  slice-of-life  
 testimonial  
 demonstration  
 lifestyle  
 humorous appeal 

 Advertising account managers:  
  write the advertising copy  
 posttest advertisements  
 determine the most efficient vehicles for delivering an advertising message  
@ are the primary liaison between the advertising agency and its client  
 do all of the above

   Advocacy advertising:  
  is primarily used to promote sales promotion activities  
@ states a firm's position on an issue to sway public opinion  
 is intended to inform customers about a new product  
 is also called public service advertising  
 does not create feedback


   Advertisers try to minimize mistakes by getting reactions to advertising messages before they are actually placed by using:  
@  pretesting  
 objective-and-task techniques  
 test sampling  
 standard deviation testing  
 hypothesis testing


   Copy testing:  
  measures selective perception variables for ads  
 calculates ad reach and frequency  
@ measures ad effectiveness  
 compares media schedules  
 does all of the above

 Cosmetics that do not need to be touched up during the day but are able to last from morning until late at night is a product that is in the growth stage of its product life cycle. Therefore, the promotion mix for such products:  
  includes the use of many different types of buying incentives  
@ shows an increase in the use of advertising  
 shows an increase in the use of sales promotions  
 tries to create product awareness  
 is accurately described by all of the above

   Delia's is a clothing retailer that targets teenage girls. It runs coordinated promotions for its catalogs, its Web site, and its retail outlets. It uses the same models in its catalog and in its print ads as well as on its Web site. Delia's works to make sure its public relations activities as well as its sales promotions harmonize with its advertising in all venues. From this information, you can infer that Delia's is using:  
  viral marketing  
 customerization  
 promotional synergy  
@ integrated marketing communications  
 synergistic communications 

   During the growth stage of the product life cycle,:  
  the use of advertising decreases  
 the use of sales promotion to encourage product trial increases  
@ promotions start stressing product benefits  
 the use of all types of incentives increases  
 none of the above occurs

  During the introduction stage of the product life cycle,:  
  personal selling is the primary promotional tool in the consumer market  
 pull strategies are commonly used  
 publicity is not effective  
@ sales promotion is used to encourage product trial  
 all of the above occur

For a typical product in the decline phase of the product life cycle,:  
  advertising expenses for it will increase  
 sales promotion expenses for it will increase  
 public relations expenses for it will increase  
 personal selling expenses for it will increase  
@ money spent on all elements of the promotion mix decreases

   For its clients, a full-service agency can:  
  create ad copy  
 do advertising research  
 select media for ads  
 create photographs and illustrations for ads  
@ do all of the above


 For which of the following products will frequency be more important than reach?  
@  a handheld computer with DSL capabilities  
 light bulbs  
 bricks  
 soda  
 generic brand pet food



For which of the following products would a continuous advertising schedule be most appropriate?  
  swimming pool chemicals  
 snow goggles  
 heaters  
 mosquito spray  
@ ready to eat canned soup 

 For which of the following products would a pulsing advertising schedule be most appropriate?  
  flour  
@ insect spray  
 gasoline  
 Candles  
 canned vegetables 

   Frequency is the:  
  total number of people who will be exposed to an ad placed in one media vehicle  
 percentage of people who will be exposed to an ad more than once  
 percentage of people who will be exposed to the media vehicle  
@ number of times a person in the target group will be exposed to an ad  
 total number of ads in one advertising campaign

   Glasurit is a type of paint made specifically for use on cars. An ad in Motor Trend magazine advising consumers to request their body shops use Glasurit paint is an example of how a company uses:  
  viral marketing  
 guerilla marketing  
 a push strategy  
 public relations advertising  
@ a pull strategy 


   In terms of the communication model, the message:  
  is the actual communication going from sender to receiver  
 may include both verbal and nonverbal elements  
 is individualized when it is delivered by a salesperson  
 must be carefully crafted to connect with a variety of people when it is delivered through advertising  
@ is accurately described by all of the above



   In the movie, Best of Show, two characters publish a magazine for lesbians who raise and show pedigreed dogs. An advertiser that selected this magazine for its advertising vehicle would:  
@ have a clearly identified target market  
 be engaged in mass marketing  
 have ignored the first step in developing an advertising campaign  
 be using internal marketing  
 not be using any segmentation strategy

  Institutional advertising:  
  is also called public service advertising  
 describes products as customer services  
 is primarily used to support sales promotion activities  
@ promotes the activities or point of view of a company  
 is used as a substitute for competitive advertising

   Jell-O ads contain the phrase, "Make some magic." All Ace Hardware store ads contain the statement, "Life is an ongoing project." The American Dairy Farmers run ads that say, "Ahh, the power of cheese." These are all examples of:  
  metaphors  
 personifications  
 similes  
@ slogans  
 analogies

   Many ads for weight loss programs use before and after photos of people who are quoted in the ad as having had great success with the program. What attention-getting device is being used in the weight-loss program ads?  
  slice-of-life  
@ testimonial  
 demonstration  
 lifestyle  
 humorous appeal 

   Promotion:  
  informs consumers about new goods and services  
 reminds consumers to continue using certain products  
 persuades consumers to choose one product over another  
 builds relationships with customers  
@ does all of the above
 
   Reach is the:  
  total number of people who will be exposed to an ad placed in one media vehicle  
 percentage of people who will be exposed to an ad more than once  
@ percentage of people who will be exposed to the media vehicle  
 number of times a person in the target group will be exposed to an ad  
 total number of ads in one advertising campaign 

Rogers Web Design is an example of a(n) _____ because it specializes in creating and placing banner and pop-up advertisements for consumer products. These are the only services the company performs for its clients.  
  creative boutique  
@ limited-service agency  
 online broker  
 online media representative  
 specialty agent 

 Sports drinks are in the maturity stage of their product life cycle. This means promotion for sports drinks:  
  concentrates on creating product awareness  
 primarily relies on public relations  
 does not use sales promotions  
@ concentrates on persuading people to switch brands  
 primarily uses push strategies  

  The ad for Maybelline age minimizing makeup in Ladies' Home Journal magazine featured actress Melina Kanakaredes and offered readers a $1-off coupon if they would try the new makeup. In terms of the communication model, the medium of this ad is:  
 Your Answer:  Maybelline and its advertising agency  
Correct Answer:  Ladies' Home Journal magazine  

 The ad for Maybelline age minimizing makeup in Ladies' Home Journal magazine featured actress Melina Kanakaredes and offered readers a $1-off coupon if they would try the new makeup. In terms of the communication model, the medium of this ad is:  
 Your Answer:  Maybelline and its advertising agency  
Correct Answer:  Ladies' Home Journal magazine

   The ad for Maybelline age minimizing makeup in Ladies' Home Journal magazine featured actress Melina Kanakaredes and offered readers a $1-off coupon if they would try the new makeup. In terms of the communication model, the source of this ad is:  
  Melina Kanakaredes  
 the target market to which Melina Kanakaredes appeals  
@ Maybelline and its advertising agency  
 readers who redeem the $1-off coupon  
 Ladies' Home Journal magazine 

 The ad for Oscar Mayer sliced ham advises consumers to, "Go ahead. Be shallow. Choose us for our looks." This wordplay on the common saying that looks are only skin deep is an example of a _____ appeal.  
  fear  
@ humorous  
 mass  
 non-AIDA  
 sex  

  The advertiser of a rifle scope used in hunting knows that 75 percent of its target market reads American Rifleman magazine every month and assumes its ad will be seen by the magazine's readers six times if it buys ads in all twelve issues. The advertiser can compare the effectiveness of American Rifleman to other hunting magazines by using:  
  average frequency  
 minimum reach  
 cost per medium (CPM)  
 average reach  
@ gross rating points (GRPs) 


   The background for the Benadryl allergy medication ad shows green grass and lovely flowers. The headline states "Benadryl is 54 percent more effective than the leading prescription." At the bottom of the ad in small print is an explanation of how these results of the effectiveness of Benadryl were determined. The ad also contains a package of Benadryl so consumers can easily recognize it at the store. The page next to the ad contains an article on a woman who gave birth only five days after learning she was pregnant. In terms of the communication model, the:  
  bright background would be classified as noise  
@ article on the opposite page is an example of noise  
 encoder does not share a mutual frame of reference with the decoder  
 larger the print type of the headline, the greater the noise of the ad  
 potential of the ad to communicate is directly related to the promotion channel  

   The central idea of an ad is expressed in its:  
  advertising layout  
 media vehicles  
 media schedule  
 implementation of the needs hierarchy model  
@ advertising appeal 

   The decision to use a cleaning genie to communicate the strength and power of Mr. Clean cleaning liquid is representative of the _____ process of the communication model.  
  receiving  
@ encoding  
 signing  
 decoding  
 sending 

The first stage in developing a promotion mix is to:  
  determine a target market  
@ establish promotion objectives  
 segment the market  
 hire a promotional manager  
 determine and allocate the promotion budget 

   The leading greeting card manufacturer in the United States is Hallmark. Its competitors include American Greeting and the smaller card manufacturer Carleton Cards. Which of the following statements concerning the relationship among these manufacturers is true?  
 @ Carleton would be most likely to benefit from the use of comparative advertising.  
 Hallmark would be the only one of the three that would benefit from comparative advertising.   All of these companies would benefit equally from comparative advertising.  
 It would be unethical for any of these companies to use comparative advertising.  
 Companies in the greeting cards industry would benefit from comparative advertising. 

The television ad for Hanes underwear was broadcast between programs on ABC. The ad showed Michael Jordan and a group of young men trying not to look at Jordan in a locker room situation. The ad indicates these men want to wear the same kind of underwear Jordan does. In terms of the communication model, the medium of this ad is:  
@  television  
 Michael Jordan  
 the locker room setting  
 all of the people in the commercial  
 the other ads which ran before and after the Hanes ad


   The _____ is the focus for companies adopting the integrated marketing communications (IMC) perspective.  
  channel partner  
@ customer  
 advertiser  
 stockholder  
 weather 


The _____ is the framework for developing, implementing, and controlling the firm's promotional activities.  
@  promotion plan  
 market audit  
 mission statement  
 organizational hierarchy  
 marketing hierarchy 

   The _____ of Batdorf and Bronson coffee is that the customer can purchase the coffee at its Web site roasted to order. It is the only coffee vendor that offers such a service.  
  creative platform  
 AIDA model  
@ unique selling proposition (USP)  
 FIGARO model  
 cooperative advantage 

The _____ philosophy recognizes the customer absorbs information about a product or organization from its advertising, the people who drive its delivery vehicles, its operating facilities, and any publicity.  
  traditional promotional  
 market-exchange  
 database marketing  
 niche marketing  
@ integrated marketing communications 

  To adopt an integrated marketing communications (IMC) philosophy, a company must be concerned about the creation of communication harmony. This means the company should:  
  take a niche approach to promotion  
 create different promotional strategies for each geographic market  
 use flexible pricing  
 use a generalized mission statement that supports multiple goals  
@ use all of the elements of its promotional mix plus product design and packaging to create a single message 

   To compare the relative cost-effectiveness of different media and of spots run on different vehicles in the same medium, media planners use:  
  average reach  
 gross rating points (GRPs)  
 cost per medium (CPM)  
@ cost per thousand (CPM)  
 average frequency

 To determine how consumers feel about the addition of the word kosher on the back of Frito-Lay chips, advertising would use which type of posttesting?  
  projective techniques  
 aided recall  
@ attitudinal measures  
 concept testing  
 unaided recall 

   To promote its 2002 summer shows, ABC ran several new programs rather than rerunning the previous season's shows. To promote the new programming, ABC ran ads in which it told viewers what NBC and CBS would be rebroadcasting and then described its own programs as new and different. To promote its summer television shows, ABC used:  
  combative advertising  
 direct selling  
 media synergy  
@ comparative advertising  
 advocacy advertising 

   To set its promotion budget, a manufacturer of neon signs decided one of its communication goals would be to create a brand identity for a product that is typically perceived as a commodity. Once the goal was set, the company allocated $350,000 to accomplish this goal. Which of the following budgeting methods is this company using?  
@  objective-task method  
 percentage-of-sales method  
 resource-allocation method  
 the top-down method  
 competitive-parity method 

   To show its toughness, Bounty paper towels uses ads which show how strong the towel is even wet and others which show how much more the towel can pick up than other brands. What attention-getting device is being used in the Bounty ads?  
  slice-of-life  
 testimonial  
@ demonstration  
 lifestyle  
 humorous appeal


  Toward the end of the fiscal year, the owner of a small company that manufactures packaging material came back from lunch concerned because he had learned that a business targeted to the same customers as his was planning on spending $150,000 on promotion. As soon as he arrived at the office, he called his financial manager and said, "I want to budget $150,000 for next year's promotion." Which method of promotional budgeting did the owner want to use?  
  objective-task method  
 percentage-of-sales method  
 resource-allocation method  
 the bottom-up method  
@ competitive-parity method 

   VHS-based movies are a product in the decline phase of the product life cycle because the product is being replaced by DVD-based movies. Which of the following statements accurately describes the promotion mix for VHS-based movies?  
  Money allocated to advertising will likely increase.  
 Money allocated to sales promotions will increase dramatically.  
 Money allocated to personal selling will increase.  
 Money allocated to public relations will increase.  
@ Money spent on all elements of the promotion mix will decrease.  

   What does a girl in a bikini have to do with auto parts? That's what many wives wondered when they saw the pin-up calendars their husbands received from auto repair shops between 1950 and 1980. The girls were used to attract attention to the ads on the calendar, and the auto repair shops were using a _____ appeal.  
  fear  
 humorous  
 mass  
 non-AIDA  
@ sex 

   What do we mean when we speak of "co-op" advertising?  
  A situation where a manufacturer and a retailer share the cost of advertising a specific product.  
 Advertising for not-for-profit agencies.  
@ A situation where one company intentionally imitates a competitor's advertising.  
 Advertising that is created using the assistance of an outside agency.  


What is the relationship between the richness of an information channel and the ability to stimulate emotions?  
  It is more expensive to stimulate an emotional response with an information-sparse medium such as print ads.  
 You can only influence people emotionally in a face-to-face situation.  
@ In order to stimulate emotions with advertising, you need an information rich medium such as television.  
 There is no relationship between the ability to stimulate emotions and the richness of the information channel.  
 The sparser the information channel, the easier it is to stimulate emotions.  
 
 When determining the media schedule for advertising, media planners must consider:  
  the match between the demographic and psychographic profile of the target audience and the people reached by a media vehicle  
 the advertising patterns of competitors  
 the capability of the medium to convey the desired information adequately  
 the compatibility of the product with editorial content  
@ all of the above

When FedEx communicates with its target markets through the use of a print medium such as Newsweek magazine, it is using which element of the promotion mix?  
  sales promotion  
@ advertising  
 telemarketing  
 personal selling  
 product 


Which media vehicle is described as the one consulted just before the consumer makes a purchase decision?  
  magazines  
 radio  
 television  
@ directories  
 newspapers 

  Which of the following factors will influence how much money a company allocates to the various elements of the promotion mix?  
  the complexity and formality of the company's decision-making processes  
 market potential  
 market size  
 past experiences with promotions  
@ all of the above

 Which of the following is a type of advertising schedule that would be appropriate for products like charcoal, plant fertilizer, and paper plates?  
  continuous  
 intermittent reinforcement  
@ pulsing  
 periodic reinforcement  
 bruiting


   Which of the following is an example of a bottom-up technique for developing promotional budgets?  
@  objective-task method  
 percentage-of-sales method  
 resource-allocation method  
 as-much-as-you-can-afford method  
 competitive-parity method 



Which of the following is an example of a top-down budgeting technique used to determine how much a firm will allocate to its promotional activities?  
  objective-task method  
@ percentage-of-sales method  
 resource-allocation method  
 the ABC method  
 economic potential method 

   Which of the following is NOT an element of the promotion mix?  
  sales promotion  
 advertising  
 public relations  
 personal selling  
@ product


  Which of the following is NOT an example of a specialized task that must be performed to create an advertising campaign?  
  creative services  
 account management  
@ new product development  
 research and marketing services  
 media planning 

   Which of the following is one of the reasons used to explain corporate resistance to the implementation of the integrated marketing communications (IMC) approach?  
@  The IMC approach puts more emphasis on aspects of the promotion mix other than advertising.  
 Corporations do not allow their advertising agencies to provide all needed promotional services.  
 The IMC approach does not change the way promotional strategies are implemented.  
 Upper-level managers tend to give their wholehearted support to IMC, and discount brand manager input.  
 All of the above are reasons used to explain corporate resistance to the implementation of integrated marketing communications (IMC) approach.  


   Which of the following is one of the reasons used to explain corporate resistance to the implementation of integrated marketing communications (IMC) approach?  
  Most industries do not have enough competitive intelligence.  
 The IMC approach cannot be used by companies operating in oligopolies.  
 Most companies are customer-driven instead of financially driven.  
@ The IMC approach puts more weight on aspects of promotion other than advertising.  
 Companies with multiple strategic business units (SBUs) cannot effectively use the IMC approach.

   Which of the following statements about advertising on television is true?  
@  Advertising on network television can be quite expensive.  
 Network television is less expensive than cable television.  
 Spot TV is ad time purchased on the national networks.  
 The cost of television media prohibits effective demographic and lifestyle segmentation.  
 None of the above statements about advertising on television is true.  


Which of the following statements about magazines as an advertising medium is true?  
@  Magazines have long lead times.  
 Magazines have not benefited from the use of computer technology.  
 Magazines typically use a mass marketing strategy.  
 Magazines do not accept local advertising.  
 None of the above statements about magazines as an advertising medium is true.  


   Which of the following statements about newspapers as an advertising medium is true?  
  Newspapers are one of the newest media vehicles.  
 Newspapers are most effective when used with national advertising campaigns.  
 Within the traditional communication process, newspapers are the only medium that does not experience noise.  
 Newspapers are inappropriate means of advertising sales promotions.  
@ Local retailers can make effective use of newspapers as an advertising medium.  

   Which of the following statements about the competitive-parity method of budgeting is true?  
  The competitive-parity method is the most commonly used bottom-up method for budgeting.  
 This method works because it relies on dynamic market shares that change annually.  
 The competitive-parity method is the only top-down budgeting method used to determine promotional budgets.  
@ In the consumer market, the competitive-parity method of budgeting would be described as "keeping up with the Joneses".  
 The use of this method guarantees a successful promotion.  

   Which of these demand curve effects is a desireble outcome of promotions?  
  Generate a shift upward in the overall demand curve  
 Decrease price elasticity when prices increaase  
 Increase price elasticity when prices decrease  
@ All of the above 

   Which of these is more likely to be the measure of success for an advertising campaign as a whole?  
  The number of people who saw the first ad in the campaign.  
 The number of people who could recall any ad in the campaign without help.  
 The number of people who recognized the first ad in the campaign.  
@ Whether or not sales increased during the period the campaign was running  
 The total number of impressions generated by the campaign.  

   Which of these quotations best illustrates the difficulty of evaluating promotions?  
  "There's a sucker born every minute, and two to take him"  
 "There has to be a pony in there somewhere"  
 "You can fool all of the people some of the time and you can fool some of the people all of the time, but you can't fool all of the people all of the time"  
@ "I know half of my advertising budget is wasted, but I don't know which half"  

   Why is it important to understand the AIDA model for persuasive communication?  
@ In order to have an effective persuasive message, you must address all four stages  
 Attention, Interest, Desire, Action  
 Because different media require different stages of the model.  
 A good persuasive message begins with a call to action.  
 

   Xiomey was shopping in the mall when a researcher approached her and asked if she had ever seen ads for Serta, Beautyrest, or Select Comfort mattresses. This question would be most likely part of which type of posttesting?  
  projective techniques  
@ aided recall  
 attitudinal measures  
 concept testing  
 unaided recall

  _____ is a problem-solving process for getting an advertising message to a specific target audience in the most effective fashion-based on the objective of the promotion and the advertising budget.  
@  Media planning  
 Guerilla marketing  
 Viral marketing  
 AIDA  
 Ad-based management

  _____ is nonpersonal communication paid for by an advertised sponsor using mass media to persuade or inform an audience.  
  Personal selling  
 Direct marketing  
 Market communication  
 Direct selling  
@ Advertising 

   _____ is the degree to which the target market will see an advertising message in a specific medium.  
@  Advertising exposure  
 Communications feedback  
 Consumer perception  
 Medium responsiveness  
 Advertising reliability

Question 5    Multiple Choice  0 of 1 point(s)    
  A 40-year-old woman and a 15-year-old boy would decode the ad for Maybelline age minimizing makeup differently because:  
 Your Answer:  [None Given]  
Correct Answer:  the subject of the message is not personally relevant to both  
 


  Question 6    Multiple Choice  0 of 1 point(s)    
  To set its promotion budget, a manufacturer of neon signs decided one of its communication goals would be to create a brand identity for a product that is typically perceived as a commodity. Once the goal was set, the company allocated $350,000 to accomplish this goal. Which of the following budgeting methods is this company using?  
 Your Answer:  [None Given]  
Correct Answer:  objective-task method  
 


  Question 7    Multiple Choice  0 of 1 point(s)    
  Glasurit is a type of paint made specifically for use on cars. An ad in Motor Trend magazine advising consumers to request their body shops use Glasurit paint is an example of how a company uses:  
 Your Answer:  [None Given]  
Correct Answer:  a pull strategy  
 


  Question 8    Multiple Choice  0 of 1 point(s)    
  According to the communication model, _____ is the process of translating an idea into a form of communication that will convey the desired meaning.  
 Your Answer:  [None Given]  
Correct Answer:  encoding  
 


  Question 9    Multiple Choice  0 of 1 point(s)    
  The ad for Maybelline age minimizing makeup in Ladies' Home Journal magazine featured actress Melina Kanakaredes and offered readers a $1-off coupon if they would try the new makeup. In terms of the communication model, the source of this ad is:  
 Your Answer:  [None Given]  
Correct Answer:  Maybelline and its advertising agency  
 


  Question 10    Multiple Choice  0 of 1 point(s)    
  Commercials for Pier 1 Imports retail stores show a television actress approaching obviously-stressed professional women between the ages of 25 and 40 and advising them to "get in touch with their senses" by visiting their nearest Pier 1 store. The end of the commercials shows the much-relaxed women leaving a Pier 1 store with all their great purchases. What ad format is Pier 1 using?  
 Your Answer:  [None Given]  
Correct Answer:  lifestyle  
 

Question 9    Multiple Choice  0 of 1 point(s)    
  An ad for Ford did not advertise any cars, but instead explained the meaning of the company's slogan, "no boundaries." This ad is an example of:  
 Your Answer:  [None Given]  
Correct Answer:  institutional advertising  
 


  Question 10    Multiple Choice  0 of 1 point(s)    
  Copy testing:  
 Your Answer:  [None Given]  
Correct Answer:  measures ad effectiveness  
 
Question 1    Multiple Choice  0 of 1 point(s)    
  An ad for a new prescription drug to protect users from a second heart attack or stroke shows an elderly man fishing and states that if you don't use this medication, another heart attack or another stroke may sneak up on you. This is an example of a(n) _____ appeal.  
 Your Answer:  [None Given]  
Correct Answer:  fear  
 


  Question 2    Multiple Choice  0 of 1 point(s)    
  In addition to advertising in teen-oriented magazines to market its new Focus disposable contact lenses, CIBA Vision produced an interactive CD-ROM with games that demonstrated the importance of depth perception, peripheral vision, and eye-hand coordination to safe driving and gave it free to schools to use in their driving courses. The interactive CD-ROM received positive publicity in many newspapers as well as educational journals. CIBA Vision sent e-mails and direct mail pieces to high school driving teachers offering them as many copies of the program as needed. When the students finished the program, they were urged to visit the company Web site for free promotional items to learn about Focus contact lenses. From this information, it would seem that CIBA Vision has adopted a(n) _____ philosophy.  
 Your Answer:  [None Given]  
Correct Answer:  integrated marketing communications  
 


  Question 3    Multiple Choice  0 of 1 point(s)    
  Sports drinks are in the maturity stage of their product life cycle. This means promotion for sports drinks:  
 Your Answer:  [None Given]  
Correct Answer:  concentrates on persuading people to switch brands  
 


  Question 4    Multiple Choice  0 of 1 point(s)    
  For its clients, a full-service agency can:  
 Your Answer:  [None Given]  
Correct Answer:  do all of the above  
 


  Question 5    Multiple Choice  0 of 1 point(s)    
  An ad for Post Selects brand cereal is an example of:  
 Your Answer:  [None Given]  
Correct Answer:  product advertising  
 


  Question 6    Multiple Choice  0 of 1 point(s)    
  According to the information presented in class, which of the following advertising media attracted the most money in 2003?  
 Your Answer:  [None Given]  
Correct Answer:  Newspapers  
 


  Question 7    Multiple Choice  0 of 1 point(s)    
  Jell-O ads contain the phrase, "Make some magic." All Ace Hardware store ads contain the statement, "Life is an ongoing project." The American Dairy Farmers run ads that say, "Ahh, the power of cheese." These are all examples of:  
 Your Answer:  [None Given]  
Correct Answer:  slogans  
 


  Question 8    Multiple Choice  0 of 1 point(s)    
  The _____ for Ford Motor Company included a series of ads on network television, on cable television, and in print ads, billboards, as well as signs for Ford dealers to post in their showroom windows.  
 Your Answer:  [None Given]  
Correct Answer:  advertising campaign  
 


  Question 9    Multiple Choice  0 of 1 point(s)    
  To show its toughness, Bounty paper towels uses ads which show how strong the towel is even wet and others which show how much more the towel can pick up than other brands. What attention-getting device is being used in the Bounty ads?  
 Your Answer:  [None Given]  
Correct Answer:  demonstration  
 


  Question 10    Multiple Choice  0 of 1 point(s)    
  Cosmetics that do not need to be touched up during the day but are able to last from morning until late at night is a product that is in the growth stage of its product life cycle. Therefore, the promotion mix for such products:  
 Your Answer:  [None Given]  
Correct Answer:  shows an increase in the use of advertising  
 

  A company that wanted its sales force to see a direct connection between their performance and how well they were paid would most likely use which of the following compensation plans?  
  objective-and-task plan  
 straight salary plan  
 quota-bonus plan  
@ straight commission plan  
 straight draw plan  

 A decade ago tourists in downtown Nashville, the capital of Tennessee, were harassed by panhandlers every few steps. The city was dirty, and the nightlife was limited in spite of the fact Nashville was the home to many entertainers. Nashville has cleaned its city and made visitors feel safe and welcomed. An article that ran in Southern Living magazine (targeted to people living in the Southeastern U.S.) invited people to come and enjoy its new entertainment venues. The objective of this article was to:  
  personalize the city's greetings to new visitors  
@ enhance the city's image  
 introduce a new product to consumers  
 influence government legislation  
 call attention to the Tennessean state government's involvement with the city 

   A loan company that wanted to build awareness and reinforce its image could do so by giving out _______________ with the name of the company on them.  
@  promotional products  
 brand reinforcers  
 incentive promotion pieces  
 samples  
 point-of-purchase displays  

A "rainmaker" is the member of a sales team responsible for developing new customers. Which of the following personality types is most likely to be the rainmaker?  
  The grouch.  
 The analyst.  
 The detail person.  
@ The schmoozer.  
 the sales manager.  


According to your instructor, what is the fundamental premise behind guerilla marketing?  
  The market leader cannot engage in guerilla marketing, everyone else in the industry has to engage in guerilla marketing.  
 Guerilla marketing is the result of the changes in advertising created by the Internet.  
@ By taking creative approaches to advertising as part of the promotional mix, you can leverage the effectiveness of your advertising budget.  
 In order to use guerilla marketing effectively, you need to have a large advertising budget. 

   According to your instructor, when is it appropriate to ask for the order when closing a sales call.  
  When the customer has expressed an interest in buying your product.  
 When the customer wants more time to think about it.  
 When the customer has hinted that she probably won't buy your product  
 When you are sure that the customer will say no.  
@ All of the above.  



   According to your instructor, which of the following techniques is the best way to maintain a relationship with a customer?  
  Send a Christmas card at Christmas.  
 Send a Holiday greeting card at Christmas.  
@ Send a birthday card on the customer's birthday.  
 Send an Easter card at Easter.  
 Send a "Wish you were here" card from Hawaii while you are on vacation. 

According to your instructor, which stage of the generic sales call is an inexperienced salesperson most likely to omit?  
  Prequalify the customer  
@ Establish rapport  
 Uncover needs  
 Create solutions  
 Close the sale 

  According to your instructor, why is it important to have information about the people who lead an organization in a press kit?  
  Because leaders have big egos and this is one way to make them feel important.  
@ Because you can include information about leaders without having to pay royalties.  
 Because people are interested in knowing about the people who lead an organization.  
 Because the more people featured in a press kit, the more favorably analysts will value the company's stock.  

 According to your instructor, why is it that marketers adopting the shotgun approach to sales can be perceived as obnoxious?  
  Because shotgun marketers use the "hard sell" approach.  
@ Because shotgun marketers may consider a campaign successful even if only a very small percentage of the people contacted actually buy the product.  
 Because the highly targeted lists used by shotgun marketers can be perceived as an invasion of privacy.  
 Because shotgun marketers contact customers several times before accepting "no" as an answer. 


   According to the material presented in class, which of the following is a selling situation?  
  The sales clerk in a jewelry store helps a man decide on an appropriate engagement ring.  
 The man asks his girlfriend to marry him.  
 The girlfriend asks for more time to think about it.  
 The man asks his marketing professor to accept a late assignment because he had personal issues that kept him from completing it on time.  
@ All of the above are selling situations.  


After salesperson Danny O'Reilly has made a sale, he asks his customer if she knows of anyone else who might also be interested in buying his products. Referrals are one method used in:  
@  prospecting  
 cold calling  
 researching the pre-approach  
 niche marketing  
 determining potential objections 

  After taking a sales job at Americhem Plastics Corp., Melissa Hertz attended company-sponsored classes where she learned about the plastics she would be selling, how they were developed, potential uses for the plastics produced by the company as well as basic selling skills. She was surprised to find several veteran salespeople also attending some of her classes. Hertz should have learned from this experience:  
  that all industries have the same requirements for their salespeople  
@ the importance of sales training-no matter how experienced the salesperson  
 that she could get paid for signing up for additional classes  
 that her company was more interested in preparation than execution  
 that she had taken a job in a reactive industry  

   An office supply store that pays a discounted price when it orders more than twelve metal filing cabinets is receiving a:  
  merchandise allowance  
 bonus pack  
@ cash allowance  
 promotional reinforcer  
 bundled promotion  


   At the sales presentation for EFTPS software for making the payment of all federal taxes secure, easy, and fast, the _______________ demonstrated how payments could be set up twelve months in advance and how easy it was to view payment history.  
  order taker  
 proactive salesperson  
 order getter  
 relationship manager  
@ technical specialist 

Bristol-Meyers Squibb Co. was sued because it filed an additional patent on its anxiety treatment BuSpar, which kept generic competition off the market for months. The lawsuit was the subject of a television news report and several newspaper articles. To counter this _______________, Bristol Meyers would have used _______________ to preserve its corporate image.  
  macroenvironmental weakness, publicity  
 negative publicity; consumer promotions  
 positive publicity; personal selling  
@ negative publicity; public relations activities  
 macroenvironmental threat; internal marketing  
 


 During the _______________ stage of the creative selling process, a salesperson might learn that the prospect is an avid soccer fan, a collector of Civil War firearms, and a father of three teenagers.  
  cold call  
 qualifying  
 sales presentation  
@ pre-approach  
 feedback  

 Each salesperson has the responsibility for a set group of customers. This group of customers is referred to as the salesperson's:  
  buying center  
 selling center  
 sales team  
@ sales territory  
 sales chain  

  Frasier, Sex and the City, and West Wing are television shows that have showed actors drinking a cup of Starbucks coffee. Starbucks coffee used which consumer promotion?  
  a point-of-view promotion  
@ product placement  
 media promotion  
 push money  
 promotional product  


   In which stage of the creative selling process would the sales clerk say, "Will you being paying cash for this purchase?"  
  pre-approach  
@ closing  
 prospecting  
 bird dogging  
 handling an objection  

  In which type of a selling situation would the sale follow-up be least important?  
  proactive marketing  
@ transactional selling  
 market segregation  
 transformational selling  
 relationship selling  


Individual salesperson performance can be evaluated by:  
  measuring it against sales quotas for individual sales territories  
 determining the number of sales closed  
 calculating the dollar amount of net sales for individual sales territories  
 comparing the number of products sold in each sales territory  
@ any of the above methods 

 
Mark often wears a t-shirt given to him by the owner of Graham Advertising Agency, a company Mark does business with. The shirt has the advertising agency's logo on it as well as its address, phone number, and Web site. The shirt is an example of a(n):  
  incentive product  
 sample  
 bonus pack  
@ promotional product  
 cash discount 

 Martha Stewart Weddings magazine and the Puerto Rico Tourism Company used _______________ to show people that Puerto Rico was the perfect site for a wedding. The magazine and the tourism bureau sponsored a "The Prefect Wedding in Puerto Rico" sweepstakes to create interest in weddings and more specifically weddings in Puerto Rico.  
  push money  
 an incentive program  
@ a cross-promotion  
 situational marketing  
 product placement  

 MobilExxon distributed to newspapers and magazines an article describing how its research team had discovered a new drug that is instrumental in the fight to stop the spread of malaria. The article was an example of a(n):  
  impressions quantification  
 press package  
@ press release  
 lobbying package  
 image sponsorship



 PeopleSoft is enterprise application software that businesses can use to create more efficient relationships with suppliers, customers, and employees. To sell the entire software package, the company uses _______________, which includes a technical specialist, an accounting expert, a human resources expert, and a sales representative.  
  a selling center  
 relationship marketing  
@ team selling  
 a buying center  
 seminar selling 

Southern Security makes safes for storing valuables. When Ed sold safes for the company, he would deal with a prospect's concerns such as the cost of the safe and then ask, "Are you ready to purchase a safe for your family, or are there other concerns that we need to discuss?" When Ed asked this question, he was:  
@  using a last objection close  
 creating rapport  
 showing customer empathy  
 still engaged in the approach stage of the creative selling process  
 using a standing-room-only close

   Southern Security makes safes for storing valuables such as jewelry, firearms, and family records. When Ed sold safes for the company, he had to be prepared with a response when a prospect said, "I have no room for a safe," or "My children would never play with my rifle." In other words, Ed had to:  
  prepare several different closings  
 use a canned sales presentation  
 treat each sale as a transaction  
 avoid cold calling  
@ know how to deal with common objections 

The basic measurement tool of public relations efforts is:  
  concept testing  
 impression counts  
 awareness and preference studies  
 SWOT analyses  
@ the counting of press clippings 

 The basic rule of good public relations is:  
  "Don't put all your eggs in one basket."  
 "A rolling stone gathers no moss."  
@ "Do something good, then talk about it."  
 "Let a smile be your umbrella."  
 "The early bird gets the worm." 

 The coupon for a free Blimpie 6-inch sub sandwich with the purchase of a 6-inch sub of equal or greater value and a 32-ounce fountain drink is an example of a:  
  trade promotion  
 promotional product  
 specialty pack  
 premium  
@ consumer promotion 


 The fact that the Red Baron Gallery has provided funding to Georgia public television to help pay for the rights to broadcast The Antiques Roadshow and is credited at the beginning of the show for its financial support is an example of:  
  image advertising  
@ sponsorship  
 disintermediation  
 a sales promotion  
 a press release  

The free can of Gillette series shave gel Matt received when he purchased a Gillette Mach3 Turbo razor is an example of:  
  internal marketing  
 green marketing  
@ a sales promotion  
 cause marketing  
 market diversification

 The manufacturer of Charmin toilet tissue sent the Charmin bear (its brand mascot) to help refurbish restrooms at fairgrounds across the country. As part of this project, each restroom was to provide fairgoers with a clean oasis featuring aromatherapy, soothing music, constant maintenance, and premium Charmin Ultra toilet tissue. Newspaper articles about Charmin's role in making restrooms more inviting was an example of:  
  image management  
 brand placement  
 a push strategy  
 a sales promotion  
@ publicity 

 The manufacturer of Charmin toilet tissue sent the Charmin bear (its brand mascot) to help refurbish restrooms at fairgrounds across the country. As part of this project, each restroom was to provide fairgoers with a clean oasis featuring aromatherapy, soothing music, constant maintenance, and premium Charmin Ultra toilet tissue. The use of Charmin in this clean-up effort is an example of:  
  sales promotion  
@ public relations  
 cause marketing  
 personal selling  
 image management  

 The most common way for public relations specialists to communicate is through the use of a(n):  
  impressions quantification  
 press package  
@ press release  
 lobbying package  
 image sponsorship 

  The salesperson for Americhem Plastics Corp. believes in winning, keeping, and developing her customers. In other words, she engages in:  
  proactive marketing  
 transactional selling  
 market segregation  
 transformational selling  
@ relationship selling  

   The _______________ promotes the firm and tries to stimulate demand for a product but does not actually complete a sale.  
  order taker  
@ missionary salesperson  
 order getter  
 relationship manager  
 purchasing agent  

 There are three steps involved in creating a public relations campaign. Step 1 is to:  
@ develop objectives  
 conduct a SWOT analysis  
 create a mission statement  
 recruit and train public relations specialists  
 test parts of the proposed PR campaign  
 

To sell _______________, a salesperson would more than likely go through a systematic series of steps called the creative selling process.  
  kites, balls, and other beach toys  
 fresh produce  
 t-shirts at a rock concert  
@ a $2.5 million printing press  
 consumer cleaning products 

 What are the four stages of the generic sales call discussed in class?  
  Prequalify the customer, uncover needs, develop solutions, close the sale.  
 Prequalify the customer, create desire, call for action, close the sale.  
 Establish rapport, uncover needs, call for action, close the sale.  
 Establish rapport, create desire, develop solutions, close the sale.  
@ Establish rapport, uncover needs, develop solutions, close the sale. 

What is the most important skill a salesperson needs to employ during the "uncover needs" stage of the selling process?  
  The ability to establish rapport  
@ The ability to listen  
 The ability to close  
 The ability to lead the conversation 


  What type of compensation plan would a company be most likely to use for its missionary salespeople?  
  objective-and-task plan  
@ straight salary plan  
 quota-commission plan  
 straight commission plan  
 straight draw plan 

 What type of sales promotion would be used with a push strategy in which the goal is to push a product through the channel of distribution so it is available to consumers?  
  coupons  
@ cash and promotional allowances  
 rebates  
 bonus packs  
 all of the above

 When Antoine purchased the Barbie dream house for his daughter, he mailed in his receipt showing how much he had paid for the toy and the proof of purchase form off the carton in which the dream house came to the manufacturer, Mattel. In six weeks, he received a check from Mattel for $5. Mattel used a _______________ to refund part of the purchase price.  
  cash allowance  
 premium  
 promotional allowance  
@ rebate  
 trade promotion 


- When Kennett called Southern Securityy to buy a safe in which to store his gun collection, he talked to a(n) _______________ who performed all the activities needed to process the transaction.  
@  order taker  
 missionary salesperson  
 order getter  
 relationship manager  
 technical specialist  


  When selling waterless cookware, Ron was given a script and instructed not to deviate from his script when making his sales calls. From this information, you know Ron was using a _______________ sales presentation.  
  FAB  
@ canned  
 simulated  
 creative  
 response-stimulus  

  When Tarra worked as a telemarketer, she would start her call with the same statement, "Our environmental specialists will be in your area tomorrow. Would you like them to tell you how you can save money?" In which part of the creative selling process was Tarra engaged?  
@  prospecting  
 e-commerce  
 researching the pre-approach  
 niche marketing  
 determining potential objections 


Which of the following is a difference between inside selling and outside selling?  
  The job may involve order taking  
@ The job may involve extensive travel  
 The job may involve consultation  
 The job may involve performance based compensation  
 The job requires asking for the order.  

 Which of the following is a common objective for public relations activities?  
  the introduction of new products to manufacturers  
 to enhance a city, region, or country  
 the introduction of new products to consumers  
 to influence government legislation  
@ all of the above 

 Which of the following is a reason why personal selling typically plays a limited role in most companies' promotion mix?  
  its personal nature  
 the difficulty associated with locating prospects  
@ its cost  
 the advantages offered by telemarketing  
 all of the above  

Which of the following is an example of a behavior goal a sales manager might set for her sales force?  
  Each salesperson must call on four customers each day.  
 Each salesperson must spend one night a week on the road.  
 Each salesperson must make ten cold calls each week.  
 Each salesperson must follow-up each sale within 24 hours of closing the sale.  
@ All of the above are examples of behavior goals a sales manager might set for her sales force. 

 Which of the following is an example of a discount promotion given to members of the distribution channel?  
  push money  
 incentive products  
 promotional products  
 samples  
@ merchandise allowance 

  Which of the following is an example of a sales promotion?  
@  a coupon for $5 off a visit to an amusement park  
 a reduced price on a movie ticket because you are willing to attend a matinee  
 a news release encouraging voters to support a new bond issue for a local elementary school  
 a listing of all the television programs to be shown by Charter Cable in the Rome area the week of March 16  
 a plan to increase the sale of ice cream by adding two new flavors  


  Which of the following is one of the three primary roles for a salesperson as discussed in class?  
  Training.  
 Performance-based compensation.  
 The AIDA model.  
@ New customer development.  
 Determining sales territories. 

 Which of the following statements about relationship selling is true?  
  Relationship selling primarily uses hard-sell tactics.  
 A salesperson selling new tires to tourists who are stranded in his community will more than likely use relationship selling.  
@ With relationship selling, the salesperson tries to develop a mutually satisfying relationship with customers.  
 People who engage in relationship selling do not prospect for new customers.  
 All of the above statements about relationship selling are true.  


Which of the following types of sales promotions can be targeted to consumers and to members of the distribution channel?  
  push money  
 trade shows  
 bonus packs  
@ contests and sweepstakes  
 coupons  

 Which of the following types of sales promotions would never be offered to a final consumer?  
  coupons  
@ push money  
 bonus packs  
 contests and sweepstakes  
 samples  

Why are video news releases a controversial issue in broadcast journalism?  
  Because companies have started asking television stations to pay royalties if they air the footage, increasing the cost of producing the news.  
 Because many television stations do not have the legal staffs needed to secure permission to edit copyrighted material.  
 Because there is no way for the television station to know whether the actors in the video were members of AFTRA, the union for television actors.  
@ Because television stations with low budgets might choose to consider a video press release to be news simply because there is some professionally produced footage available for free. 

 Why do many organizations rely heavily on personal selling?  
  to keep their products from moving from the introduction stage to the growth stage of their product life cycle  
 to make more efficient use of the promotional mix  
 to quickly move a product to the last stage of its product life cycle  
 to avoid use of the "personal touch"  
@ to more efficiently sell complex and/or expensive products  


Why is it an advantage to have the material in your press kit "professional quality"?  
@  Because it increases the chances of a busy reporter using your material "as is".  
 Because reporters prefer not to have to write their own stories.  
 Because it is less expensive to have things professionally produced than it is to use amateurs for the work.  
 Because there is less chance of having the news story "slanted" in your favor 

 Why is it that many companies continue to use the "hard sell" approach to sales to this day?  
@  Because there are many situations where the hard sell approach is more effective than relationship selling.  
 Because relationship selling is primarily used in business to business marketing.  
 Because you can't use performance based compensation for relational selling.  
 Because it is easier to use behavioral measures of performance with the hard sell approach. 



 Without prior introduction or arrangement, telemarketers telephone your home and business offering to sell you a variety of products. Telemarketers engage in what form of prospecting?  
  spamming  
@ cold calling  
 bird dogging  
 bridge spanning  
 referral chain 

 

  _______________ are useful if a firm has an immediate objective, such as bolstering sales for a brand quickly or encouraging customers to try a new product.  
  Public relations activities  
@ Sales promotions  
 Billboard advertisements  
 Sponsorships  
 All of the above  

 _______________ attempts to influence consumers while they are in the store by catching their attention with displays or signs.  
@  A point-of-purchase promotion  
 Brand placement  
 A trade show  
 Push money  
 Transit signage  

  _______________ attempts to influence the attitudes and perceptions of consumers, stockholders, and others towards companies, brands, politicians, celebrities, not-for-profit organizations, etc.  
  Sales promotion  
@ Public relations  
 Cause marketing  
 Personal selling  
 Image management 


 _______________ is a sales technique that focuses on making an immediate sale with little or no attempt to develop a relationship with the customer.  
  Proactive marketing  
@ Transactional selling  
 Market segregation  
 Transformational selling  
 Relationship selling 

 _______________ is the process of planning, implementing, and controlling the personal selling function.  
  Operations management  
 Marketing management  
 Promotion management  
@ Sales management  
 The management of mass marketing activities  

 _______________ occurs when a company representative contacts a prospect directly regarding a good or service.  
  Cross-promotion  
 Sales management  
 Mass selling  
@ Personal selling  
 Prospecting  

 Question 1    Multiple Choice  1 of 1 point(s)    
  PeopleSoft is enterprise application software that businesses can use to create more efficient relationships with suppliers, customers, and employees. To sell the entire software package, the company uses _______________, which includes a technical specialist, an accounting expert, a human resources expert, and a sales representative.  
 Your Answer:  team selling  
Correct Answer:  team selling  
 


  Question 2    Multiple Choice  0 of 1 point(s)    
  In terms of the promotion mix, telemarketing is most closely related to:  
 Your Answer:  [None Given]  
Correct Answer:  personal selling  
 


  Question 3    Multiple Choice  0 of 1 point(s)    
  Which of the following is an example of a qualitative measure a sales manager could use to evaluate his or her salespeople?  
 Your Answer:  [None Given]  
Correct Answer:  the salesperson's product knowledge  
 


  Question 4    Multiple Choice  0 of 1 point(s)    
  _______________ occurs when a company representative contacts a prospect directly regarding a good or service.  
 Your Answer:  [None Given]  
Correct Answer:  Personal selling  
 


  Question 5    Multiple Choice  0 of 1 point(s)    
  When Kennett called Southern Security to buy a safe in which to store his gun collection, he talked to a(n) _______________ who performed all the activities needed to process the transaction.  
 Your Answer:  [None Given]  
Correct Answer:  order taker  
 


  Question 6    Multiple Choice  0 of 1 point(s)    
  Individual salesperson performance can be evaluated by:  
 Your Answer:  [None Given]  
Correct Answer:  any of the above methods  
 


  Question 7    Multiple Choice  0 of 1 point(s)    
  Which of the following is a difference between inside selling and outside selling?  
 Your Answer:  [None Given]  
Correct Answer:  The job may involve extensive travel  
 


  Question 8    Multiple Choice  0 of 1 point(s)    
  The manufacturer of Charmin toilet tissue sent the Charmin bear (its brand mascot) to help refurbish restrooms at fairgrounds across the country. As part of this project, each restroom was to provide fairgoers with a clean oasis featuring aromatherapy, soothing music, constant maintenance, and premium Charmin Ultra toilet tissue. Newspaper articles about Charmin's role in making restrooms more inviting was an example of:  
 Your Answer:  [None Given]  
Correct Answer:  publicity  
 


  Question 9    Multiple Choice  0 of 1 point(s)    
  What are the four stages of the generic sales call discussed in class?  
 Your Answer:  [None Given]  
Correct Answer:  Establish rapport, uncover needs, develop solutions, close the sale.  
 


  Question 10    Multiple Choice  0 of 1 point(s)    
  According to your instructor, which stage of the generic sales call is an inexperienced salesperson most likely to omit?  
 Your Answer:  [None Given]  
Correct Answer:  Establish rapport  

Question 1    Multiple Choice  0 of 1 point(s)    
  According to you instructor, why are sales teams becoming more popular in personal selling?  
 Your Answer:  [None Given]  
Correct Answer:  Because it increases the number of potential employees for a sales organization.  
 


  Question 2    Multiple Choice  0 of 1 point(s)    
  A loan company that wanted to build awareness and reinforce its image could do so by giving out _______________ with the name of the company on them.  
 Your Answer:  [None Given]  
Correct Answer:  promotional products  
 


  Question 3    Multiple Choice  0 of 1 point(s)    
  The coupon for a free Blimpie 6-inch sub sandwich with the purchase of a 6-inch sub of equal or greater value and a 32-ounce fountain drink is an example of a:  
 Your Answer:  [None Given]  
Correct Answer:  consumer promotion  
 


  Question 4    Multiple Choice  0 of 1 point(s)    
  In which stage of the creative selling process would the sales clerk say, "Will you being paying cash for this purchase?"  
 Your Answer:  [None Given]  
Correct Answer:  closing  
 


  Question 5    Multiple Choice  0 of 1 point(s)    
  A company that wanted its sales force to see a direct connection between their performance and how well they were paid would most likely use which of the following compensation plans?  
 Your Answer:  [None Given]  
Correct Answer:  straight commission plan  
 


  Question 6    Multiple Choice  0 of 1 point(s)    
  When Tarra worked as a telemarketer, she would start her call with the same statement, "Our environmental specialists will be in your area tomorrow. Would you like them to tell you how you can save money?" In which part of the creative selling process was Tarra engaged?  
 Your Answer:  [None Given]  
Correct Answer:  prospecting  
 


  Question 7    Multiple Choice  0 of 1 point(s)    
  The basic rule of good public relations is:  
 Your Answer:  [None Given]  
Correct Answer:  "Do something good, then talk about it."  
 


  Question 8    Multiple Choice  0 of 1 point(s)    
  Which of the following types of sales promotions would never be offered to a final consumer?  
 Your Answer:  [None Given]  
Correct Answer:  push money  
 


  Question 9    Multiple Choice  0 of 1 point(s)    
  Individual salesperson performance can be evaluated by:  
 Your Answer:  [None Given]  
Correct Answer:  any of the above methods  
 


  Question 10    Multiple Choice  0 of 1 point(s)    
  Why is it that many companies continue to use the "hard sell" approach to sales to this day?  
 Your Answer:  [None Given]  
Correct Answer:  Because there are many situations where the hard sell approach is more effective than relationship selling.  
 


Question 1    Multiple Choice  1 of 1 point(s)    
  According to you instructor, why are sales teams becoming more popular in personal selling?  
 Your Answer:  Because it increases the number of potential employees for a sales organization.  
Correct Answer:  Because it increases the number of potential employees for a sales organization.  
 


  Question 2    Multiple Choice  0 of 1 point(s)    
  When Virginia purchased a bottle of Simple Green cleaning liquid, she also received a free bottle of Simple Green hand soap. The two items were shrink-wrapped together to create a:  
 Your Answer:  [None Given]  
Correct Answer:  bonus pack  
 


  Question 3    Multiple Choice  0 of 1 point(s)    
  The most common way for public relations specialists to communicate is through the use of a(n):  
 Your Answer:  [None Given]  
Correct Answer:  press release  
 


  Question 4    Multiple Choice  0 of 1 point(s)    
  _______________ is the process of planning, implementing, and controlling the personal selling function.  
 Your Answer:  [None Given]  
Correct Answer:  Sales management  
 


  Question 5    Multiple Choice  0 of 1 point(s)    
  Bristol-Meyers Squibb Co. was sued because it filed an additional patent on its anxiety treatment BuSpar, which kept generic competition off the market for months. The lawsuit was the subject of a television news report and several newspaper articles. To counter this _______________, Bristol Meyers would have used _______________ to preserve its corporate image.  
 Your Answer:  [None Given]  
Correct Answer:  negative publicity; public relations activities  
 


  Question 6    Multiple Choice  0 of 1 point(s)    
  The first step in the creative selling process is:  
 Your Answer:  [None Given]  
Correct Answer:  prospecting and qualifying  
 


  Question 7    Multiple Choice  0 of 1 point(s)    
  Martha Stewart Weddings magazine and the Puerto Rico Tourism Company used _______________ to show people that Puerto Rico was the perfect site for a wedding. The magazine and the tourism bureau sponsored a "The Prefect Wedding in Puerto Rico" sweepstakes to create interest in weddings and more specifically weddings in Puerto Rico.  
 Your Answer:  [None Given]  
Correct Answer:  a cross-promotion  
 


  Question 8    Multiple Choice  0 of 1 point(s)    
  Which of the following is a common objective for public relations activities?  
 Your Answer:  [None Given]  
Correct Answer:  all of the above  
 


  Question 9    Multiple Choice  0 of 1 point(s)    
  Which of the following is an example of a sales promotion?  
 Your Answer:  [None Given]  
Correct Answer:  a coupon for $5 off a visit to an amusement park  
 


  Question 10    Multiple Choice  0 of 1 point(s)    
  In terms of the promotion mix, telemarketing is most closely related to:  
 Your Answer:  [None Given]  
Correct Answer:  personal selling  
 

Question 4    Multiple Choice  0 of 1 point(s)    
  As Humphrey searched through the supermarket to find marshmallows, he was enjoying the music being played by the store. Then he heard an announcer saying, "Visit our deli department right now. Buy some just-cooked dinner rolls for your supper tonight." What Humphrey heard is an example of:  
 Your Answer:  [None Given]  
Correct Answer:  a point-of-purchase promotion  
 


  Question 5    Multiple Choice  0 of 1 point(s)    
  To promote her realty company, a realtor purchased a small moving van and had her logo painted on the truck as well as a message that read, "Buy or sell a home with me, and use this truck for your move free." The realtor was able to use this mobile ad to "ambush" potential customers by touring neighborhoods where homes were changing hands. The realtor used:  
 Your Answer:  [None Given]  
Correct Answer:  guerrilla marketing  
 


  Question 6    Multiple Choice  0 of 1 point(s)    
  There are three steps involved in creating a public relations campaign. Step 1 is to:  
 Your Answer:  [None Given]  
Correct Answer:  develop objectives  
 


  Question 7    Multiple Choice  0 of 1 point(s)    
  The salesperson for Americhem Plastics Corp. believes in winning, keeping, and developing her customers. In other words, she engages in:  
 Your Answer:  [None Given]  
Correct Answer:  relationship selling  
 


  Question 8    Multiple Choice  0 of 1 point(s)    
  According to your instructor, what is the fundamental premise behind guerilla marketing?  
 Your Answer:  [None Given]  
Correct Answer:  By taking creative approaches to advertising as part of the promotional mix, you can leverage the effectiveness of your advertising budget.  
 


  Question 9    Multiple Choice  0 of 1 point(s)    
  Why is it that many companies continue to use the "hard sell" approach to sales to this day?  
 Your Answer:  [None Given]  
Correct Answer:  Because there are many situations where the hard sell approach is more effective than relationship selling.  
 


  Question 10    Multiple Choice  0 of 1 point(s)    
  According to your instructor, when is it appropriate to ask for the order when closing a sales call.  
 Your Answer:  [None Given]  
Correct Answer:  All of the above.  
 

Question 1    Multiple Choice  0 of 1 point(s)    
  Like advertising, sales promotions for athletic shoes can target:  
 Your Answer:  [None Given]  
Correct Answer:  all of the above  
 


  Question 2    Multiple Choice  0 of 1 point(s)    
  After taking a sales job at Americhem Plastics Corp., Melissa Hertz attended company-sponsored classes where she learned about the plastics she would be selling, how they were developed, potential uses for the plastics produced by the company as well as basic selling skills. She was surprised to find several veteran salespeople also attending some of her classes. Hertz should have learned from this experience:  
 Your Answer:  [None Given]  
Correct Answer:  the importance of sales training-no matter how experienced the salesperson  
 


  Question 3    Multiple Choice  0 of 1 point(s)    
  What is the most important skill a salesperson needs to employ during the "uncover needs" stage of the selling process?  
 Your Answer:  [None Given]  
Correct Answer:  The ability to listen  
 


  Question 4    Multiple Choice  0 of 1 point(s)    
  According to your instructor, which of the following techniques is the best way to maintain a relationship with a customer?  
 Your Answer:  [None Given]  
Correct Answer:  Send a birthday card on the customer's birthday.  
 


  Question 5    Multiple Choice  0 of 1 point(s)    
  The basic rule of good public relations is:  
 Your Answer:  [None Given]  
Correct Answer:  "Do something good, then talk about it."  
 


  Question 6    Multiple Choice  0 of 1 point(s)    
  To sell _______________, a salesperson would more than likely go through a systematic series of steps called the creative selling process.  
 Your Answer:  [None Given]  
Correct Answer:  a $2.5 million printing press  
 


  Question 7    Multiple Choice  0 of 1 point(s)    
  When Kennett called Southern Security to buy a safe in which to store his gun collection, he talked to a(n) _______________ who performed all the activities needed to process the transaction.  
 Your Answer:  [None Given]  
Correct Answer:  order taker  
 


  Question 8    Multiple Choice  0 of 1 point(s)    
  Without prior introduction or arrangement, telemarketers telephone your home and business offering to sell you a variety of products. Telemarketers engage in what form of prospecting?  
 Your Answer:  [None Given]  
Correct Answer:  cold calling  
 


  Question 9    Multiple Choice  0 of 1 point(s)    
  Keely was invited to examine a free copy of a new magazine called Coastal Living. If she did not enjoy the magazine, she could write "Refused" on the invoice for her subscription that would arrive about three weeks after the free magazine. The magazine's publisher was using:  
 Your Answer:  [None Given]  
Correct Answer:  sampling  
 


  Question 10    Multiple Choice  0 of 1 point(s)    
  The manufacturer of Charmin toilet tissue sent the Charmin bear (its brand mascot) to help refurbish restrooms at fairgrounds across the country. As part of this project, each restroom was to provide fairgoers with a clean oasis featuring aromatherapy, soothing music, constant maintenance, and premium Charmin Ultra toilet tissue. The use of Charmin in this clean-up effort is an example of:  
 Your Answer:  [None Given]  
Correct Answer:  public relations  
 


Question 1    Multiple Choice  0 of 1 point(s)    
  As the salesperson entered the prospect's office, the salesperson extended his hand and said, "Your old college roommate Tiara Johns suggested I call on you." This action would occur in which stage of the creative selling process?  
 Your Answer:  [None Given]  
Correct Answer:  establishing rapport  
 


  Question 2    Multiple Choice  0 of 1 point(s)    
  According to your instructor, when is it appropriate to ask for the order when closing a sales call.  
 Your Answer:  [None Given]  
Correct Answer:  All of the above.  
 


  Question 3    Multiple Choice  0 of 1 point(s)    
  When Antoine purchased the Barbie dream house for his daughter, he mailed in his receipt showing how much he had paid for the toy and the proof of purchase form off the carton in which the dream house came to the manufacturer, Mattel. In six weeks, he received a check from Mattel for $5. Mattel used a _______________ to refund part of the purchase price.  
 Your Answer:  [None Given]  
Correct Answer:  rebate  
 


  Question 4    Multiple Choice  0 of 1 point(s)    
  Which of the following types of sales promotions can be targeted to consumers and to members of the distribution channel?  
 Your Answer:  [None Given]  
Correct Answer:  contests and sweepstakes  
 


  Question 5    Multiple Choice  0 of 1 point(s)    
  When Kennett called Southern Security to buy a safe in which to store his gun collection, he talked to a(n) _______________ who performed all the activities needed to process the transaction.  
 Your Answer:  [None Given]  
Correct Answer:  order taker  
 


  Question 6    Multiple Choice  0 of 1 point(s)    
  The manufacturer of Charmin toilet tissue sent the Charmin bear (its brand mascot) to help refurbish restrooms at fairgrounds across the country. As part of this project, each restroom was to provide fairgoers with a clean oasis featuring aromatherapy, soothing music, constant maintenance, and premium Charmin Ultra toilet tissue. Newspaper articles about Charmin's role in making restrooms more inviting was an example of:  
 Your Answer:  [None Given]  
Correct Answer:  publicity  
 


  Question 7    Multiple Choice  0 of 1 point(s)    
  Each salesperson has the responsibility for a set group of customers. This group of customers is referred to as the salesperson's:  
 Your Answer:  [None Given]  
Correct Answer:  sales territory  
 


  Question 8    Multiple Choice  0 of 1 point(s)    
  A decade ago tourists in downtown Nashville, the capital of Tennessee, were harassed by panhandlers every few steps. The city was dirty, and the nightlife was limited in spite of the fact Nashville was the home to many entertainers. Nashville has cleaned its city and made visitors feel safe and welcomed. An article that ran in Southern Living magazine (targeted to people living in the Southeastern U.S.) invited people to come and enjoy its new entertainment venues. The objective of this article was to:  
 Your Answer:  [None Given]  
Correct Answer:  enhance the city's image  
 


  Question 9    Multiple Choice  0 of 1 point(s)    
  There are three steps involved in creating a public relations campaign. Step 1 is to:  
 Your Answer:  [None Given]  
Correct Answer:  develop objectives  
 


  Question 10    Multiple Choice  0 of 1 point(s)    
  According to your instructor, what is the fundamental premise behind guerilla marketing?  
 Your Answer:  [None Given]  
Correct Answer:  By taking creative approaches to advertising as part of the promotional mix, you can leverage the effectiveness of your advertising budget.  
 
 A bookstore that sells coffee, pastries, and doughnuts as well as books is using a strategy of:  
  retail diversity  
@ scrambled merchandising  
 merchandise diversification  
 retail overutilization  
 merchandise dependency

A firm that is a channel leader has power relative to the other channel members. This power comes from:  
  the ability to control resources  
 the ability to let channel members handle its most profitable products  
 the ability to take profitable product lines away from intermediaries  
 legal authority granted by a contract  
@ all of the above 

 A retailer looking to locate a store in an area in which the needs of the population are not being adequately met by existing stores is looking for a(n) _______________ trade area.  
  overstored  
 secondary  
 saturated  
 primary  
@ understored

 A shoe department carries more than 55 different styles of white sandals. This is a description of the shoe department's:  
  merchandise breadth  
 retail diversity  
 store inventory depth  
@ merchandise depth  
 scrambled merchandising  

A store using an everyday low-pricing (EDLP) strategy:  
  uses its pricing strategy to create a differential advantage and will engage in a price war if necessary to maintain this advantage  
 has weekly sales  
@ sets prices between the list price suggested by the retailer and the deeply discounted prices offered by stores that compete on price alone  
 never experiences a stock-out  
 follows prices as specified by the Robinson-Patman Act

 A successful television show like FX's The Shield can only be seen on that network and on no other network. The production company that developed and created the series uses _____ distribution.  
  extensive  
 selective  
@ exclusive  
 relational  
 intensive

A _______________ is how a target market perceives the store.  
  store layout  
 merchandise assortment  
 market segmentation strategy  
 subliminal retail perception  
@ store image 

 According to your instructor, what is the goal of a channel of distribution?  
  To minimize channel conflict among members.  
 To create form utility.  
@ To create the most efficient way to move products from producer to consumer.  
 To create barriers to entry for international competitors.  
 To protect channel members from product liability claims. 

According to your instructor, what was the most significant technological innovation of the 20th century for retail stores?  
  The escalator.  
 Electricity.  
@ Air conditioning.  
 The printing press.  
 The Internet. 

 According to the text, which of the following products would be most effectively sold using direct selling?  
  clothes  
 pots and pans  
 baby furniture  
@ nutritional products  
 pet food 



 According to your instructor, why should we be paying attention to changes in logistics?  
  Because logistics are interesting, and understanding the principles will improve your quality of life.  
@ There are tremendous opportunities to make money by figuring out how to make shipments of physical products as efficient as the internet has made ordering them.  
 Because changes in physical distribution are so rare that it is critical to be alert for when they happen.  
 Because logistics is a relatively small expense when you take into consideration the entire marketing mix.  
 In order to protect against shortages and surpluses caused by "boom or bust" logistics.

 At some catalog retailers, employees on roller skates called pickers are handed a printout of the products a customer wants and then the pickers skate through large warehouses accumulating the desired products into one central location. The pickers are engaged in:  
  telemarketing  
 breakeven analysis  
@ materials handling  
 transportation  
 trend analyses  

Burger King, Holiday Inn, Avis, and H&R Block tax service are all franchises, which means they are also members of a _____ vertical marketing system (VMS).  
  administered  
 conventional  
 corporate  
 horizontal  
@ contractual 

 Cash-and-carry wholesalers:  
  are types of full-service merchant wholesalers  
 carry products to small business customer locations for their inspection and selection  
@ provide low-cost merchandise for retailers and industrial customers who are too small for other wholesalers' sales representatives to call on them  
 typically handle specialty items like magazines or health and beauty aids  
 typically sell bulk products like grain and coal  

 Channels of distribution:  
@  create time, place and ownership utility  
 create promotion, product, and place utility  
 create time, place, ownership, and form utility  
 do not create time or place utility  
 do not create any kind of utility 

Convenience stores:  
  primarily carry shopping goods  
 charge lower-than-normal prices for staple goods  
@ create time utility for their customers  
 create form, time, and place utility for their customers  
 are accurately described by all of the above  


 Distribution channels create efficiencies by reducing the number of transactions necessary for goods to flow from different manufacturers to large numbers of customers. This occurs in part through:  
  the creation of atmospherics  
@ creating assortments  
 breaking bulk  
 the absence of economies of scale  
 relationship management



 During the Civil War, sutlers set up their wagons full of merchandise on the edge of army camps and sold the soldiers items like stationary, shaving mirrors, and tobacco that the U.S. government did not supply in the soldier's rations. These sutlers were early:  
  e-tailers  
 disintermediaries  
 direct channels  
 demarketers  
@ retailers 

Eby-Brown is a business that buys cigarettes, candy, and gum from many different manufacturers and then resells the items to a variety of different stores. Eby-Brown owns the goods it sells and suffers loses if the products are stolen or become out-of-date. It has developed its own marketing strategy and is an example of a:  
  rack shipper  
 job racker  
@ merchant wholesaler  
 merchandise broker  
 merchandise agent 

 Eby-Brown is a business that buys cigarettes, candy, and gum from many different manufacturers and then resells the items to a variety of different stores. Eby-Brown is an example of a(n):  
@  independent intermediary  
 reverse marketer  
 functional provider  
 disintermediator  
 brokerage company 



Five years ago, the Paradise Surf Shop was the only surf-gear store catering to women. It carried everything the serious female surfer needed to ride the waves. It is an example of which form of retailer?  
@  specialty store  
 convenience store  
 warehouse club  
 hypermarket  
 mass merchandiser 

Five years ago, the Paradise Surf Shop was the only surf-gear store catering to women. It carried everything the serious female surfer needed to ride the waves. In terms of its merchandise assortment, it had a:  
  shallow and deep assortment  
@ narrow and deep assortment  
 shallow and broad assortment  
 broad and deep assortment


 Five years ago, the Paradise Surf Shop was the only surf-gear store catering to women. Its _______________ included surfboards smaller and lighter than those used by men.  
@  merchandise mix  
 scrambled merchandising strategy  
 market segmentation strategy  
 inventory classification  
 retail format

 Five years ago, the Paradise Surf Shop was the only surf-gear store catering to women. It carried everything the serious female surfer needed to ride the waves. The Paradise Surf Shop carried lightweight surfboards with flowers painted on them stacked around the wall of the store. It also had clean, well-lit dressing rooms for trying on wetsuits and knowledgeable staff who knew what a woman needed to succeed in an essentially male-dominated sport. These items together created its:  
  store layout  
 merchandise assortment  
 media plan  
 subliminal retail perception  
@ store image  

For which of the following products is the manufacturer most likely to want to use intensive distribution?  
  Herman Miller office furniture  
@ bottled water  
 computer memory chips  
 barbecue grills  
 needlepoint kits

Freestanding retailers:  
  benefit from the close proximity to stores selling similar merchandise  
 rely on spillover from other stores to attract traffic to the stores  
 are typically small entrepreneurial operations  
@ are located by themselves in separate buildings  
 are found anchoring larger shopping centers  

 Full-service merchant wholesalers:  
  often provide delivery, credit, and product-use assistance for customers  
 can carry general merchandise or specialize in one area  
 set the price they charge to customers  
 often have their own sales force to call on retailers  
@ are accurately described by all of the above 

Herb and Lajuanna Hicks have decided to market condiments typically found in Hispanic kitchens and not readily available in the United States. As innovative entrepreneurs, they will most likely start their operation in a:  
  large shopping mall  
 superregional center  
 power strip  
 freestanding store on the edge of a major mall  
@ kiosk or cart 

 In a _____ a single channel member controls channel behavior in order to avoid channel conflict and to create economies through size, eliminating duplication of services, and creating buying bargaining power.  
  functional organizational structure  
@ vertical marketing system  
 horizontal marketing system  
 mechanistic organizational structure  
 horizontal marketing system  

 In a(n) _______________, a master distributor recruits other people to become distributors. The master distributor sells the company's products to the people she entices to join and receives commissions on all the merchandise sold by the people she recruits.  
  party plan system  
@ multilevel network  
 integrated sales system  
 hierarchical sales network  
 integrated distribution channel 


  In terms of the basic types of retail locations, the traditional downtown business area found in most towns and cities is referred to as a:  
  market trade area  
@ central business district  
 shopping center  
 freestanding entity  
 power center 

In the _______________ stage of the retail life cycle, retail businesses become obsolete as newer ways of doing business emerge.  
@  decline  
 overexpansion  
 maturity  
 adult  
 gray marketing  

In the _______________ stage of the retail life cycle, the new retailer often is an aggressive entrepreneur who takes a unique approach to doing business by creating a differential advantage.  
  expansion  
@ introduction  
 extension  
 maturity  
 growth

 In which kind of retail store are you most likely to see a free-flow layout?  
  a warehouse store like Sam's Club  
@ a clothing specialty store like The Gap  
 a supermarket like Safeway  
 a superstore  
 a hypermarket 


Increasingly commonplace today is the appearance of two different franchise chains under the same roof, such as Del Taco and Mrs. Winner's chicken or Pizza-Hut and Baskin-Robbins ice cream. These would be examples of _____ marketing systems.  
  administered  
 conventional  
 corporate  
@ horizontal  
 contractual 


Manufacturers' agents are:  
  also called manufacturers' brokers  
 most often found working for large, international corporations  
@ compensated with commissions based on a percentage of what they sell  
 manufacturer-owned intermediaries  
 completely supervised by the manufacturer for which they sell

 Many manufacturers such as Weber (manufacturer of grills) and the VF Corporation (manufacturer of Bali, Playtex and Hanes brand products) sell some of their products directly to consumers and eliminate distribution levels from their channels. In other words, these manufacturers are engaged in:  
  cross-docking  
 piggybacking  
 demarketing  
@ disintermediation  
 reverse distribution

 Once an organization has developed its distribution objectives, it should next:  
  vote on who will be the channel leader  
 determine a distribution budget  
 define its target market and its market segmentation strategy  
@ evaluate the internal and external environments  
 determine the desired level of distribution intensity 

 Sales branches, sales offices, and manufacturers' showrooms:  
@  are all examples of manufacturer-owned intermediaries  
 all determine the marketing strategy for the products which they sell  
 are all examples of independent intermediaries  
 all carry inventory and take title to the inventory they carry  
 all perform the same functions as self-service wholesalers 


 Selective distribution strategy is most often used with which consumer product category?  
  convenience goods  
 supplies  
 specialty goods  
 unsought goods  
@ shopping goods 

The development of the automobile and the refrigerator allowed for the successful development of which retail form?  
  specialty stores  
 factory outlets  
 department stores  
 general merchandise discount stores  
@ supermarkets

The fact Burt can visit his local hardware store and buy one four-inch paint brush and not the case of 24 that the retailer had to buy is an example of how the channel of distribution facilitates:  
  the creation of atmospherics  
 risk taking  
@ breaking bulk  
 creating assortments  
 relationship management

 The fact Nita can visit her local supermarket and buy a dozen roses, a gallon of milk, two birthday cards, a bag of cat litter, and a pan for roasting a turkey illustrates how channels of distribution benefit consumers by:  
  the creation of atmospherics  
@ creating assortments  
 piggybacking  
 the absence of economies of scale  
 relationship management 

The fastest and most expensive mode of transportation is:  
  trucking  
 pipeline  
 railroad  
@ air  
 water

 The first step in deciding on a distribution plan is to:  
  vote on who will be the channel leader  
 determine a distribution budget  
@ develop appropriate distribution objectives that support the organization's overall marketing plan  
 evaluate the internal and external environments  
 determine the desired level of distribution intensity  

 The Healthy Bites Grills are fast food restaurants that sell burgers made from lean buffalo meat, fries that aren't fried, as well as veggie pockets with soy cheese. The company is four years old and already has outlets in New York, Japan, Canada, and Kuwait. In which stage of the retail life cycle are the Healthy Bites Grills?  
@  growth  
 maturity  
 expansion  
 introduction  
 extension  

 The Home Depot entered the market as a store for professional and amateur plumbers, electricians, and carpenters. Then it introduced its garden centers. Next, came its home design center, which provides help with all kinds of interior decorating problems. Now it is enlarging its garden centers and hiring knowledgeable gardeners to staff these centers. The addition of more amenities and greater assortment can be explained by the:  
  interconnectedness of the retail life cycle  
 trend toward customerization  
 trend toward disintermediation  
@ wheel-of-retailing hypothesis  
 80/20 rule of retailing  

 The lighting in the restaurant was muted as the server led the couple to chairs, covered in leather, and solid enough to last a lifetime, where they were seated at a table covered with a white linen cloth. The colors were limited to earth tones with an occasional splash of red. Patrons could not hear any kitchen noises, only a pleasant melody that seemed familiar but unrecognizable. The smells were extraordinarily varied and made the couple eager to dine. This is a description of the restaurant's:  
  subliminal retail perception  
 merchandise layout  
@ atmospherics  
 demographically-oriented format  
 formatted image  

 The logic used for selecting a market segment is the logic that should be used when a retailer is determining:  
@ its merchandise mix  
 its brand strategy  
 how it will create form utility  
 how to classify its inventory  
 its retail format 

  The manufacturer of Heinz ketchup distributes the condiment to food retailers like supermarkets as well as to restaurants and other providers of food service. The manufacturer of Heinz uses:  
  EDI distribution channels  
 a hierarchical channel  
 a pyramid distribution system  
@ a multiple distribution system  
 multilevel networking  

 The only place you can buy See's candy in Houston, Texas, is at Rice Epicurean stores. This is an example of _____ distribution.  
  extensive  
 selective  
@ exclusive  
 relational  
 intensive

The Paper Moon sells gift-wrapping paper, stationary, and greeting cards. It does not sell any other products. The number of different kinds of wrapping paper, stationary, and cards make up its:  
  retail diversity  
 merchandise diversification  
 retail atmospherics  
@ merchandise assortment  
 retail format 



 The _____ vertical marketing system (VMS) gives an organization more control over both supply sources and resale of its products. It also increases the organization's capital investment and its fixed costs.  
  administered  
 conventional  
@ corporate  
 horizontal  
 contractual  



 Thousands of companies are involved in creating and transporting the approximately 5,000 components that go into manufacturing a typical automobile. These organizations plus its manufacturer and the company that transported the automobile from its factory to the retailer and the automobile retailer make up the _____ for the automobile.  
  distribution pattern  
 organizational chart  
 direct channel of distribution  
@ supply chain  
 materials handling system 


Through a process of _____, traditional intermediaries are being eliminated as companies question the value added by layers in the distribution channel.  
  cross-docking  
 piggybacking  
 demarketing  
@ disintermediation  
 reverse distribution 

 Question 1    Multiple Choice  1 of 1 point(s)    
  When Beth went shopping at her local Target, she wanted to buy a birthday gift in the $15 to $20 price range. She located a Rapunzel Barbie that seemed to be an appropriate gift, but the item was not priced. She went to a convenient UPC scanner where she scanned the doll, and found it was priced at $15.99. This is one way that _______________ has changed how people shop.  
 Your Answer:  technology  
Correct Answer:  technology  
 


  Question 2    Multiple Choice  1 of 1 point(s)    
  When the golf pro learned ThermoCare air-activated heat wraps for the relief of sore muscles were sale priced at $3 less than usual, he went to the drugstore to buy himself several. He was disappointed to learn the store had not expected the demand for the heat wraps, and therefore, did not order a large enough supply. The golf pro's disappointment was due to:  
 Your Answer:  a stock-out  
Correct Answer:  a stock-out  
 


  Question 3    Multiple Choice  1 of 1 point(s)    
  A successful television show like FX's The Shield can only be seen on that network and on no other network. The production company that developed and created the series uses _____ distribution.  
 Your Answer:  exclusive  
Correct Answer:  exclusive  
 


  Question 4    Multiple Choice  1 of 1 point(s)    
  Distribution channels create efficiencies by reducing the number of transactions necessary for goods to flow from different manufacturers to large numbers of customers. This occurs in part through:  
 Your Answer:  creating assortments  
Correct Answer:  creating assortments  
 


  Question 5    Multiple Choice  1 of 1 point(s)    
  Eby-Brown is a business that buys cigarettes, candy, and gum from many different manufacturers and then resells the items to a variety of different stores. Eby-Brown owns the goods it sells and suffers loses if the products are stolen or become out-of-date. It has developed its own marketing strategy and is an example of a:  
 Your Answer:  merchant wholesaler  
Correct Answer:  merchant wholesaler  
 


  Question 6    Multiple Choice  0 of 1 point(s)    
  Store location planners look at a store's _______________, the geographic zone that accounts for the majority of its sales and customers.  
 Your Answer:  target market  
Correct Answer:  trade area  
 


  Question 7    Multiple Choice  1 of 1 point(s)    
  37.To sell the wind chimes he makes, Will Devon set up a display in his front yard and sells to people passing by his home. This is an example of _____ distribution.  
 Your Answer:  direct  
Correct Answer:  direct  
 


  Question 8    Multiple Choice  1 of 1 point(s)    
  Five years ago, the Paradise Surf Shop was the only surf-gear store catering to women. Its _______________ included surfboards smaller and lighter than those used by men.  
 Your Answer:  merchandise mix  
Correct Answer:  merchandise mix  
 


  Question 9    Multiple Choice  1 of 1 point(s)    
  In the _______________ stage of the retail life cycle, retail businesses become obsolete as newer ways of doing business emerge.  
 Your Answer:  decline  
Correct Answer:  decline  
 


  Question 10    Multiple Choice  1 of 1 point(s)    
  For which of the following products is the manufacturer most likely to want to use intensive distribution?  
 Your Answer:  bottled water  
Correct Answer:  bottled water  
 


 Question 1    Multiple Choice  1 of 1 point(s)    
  Freestanding retailers:  
 Your Answer:  are located by themselves in separate buildings  
Correct Answer:  are located by themselves in separate buildings  
 


  Question 2    Multiple Choice  0 of 1 point(s)    
  A successful television show like FX's The Shield can only be seen on that network and on no other network. The production company that developed and created the series uses _____ distribution.  
 Your Answer:  extensive  
Correct Answer:  exclusive  
 


  Question 3    Multiple Choice  1 of 1 point(s)    
  The manufacturer-retailer-consumer channel is:  
 Your Answer:  the shortest indirect channel  
Correct Answer:  the shortest indirect channel  
 


  Question 4    Multiple Choice  1 of 1 point(s)    
  The only place you can buy See's candy in Houston, Texas, is at Rice Epicurean stores. This is an example of _____ distribution.  
 Your Answer:  exclusive  
Correct Answer:  exclusive  
 


  Question 5    Multiple Choice  1 of 1 point(s)    
  When The Home Depot opened stores in Mexico and Canada it was showing how _______________ has influenced retailing.  
 Your Answer:  the globalization of the market  
Correct Answer:  the globalization of the market  
 


  Question 6    Multiple Choice  1 of 1 point(s)    
  Which of the following is not an example of a major demographic factor that is altering the face of retailing today?  
 Your Answer:  rapidly increasing gross national product in United States  
Correct Answer:  rapidly increasing gross national product in United States  
 


  Question 7    Multiple Choice  1 of 1 point(s)    
  Waverly is a manufacturer of wallpaper and matching fabrics, which can be used to make curtains, bedspreads, tablecloths, and chair coverings. Its products are available at its Web site, www.waverly.com, or at large retail stores like The Home Depot and Target. Waverly uses:  
 Your Answer:  a multiple distribution system  
Correct Answer:  a multiple distribution system  
 


  Question 8    Multiple Choice  1 of 1 point(s)    
  _______________ is the process by which goods and services are sold to consumers for their personal use.  
 Your Answer:  Retailing  
Correct Answer:  Retailing  
 


  Question 9    Multiple Choice  1 of 1 point(s)    
  Why do manufacturers use a direct channel of distribution?  
 Your Answer:  to have more control over product distribution  
Correct Answer:  to have more control over product distribution  
 


  Question 10    Multiple Choice  1 of 1 point(s)    
  Which of the following statements about retailers and retailing activities is true?  
 Your Answer:  Retailers are members of channels of distribution.  
Correct Answer:  Retailers are members of channels of distribution.  
 

 Question 1    Multiple Choice  1 of 1 point(s)    
  In terms of the basic types of retail locations, the traditional downtown business area found in most towns and cities is referred to as a:  
 Your Answer:  central business district  
Correct Answer:  central business district  
 


  Question 2    Multiple Choice  1 of 1 point(s)    
  When The Home Depot opened stores in Mexico and Canada it was showing how _______________ has influenced retailing.  
 Your Answer:  the globalization of the market  
Correct Answer:  the globalization of the market  
 


  Question 3    Multiple Choice  1 of 1 point(s)    
  The Healthy Bites Grills are fast food restaurants that sell burgers made from lean buffalo meat, fries that aren't fried, as well as veggie pockets with soy cheese. The company is four years old and already has outlets in New York, Japan, Canada, and Kuwait. In which stage of the retail life cycle are the Healthy Bites Grills?  
 Your Answer:  growth  
Correct Answer:  growth  
 


  Question 4    Multiple Choice  1 of 1 point(s)    
  Which of the following statements about store layout is true?  
 Your Answer:  All of the above statements about store layout are true.  
Correct Answer:  All of the above statements about store layout are true.  
 


  Question 5    Multiple Choice  0 of 1 point(s)    
  According to the _____, the Lincoln Gift Shop adds value to Department 59 Christmas collectibles by arranging them in a carefully lit large display that shows how attractively the items can be grouped.  
 Your Answer:  80/20 rule  
Correct Answer:  value chain concept  
 


  Question 6    Multiple Choice  1 of 1 point(s)    
  Cash-and-carry wholesalers:  
 Your Answer:  provide low-cost merchandise for retailers and industrial customers who are too small for other wholesalers' sales representatives to call on them  
Correct Answer:  provide low-cost merchandise for retailers and industrial customers who are too small for other wholesalers' sales representatives to call on them  
 


  Question 7    Multiple Choice  1 of 1 point(s)    
  Which of the following is illegal?  
 Your Answer:  a pyramid scheme  
Correct Answer:  a pyramid scheme  
 


  Question 8    Multiple Choice  1 of 1 point(s)    
  Which of the following modes of transportation would most likely be used to haul several tons of bananas from South America where it is grown to markets in the Great Lakes region?  
 Your Answer:  fishyback  
Correct Answer:  fishyback  
 


  Question 9    Multiple Choice  1 of 1 point(s)    
  _____ distribution aims at maximizing market coverage by selling a product through all wholesalers and retailers that will stock and sell the product.  
 Your Answer:  Intensive  
Correct Answer:  Intensive  
 


  Question 10    Multiple Choice  1 of 1 point(s)    
  Waverly is a manufacturer of wallpaper and matching fabrics, which can be used to make curtains, bedspreads, tablecloths, and chair coverings. Wholesalers buy the Waverly products and distribute them to leading department stores, major specialty stores, and independent paint and wallpaper stores. This is an example of a(n):  
 Your Answer:  channel of distribution  
Correct Answer:  channel of distribution  
 


Question 1    Multiple Choice  1 of 1 point(s)    
  A _______________ is how a target market perceives the store.  
 Your Answer:  store image  
Correct Answer:  store image  
 


  Question 2    Multiple Choice  1 of 1 point(s)    
  Convenience stores:  
 Your Answer:  create time utility for their customers  
Correct Answer:  create time utility for their customers  
 


  Question 3    Multiple Choice  1 of 1 point(s)    
  V&E Bridal Design Shop caters to engaged couples who desire a traditional Eastern Asian wedding ceremony. The brides-to-be who shop at this store are typically Japanese, Laotian, Vietnamese, Korean, or Chinese. This retailer has used _______________ to locate a market for its products.  
 Your Answer:  demographics  
Correct Answer:  demographics  
 


  Question 4    Multiple Choice  1 of 1 point(s)    
  _______________ is the number of different product lines available at a particular retail store.  
 Your Answer:  Merchandise breadth  
Correct Answer:  Merchandise breadth  
 


  Question 5    Multiple Choice  1 of 1 point(s)    
  A firm that is a channel leader has power relative to the other channel members. This power comes from:  
 Your Answer:  all of the above  
Correct Answer:  all of the above  
 


  Question 6    Multiple Choice  1 of 1 point(s)    
  Which of the following design factors can be used to create store atmospherics?  
 Your Answer:  store layout  
Correct Answer:  store layout  
 


  Question 7    Multiple Choice  1 of 1 point(s)    
  Eby-Brown is a business that buys cigarettes, candy, and gum from many different manufacturers and then resells the items to a variety of different stores. Eby-Brown is an example of a(n):  
 Your Answer:  independent intermediary  
Correct Answer:  independent intermediary  
 


  Question 8    Multiple Choice  1 of 1 point(s)    
  Which of the following statements about store layout is true?  
 Your Answer:  All of the above statements about store layout are true.  
Correct Answer:  All of the above statements about store layout are true.  
 


  Question 9    Multiple Choice  1 of 1 point(s)    
  Which of the following statements about retailers and retailing activities is true?  
 Your Answer:  Retailers are members of channels of distribution.  
Correct Answer:  Retailers are members of channels of distribution.  
 


  Question 10    Multiple Choice  1 of 1 point(s)    
  During the maturity stage of the retail life cycle:  
 Your Answer:  all of the above occur  
Correct Answer:  all of the above occur  
 
Question 5    Multiple Choice  0 of 1 point(s)    
  According to your instructor, why do intermediaries (middlemen) exist.  
 Your Answer:  [None Given]  
Correct Answer:  Because some companies are better at performing one or more of these functions than other companies are.  
 


  Question 6    Multiple Choice  0 of 1 point(s)    
  Which of the following is not an example of a major demographic factor that is altering the face of retailing today?  
 Your Answer:  [None Given]  
Correct Answer:  rapidly increasing gross national product in United States  
 


  Question 7    Multiple Choice  0 of 1 point(s)    
  In a _____ a single channel member controls channel behavior in order to avoid channel conflict and to create economies through size, eliminating duplication of services, and creating buying bargaining power.  
 Your Answer:  [None Given]  
Correct Answer:  vertical marketing system  
 


  Question 8    Multiple Choice  0 of 1 point(s)    
  Which of the following population characteristics might location planners consider when determining where to locate a new store?  
 Your Answer:  [None Given]  
Correct Answer:  all of the above  
 


  Question 9    Multiple Choice  0 of 1 point(s)    
  _____ is a logistics activity, which involves developing and implementing a process to ensure the firm always has goods available to meet customers' demands-no more and no less.  
 Your Answer:  [None Given]  
Correct Answer:  Inventory control  
 


  Question 10    Multiple Choice  0 of 1 point(s)    
  V&E Bridal Design Shop caters to engaged couples who desire a traditional Eastern Asian wedding ceremony. The brides-to-be who shop at this store are typically Japanese, Laotian, Vietnamese, Korean, or Chinese. This retailer has used _______________ to locate a market for its products.  
 Your Answer:  [None Given]  
Correct Answer:  demographics  
 

Question 3    Multiple Choice  0 of 1 point(s)    
  Which of the following is the best example of a direct channel?  
 Your Answer:  [None Given]  
Correct Answer:  Farmer Mintz sells corn to the Hughes household who will eat it for supper.  
 


  Question 4    Multiple Choice  0 of 1 point(s)    
  Five years ago, the Paradise Surf Shop was the only surf-gear store catering to women. It carried everything the serious female surfer needed to ride the waves. It is an example of which form of retailer?  
 Your Answer:  [None Given]  
Correct Answer:  specialty store  
 


  Question 5    Multiple Choice  0 of 1 point(s)    
  During the maturity stage of the retail life cycle:  
 Your Answer:  [None Given]  
Correct Answer:  all of the above occur  
 


  Question 6    Multiple Choice  0 of 1 point(s)    
  Which of the following design factors can be used to create store atmospherics?  
 Your Answer:  [None Given]  
Correct Answer:  store layout  
 


  Question 7    Multiple Choice  0 of 1 point(s)    
  For which of the following products is the manufacturer most likely to want to use intensive distribution?  
 Your Answer:  [None Given]  
Correct Answer:  bottled water  
 


  Question 8    Multiple Choice  0 of 1 point(s)    
  According to your instructor, what is the goal of a channel of distribution?  
 Your Answer:  [None Given]  
Correct Answer:  To create the most efficient way to move products from producer to consumer.  
 


  Question 9    Multiple Choice  0 of 1 point(s)    
  Which of the following stores is most likely to have a narrow assortment of merchandise?  
 Your Answer:  [None Given]  
Correct Answer:  a store that only sells men's ties  
 


  Question 10    Multiple Choice  0 of 1 point(s)    
  Which of the following population characteristics might location planners consider when determining where to locate a new store?  
 Your Answer:  [None Given]  
Correct Answer:  all of the above  
 


 Question 1    Multiple Choice  1 of 1 point(s)    
  In terms of the basic types of retail locations, the traditional downtown business area found in most towns and cities is referred to as a:  
 Your Answer:  central business district  
Correct Answer:  central business district  
 


  Question 2    Multiple Choice  1 of 1 point(s)    
  A retailer looking to locate a store in an area in which the needs of the population are not being adequately met by existing stores is looking for a(n) _______________ trade area.  
 Your Answer:  understored  
Correct Answer:  understored  
 


  Question 3    Multiple Choice  0 of 1 point(s)    
  Which of the following is a reason why a company may choose to have more than one channel of distribution for the same product?  
 Your Answer:  [None Given]  
Correct Answer:  Because the product has two distinct market segments that perceive the product differently.  
 


  Question 4    Multiple Choice  0 of 1 point(s)    
  Which of the following statements about a vertical marketing system (VMS) is true?  
 Your Answer:  [None Given]  
Correct Answer:  A VMS is a channel in which there is cooperation among channel members at two or more different levels of the channel.  
 


  Question 5    Multiple Choice  0 of 1 point(s)    
  Freestanding retailers:  
 Your Answer:  [None Given]  
Correct Answer:  are located by themselves in separate buildings  
 


  Question 6    Multiple Choice  0 of 1 point(s)    
  The only place you can buy See's candy in Houston, Texas, is at Rice Epicurean stores. This is an example of _____ distribution.  
 Your Answer:  [None Given]  
Correct Answer:  exclusive  
 


  Question 7    Multiple Choice  0 of 1 point(s)    
  Five years ago, the Paradise Surf Shop was the only surf-gear store catering to women. It carried everything the serious female surfer needed to ride the waves. It is an example of which form of retailer?  
 Your Answer:  [None Given]  
Correct Answer:  specialty store  
 


  Question 8    Multiple Choice  0 of 1 point(s)    
  Wholesalers and retailers purchase large quantities (usually cases) of goods from manufacturers but sell only one or a few at a time to many different customers. This process is called:  
 Your Answer:  [None Given]  
Correct Answer:  breaking bulk  
 


  Question 9    Multiple Choice  0 of 1 point(s)    
  A(n) _____ is a series of firms or individuals that facilitates the movement of a product from the producer to the final customers.  
 Your Answer:  [None Given]  
Correct Answer:  channel of distribution  
 


  Question 10    Multiple Choice  0 of 1 point(s)    
  During the maturity stage of the retail life cycle:  
 Your Answer:  [None Given]  
Correct Answer:  all of the above occur  
 


Question 1    Multiple Choice  1 of 1 point(s)    
  A firm that is a channel leader has power relative to the other channel members. This power comes from:  
 Your Answer:  all of the above  
Correct Answer:  all of the above  
 


  Question 2    Multiple Choice  1 of 1 point(s)    
  _______________ is the process by which goods and services are sold to consumers for their personal use.  
 Your Answer:  Retailing  
Correct Answer:  Retailing  
 


  Question 3    Multiple Choice  0 of 1 point(s)    
  When Harris Pool Chemical Co. purchased 144 buckets of chlorine tablets over the Internet from ChemTech, the company placed the order with the manufacturer and arranged for its transportation by truck to the store. As a limited-service wholesaler, ChemTech took title to the pool chemicals but never actually took possession of them. ChemTech is an example of a:  
 Your Answer:  [None Given]  
Correct Answer:  drop shipper  
 


  Question 4    Multiple Choice  0 of 1 point(s)    
  In terms of the basic types of retail locations, the traditional downtown business area found in most towns and cities is referred to as a:  
 Your Answer:  [None Given]  
Correct Answer:  central business district  
 


  Question 5    Multiple Choice  0 of 1 point(s)    
  Distribution channels create efficiencies by reducing the number of transactions necessary for goods to flow from different manufacturers to large numbers of customers. This occurs in part through:  
 Your Answer:  [None Given]  
Correct Answer:  creating assortments  
 


  Question 6    Multiple Choice  0 of 1 point(s)    
  Which of the following is not an example of a major demographic factor that is altering the face of retailing today?  
 Your Answer:  [None Given]  
Correct Answer:  rapidly increasing gross national product in United States  
 


  Question 7    Multiple Choice  0 of 1 point(s)    
  Manufacturers' agents are:  
 Your Answer:  [None Given]  
Correct Answer:  compensated with commissions based on a percentage of what they sell  
 


  Question 8    Multiple Choice  0 of 1 point(s)    
  Which of the following design factors can be used to create store atmospherics?  
 Your Answer:  [None Given]  
Correct Answer:  store layout  
 


  Question 9    Multiple Choice  0 of 1 point(s)    
  Thousands of companies are involved in creating and transporting the approximately 5,000 components that go into manufacturing a typical automobile. These organizations plus its manufacturer and the company that transported the automobile from its factory to the retailer and the automobile retailer make up the _____ for the automobile.  
 Your Answer:  [None Given]  
Correct Answer:  supply chain  
 


  Question 10    Multiple Choice  0 of 1 point(s)    
  When you receive a telephone call from a salesperson asking you to buy a subscription to an educational magazine designed to help children improve their math skills, it is an example of:  
 Your Answer:  [None Given]  
Correct Answer:  direct selling  
 

Question 1    Multiple Choice  1 of 1 point(s)    
  Cash-and-carry wholesalers:  
 Your Answer:  provide low-cost merchandise for retailers and industrial customers who are too small for other wholesalers' sales representatives to call on them  
Correct Answer:  provide low-cost merchandise for retailers and industrial customers who are too small for other wholesalers' sales representatives to call on them  
 


  Question 2    Multiple Choice  1 of 1 point(s)    
  Which of the following modes of transportation would most likely be used to haul several tons of bananas from South America where it is grown to markets in the Great Lakes region?  
 Your Answer:  fishyback  
Correct Answer:  fishyback  
 


  Question 3    Multiple Choice  1 of 1 point(s)    
  For which of the following products is the manufacturer most likely to want to use intensive distribution?  
 Your Answer:  bottled water  
Correct Answer:  bottled water  
 


  Question 4    Multiple Choice  1 of 1 point(s)    
  _____ distribution aims at maximizing market coverage by selling a product through all wholesalers and retailers that will stock and sell the product.  
 Your Answer:  Intensive  
Correct Answer:  Intensive  
 


  Question 5    Multiple Choice  0 of 1 point(s)    
  In which kind of retail store are you most likely to see a grid layout?  
 Your Answer:  a clothing specialty store  
Correct Answer:  a supermarket  
 


  Question 6    Multiple Choice  1 of 1 point(s)    
  Five years ago, the Paradise Surf Shop was the only surf-gear store catering to women. It carried everything the serious female surfer needed to ride the waves. The Paradise Surf Shop carried lightweight surfboards with flowers painted on them stacked around the wall of the store. It also had clean, well-lit dressing rooms for trying on wetsuits and knowledgeable staff who knew what a woman needed to succeed in an essentially male-dominated sport. These items together created its:  
 Your Answer:  store image  
Correct Answer:  store image  
 


  Question 7    Multiple Choice  1 of 1 point(s)    
  The fastest and most expensive mode of transportation is:  
 Your Answer:  air  
Correct Answer:  air  
 


  Question 8    Multiple Choice  0 of 1 point(s)    
  Which of the following statements about merchandise brokers is true?  
 Your Answer:  [None Given]  
Correct Answer:  Brokers never take title to the merchandise they sell.  
 


  Question 9    Multiple Choice  0 of 1 point(s)    
  Five years ago, the Paradise Surf Shop was the only surf-gear store catering to women. It carried everything the serious female surfer needed to ride the waves. It is an example of which form of retailer?  
 Your Answer:  [None Given]  
Correct Answer:  specialty store  
 


  Question 10    Multiple Choice  0 of 1 point(s)    
  The fact Nita can visit her local supermarket and buy a dozen roses, a gallon of milk, two birthday cards, a bag of cat litter, and a pan for roasting a turkey illustrates how channels of distribution benefit consumers by:  
 Your Answer:  [None Given]  
Correct Answer:  creating assortments  
 
Question 1    Multiple Choice  1 of 1 point(s)    
  Waverly is a manufacturer of wallpaper and matching fabrics, which can be used to make curtains, bedspreads, tablecloths, and chair coverings. Wholesalers buy the Waverly products and distribute them to leading department stores, major specialty stores, and independent paint and wallpaper stores. This is an example of a(n):  
 Your Answer:  channel of distribution  
Correct Answer:  channel of distribution  
 


  Question 2    Multiple Choice  0 of 1 point(s)    
  The Paper Moon sells gift-wrapping paper, stationary, and greeting cards. It does not sell any other products. The number of different kinds of wrapping paper, stationary, and cards make up its:  
 Your Answer:  merchandise diversification  
Correct Answer:  merchandise assortment  
 


  Question 3    Multiple Choice  0 of 1 point(s)    
  ________________ is any method a firm uses to complete an exchange that does not require a customer to visit a store.  
 Your Answer:  [None Given]  
Correct Answer:  Nonstore retailing  
 


  Question 4    Multiple Choice  0 of 1 point(s)    
  Which of the following statements about business-to-business channels is true?  
 Your Answer:  [None Given]  
Correct Answer:  Direct channels are more common in business-to-business markets than consumer markets.  
 


  Question 5    Multiple Choice  0 of 1 point(s)    
  With the _______________ of direct selling, people tend to get caught up in the "group spirit" and buy things they would not normally buy if alone.  
 Your Answer:  [None Given]  
Correct Answer:  party plan system  
 


  Question 6    Multiple Choice  0 of 1 point(s)    
  V&E Bridal Design Shop caters to engaged couples who desire a traditional Eastern Asian wedding ceremony. The brides-to-be who shop at this store are typically Japanese, Laotian, Vietnamese, Korean, or Chinese. This retailer has used _______________ to locate a market for its products.  
 Your Answer:  [None Given]  
Correct Answer:  demographics  
 


  Question 7    Multiple Choice  0 of 1 point(s)    
  _______________ is the process by which goods and services are sold to consumers for their personal use.  
 Your Answer:  [None Given]  
Correct Answer:  Retailing  
 


  Question 8    Multiple Choice  0 of 1 point(s)    
  A retailer looking to locate a store in an area in which the needs of the population are not being adequately met by existing stores is looking for a(n) _______________ trade area.  
 Your Answer:  [None Given]  
Correct Answer:  understored  
 


  Question 9    Multiple Choice  0 of 1 point(s)    
  Store location planners look at a store's _______________, the geographic zone that accounts for the majority of its sales and customers.  
 Your Answer:  [None Given]  
Correct Answer:  trade area  
 


  Question 10    Multiple Choice  0 of 1 point(s)    
  _____ are intermediaries that facilitate transactions in markets such as real estate, food, and used equipment in which there are a lot of small buyers and sellers. They identify likely buyers and sellers and bring the two together in return for a fee received when the transaction is complete.  
 Your Answer:  [None Given]  
Correct Answer:  Merchandise brokers  
 




Question 1    Multiple Choice  1 of 1 point(s)    
  Selective distribution strategy is most often used with which consumer product category?  
 Your Answer:  shopping goods  
Correct Answer:  shopping goods  
 


  Question 2    Multiple Choice  1 of 1 point(s)    
  When you receive a telephone call from a salesperson asking you to buy a subscription to an educational magazine designed to help children improve their math skills, it is an example of:  
 Your Answer:  direct selling  
Correct Answer:  direct selling  
 


  Question 3    Multiple Choice  0 of 1 point(s)    
  Vending machines are a tried-and-true way to sell:  
 Your Answer:  [None Given]  
Correct Answer:  convenience goods  
 


  Question 4    Multiple Choice  0 of 1 point(s)    
  In which kind of retail store are you most likely to see a free-flow layout?  
 Your Answer:  [None Given]  
Correct Answer:  a clothing specialty store like The Gap  
 


  Question 5    Multiple Choice  0 of 1 point(s)    
  Waverly is a manufacturer of wallpaper and matching fabrics, which can be used to make curtains, bedspreads, tablecloths, and chair coverings. Wholesalers buy the Waverly products and distribute them to leading department stores, major specialty stores, and independent paint and wallpaper stores. This is an example of a(n):  
 Your Answer:  [None Given]  
Correct Answer:  channel of distribution  
 


  Question 6    Multiple Choice  0 of 1 point(s)    
  According to your instructor, what is the goal of a channel of distribution?  
 Your Answer:  [None Given]  
Correct Answer:  To create the most efficient way to move products from producer to consumer.  
 


  Question 7    Multiple Choice  0 of 1 point(s)    
  Which of the following is a reason why a company may choose to have more than one channel of distribution for the same product?  
 Your Answer:  [None Given]  
Correct Answer:  Because the product has two distinct market segments that perceive the product differently.  
 


  Question 8    Multiple Choice  0 of 1 point(s)    
  Which of the following statements about business-to-business channels is true?  
 Your Answer:  [None Given]  
Correct Answer:  Direct channels are more common in business-to-business markets than consumer markets.  
 


  Question 9    Multiple Choice  0 of 1 point(s)    
  Through a process of _____, traditional intermediaries are being eliminated as companies question the value added by layers in the distribution channel.  
 Your Answer:  [None Given]  
Correct Answer:  disintermediation  
 


  Question 10    Multiple Choice  0 of 1 point(s)    
  Five years ago, the Paradise Surf Shop was the only surf-gear store catering to women. It carried everything the serious female surfer needed to ride the waves. The Paradise Surf Shop carried lightweight surfboards with flowers painted on them stacked around the wall of the store. It also had clean, well-lit dressing rooms for trying on wetsuits and knowledgeable staff who knew what a woman needed to succeed in an essentially male-dominated sport. These items together created its:  
 Your Answer:  [None Given]  
Correct Answer:  store image  
 




Question 1    Multiple Choice  1 of 1 point(s)    
  Five years ago, the Paradise Surf Shop was the only surf-gear store catering to women. Its _______________ included surfboards smaller and lighter than those used by men.  
 Your Answer:  merchandise mix  
Correct Answer:  merchandise mix  
 


  Question 2    Multiple Choice  1 of 1 point(s)    
  Waverly is a manufacturer of wallpaper and matching fabrics, which can be used to make curtains, bedspreads, tablecloths, and chair coverings. Its products are available at its Web site, www.waverly.com, or at large retail stores like The Home Depot and Target. Waverly uses:  
 Your Answer:  a multiple distribution system  
Correct Answer:  a multiple distribution system  
 


  Question 3    Multiple Choice  1 of 1 point(s)    
  Burger King, Holiday Inn, Avis, and H&R Block tax service are all franchises, which means they are also members of a _____ vertical marketing system (VMS).  
 Your Answer:  contractual  
Correct Answer:  contractual  
 


  Question 4    Multiple Choice  0 of 1 point(s)    
  Sales branches:  
 Your Answer:  [None Given]  
Correct Answer:  are manufacturer-owned facilities that carry inventory and provide sales and services to customers  
 


  Question 5    Multiple Choice  0 of 1 point(s)    
  Which of the following statements about retailers and retailing activities is true?  
 Your Answer:  [None Given]  
Correct Answer:  Retailers are members of channels of distribution.  
 


  Question 6    Multiple Choice  0 of 1 point(s)    
  A _______________ is how a target market perceives the store.  
 Your Answer:  [None Given]  
Correct Answer:  store image  
 


  Question 7    Multiple Choice  0 of 1 point(s)    
  Selective distribution strategy is most often used with which consumer product category?  
 Your Answer:  [None Given]  
Correct Answer:  shopping goods  
 


  Question 8    Multiple Choice  0 of 1 point(s)    
  Which of the following is a reason why a company may choose to have more than one channel of distribution for the same product?  
 Your Answer:  [None Given]  
Correct Answer:  Because the product has two distinct market segments that perceive the product differently.  
 


  Question 9    Multiple Choice  0 of 1 point(s)    
  _____ is a logistics activity, which involves developing and implementing a process to ensure the firm always has goods available to meet customers' demands-no more and no less.  
 Your Answer:  [None Given]  
Correct Answer:  Inventory control  
 


  Question 10    Multiple Choice  0 of 1 point(s)    
  A shoe department carries more than 55 different styles of white sandals. This is a description of the shoe department's:  
 Your Answer:  [None Given]  
Correct Answer:  merchandise depth  
 
  Question 1    Multiple Choice  1 of 1 point(s)    
  Five years ago, the Paradise Surf Shop was the only surf-gear store catering to women. It carried everything the serious female surfer needed to ride the waves. In terms of its merchandise assortment, it had a:  
 Your Answer:  narrow and deep assortment  
Correct Answer:  narrow and deep assortment  
 


  Question 2    Multiple Choice  0 of 1 point(s)    
  Which of the following statements about channel functions is true?  
 Your Answer:  [None Given]  
Correct Answer:  All channel functions have to be performed by somebody.  
 


  Question 3    Multiple Choice  0 of 1 point(s)    
  The fact Nita can visit her local supermarket and buy a dozen roses, a gallon of milk, two birthday cards, a bag of cat litter, and a pan for roasting a turkey illustrates how channels of distribution benefit consumers by:  
 Your Answer:  [None Given]  
Correct Answer:  creating assortments  
 


  Question 4    Multiple Choice  0 of 1 point(s)    
  Wholesalers and retailers purchase large quantities (usually cases) of goods from manufacturers but sell only one or a few at a time to many different customers. This process is called:  
 Your Answer:  [None Given]  
Correct Answer:  breaking bulk  
 


  Question 5    Multiple Choice  0 of 1 point(s)    
  According to your instructor, why should we be paying attention to changes in logistics?  
 Your Answer:  [None Given]  
Correct Answer:  There are tremendous opportunities to make money by figuring out how to make shipments of physical products as efficient as the internet has made ordering them.  
 


  Question 6    Multiple Choice  0 of 1 point(s)    
  Why is it important for a marketer to know the ownership structure of a retail store?  
 Your Answer:  [None Given]  
Correct Answer:  Because the ownership structure determines who you need to approach in order to sell to the store.  
 


  Question 7    Multiple Choice  0 of 1 point(s)    
  The logic used for selecting a market segment is the logic that should be used when a retailer is determining:  
 Your Answer:  [None Given]  
Correct Answer:  its merchandise mix  
 


  Question 8    Multiple Choice  0 of 1 point(s)    
  When Beth went shopping at her local Target, she wanted to buy a birthday gift in the $15 to $20 price range. She located a Rapunzel Barbie that seemed to be an appropriate gift, but the item was not priced. She went to a convenient UPC scanner where she scanned the doll, and found it was priced at $15.99. This is one way that _______________ has changed how people shop.  
 Your Answer:  [None Given]  
Correct Answer:  technology  
 


  Question 9    Multiple Choice  0 of 1 point(s)    
  In the _______________ stage of the retail life cycle, the new retailer often is an aggressive entrepreneur who takes a unique approach to doing business by creating a differential advantage.  
 Your Answer:  [None Given]  
Correct Answer:  introduction  
 


  Question 10    Multiple Choice  0 of 1 point(s)    
  A _______________ is how a target market perceives the store.  
 Your Answer:  [None Given]  
Correct Answer:  store image  
 




 A firm's _____ is made up of the ethical values, norms, and beliefs that influence the behavior of everyone in the organization.  
  cultural autonomy  
 managerial portfolio  
@ corporate culture  
 management hierarchy  
 organizational chart 

 A firm has _____ when it is able to outperform the competition, providing customers with a benefit the competition can't.  
  synergy  
@ a competitive advantage  
 a quality-value paradigm  
 a comparative niche  
 a competitive orientation  

A new California law requires retailers such as Home Depot and Sam's Club to use netting, rails, or other restraints to prevent merchandise from falling off tall storage racks. This is an example of a change in the _____ environment for stores operating in that state.  
  economic  
 natural  
 technological  
@ legal  
 sociocultural  

 A New Era company that wishes to tackle a social problem by joining forces with a not-for-profit organization like The Nature Conservancy or the American Cancer Society should use a(n) _____ strategy.  
  gray marketing  
 ethical stewardship  
@ cause marketing  
 black marketing  
 green marketing 


A new windshield wiper that uses computer sensors to determine when it needs to operate and how fast it needs to operate is a change in the _____ environment for car manufacturers.  
  economic  
 natural  
@ technological  
 legal  
 sociocultural 

 A slotting allowance is:  
  most closely related to the product element of the marketing mix  
 additional warehousing costs engendered by poor sales  
 the cost of buying advertising time on specific television shows  
 the cost associated with introducing a new product to an existing product line  
@ a fee paid in exchange for agreeing to place the manufacturer's products on the retailer's valuable shelf space  

 A slotting allowance is most closely related to which element of the marketing mix?  
  pricing  
 promotion  
 production  
@ distribution  
 product  

 A small independent retailer set the following marketing objective: "To make the store more inviting to customers during the next year." This would be an example of a(n) _____ objective.  
@  qualitative  
 functional  
 tangible  
 intangible  
 quantitative 

 An employee of an international company could look to the organization's _____ to determine the company's moral stance on dealing with government officials who expect illicit monetary donations in exchange for needed operation permits and licenses.  
@  code of ethics  
 mission statement  
 marketing strategy  
 marketing communications  
 social profitability statement

According to the BCG growth-share matrix, _____ are strategic business units with products that have a dominant market share in a high-growth market.  
  problem children  
 exclamation points  
 cash cows  
@ stars  
 question marks 



According to the Consumer Bill of Rights, consumers have the right to all of the following EXCEPT:  
  safety  
@ equal treatment  
 be informed  
 be heard  
 choose freely 

 According to your instructor, what is the point you should learn from the Boston Consulting Group matrix?  
  You can organize your products according to market share and industry attractiveness.  
 There are only four basic approaches to portfolio management.  
@ You need a balance of cash cows and stars to assure long-term success.  
 Strategic business units can be characterized as cash cows, dogs, stars and question marks.  
 The more stars you have, the better off your business will be.  

 According to your instructor, which of the following is a reason why environmental scans are so important to you as a marketing manager?  
  Because you need them in order to correct the organization's vision statement.  
@ Because the quality of the environmental scan affects the quality of the marketing strategies you develop.  
 Environmental scans include both macro and micro environments.  
 Environmental scans include both internal and external factors.  
 Because you need to consider internal, external, macro and micro factors.  


 According to your instructor which of the following is a reason why environmental scans are so important to marketing managers.  
  Reading the paper and a variety of trade magazines are important ways to monitor events. 
 Because technology is no longer changing rapidly.  
 Because you need an environmental scan in order to make a good decision.  
 Because marketers are expected to perform a boundary-spanning function in an organization.  
@ They can help you avoid being "blindsided" by unanticipated market factors.  

 After investigating several diet-program companies, the FTC required them to run _____, which reflected the programs' successes with helping customers keep off the weight they had lost.  
@  corrective advertising  
 puffery  
 dynamic advertising  
 fair and equitable advertising  
 remedial promotion  


Archetype is a Unilever SBU that markets a number of different tea brands around the world. Each brand addresses the taste and emotions of local tea cultures. How would you classify the marketing Archetype does to create specific products for the more than 100 countries in which it is sold?  
  a weakness  
@ a strength  
 a threat  
 an opportunity


 Archetype is a Unilever SBU that markets a number of different tea brands around the world. Each brand addresses the taste and emotions of local tea cultures. Which of the following would be part of Archetype's internal environment?  
  changes in the way people drink tea  
 increased competition from a local tea producer  
 government regulation of information on package labels  
@ the marketing Archetype does to reach consumers and satisfy their cultural needs  
 a shortage of tea leaves due to unusual weather conditions  

Argo is the world's leading brand of small vehicles that are designed to be driven on all types of terrains and in all types of weather. The vehicles can carry up to six passengers. The market for such vehicles has become stagnant due to the recent economic downturn. In terms of the BCG growth-market share matrix, Argo would be classified as a:  
  problem child  
 dog  
@ cash cow  
 star  
 question mark 

Assume a manufacturer of protein bars engaged in marketing planning to determine how to increase sales revenues. In the first step of the marketing planning process, it would:  
  decide if it wants to use either a market development or a product development strategy  
 plan how it will develop new products  
@ examine its SWOT analysis and carefully study the marketing environment  
 set organizational objectives  
 create a cross-functional team 

 At a broad level, many marketers compete for consumers' _____, the amount of money people have left after paying for necessities such as housing, utilities, food, and clothing.  
  gross income  
@ discretionary income  
 disposable revenue  
 net revenue  
 disposable diapers  

AT&T, WorldCom, and Sprint control about 80 percent of the $16 billion international long-distance telephone service market. These three businesses operate in:  
  a monopoly  
 monopolistic competition  
 perfect competition  
@ an oligopoly  
 oligopolistic competition 

A _____ is a tool management uses to assess the potential of a firm's products or businesses. Which of the following is an example of a proactive approach to international marketing?  
  SWOT analysis  
@ portfolio analysis  
 value audit  
 situational audit  
 compatibility assessment  



Biopure is a company that manufactures and markets oxygen therapeutics. Its products are Hemopure for human use and Oxyglobin for animal use. Both have been developed as alternatives to red blood cell transfusions. Which of the following would be part of Biopure's internal environment?  
  approval by the U.S. Food and Drug Administration to allow veterinarians to use Oxyglobin  
 the raw materials needed to make Hemopure and Oxyglobin  
@ the patented manufacturing process that it uses to produce Hemopure and Oxyglobin  
 a competitor developing a similar product  
 all of the above 

 Birmingham, Alabama, is in the heartland of the South. During the last decade, more than 75,000 Hispanics have moved into the city and its neighboring areas. This Spanish-speaking population has led to car dealerships, restaurants, and other retailers in the area using Spanish-language signs and hiring Spanish-speaking employees. These Birmingham businesses are adopting _____ programs.  
  philanthropic  
@ cultural diversity  
 ethical stewardship  
 economic responsibility  
 cause marketing 



 Both Ford Motor Company and General Motors Corporation have entered the growing Chinese market. Both automotive companies plan to sell their cars for $12,000 because that is what the Chinese consumer expects to pay for a car. This is the _____ strategy both companies will use.  
  production  
 promotion  
 distribution  
@ pricing  
 product 



 "Challenge Day provides youth and their communities with experiential workshops and programs that demonstrate the possibility of love and connection through the celebration of diversity, truth and full expression." The preceding statement is Challenge Day's:  
  vision  
 strategic objective  
 marketing objective  
 strategic plan  
@ mission statement

Changes in demographics reflect changes in the _____ environment.  
@  sociocultural  
 competitive  
 technological  
 legal  
 economic 

 Cherokee Electronics is and has been for years the United States's leading manufacturer of two-way radios, but with the growth of handheld communications devices, Cherokee has seen its market gradually disappear. In terms of the BCG growth-market share matrix, Cherokee's two-way radio would be classified as a:  
  problem child  
 dog  
@ cash cow  
 star  
 question mark 

 Cherokee Electronics is and has been for years the United States's leading manufacturer of two-way radios, but with the growth of handheld communications devices, the market for two-way radios is disappearing. Worldwide Electronics, one of Cherokee's competitors, has a small share of the two-way radio market. In terms of the BCG growth-market share matrix, Worldwide's two-way radio would be classified as a:  
  problem child  
@ dog  
 cash cow  
 star  
 question mark 

Circuit City announced that it will no longer sell VHS movie titles due to the growing popularity of DVDs. This change in products being sold at Circuit City stores reflects a change in the _____ environment.  
  economic  
 natural  
@ technological  
 legal  
 sociocultural 

Cleaner-by-Nature, Eco-Mat, and Greener Cleaners are all dry cleaning retail chains that use environmentally safe chemicals to clean clothes. (Traditional dry cleaners use percloroethylene, which is not earth-friendly.) There are approximately 6,000 of these nontraditional cleaners in the United States that are committed to a(n) _____ strategy.  
  gray marketing  
 ethical diversity  
 cause marketing  
 black marketing  
@ green marketing  

 Codes of ethics:  
  are legal requirements for all companies doing business in the United States  
@ are written standards of behaviors to which everyone in the organization must adhere  
 often perform the same function as an organizational mission statement  
 are generalities that can be interpreted in various ways  
 are standardized across all U.S.-based companies 


 Companies that want to be suppliers for firms in countries that are members of the European Union need to:  
  show evidence of a serious long-term commitment to cause marketing  
@ have ISO 9000 certification  
 institute a production-orientation  
 keep prices low and not be concerned about quality issues  
 build firms in Europe to avoid paying tariffs

 Consumerism is the:  
  study of consumer behavior  
 orientation of forward-thinking businesses who are interested in developing transactional relationships with their customers  
@ social movement directed toward protecting consumers from harmful business practices  
 enforcement of the Consumer Bill of Rights by the U.S. Justice Department  
 political movement directed toward limiting consumers' right to litigation  


Delta Apparel's cotton t-shirts, baseball jerseys, and tank tops are produced in 32 different colors and often sport corporate logos. Its primary competitors are Hanes, Russell, Gildan, and Fruit of the Loom. Delta Apparel succeeds in this highly competitive market because it is the most cost-efficient manufacturer. This ability to be cost-efficient is its:  
  lack of a mission statement  
 quality-value paradigm  
@ distinctive competency  
 comparative synergy  
 marketing myopia 

Djibouti is a former French colony in eastern Africa, bordering the Gulf of Aden and the Red Sea. It is between Eritrea and Somalia. The nation is heavily dependent on foreign assistance to help support its balance of payments and to finance development projects. Djibouti has an unemployment rate of about 50 percent. Over the last seven years, per capita consumption has dropped an estimated 35 percent due to a recession. This is a description of the country's _____ environment.  
@  economic  
 natural  
 technological  
 legal  
 sociocultural 

 Dreyer's produces several kinds of premium ice cream including Dreyer's Dreamery, Starbucks, and Godiva. The distribution system it has set up with its retailers allows Dreyer's to ship ice cream directly from its manufacturing plants to stores without using warehouses. This distribution method reduces the use of natural resources because the company does not have to carry its products to and from warehouses. Dreyer's distribution system indicates the company has opted for a position of:  
  economic responsiveness  
 ethical commitment  
 ethical democracy  
@ environmental stewardship  
 gray marketer 

Each year, Honeywell asks every department manager to rate his or her department's strengths and weaknesses as well as those of the other departments with which the department interacts. Then each department manager is asked what he or she sees as the greatest threat and/or opportunity for the company. Honeywell has its department managers engage in an annual:  
@  SWOT analysis  
 portfolio analysis  
 value audit  
 functional audit  
 compatibility assessment  

 For years, Christian music was ignored, but in 2001, the Christian music industry boomed. Although sales of mainstream music dropped by 3 percent, the sale of Christian music grew by 13.5 percent. One reason for the growing popularity of Christian music is "the realization that Christian lyrics try to point to something that eases the pain while secular music just wallows in it." The growing popularity of Christian music is an example of a change in the _____ environment.  
  economic  
 natural  
 technological  
 legal  
@ sociocultural 

Goodwin Enterprises makes cloth dolls like the ones children played with in the nineteenth century. The company has been approached by the Cracker Barrel restaurant chain to see if it would like to sell its dolls in Cracker Barrel stores. The decision whether to make the dolls available at so many outlets across the United States would be part of which marketing mix element?  
  production  
 promotion  
@ distribution  
 pricing  
 human resources 

In 1998, the Federal Trade Commission implemented the Children's Online Privacy Protection Act, which stipulates methods businesses must use to get parental consent for children to visit their Web sites and protects any information gained from children in the course of their visiting the site. The law was the direct result of:  
  utilitarianism  
 consumer idealism  
 social responsibility  
@ consumerism  
 individualism  

In 2002, Home Depot, the building and gardening supply retail giant, voluntarily discontinued the sale of wood products made from endangered forests as part of its _____ strategy.  
  gray marketing  
 ethical diversity  
 cause marketing  
 black marketing  
@ green marketing 

In 2002, Krispy Kreme doughnuts became available outside of North America for the first time with the opening of a Krispy Kreme shop in Australia. Krispy Kreme used a _____ strategy.  
@  market development  
 product penetration  
 market penetration  
 diversification  
 product development

 In 2002, Wendy's announced plans to buy a chain of Mexican food restaurants. Wendy's International, the nation's third largest hamburger chain, purchased the 169-restaurant Baja Fresh chain to appeal to consumers who do not like fast food. Wendy's used a _____ strategy.  
  market development  
 product penetration  
 market penetration  
@ diversification  
 product development 

In its advertising, Serta claims, "We make the world's best mattress." This claim is an example of:  
  status quo advertising  
@ puffery  
 dynamic advertising  
 fair and equitable advertising  
 metaphoric promotion  



In _____ there are a relatively small number of sellers, each holding substantial market share in a market with many buyers.  
  a monopoly  
 monopolistic competition  
 perfect competition  
@ an oligopoly  
 oligopolistic competition 

 Interactive Custom Clothes Company offers shoppers the ability to custom design bags, dresses, jackets, pants, and other items of clothing aided by the benefit of try-before-you-buy visual sampling. The site guides you step-by-step through all the choices and measurements you'll need to create your online ensemble. Or you can send the company a good-fitting sample, and it will replicate it in whatever fabric you select from its on-screen catalog. The company's ability to respond to specific customers' needs gives it:  
  synergy  
@ a competitive advantage  
 a quality-value paradigm  
 a comparative niche  
 a competitive orientation 

John H. Harland Company is best known for printing personal and business checks, but it also owns Harland Analytical Services, a technology company that produces software, which enables banks to gauge the behavior of their customers by tracking their spending habits. In addition, Harland owns Scantron, a computerized testing and assessment company. Harland operates Harland Analytical and Scantron as individual units within the company with their own mission, objectives, and strategies. Harland Analytical and Scantron are examples of:  
  strategic alliances  
 target markets  
@ strategic business units  
 micromarketing  
 operational units 

John H. Harland Company is a large company with 5,200 employees and almost $800 million in sales in 2001. The company is best known for printing personal and business checks. Harland also owns Harland Analytical Services, a technology company that produces software, which enables banks to gauge the behavior of their customers by tracking their spending habits. In addition, Harland owns Scantron, a computerized testing and assessment company. The _____ for John H. Harland Company includes its check-printing business, its financial software business, and its testing and assessment business.  
@  business portfolio  
 marketing mix  
 market penetration  
 tactical structure  
 market classification 

John H. Harland Company is a large company with 5,200 employees and almost $800 million in sales in 2001. The company is best known for printing personal and business checks. Harland also owns Harland Analytical Services, a technology company that produces software, which enables banks to gauge the behavior of their customers by tracking their spending habits. Harland Analytical Services has only a small share of a rapidly growing market. It would be an example of a:  
@  problem child  
 dog  
 cash cow  
 star  
 harvest 

 John H. Harland Company is best known for printing personal and business checks, but it also owns Scantron, a computerized testing and assessment company. Harland's purchase of Scantron in 1988 was an example of the implementation of a _____ strategy.  
  market development  
 product penetration  
 market penetration  
@ diversification  
 product development 


 Levi-Strauss decided to end its business dealings with China because of what the company called "pervasive human rights abuses." According to the company's vice president for corporate marketing, "There are wonderful commercial opportunities in China. But when ethical issues collide with commercial appeal, we try to ensure ethics as the trump card. For us, ethical issues precede all others." The attitude reflected by Levi-Strauss's executive indicates the company's commitment to:  
  moral utilitarianism  
 ethical democracy  
@ social responsibility  
 economic responsibility  
 moral idealism 

 Managers use SWOT analyses to:  
@  assess their companies' internal and external environments  
 select channels of distribution for their products  
 gauge response to advertising campaigns  
 implement strategic plans  
 create cross-functional job descriptions for their human resources  

 Many firms practice _____, which means that instead of working alone middle managers work together as a team to develop tactical plans that consider the objectives of all the functional areas.  
  role playing  
 organizational superordination  
@ cross-functional planning  
 team synergy  
 hierarchical implementation 



Market analysts use the term _____ to describe companies that have a shortsighted view of their company and their product.  
  paralysis by analysis  
 MBWA  
 entropic waste  
@ marketing myopia  
 market astigmatism

 Marketing mix strategies are crucial to a firm's success in achieving its objectives, but not all marketing mix strategies are ethical. Which of the following is an example of an unethical marketing mix strategy?  
  The company uses both radio and newspaper advertising.  
 The company decides to change its name to support an expanded role in its industry.  
 The company offers lower prices if consumers will buy six or more of its product.  
 The company develops a short, nonspecific code of ethics.  
@ The company rushes production on a product that has certain inherent safety flaws, which it does not reveal to the public.

 Mohawk Industries manufactures carpeting, hardwood flooring, and ceramic tile. Although there are several companies that produce similar products, Mohawk is credited with having the most efficient warehousing and distribution system. Its distribution strategy is its:  
  lack of a mission statement  
 quality-value paradigm  
@ distinctive competency  
 comparative synergy  
 marketing myopia  

Most ice cream lovers will tell you Edy's is a high-quality brand of ice cream. In 2002, Edy's introduced Godiva ice cream and Starbucks ice cream to tempt ice cream lovers to eat even more ice cream. Edy's used a _____ strategy.  
  market development  
 product penetration  
 market penetration  
 diversification  
@ product development  


Nearly everyone who has ever needed a flashlight has experienced problems with the batteries that seem to coincide with the need for the flashlight. J&K Enterprises has developed magnetic-powered batteries. The flashlight is recharged by shaking it. The ability of J&K Enterprises to develop new products such as this flashlight that satisfy real consumer needs illustrates its _____, something the organization does well.  
  synergy  
@ competency  
 benefitization  
 entropy  
 resource-expertise 



Once managers have a thorough understanding of the marketing environment, the next step in the marketing planning process is to:  
  develop a marketing strategy  
 brainstorm  
 develop action programs for modifying the marketing environment  
 modify the organizational mission  
@ set marketing objectives  

 Serengetti makes lightweight sunglasses with 100 percent UV protection for people who love to hunt, hike, and bike ride. Its long-term plans include the development of lenses for people who fish that in addition to protecting users from UV rays will have effective water-sheeting action to reduce lens spotting. Serengetti has determined that fishing is one of the fastest growing sports in the United States. In terms of a SWOT analysis, Serengetti has recognized a market:  
  weakness  
 strength  
 threat  
@ opportunity 

 Serengetti makes lightweight sunglasses with 100 percent UV protection for people who love to hunt, hike, and bike ride. Its long-term plans include the development of lenses for people who fish that in addition to protecting users from UV rays will have effective water-sheeting action to reduce lens spotting. The company believes it can use its reputation as a sunglasses manufacturer for a specialized market in combination with its current manufacturing facilities and labor force to enter this new market. In terms of a SWOT analysis, Serengetti will be relying on its marketing _____ to make its new product a success.  
  weaknesses  
@ strengths  
 threats  
 opportunities 

Some Web sites are well-organized, have the information visitors are most likely to need readily available, and are easy to navigate. These well-designed Web sites realize customers want information that is convenient for them and illustrate how:  
  gray marketing can influence product design  
 distribution can create quality  
 product concerns can be overcome with cause marketing  
 ISO 14000 standards are used on the Internet  
@ quality concerns can influence marketing communications  
 

Starkist, Chicken of the Sea, and Bumble Bee brands of canned tuna are all part of each other's _____ environment.  
  economic  
 natural  
 technological  
 legal  
@ competitive  

 Teddy Grahams are small graham cookies shaped like teddy bears. Nabisco, which makes Teddy Grahams, created Teddy Graham Bearwiches cookies (two Teddy Grahams stuck together with a cream filling) to implement its _____ strategy. Nabisco believed its Teddy Grahams customers would buy the new product as well.  
  market development  
 product penetration  
 market penetration  
 diversification  
@ product development 

The acronym TQM refers to:  
  targeted quantity marketing  
 trend and quality measurement  
 tactical quantification of markets  
 tactical quota management  
@ total quality management

 The business cycle includes periods of:  
  prosperity, recession, and inflation  
 recession, depression, and inflation  
@ prosperity, recession, and recovery  
 recession, depression, and prosperity  
 inflation, recovery, and prosperity  

 The decision by the manufacturer of Edy's ice cream to develop and market Starbucks Fresh Roasted Ice Cream and Frappuccino Blended Coffee Bars along with the company's other types of ice cream was an example of a _____ strategy.  
  production  
 promotion  
 distribution  
 pricing  
@ product

The external macro business environment for a company that manufactures gasoline-powered electric generators for campers would include:  
@  the economy in which it operates  
 its technological competencies  
 its financial resources  
 the relationships it has established with its suppliers and its intermediaries  
 all of the above 

The external microenvironment for a company that manufactures gasoline-powered electric generators for campers would include:  
@  a target market of campers who buy the generators  
 a back-to-nature movement that encourages wilderness camping  
 changes in the economy that make luxury vacations unaffordable for most families  
 overcrowding in some national state parks  
 all of the above 

 The formal marketing plan:  
  describes the marketing environment  
 identifies who will be responsible for carrying out each part of the marketing strategy  
 outlines the marketing objectives and strategies  
 is often a key factor in attracting investors to the company  
@ is accurately described by all of the above 

The growth-market share matrix developed by the Boston Consulting Group (BCG) is an example of a type of:  
  SWOT analysis  
@ portfolio analysis  
 value audit  
 situational audit  
 compatibility assessment 

 The Malcolm Baldrige Award is most closely related to:  
  green marketing  
 cultural diversity programs  
 environmental stewardships  
@ total quality management  
 cause marketing 

 The outbreak of diseases in Europe forced the widespread slaughter of many farmers' entire herds of livestock. In addition, the fear of eating contaminated meat caused a large number of tourists to cancel their plans to visit Europe. This is a description of how a serious animal epidemic can influence the _____ environment.  
@  economic  
 natural  
 technological  
 legal  
 sociocultural

 The Product-Market Growth Matrix:  
@  provides managers with four different fundamental marketing strategies-market penetration, market development, product development, and diversification  
 is a graphic illustration of what tactics should be used within a marketing strategy for products at different growth stages  
 graphically illustrates whether a manager should use personal marketing communications or mass marketing communications  
 helps top management identify and label their various SBUs  
 lists strengths, weaknesses, opportunities, and threats that can impede the growth of a market 

"The purpose of Motorola is to honorably serve the needs of the community by providing products and services of superior quality at a fair price to our customers; to do this so as to earn an adequate profit which is required for the total enterprise to grow; and by doing so provide the opportunity for our employees and shareholders to achieve their reasonable personal objectives." The preceding statement is Motorola's:  
  tactical plan  
 strategic objective  
 marketing objective  
 strategic plan  
@ mission statement 

 The U.S. Food and Drug Administration and the Environmental Protection Agency are part of the _____ macroenvironment for many companies.  
  economic  
 customer  
 competitive  
---->robert-->legal  
---->kai@ sociocultural

 The way the U.S. Food and Drug Administration carefully tests new drugs to make sure they (1) do what the manufacturers claim and (2) have no serious side effects is an example of the implementation of which of the rights identified by the Consumer Bill of Rights?  
@  right to safety  
 right to equal treatment  
 right to be informed  
 right to be heard  
 right to choose freely 

 To develop strategic plans, top-level corporate managers follow three steps. The first of these steps is to:  
  recruit and hire the right personnel  
 examine historical data  
@ develop a mission for the total corporation  
 allocate resources to the company's various SBUs  
 establish the corporation's long-term objectives 

Unilever's decision to create specialized tea mixes that appeal to the culture of the country in which they are sold is an example of a _____ strategy.  
  production  
 promotion  
 distribution  
 pricing  
@ product 



 Unilever makes Rexona deodorant and antiperspirant, and it is the world's number one brand of deodorant and antiperspirant. Because the worldwide market for deodorants and antiperspirants is growing rapidly, Unilever would classify Rexona as a _____ on the BCG growth-share matrix.  
  problem child  
 dog  
 cash cow  
@ star  
 question mark 

 Waverly is a manufacturer of wallpaper and matching fabrics, which can be used to make curtains, bedspreads, tablecloths, and chair coverings. You can purchase Waverly products on its Web site and at leading department stores, major specialty stores, and independent paint and wallpaper stores. Providing the ability to purchase Waverly products in so many outlets is part of the company's _____ strategy.  
  production  
 promotion  
@ distribution  
 pricing  
 product 

Weitz and Leupold make optical lenses. The company's Germanic origin has caused it to develop an almost Puritanical-like ethics code that tolerates no deviation from expected behavior. Employees are expected to report on fellow workers who do not adhere to the company norms. This is a brief description of the company's:  
  cultural autonomy  
 managerial portfolio  
@ corporate culture  
 management hierarchy  
 organizational chart 

 When a firm adopts a(n) _____ program, it makes sure its marketing policies and hiring practices give all people an equal chance to buy its products and to work for the company.  
  philanthropic  
@ cultural diversity  
 ethical stewardship  
 economic responsibility  
 cause marketing 

 When Lanae went to the supermarket, she was planning on buying one box of Celestial Seasonings herbal tea. When she got to the store, she discovered that if she purchased two boxes of Celestial Seasonings tea, she would receive a third box for free. Celestial Seasonings was using a _____ strategy.  
  market development  
 product penetration  
@ market penetration  
 diversification  
 product development 

When the FTC determines a company is using deceptive advertising, it can require the company to run _____, which clarifies or qualifies previous promotional messages.  
@  corrective advertising  
 puffery  
 dynamic advertising  
 fair and equitable advertising  
 remedial promotion 

Whenever you purchase an antibiotic or some other prescription medication, you are given a computer printout that explains how to take the medication, possible side effects, and what to do if you experience one of the side effects. This information illustrates the implementation of which of the rights identified by the Consumer Bill of Rights?  
  right to safety  
 right to equal treatment  
@ right to be informed  
 right to be heard  
 right to choose freely  

 Which of the following is a common criticism of marketing?  
@  Marketing promises miracles.  
 Products meet existing needs, and advertising helps to communicate their availability.  
 Marketing teaches us to value people for who they are rather than what they own.  
 Marketing activities are used to clean up the environment.  
 Marketers recommend ways to satisfy consumers' needs.  


 Which of the following is an example of brand competition?  
  Domino's pizza, Papa John's pizza, and Pizza Hut pizza  
 Disneyland, Panama City, Florida, and New York City  
 Starkist tuna, Oscar Mayer bologna, Peter Pan peanut butter, and Kraft mayonnaise  
 flour, sugar, baking powder, and eggs  
@ Huggies disposable diapers, Gerber baby food, and Enfamil baby formula  


 Which of the following is an example of product competition?  
  a movie theater and a video rental store  
 a sports drink, a soda, and a bottle of water  
 an amusement park, a Caribbean cruise, and a beach resort  
 pizza, hamburgers, deli sandwiches  
@ all of the above
 
Which of the following elements of the marketing mix strategy has the greatest potential for unethical use?  
@  promotion  
 product  
 distribution  
 form utility  
 production 

 Which of the following statements about ethical business behavior is true?  
  The tenets of ethical business behavior are the same worldwide.  
 Culture does not influence business ethics.  
@ Because each culture has its own set of values, beliefs, and customs, ethical business behaviors vary in different parts of the world.  
 Successful international companies typically adopt the ethical business behavior of the country in which they are doing business.  
 All companies in a single culture will have similar ethical business behavior.  

 Which of the following statements about social responsibility is true?  
@  Social responsibility includes cause marketing and environmental stewardship.  
 Firms that believe in social responsibility possess a value system that focuses on short-term profits.  
 Firms that believe in social responsibility believe it is their duty to earn as much profit as possible for their employees and their investors.  
 Cultural diversity is not a concern for firms that believe in social responsibility.  
 all of the above 

Which of the following types of managers would be most likely to engage in operational planning on a daily basis?  
  the company's CEO  
@ the company's sales manager  
 the company's director of product research and development  
 the company's vice-president of human resources  
 the company's chief financial officer 

  Xerox exemplifies the use of a(n) _____ strategy with its "Design for the Environment" program, which focuses on ways to make its products recyclable and remanufacturable.  
  gray marketing  
 ethical diversity  
 cause marketing  
 black marketing  
@ green marketing 

 Zeal Media is a community-driven Web directory that features Web site ratings and reviews. Because the site was designed to be of the people, by the people, and for the people, Zeal Media needed people not only to visit the site but also to contribute to it by sending in reviews. The company instituted a policy of donating 2 cents for each new review and 20 cents for each new directory listing to a not-for-profit organization selected by the community. Zeal Media used a(n) _____ strategy to increase its service offering.  
  gray marketing  
 ethical stewardship  
@ cause marketing  
 black marketing  
 green marketing 

 _____ is a management philosophy in which organizations engage in activities that have a positive effect on society and promote the public good.  
  Moral utilitarianism  
 Ethical democracy  
@ Social responsibility  
 Economic responsibility  
 Moral idealism 

 _____ is the managerial decision process that matches the organization's resources and capabilities to its market opportunities for long-term growth.  
  Operational planning  
@ Strategic planning  
 Strategic implementation  
 Tactical implementation  
 Tactical planning 

 _____ is a philosophy that calls for companywide dedication to the development, maintenance, and continuous improvement of all aspects of the company's operations.  
  Customer relationship marketing (CRM)  
 Efficient organizational management (EOM)  
 Cause-related marketing (CRM)  
 Ethical marketing  
@ Total quality management (TQM) 


 _____ is the social movement directed toward protecting consumers from harmful business practices.  
  Utilitarianism  
 Consumer idealism  
 Social responsibility  
@ Consumerism  
 Individualism 

 _____ refers to claims of superiority that neither sponsors nor critics of the ads can prove as true or untrue.  
  Status quo advertising  
@ Puffery  
 Dynamic advertising  
 Fair and equitable advertising  
 Metaphoric promotions 

3M instituted the "Pollution Prevention Pays" program to reduce pollution and recycle materials. Since its inception, this program has generated more than 4,000 ideas that eliminated more than 1.6 billion pounds of air, water, and sold waste pollutants from the environment. The "Pollution Prevention Pays" program indicates 3M has assumed a position of:  
  economic responsiveness  
 ethical commitment  
 ethical democracy  
@ environmental stewardship  
 gray marketer 

 Question 6    Multiple Choice  0 of 1 point(s)    
  Orkin knows homeowners skimp on maintenance services such as pest control during recessions. It also knows that the more consumers know about bugs, the more likely they are to buy its services. So it teamed with the Smithsonian Institution on the Insect Safari, a traveling exhibit designed to teach people about the critters. The Safari truck visited 108 cities during one year, leaving a wake of creeped-out insectophobes reaching for the phone numbers of the local Orkin franchise. Business increased in each city visited. The Insect Safari was an example of a _____ strategy.  
 Your Answer:  [None Given]  
Correct Answer:  promotion  
 


  Question 7    Multiple Choice  0 of 1 point(s)    
  The external microenvironment for a company that manufactures gasoline-powered electric generators for campers would include:  
 Your Answer:  [None Given]  
Correct Answer:  a target market of campers who buy the generators  
 


  Question 8    Multiple Choice  0 of 1 point(s)    
  The growth-market share matrix developed by the Boston Consulting Group (BCG) is an example of a type of:  
 Your Answer:  [None Given]  
Correct Answer:  portfolio analysis  
 


  Question 9    Multiple Choice  0 of 1 point(s)    
  When the FTC determines a company is using deceptive advertising, it can require the company to run _____, which clarifies or qualifies previous promotional messages.  
 Your Answer:  [None Given]  
Correct Answer:  corrective advertising  
 


  Question 10    Multiple Choice  0 of 1 point(s)    
  "Challenge Day provides youth and their communities with experiential workshops and programs that demonstrate the possibility of love and connection through the celebration of diversity, truth and full expression." The preceding statement is Challenge Day's:  
 Your Answer:  [None Given]  
Correct Answer:  mission statement  
 


Question 1    Multiple Choice  1 of 1 point(s)    
  Which of the following statements about ethical business behavior is true?  
 Your Answer:  Because each culture has its own set of values, beliefs, and customs, ethical business behaviors vary in different parts of the world.  
Correct Answer:  Because each culture has its own set of values, beliefs, and customs, ethical business behaviors vary in different parts of the world.  
 


  Question 2    Multiple Choice  1 of 1 point(s)    
  Mohawk Industries manufactures carpeting, hardwood flooring, and ceramic tile. Although there are several companies that produce similar products, Mohawk is credited with having the most efficient warehousing and distribution system. Its distribution strategy is its:  
 Your Answer:  distinctive competency  
Correct Answer:  distinctive competency  
 


  Question 3    Multiple Choice  1 of 1 point(s)    
  The growth-market share matrix developed by the Boston Consulting Group (BCG) is an example of a type of:  
 Your Answer:  portfolio analysis  
Correct Answer:  portfolio analysis  
 


  Question 4    Multiple Choice  1 of 1 point(s)    
  In 2002, Krispy Kreme doughnuts became available outside of North America for the first time with the opening of a Krispy Kreme shop in Australia. Krispy Kreme used a _____ strategy.  
 Your Answer:  market development  
Correct Answer:  market development  
 


  Question 5    Multiple Choice  1 of 1 point(s)    
  Argo is the world's leading brand of small vehicles that are designed to be driven on all types of terrains and in all types of weather. The vehicles can carry up to six passengers. The market for such vehicles has become stagnant due to the recent economic downturn. In terms of the BCG growth-market share matrix, Argo would be classified as a:  
 Your Answer:  cash cow  
Correct Answer:  cash cow  
 


  Question 6    Multiple Choice  1 of 1 point(s)    
  Interactive Custom Clothes Company offers shoppers the ability to custom design bags, dresses, jackets, pants, and other items of clothing aided by the benefit of try-before-you-buy visual sampling. The site guides you step-by-step through all the choices and measurements you'll need to create your online ensemble. Or you can send the company a good-fitting sample, and it will replicate it in whatever fabric you select from its on-screen catalog. The company's ability to respond to specific customers' needs gives it:  
 Your Answer:  a competitive advantage  
Correct Answer:  a competitive advantage  
 


  Question 7    Multiple Choice  0 of 1 point(s)    
  The U.S. Food and Drug Administration and the Environmental Protection Agency are part of the _____ macroenvironment for many companies.  
 Your Answer:  physical  
Correct Answer:  legal  
 


  Question 8    Multiple Choice  1 of 1 point(s)    
  Archetype is a Unilever SBU that markets a number of different tea brands around the world. Each brand addresses the taste and emotions of local tea cultures. Which of the following would be part of Archetype's internal environment?  
 Your Answer:  the marketing Archetype does to reach consumers and satisfy their cultural needs  
Correct Answer:  the marketing Archetype does to reach consumers and satisfy their cultural needs  
 


  Question 9    Multiple Choice  1 of 1 point(s)    
  When the FTC determines a company is using deceptive advertising, it can require the company to run _____, which clarifies or qualifies previous promotional messages.  
 Your Answer:  corrective advertising  
Correct Answer:  corrective advertising  
 


  Question 10    Multiple Choice  1 of 1 point(s)    
  Xerox exemplifies the use of a(n) _____ strategy with its "Design for the Environment" program, which focuses on ways to make its products recyclable and remanufacturable.  
 Your Answer:  green marketing  
Correct Answer:  green marketing  
 


 Question 2    Multiple Choice  0 of 1 point(s)    
  _____ is a practice where a seller sets different prices for different customers based on the customers' assumed ability or willingness to pay. The practice is considered legal but unfair to customers.  
 Your Answer:  [None Given]  
Correct Answer:  Dynamic pricing  
 


  Question 3    Multiple Choice  0 of 1 point(s)    
  Waverly is a manufacturer of wallpaper and matching fabrics, which can be used to make curtains, bedspreads, tablecloths, and chair coverings. You can purchase Waverly products on its Web site and at leading department stores, major specialty stores, and independent paint and wallpaper stores. Providing the ability to purchase Waverly products in so many outlets is part of the company's _____ strategy.  
 Your Answer:  [None Given]  
Correct Answer:  distribution  
 


  Question 4    Multiple Choice  0 of 1 point(s)    
  A New Era company that wishes to tackle a social problem by joining forces with a not-for-profit organization like The Nature Conservancy or the American Cancer Society should use a(n) _____ strategy.  
 Your Answer:  [None Given]  
Correct Answer:  cause marketing  
 


  Question 5    Multiple Choice  0 of 1 point(s)    
  A small independent retailer set the following marketing objective: "To make the store more inviting to customers during the next year." This would be an example of a(n) _____ objective.  
 Your Answer:  [None Given]  
Correct Answer:  qualitative  
 


  Question 6    Multiple Choice  0 of 1 point(s)    
  Changes in demographics reflect changes in the _____ environment.  
 Your Answer:  [None Given]  
Correct Answer:  sociocultural  
 


  Question 7    Multiple Choice  0 of 1 point(s)    
  Progresso soup ran a series of commercials targeted to young adults in which it compared its chicken noodle soup to "that soup you ate when you were a kid." The ads were trying to get people who regularly eat soup to buy Progresso brand soup instead of Campbell's. A strategy that attracts users of competitive brands is defined as a _____ strategy.  
 Your Answer:  [None Given]  
Correct Answer:  market penetration  
 


  Question 8    Multiple Choice  0 of 1 point(s)    
  Which of the following types of managers would be most likely to engage in operational planning on a daily basis?  
 Your Answer:  [None Given]  
Correct Answer:  the company's sales manager  
 


  Question 9    Multiple Choice  0 of 1 point(s)    
  Each year, Honeywell asks every department manager to rate his or her department's strengths and weaknesses as well as those of the other departments with which the department interacts. Then each department manager is asked what he or she sees as the greatest threat and/or opportunity for the company. Honeywell has its department managers engage in an annual:  
 Your Answer:  [None Given]  
Correct Answer:  SWOT analysis  
 


  Question 10    Multiple Choice  0 of 1 point(s)    
  Which of the following statements about ethical business behavior is true?  
 Your Answer:  [None Given]  
Correct Answer:  Because each culture has its own set of values, beliefs, and customs, ethical business behaviors vary in different parts of the world.  
 

According to the instructor, what is a major weakness in strategic plans?  
  Strategic plans can only be developed for long-range time horizons.  
@ There is no clear linkage between long-range strategic goals and short-term tactical goals  
 There are no outcome measures associated with strategic plans.  
 Strategic goals are not measurable.  
 Strategic plans are only as good as the people who create them.  
 


  Question 2    Multiple Choice  1 points    
   Bruce is designing an introductory marketing class. He wants a class where students learn a lot about marketing and are satisfied with the educational experience. Which of the following would be a PROCESS measure for the class?  
  The scores for the final examination.  
@ The percentage of students who take the practice tests.  
 The change in the class evaluations from one semester to the next.  
 The percentage of students who report they are highly satisfied with the class on an exit survey.  
 


  Question 3    Multiple Choice  1 points    
   Djibouti is located on the eastern coast of Africa where it is strategically located near the world's busiest shipping lanes. Its port and railroad system provides easy access to Ethiopia and other landlocked African nations in the region. The ability to use the Djibouti port would give a distribution company that targeted Ethiopia a:  
@  competitive advantage  
 localization basis  
 local personalization  
 SWOT analysis opportunity  
 disintermediation opportunity  
 


  Question 4    Multiple Choice  1 points    
   In February 2000, the United States imposed a tax on imported steel in an effort to protect about 5,000 U.S. jobs. Another name for this levied tax is a(n):  
  embargo  
 gray tax  
@ tariff  
 import quota  
 boycott  
 


  Question 5    Multiple Choice  1 points    
   The North American Free Trade Agreement (NAFTA):  
  is the world's smallest economic community  
 will never be expanded to include other members  
@ has only three members  
 is one of the few such organizations that has been free of controversy  
 is accurately described by all of the above  
 


  Question 6    Multiple Choice  1 points    
   If tactical planning is to succeed, each "management unit" should know:  
  What time lunch is served.  
@ What the unit is expected to do in order to achieve the goal.  
 The budget allocated to all functional areas in the organization.  
 Who their "coopitors" in the organization are.  
 


  Question 7    Multiple Choice  1 points    
   Which of the following statements about traditional organizational structure is true?  
  Functional silos are eliminated.  
@ There are clear lines of authority.  
 Marketing is a function of the sales department.  
 Traditional organization structure is necessary for very large organizations.  
 


  Question 8    Multiple Choice  1 points    
   Bruce is designing an introductory marketing class. He wants a class where students learn a lot about marketing and are satisfied with the educational experience. Which of the following would be a PROCESS measure for the class?  
  The score on a marketing test given on the first day of class.  
 The score on a marketing test given on the last day of class.  
 the difference between the marketing test scores from the first and the last day of class  
@ The percentage of students who buy the textbook required for the class.  
 


  Question 9    Multiple Choice  1 points    
   If tactical planning is to succeed, each "management unit" should know:  
@  The deadline for achieving the assigned task.  
 Whether or not the organization is organized along functional or traditional lines  
 The budget allocated to all functional areas in the organization.  
 Whether a specific metric is a process measure or an outcome measure.  
 


  Question 10    Multiple Choice  1 points    
   After exporting, the next level of commitment a firm can make to a foreign market is a contractual agreement. Two of the most common types of such agreements are:  
  contract manufacturing and contract marketing  
 contract manufacturing and strategic alliances  
 joint ventures and franchising  
 warehousing and financing  
@ licensing and franchising  
 
============

All of the following are recognized by Michael Porter as keys to a nation's competitive advantage EXCEPT:  
 Your Answer:  company strategy, structure, and rivalry  
Correct Answer:  reintermediation  
 


  Question 2    Multiple Choice  1 of 1 point(s)    
  Which of the following is a form of protectionism?  
 Your Answer:  local content rules  
Correct Answer:  local content rules  
 


  Question 3    Multiple Choice  1 of 1 point(s)    
  Which of the following statements about traditional organizational structure is true?  
 Your Answer:  Sales is a function of the marketing department  
Correct Answer:  Sales is a function of the marketing department  
 


  Question 4    Multiple Choice  0 of 1 point(s)    
  According to the instructor, what is a major weakness in strategic plans?  
 Your Answer:  Strategic goals are not measurable.  
Correct Answer:  There is no clear linkage between long-range strategic goals and short-term tactical goals  
 


  Question 5    Multiple Choice  1 of 1 point(s)    
  According to your instructor, what are the two things you need to know about a number used as a measurement to understand it?  
 Your Answer:  Is it good or bad, is it getting better?  
Correct Answer:  Is it good or bad, is it getting better?  
 


  Question 6    Multiple Choice  0 of 1 point(s)    
  In February 2000, the United States imposed a tax on imported steel in an effort to protect about 5,000 U.S. jobs. Another name for this levied tax is a(n):  
 Your Answer:  gray tax  
Correct Answer:  tariff  
 


  Question 7    Multiple Choice  0 of 1 point(s)    
  According to your instructor, why do you have so many group projects in the college of busines?  
 Your Answer:  Because students learn better in a group setting.  
Correct Answer:  Because businesses want students who can work in cross-functional teams  
 


  Question 8    Multiple Choice  0 of 1 point(s)    
  Bruce is designing an introductory marketing class. He wants a class where students learn a lot about marketing and are satisfied with the educational experience. Which of the following would be an outcome measure for the class?  
 Your Answer:  The number of times each student takes mini test 6.  
Correct Answer:  The college of business class evaluation results  
 


  Question 9    Multiple Choice  0 of 1 point(s)    
  In a trend analysis, marketers:  
 Your Answer:  create the perfect target market and then try to locate markets that have many, if not all, of the same characteristics  
Correct Answer:  use data such as company and industry sales over a period of years to understand past directions that may continue in the future  
 


  Question 10    Multiple Choice  0 of 1 point(s)    
  Which of the following statements about budgeting is consistent with the information presented in class?  
 Your Answer:  Budgeting is based on a top-down process.  
Correct Answer:  Budgeting is the process through which tactical plans are made real.  
 
===========

 U.S. sugar import quotas have existed for over 50 years and preserve about half of the U.S. sugar market for domestic producers. These quotas are examples of:  
  a political asylum  
 synergistic manipulation  
@ protectionism  
 world-centrism  
 disintermediation  
 


  Question 2    Multiple Choice  1 points    
   Which of the following statements about the European Union (EU) is true?  
@  The EU is an example of an economic community.  
 Every country in Europe is a member of the EU.  
 Not only is every European country a member of the EU, but also every other country in the world that is currently ruled by a European country.  
 In the beginning, the European Union had more than twenty member nations.  
 None of the above statements about the EU is true.  
 


  Question 3    Multiple Choice  1 points    
   Prior to the creation of the European Union (EU), the German government required all beer manufacturers (foreign and domestic) who wished to sell to the German market to contain a high percentage of German-grown hops. This law was an example of a(n):  
  environmental protection law  
 80/20 ruling  
 import quota  
 export quota  
@ local content rule  
 


  Question 4    Multiple Choice  1 points    
   Several Arab nations completely prohibit the importation of Coca-Cola because it maintains distributors in Israel. The Arab nations have instituted a(n) _____ against Coke.  
  gray tax  
@ embargo  
 barrier to entry  
 supply-based infrastructure  
 export prohibition  
 


  Question 5    Multiple Choice  1 points    
   What was your instructor talking about when he suggested getting buy-in from stakeholders when developing metrics.  
  The budget for implementing the evaluation can come from contributions from the divisions being evaluated.  
 The un-aimed arrow never misses.  
 It is important for supervisors to determine exactly how their subordinates will be evaluated before the plan is implemented.  
@ In order to be effective, both the measurement and how it will be interpreted have to be agreed to in advance.  
 I have no clue, the instructor wasn't exactly clear on this point.  
 


  Question 6    Multiple Choice  1 points    
   _____ allow companies easy access to new markets, especially because their partnerships often bring with them preferential treatment in the partner's home country.  
  Transactional relationships  
 Product-oriented relationships  
 Exchange relationships  
@ Strategic alliances  
 Expropriation agreements  
 


  Question 7    Multiple Choice  1 points    
   According to your instructor, why do you have so many group projects in the college of busines?  
  Because teachers are too lazy to grade individual papers  
 Because students enjoy them so much  
@ Because businesses want students who can work in cross-functional teams  
 Because it is important for students to meet and to get to know each other.  
 Because students learn better in a group setting.  
 


  Question 8    Multiple Choice  1 points    
   A market audit:  
  typically does not involve nonmarketing personnel in any way  
 is necessarily subjective because it is prepared by the department being audited  
 cannot provide feedback on specific marketing plans  
@ is used to determine if a firm can improve its marketing programs  
 is all of the above  
 


  Question 9    Multiple Choice  1 points    
   According to your instructor, what are the two things you need to know about a number used as a measurement to understand it?  
  Is it up or down, is it getting better?  
@ Is it good or bad, is it getting better?  
 Is it objective or subjective, is it getting better?  
 Is is numeric or percentage, is it getting better?  
 Is it continuous or interval, was it good for you too?  
 


  Question 10    Multiple Choice  1 points    
   Which of the following statements about world trade is true?  
  World trade refers to the flow of goods and services among different countries.  
 World trade activity is steadily increasing year by year.  
 World trade refers to the value of all exports and imports of the world's nations.  
 Not all countries participate equally in world trade.  
@ All of the above statements about world trade are true.  
 
=============

To enter the European market, Starbucks joined in a cooperative venture with Bon Appetit Group A.G. in Switzerland. Bon Appetit has the recognized brand name and Starbucks has the product and the expertise to run coffeehouses. This is an example of a(n):  
  transactional relationship  
 product-oriented relationship  
 exchange relationship  
@ strategic alliance  
 expropriation agreement  
 


  Question 2    Multiple Choice  1 points    
   If tactical planning is to succeed, each "management unit" should know:  
@  The deadline for achieving the assigned task.  
 Whether or not the organization is organized along functional or traditional lines  
 The budget allocated to all functional areas in the organization.  
 Whether a specific metric is a process measure or an outcome measure.  
 


  Question 3    Multiple Choice  1 points    
   Several Arab nations completely prohibit the importation of Coca-Cola because it maintains distributors in Israel. The Arab nations have instituted a(n) _____ against Coke.  
  gray tax  
@ embargo  
 barrier to entry  
 supply-based infrastructure  
 export prohibition  
 


  Question 4    Multiple Choice  1 points    
   Which of the following statements about sales volume measures is true?  
  Dollar sales gives you a better picture of performance than unit sales figures.  
 Sales volume is always an outcome measure.  
 Sales volume is a process measure, dollar sales is an outcome measure.  
@ You can only tell if sales volume is good or bad by comparing the results to a benchmark such as a marketing plan.  
 


  Question 5    Multiple Choice  1 points    
   Which of the following is an advantage of a multinational corporation?  
  Exempt from local regulations.  
@ The ability to locate production facilities where there is an economic advantage.  
 The ability to quickly adapt to changing local conditions.  
 Immunity from expropriation or nationalization.  
 


  Question 6    Multiple Choice  1 points    
   Why is it important to measure market share relative to the competition?  
@  Because the company with the largest market share usually has an advantage in the marketplace.  
 Companies with the largest market share tend to employ "push" strategies.  
 Companies with the largest market share tend to employ "pull" strategies.  
 Because if you have a smaller market share than your competitor, it is important to allocate advertising dollars more efficiently.  
 


  Question 7    Multiple Choice  1 points    
   A market audit:  
  typically does not involve nonmarketing personnel in any way  
 is necessarily subjective because it is prepared by the department being audited  
 cannot provide feedback on specific marketing plans  
@ is used to determine if a firm can improve its marketing programs  
 is all of the above  
 


  Question 8    Multiple Choice  1 points    
   If tactical planning is to succeed, each "management unit" should know:  
  What time lunch is served.  
@ What the unit is expected to do in order to achieve the goal.  
 The budget allocated to all functional areas in the organization.  
 Who their "coopitors" in the organization are.  
 


  Question 9    Multiple Choice  1 points    
   According to your instructor, why do you have so many group projects in the college of busines?  
  Because teachers are too lazy to grade individual papers  
 Because students enjoy them so much  
@ Because businesses want students who can work in cross-functional teams  
 Because it is important for students to meet and to get to know each other.  
 Because students learn better in a group setting.  
 


  Question 10    Multiple Choice  1 points    
   Which of the following statements about the European Union (EU) is true?  
@  The EU is an example of an economic community.  
 Every country in Europe is a member of the EU.  
 Not only is every European country a member of the EU, but also every other country in the world that is currently ruled by a European country.  
 In the beginning, the European Union had more than twenty member nations.  
 None of the above statements about the EU is true.  
 
=============

What does it mean when we speak of flatter organizational structures?  
@  There are fewer people you have to talk to in order to get approval for a decision.  
 There are too many middle managers who say nice things about their superiors in order to get a promition.  
 The hierarchical structure is more rigid than traditional organizations.  
 Information learned from the customer is carefully filtered and interpreted in order to help senior managers mage the right decision.  
 


  Question 2    Multiple Choice  1 points    
   In a trend analysis, marketers:  
@  use data such as company and industry sales over a period of years to understand past directions that may continue in the future  
 use the Boston Consulting Group growth-market share matrix to identify strategic business units  
 employ computer simulation software to evaluate contingency plans  
 create the perfect target market and then try to locate markets that have many, if not all, of the same characteristics  
 determine which sales promotion works best for each product  
 


  Question 3    Multiple Choice  1 points    
   After exporting, the next level of commitment a firm can make to a foreign market is a contractual agreement. Two of the most common types of such agreements are:  
  contract manufacturing and contract marketing  
 contract manufacturing and strategic alliances  
 joint ventures and franchising  
 warehousing and financing  
@ licensing and franchising  
 


  Question 4    Multiple Choice  1 points    
   Which of the following is a consequence of a trade deficit?  
  Currency strengthens  
@ Negative impact on the economy  
 Gray market dumping  
 Increased popularity for local politicians  
 


  Question 5    Multiple Choice  1 points    
   Which of the following is NOT an element of controlling the marketing plan as discussed in class?  
@  Establishing the goals  
 Measuring actual performance  
 Comparing actual to planned performance  
 Making necessary changes  
 All of the above are elements of controlling the marketing plan  
 


  Question 6    Multiple Choice  1 points    
   Bruce is designing an introductory marketing class. He wants a class where students learn a lot about marketing and are satisfied with the educational experience. Which of the following would be an outcome measure for the class?  
  The number of students who attend class on the first day  
 the number of students who attend class on the last day  
@ The college of business class evaluation results  
 The number of times each student takes mini test 6.  
 


  Question 7    Multiple Choice  1 points    
   A _____ is a comprehensive review of a firm's marketing function, and it is used to determine if a firm can improve its marketing programs. It can also give feedback on specific marketing plans.  
  value analysis  
 SWOT analysis  
 profit-growth matrix  
 marketing plan  
@ market audit  
 


  Question 8    Multiple Choice  1 points    
   In a _____, a firm gives another firm the right to produce and market its product in a specific country or region in return for royalties.  
@  licensing agreement  
 franchising agreement  
 joint venture  
 marketing mix alliance  
 direct investment  
 


  Question 9    Multiple Choice  1 points    
   In the British Isles, Guinness brewery was given the right to produce and market Budweiser beer by Anheuser-Busch, its U.S.-brand owner. Since Guinness pays Anheuser-Busch royalties for the use of its brand, this would be an example of a:  
@  licensing agreement  
 franchising agreement  
 joint venture  
 marketing mix alliance  
 direct investment by Anheuser-Busch  
 


  Question 10    Multiple Choice  1 points    
   Bruce is designing an introductory marketing class. He wants a class where students learn a lot about marketing and are satisfied with the educational experience. Which of the following would be a PROCESS measure for the class?  
  The scores for the final examination.  
@ The percentage of students who take the practice tests.  
 The change in the class evaluations from one semester to the next.  
 The percentage of students who report they are highly satisfied with the class on an exit survey. 
 
===========

Which of the following is a form of protectionism?  
  disintermediation  
 comparative advertising  
 direct investment opportunities  
 parliamentarian laws  
@ local content rules  
 


  Question 2    Multiple Choice  1 points    
   According to the instructor, what is a major weakness in strategic plans?  
  Strategic plans can only be developed for long-range time horizons.  
@ There is no clear linkage between long-range strategic goals and short-term tactical goals  
 There are no outcome measures associated with strategic plans.  
 Strategic goals are not measurable.  
 Strategic plans are only as good as the people who create them.  
 


  Question 3    Multiple Choice  1 points    
   Why is it important to measure market share relative to the competition?  
@  Because the company with the largest market share usually has an advantage in the marketplace.  
 Companies with the largest market share tend to employ "push" strategies.  
 Companies with the largest market share tend to employ "pull" strategies.  
 Because if you have a smaller market share than your competitor, it is important to allocate advertising dollars more efficiently.  
 


  Question 4    Multiple Choice  1 points    
   In February 2000, the United States imposed a tax on imported steel in an effort to protect about 5,000 U.S. jobs. Another name for this levied tax is a(n):  
  embargo  
 gray tax  
@ tariff  
 import quota  
 boycott  
 


  Question 5    Multiple Choice  1 points    
   What is one of the difficulties in using cost data as a marketing metric?  
@  It is sometimes hard to decide what a cost is.  
 Over-emphasis on cost minimizes the importance of revenue.  
 Fixed costs are the same regardless of how many units are sold.  
 It is easy to allocate advertising expenses to unit sales.  
 


  Question 6    Multiple Choice  1 points    
   According to your instructor, what is the purpose of a contingency plan?  
  To tell you whether or not to proceed to the next stage of the plan when you reach a critical checkpoint.  
 To let you minimize your losses if things go wrong with the plan.  
 To let you maximize your profits if things go better than expected with the plan.  
 To provide the basis for continuous improvement for iterative marketing processes.  
@ All of the above.  
 


  Question 7    Multiple Choice  1 points    
   _____ allow companies easy access to new markets, especially because their partnerships often bring with them preferential treatment in the partner's home country.  
  Transactional relationships  
 Product-oriented relationships  
 Exchange relationships  
@ Strategic alliances  
 Expropriation agreements  
 


  Question 8    Multiple Choice  1 points    
   Which of the following statements about traditional organizational structure is true?  
  Functional silos are eliminated.  
@ There are clear lines of authority.  
 Marketing is a function of the sales department.  
 Traditional organization structure is necessary for very large organizations.  
 


  Question 9    Multiple Choice  1 points    
   Which of the following is an advantage of a multinational corporation?  
  Exempt from local regulations.  
@ The ability to locate production facilities where there is an economic advantage.  
 The ability to quickly adapt to changing local conditions.  
 Immunity from expropriation or nationalization.  
 


  Question 10    Multiple Choice  1 points    
   Bruce is designing an introductory marketing class. He wants a class where students learn a lot about marketing and are satisfied with the educational experience. Which of the following would be a PROCESS measure for the class?  
  The score on a marketing test given on the first day of class.  
 The score on a marketing test given on the last day of class.  
 the difference between the marketing test scores from the first and the last day of class  
@ The percentage of students who buy the textbook required for the class. 
 
===========

What is one of the difficulties in using cost data as a marketing metric?  
@  It is sometimes hard to decide what a cost is.  
 Over-emphasis on cost minimizes the importance of revenue.  
 Fixed costs are the same regardless of how many units are sold.  
 It is easy to allocate advertising expenses to unit sales.  
 


  Question 2    Multiple Choice  1 points    
   According to your instructor, what are the two ways we measure sales volume.  
  Dollars, donuts.  
@ Dollars, units.  
 Percentage, increase  
 Increase, decrease  
 Market share, gross sales  
 


  Question 3    Multiple Choice  1 points    
   Prior to the creation of the European Union (EU), the German government required all beer manufacturers (foreign and domestic) who wished to sell to the German market to contain a high percentage of German-grown hops. This law was an example of a(n):  
  environmental protection law  
 80/20 ruling  
 import quota  
 export quota  
@ local content rule  
 


  Question 4    Multiple Choice  1 points    
   In February 2000, the United States imposed a tax on imported steel in an effort to protect about 5,000 U.S. jobs. Another name for this levied tax is a(n):  
  embargo  
 gray tax  
@ tariff  
 import quota  
 boycott  
 


  Question 5    Multiple Choice  1 points    
   A(n) _____ is an extreme form of quota that prevents the importing of specified foreign goods completely.  
  gray tax  
 discount  
 barrier to entry  
 supply-based infrastructure  
@ embargo  
 


  Question 6    Multiple Choice  1 points    
   Bruce is designing an introductory marketing class. He wants a class where students learn a lot about marketing and are satisfied with the educational experience. Which of the following would be a PROCESS measure for the class?  
  The scores for the final examination.  
@ The percentage of students who take the practice tests.  
 The change in the class evaluations from one semester to the next.  
 The percentage of students who report they are highly satisfied with the class on an exit survey.  
 


  Question 7    Multiple Choice  1 points    
   Which of the following statements about world trade is true?  
  World trade refers to the flow of goods and services among different countries.  
 World trade activity is steadily increasing year by year.  
 World trade refers to the value of all exports and imports of the world's nations.  
 Not all countries participate equally in world trade.  
@ All of the above statements about world trade are true.  
 


  Question 8    Multiple Choice  1 points    
   Which of the following is a consequence of a trade surplus?  
  Negative economic impact  
 Currency weakens  
@ Increased popularity for local politicians  
 Cash surplus equals cash sdeficit.  
 


  Question 9    Multiple Choice  1 points    
   Several Arab nations completely prohibit the importation of Coca-Cola because it maintains distributors in Israel. The Arab nations have instituted a(n) _____ against Coke.  
  gray tax  
@ embargo  
 barrier to entry  
 supply-based infrastructure  
 export prohibition  
 


  Question 10    Multiple Choice  1 points    
   According to your instructor, what is the purpose of a contingency plan?  
  To tell you whether or not to proceed to the next stage of the plan when you reach a critical checkpoint.  
 To let you minimize your losses if things go wrong with the plan.  
 To let you maximize your profits if things go better than expected with the plan.  
 To provide the basis for continuous improvement for iterative marketing processes.  
@ All of the above.  
 
================

Firms seeking a deeper commitment to a foreign market than is possible with exporting or contractual agreements, develop _____ with one or more domestic firms in the target market.  
  transactional relationships  
 product-oriented relationships  
 exchange relationships  
@ strategic alliances  
 expropriation agreements  
 


  Question 2    Multiple Choice  1 points    
   All of the following are barriers to successful exporting EXCEPT:  
  tariffs  
 local content laws  
 import quotas  
@ export merchants  
 economic sanctions  
 


  Question 3    Multiple Choice  1 points    
   What is one of the difficulties in using cost data as a marketing metric?  
@ It is sometimes hard to decide what a cost is.  
 Over-emphasis on cost minimizes the importance of revenue.  
 Fixed costs are the same regardless of how many units are sold.  
 It is easy to allocate advertising expenses to unit sales.  
 


  Question 4    Multiple Choice  1 points    
   What do we mean in business when we speak of a "worst case scenario"  
  A situation where everything goes wrong.  
 A forecast that includes some variability to take allow for uncertainty.  
 A forecast that is discounted to compensate for the natural optimism of people in the planning process.  
@ A forecast that is based on the low-end of all the assumptions and forecasts evaluated as part of the decision.  
 


  Question 5    Multiple Choice  1 points    
   Which of the following statements about budgeting is consistent with the information presented in class?  
  The budget is determined at the corporate level and passed down to the marketing manager.  
 The budget is determined by the marketing manager and passed up to corporate.  
 The budget is based on negotiations between corporate and the marketing manager  
@ All of the above are true  
 


  Question 6    Multiple Choice  1 points    
   The North American Free Trade Agreement (NAFTA):  
  is the world's smallest economic community  
 will never be expanded to include other members  
@ has only three members  
 is one of the few such organizations that has been free of controversy  
 is accurately described by all of the above  
 


  Question 7    Multiple Choice  1 points    
   A market audit:  
  typically does not involve nonmarketing personnel in any way  
 is necessarily subjective because it is prepared by the department being audited  
 cannot provide feedback on specific marketing plans  
@ is used to determine if a firm can improve its marketing programs  
 is all of the above  
 


  Question 8    Multiple Choice  1 points    
   If tactical planning is to succeed, each "management unit" should know:  
@  The deadline for achieving the assigned task.  
 Whether or not the organization is organized along functional or traditional lines  
 The budget allocated to all functional areas in the organization.  
 Whether a specific metric is a process measure or an outcome measure.  
 


  Question 9    Multiple Choice  1 points    
   To enter the European market, Starbucks joined in a cooperative venture with Bon Appetit Group A.G. in Switzerland. Bon Appetit has the recognized brand name and Starbucks has the product and the expertise to run coffeehouses. This is an example of a(n):  
  transactional relationship  
 product-oriented relationship  
 exchange relationship  
@ strategic alliance  
 expropriation agreement  
 


  Question 10    Multiple Choice  1 points    
   According to your instructor, what is the difference between a process measure and an outcome measure?  
  A process measure is a metric, while an outcome measure is a measurement.  
 There is no difference between a process measure and an outcome measure.  
 Process measures occur at the beginning of a planning period, outcome measures occur at the end of the planning period.  
 Process measures deal with indirect costs, outcome measures deal with direct costs.  
@ Process measures tell you whether or not the elements of the plan are working, outcome measures tell you whether or not the plan as a whole is succeeding.  
 
=============

 A _____ is a comprehensive review of a firm's marketing function, and it is used to determine if a firm can improve its marketing programs. It can also give feedback on specific marketing plans.  
  value analysis  
 SWOT analysis  
 profit-growth matrix  
 marketing plan  
@ market audit  
 


  Question 2    Multiple Choice  1 points    
   Which of the following statements about traditional organizational structure is true?  
  Functional silos are eliminated.  
@ There are clear lines of authority.  
 Marketing is a function of the sales department.  
 Traditional organization structure is necessary for very large organizations.  
 


  Question 3    Multiple Choice  1 points    
   According to the instructor, what is a major weakness in strategic plans?  
  Strategic plans can only be developed for long-range time horizons.  
@ There is no clear linkage between long-range strategic goals and short-term tactical goals  
 There are no outcome measures associated with strategic plans.  
 Strategic goals are not measurable.  
 Strategic plans are only as good as the people who create them.  
 


  Question 4    Multiple Choice  1 points    
   Which of the following statements about budgeting is consistent with the information presented in class?  
  The budget is determined at the corporate level and passed down to the marketing manager.  
 The budget is determined by the marketing manager and passed up to corporate.  
 The budget is based on negotiations between corporate and the marketing manager  
@ All of the above are true  
 


  Question 5    Multiple Choice  1 points    
   According to your instructor, a measurement should be:  
@  Objective, quantifiable, timely and relevant.  
 Something you can express in a short sentence or "sound bite"  
 Something that includes all of the information available at the time the measurement is made.  
 Something that includes all of the information available at the time the decision is made.  
 Something developed at the time the decision was made that predicts the information available at the time the measurement is made.  
 


  Question 6    Multiple Choice  1 points    
   Which of the following is an example of a proactive approach to international marketing?  
  Starbucks decides to enter Europe because there are no more good locations in the United States  
@ Gillette corporation creates a competitive advantage by investing in the high-volume machinery needed to supply the world with razor blades.  
 A pharmaceutical company sells drugs that have passed their expiration date in mexico.  
 Bruceco hires an export merchant to see if there is a market for excess production in Bulgaria.  
 


  Question 7    Multiple Choice  1 points    
   Capital Radio PLC, the United Kingdom's biggest commercial radio firm announced in 2002, that it would partner with Walt Disney Co. to create a radio network for English-speaking children under the age of 16. The new company created by this agreement is an example of a(n):  
@  joint venture  
 tactical alliance  
 exchange relationship  
 transactional relationship  
 networking arrangement  
 


  Question 8    Multiple Choice  1 points    
   Firms seeking a deeper commitment to a foreign market than is possible with exporting or contractual agreements, develop _____ with one or more domestic firms in the target market.  
  transactional relationships  
 product-oriented relationships  
 exchange relationships  
@ strategic alliances  
 expropriation agreements  
 


  Question 9    Multiple Choice  1 points    
   Bruce is designing an introductory marketing class. He wants a class where students learn a lot about marketing and are satisfied with the educational experience. Which of the following would be a PROCESS measure for the class?  
  The scores for the final examination.  
@ The percentage of students who take the practice tests.  
 The change in the class evaluations from one semester to the next.  
 The percentage of students who report they are highly satisfied with the class on an exit survey.  
 


  Question 10    Multiple Choice  1 points    
   Which of the following is a consequence of a trade deficit?  
  Currency strengthens  
@ Negative impact on the economy  
 Gray market dumping  
 Increased popularity for local politicians  
 
==============

How does knowledge of the 80-20 rule affect the way we use market share data?  
  In order to be reliable, a market share estimate has to be at least 80% accurate.  
 The goal is to have the largest market share in at least 20% of your target markets.  
@ It is important to look at market share among the heavy users within a market segment.  
 We know that 80% or current customers will be loyal.  
 
 
 
 Wal-Mart's purchase of 74 superstores from the Spar supermarket chain in Germany dramatically increased the presence of the U.S. retail giant in Europe. Which market entry strategy did Wal-Mart use?  
  joint venture  
 tactical alliance  
 direct exporting  
@ direct investment  
 indirect exporting  
 

 U.S. sugar import quotas have existed for over 50 years and preserve about half of the U.S. sugar market for domestic producers. Why does the U.S. continue to have these quotas?  
@  to reduce competition for its domestic sugar industry  
 to increase the cost of sugar  
 to control the supply and demand for sugar  
 to support a mass market for sugar  
 to do all of the above  
 
===========

You just received a report that sales for the current quarter were $27 million. What is the first question you should ask yourself in order to understand use the report as a metric?  
  How many units is that?  
 Can I believe the person who wrote the report.  
 Is that American dollars or Canadian dollars?  
@ Is it good or bad?  
 
One difficulty associated with a strategic alliance is the:  
  absence of common goals  
 absence of preferential treatment by targeted countries  
@ problems associated with the blending of two different corporate cultures  
 elimination of all economies of scale  
 elimination of all operational goals 

=============

 According to your instructor, what two questions do you want to answer when comparing actual results to planned performance?  
  Was communication accurate, did everyone in the organization have a voice?  
 Did we make a profit, is it getting better or worse?  
 Did we accomplish our objectives, what things could we have done better?  
 Did we succeed, did people in upper management notice our success?  
 


Question 1    Multiple Choice  0 of 1 point(s)    
  The ViVi brand is a line of designer clothing for pregnant businesswomen. If its owner believes that professional women in other countries have a need for conservative maternity wear, the small company could hire a(n) _____ to locate and negotiate with buyers in other countries.  
 Your Answer:  [None Given]  
Correct Answer:  export merchant  
 


  Question 2    Multiple Choice  0 of 1 point(s)    
  Wal-Mart's purchase of 74 superstores from the Spar supermarket chain in Germany dramatically increased the presence of the U.S. retail giant in Europe. Which market entry strategy did Wal-Mart use?  
 Your Answer:  [None Given]  
Correct Answer:  direct investment  
 


  Question 3    Multiple Choice  0 of 1 point(s)    
  Bruce is designing an introductory marketing class. He wants a class where students learn a lot about marketing and are satisfied with the educational experience. Which of the following would be an outcome measure for the class?  
 Your Answer:  [None Given]  
Correct Answer:  The college of business class evaluation results  
 


  Question 4    Multiple Choice  0 of 1 point(s)    
  Prior to the creation of the European Union (EU), the German government required all beer manufacturers (foreign and domestic) who wished to sell to the German market to contain a high percentage of German-grown hops. This law was an example of a(n):  
 Your Answer:  [None Given]  
Correct Answer:  local content rule  
 


  Question 5    Multiple Choice  0 of 1 point(s)    
  If tactical planning is to succeed, each "management unit" should know:  
 Your Answer:  [None Given]  
Correct Answer:  What the unit is expected to do in order to achieve the goal.  
 


  Question 6    Multiple Choice  0 of 1 point(s)    
  Which of the following statements about the European Union (EU) is true?  
 Your Answer:  [None Given]  
Correct Answer:  The EU is an example of an economic community.  
 


  Question 7    Multiple Choice  0 of 1 point(s)    
  What do we mean when we say empowerment is a trend in organization structure?  
 Your Answer:  [None Given]  
Correct Answer:  Decision-making authority is located close to the customer  
 


  Question 8    Multiple Choice  0 of 1 point(s)    
  A market audit:  
 Your Answer:  [None Given]  
Correct Answer:  is used to determine if a firm can improve its marketing programs  
 


  Question 9    Multiple Choice  0 of 1 point(s)    
  You just received a report that sales for the current quarter were $27 million. What is the first question you should ask yourself in order to understand use the report as a metric?  
 Your Answer:  [None Given]  
Correct Answer:  Is it good or bad?  
 


  Question 10    Multiple Choice  0 of 1 point(s)    
  In February 2000, the United States imposed a tax on imported steel in an effort to protect about 5,000 U.S. jobs. Another name for this levied tax is a(n):  
 Your Answer:  [None Given]  
Correct Answer:  tariff  
 


 Question 10    Multiple Choice  0 of 1 point(s)    
  Djibouti is located on the eastern coast of Africa where it is strategically located near world's busiest shipping lanes. Its port and railroad system provides easy access to Ethiopia and other landlocked African nations in the region. According to Michael Porter, the source of any competitive advantage a distribution company could have from setting up operation in the port of Djibouti is:  
 Your Answer:  [None Given] 
Correct Answer:  factor conditions  
 

 In many cultures, the act of adultery is a(n) _____, and a person found guilty of adultery can be either ostracized or in some cases executed.  
  nationalized convention  
 ethical dilemma  
@ more  
 accultural edict  
 expropriated rule  
 


  Question 2    Multiple Choice  1 points    
   In some situations, internal pressures prompt governments to take over the operations of foreign companies doing business within their national borders. This process is called _____ when the domestic government reimburses a foreign company for its assets after the takeover.  
  divestment  
 diversification  
@ nationalization  
 expropriation  
 national aggregation  
 


  Question 3    Multiple Choice  1 points    
   _____ consumers believe that the purchase of imported products is wrong, hurt domestic industry, and cause domestic unemployment.  
@  Ethnocentric  
 Expatriated  
 Semiotic  
 Domesticated  
 Naturalized  
 


  Question 4    Multiple Choice  1 points    
   Expropriation occurs when:  
  domestic companies export more than they sell within national borders  
 a government reimburses a foreign company that has lost facilities due to natural causes  
 a government refuses to allow its businesses to engage in exporting  
@ a government takes over foreign operations and provides the seized operation with no reimbursements  
 a nation imports more than it exports  
 


  Question 5    Multiple Choice  1 points    
   The same-tasting Johnnie Walker whiskey that is available at a bar in the United States can also be purchased in many locations around the world. The distillery that produces Johnnie Walker whiskey uses a(n) _____ strategy.  
  product adaptation  
 localization  
@ straight extension  
 product invention  
 ethnocentric  
 


  Question 6    Multiple Choice  1 points    
   A _____ exists when an unauthorized party imports products and then sells them for a fraction of the price of authorized products.  
@  gray market  
 dual distribution system  
 disintermediation system  
 black market  
 dumping ground  
 


  Question 7    Multiple Choice  1 points    
   Krispy Kreme Doughnuts is a popular doughnut chain. When it opened Krispy Kreme outlets in Canada, it used the same marketing mix that has proven successful in the United States where the company began. Krispy Kreme has chosen a to use _____ in its global marketing strategy.  
  localization  
 ethnocentrism  
 expropriation  
 nationalism  
@ standardization  
 


  Question 8    Multiple Choice  1 points    
   Advocates of _____ can point to the fact that the production company that developed Who Wants to Be a Millionaire exported the same show format to more than 50 different countries around the world.  
  localization  
 ethnocentrism  
 expropriation  
 nationalism  
@ standardization  
 


  Question 9    Multiple Choice  1 points    
   Liberia, Vietnam, and Mexico have a visible middle class that is largely composed of entrepreneurs who are working hard to run successful small businesses. In other words, these three countries are examples of:  
  preindustrial countries  
 less developed countries (LDCs)  
 industrializing countries  
@ developing countries  
 traditional societies  
 


  Question 10    Multiple Choice  1 points    
   To successfully market the Corvette to the European market, General Motors had to make the automobile shorter and narrower. GM chose to use a _____ strategy with the Corvette.  
@ product adaptation  
 expropriation  
 straight extension  
 product invention  
 domestication  
 

=============

 The maker of Jell-O gelatin tried to offer the same product in England that is offered in the United States but failed because British consumers do not buy their gelatin in a powder format. To sell in England, the gelatin manufacturer had to use a _____ strategy.  
@  product adaptation  
 expropriation  
 straight extension  
 product invention  
 domestication  
 


  Question 2    Multiple Choice  1 points    
   Advocates of localization believe each culture is unique and has its own _____, a distinctive set of behavioral and personality characteristics.  
  opinion leadership  
 reference group  
 consumer behavior set  
@ national character  
 self-concept  
 


  Question 3    Multiple Choice  1 points    
   The Foreign Corrupt Practices Act (PCPA):  
  lists which forms of bribery are acceptable for U.S. companies to use  
 protects U.S. citizens from foreign-made products with shoddy workmanship  
 prohibits the use of deceptive pricing practices in the U.S.  
@ bars U.S. businesses from paying bribes to sell overseas  
 eliminated the exploitation of workers by U.S.-based manufacturers and any manufacturers wanting to do business in the U.S  
 


  Question 4    Multiple Choice  1 points    
   Some countries outlaw _____, a practice in which a company prices its products lower than they are priced at home.  
  gray marketing  
 parallel importing  
 black marketing  
@ dumping  
 countertrading  
 


  Question 5    Multiple Choice  1 points    
   La mordida, schmiergeld, kuroi kiri, and pot-de-vin are all:  
  expropriation processes  
@ forms of bribery  
 means of trading with the European Union (EU)  
 degrees of nationalism  
 European pricing practices  
 


  Question 6    Multiple Choice  1 points    
   France, Italy, and Germany boast sophisticated marketing systems, and are three good examples of:  
  preindustrial countries  
 less developed countries (LDCs)  
 industrializing countries  
@ developed countries  
 traditional societies  
 


  Question 7    Multiple Choice  1 points    
   Estonia is a past member of the Soviet Union. It is a developing country. This means:  
  the nation's wealth is in the hands of a few  
 its use of technology is very limited  
 the nation has an economy based on agriculture  
@ it has a middle class composed primarily of entrepreneurs  
 it has an economy based on providing services  
 


  Question 8    Multiple Choice  1 points    
   McDonald's franchisees in India steers clear of all beef and pork products and use an eggless recipe for their mayonnaise. Instead of the Big Mac, they sell mutton burgers called Maharaja Mac. This adaptation to local taste is a common practice for McDonald's and is called:  
@  localization  
 ethnocentrism  
 expropriation  
 nationalism  
 standardization  
 


  Question 9    Multiple Choice  1 points    
   Krispy Kreme Doughnuts is a popular doughnut chain. When it opened Krispy Kreme outlets in Canada, it used the same marketing mix that has proven successful in the United States where the company began. Krispy Kreme has chosen a to use _____ in its global marketing strategy.  
  localization  
 ethnocentrism  
 expropriation  
 nationalism  
@ standardization  
 


  Question 10    Multiple Choice  1 points    
   U.S. marketing campaigns that stress the desirability of "Made in America" products are more likely to appeal to _____ consumers.  
@  ethnocentric  
 expatriated  
 semiotic  
 domesticated  
 naturalized  
 
=============


Liberia, Vietnam, and Mexico have a visible middle class that is largely composed of entrepreneurs who are working hard to run successful small businesses. In other words, these three countries are examples of:  
  preindustrial countries  
 less developed countries (LDCs)  
 industrializing countries  
@ developing countries  
 traditional societies  
 


  Question 2    Multiple Choice  1 points    
   In some situations, internal pressures prompt governments to take over the operations of foreign companies doing business within their national borders. This process is called _____ when the domestic government reimburses a foreign company for its assets after the takeover.  
  divestment  
 diversification  
@ nationalization  
 expropriation  
 national aggregation  
 


  Question 3    Multiple Choice  1 points    
   U.S. marketing campaigns that stress the desirability of "Made in America" products are more likely to appeal to _____ consumers.  
@  ethnocentric  
 expatriated  
 semiotic  
 domesticated  
 naturalized  
 


  Question 4    Multiple Choice  1 points    
   Some countries outlaw _____, a practice in which a company prices its products lower than they are priced at home.  
  gray marketing  
 parallel importing  
 black marketing  
@ dumping  
 countertrading  
 


  Question 5    Multiple Choice  1 points    
   People who live in _____ cultures tend to attach more importance to personal goals. In this type of culture, people are likely to change memberships when the demands of the group become too costly.  
@  individualist  
 synergistic  
 communistic  
 socialistic  
 collectivist  
 


  Question 6    Multiple Choice  1 points    
   The Foreign Corrupt Practices Act (PCPA):  
  lists which forms of bribery are acceptable for U.S. companies to use  
 protects U.S. citizens from foreign-made products with shoddy workmanship  
 prohibits the use of deceptive pricing practices in the U.S.  
@ bars U.S. businesses from paying bribes to sell overseas  
 eliminated the exploitation of workers by U.S.-based manufacturers and any manufacturers wanting to do business in the U.S  
 


  Question 7    Multiple Choice  1 points    
   Competition for control of the rich resources in the Democratic Republic of Congo (DRC) has led to a war that has killed more than 2.5 million people since 1998 (mostly of disease and malnutrition). In this example, foreign development of the country has been stymied by its _____ environments.  
@  natural and political  
 economic and technological  
 cultural and economic  
 natural, technological, and socioeconomic  
 social, cultural  
 


  Question 8    Multiple Choice  1 points    
   Advocates of localization believe each culture is unique and has its own _____, a distinctive set of behavioral and personality characteristics.  
  opinion leadership  
 reference group  
 consumer behavior set  
@ national character  
 self-concept  
 


  Question 9    Multiple Choice  1 points    
   A less developed country (LDC):  
@  has an economy based on agriculture  
 is by definition landlocked  
 has a visible middle class  
 has a large number of entrepreneurs  
 has an advanced economic infrastructure  
 


  Question 10    Multiple Choice  1 points    
   When Johnnie Walker entered the European market, its manufacturer wanted an ad that was uniquely European. Its ad agency came up with an ad showed a man running with the bulls at Pamplona (an annual event in Spain) and then celebrating his narrow escape from death by drinking a glass of Johnnie Walker. The ad was a total failure in Germany because the Germans consider such activities stupid and reckless while the Spanish see them as brave and daring. The failure of the Johnnie Walker ad to have wide European appeal was largely due to the _____ environment.  
  socioeconomic  
@ cultural  
 technological  
 legal  
 natural  
 

===========

 _____ is the tendency to prefer products or people of one's own culture over those from other countries.  
  Nationalization  
@ Ethnocentricism  
 Domestication  
 Expropriation  
 Democratization  
 


  Question 2    Multiple Choice  1 points    
   The "thumbs up" sign in the United States is a positive sign. However, in Russia and Poland, this gesture has an offensive meaning especially when shown with the palm exposed. To learn how different cultures react to symbols, marketers need to make use of:  
  associative meanings  
 cultural sociology  
 social services  
 linguistics  
@ semiotics  
 


  Question 3    Multiple Choice  1 points    
   The population of Djibouti is approximately 460,700. To serve the communication needs of this nation, there is one Internet service provider, two AM radio stations, two FM radio stations, and one television station. There are only 8,000 telephone lines in the whole country. This is a partial description of Djibouti's:  
  net national product (NNP)  
 economic liquidity  
@ economic infrastructure  
 level of commercialization  
 gross domestic product (GDP)  
 


  Question 4    Multiple Choice  1 points    
   According to one international advertising agency representative, "You can't show a goldfish in a goldfish bowl in Germany because of the national support of animal rights." The agency representative is referring to a(n):  
  internal marketing dilemma  
 cultural aberration  
@ cultural value  
 domestic barrier  
 ethnocentric attitude  
 


  Question 5    Multiple Choice  1 points    
   When U.S.-based Wal-Mart opened its retail stores in Germany, German merchants accused the chain of dumping. In other words, the store was accused of:  
  selling shoddy merchandise  
 maintaining its "made in America" superiority  
 selling no brand name merchandise  
@ selling products at prices lower than it did in its U.S. stores  
 selling counterfeit products  
 


  Question 6    Multiple Choice  1 points    
   The Japanese who give currency as gifts will typically wash the money before making the presentation. This is a(n) _____ with a nation of people who place high importance on cleanliness.  
@  convention  
 expropriated more  
 nationalized edict  
 ethical dilemma  
 customized stimulus/response  
 


  Question 7    Multiple Choice  1 points    
   _____ consumers believe that the purchase of imported products is wrong, hurt domestic industry, and cause domestic unemployment.  
@  Ethnocentric  
 Expatriated  
 Semiotic  
 Domesticated  
 Naturalized  
 


  Question 8    Multiple Choice  1 points    
   To successfully market the Corvette to the European market, General Motors had to make the automobile shorter and narrower. GM chose to use a _____ strategy with the Corvette.  
@  product adaptation  
 expropriation  
 straight extension  
 product invention  
 domestication  
 


  Question 9    Multiple Choice  1 points    
   A less developed country (LDC):  
@  has an economy based on agriculture  
 is by definition landlocked  
 has a visible middle class  
 has a large number of entrepreneurs  
 has an advanced economic infrastructure  
 


  Question 10    Multiple Choice  1 points    
   The maker of Jell-O gelatin tried to offer the same product in England that is offered in the United States but failed because British consumers do not buy their gelatin in a powder format. To sell in England, the gelatin manufacturer had to use a _____ strategy.  
@  product adaptation  
 expropriation  
 straight extension  
 product invention  
 domestication  
 
===========

 In _____, people subordinate their personal goals to those of a stable community.  
  individualist cultures  
 cultural synergies  
 social classes  
@ collectivist cultures  
 democratic cultures  
 

Traditionally, the Afghans have celebrated the coming of Nowruz, or the Persian New Year, and the advent of spring with parades of animals, floats, and people performing the native dance Atan. This form of celebration is an example of a(n):  
@  convention  
 workplace norm  
 environmental more  
 ethical dilemma  
 custom 
 

 The Japanese who give currency as gifts will typically wash the money before making the presentation. This is a(n) _____ with a nation of people who place high importance on cleanliness.  
@  convention  
 expropriated more  
 nationalized edict  
 ethical dilemma  
 customized stimulus/response  
 
================

 The fact people in the U.S. think of Australia when they see pictures of cuddly-looking koala bears would be explained through the study of:  
  associative meanings  
 cultural sociology  
 social services  
 linguistics  
@ semiotics  
 
 The _____ refers to the quality of a country's distribution, financial, and communications systems.  
  net national product (NNP)  
 economic liquidity  
@ economic infrastructure  
 level of commercialization  
 gross domestic product (GDP) 
 

===============

Question 10    Multiple Choice  0 of 1 point(s)    
  The National Cash Register Company had to redevelop its hand-crank-operated cash registers to sell to Latin American and African countries where electricity is a luxury. The company used a _____ strategy to enter those global markets.  
 Your Answer:  straight extension  
Correct Answer:  product invention  
 
~~~~~~~~~~~~~~~~~~~~~~~~~~~~~~~~~~~~~~~
pFIN

Question 8    Multiple Choice  1 of 1 point(s)    
  Often when two people watch the same television programming, only one of them will notice an ad for a specific product, and the other will be oblivious to the ad. _______________ can be used to explain why this happens.  
 Your Answer:  Selective attention  
Correct Answer:  Selective attention  
 
 Question 19    Multiple Choice  0 of 1 point(s)    
  Traffic in major metropolitan areas is too often brought to a standstill after the wreckage has been cleared away because investigators must painstakingly measure skid marks and determine culpability. After a Nashville police chief missed an important meeting because of a wreck investigation, he was pleased to find an article about a company that markets a new high-tech crash investigation system that shortens the time required for these investigations by 50 percent. In which stage of the adoption process is the police chief?  
 Your Answer:  evaluation because he is looking for similar products  
Correct Answer:  interest because he can see how the system will benefit him and the city  
 


  Question 20    Multiple Choice  0 of 1 point(s)    
  The marketing goal for a product in the growth stage of its product life cycle is to:  
 Your Answer:  profitably phase out the product  
Correct Answer:  encourage brand loyalty  
 

 Question 28    Multiple Choice  0 of 1 point(s)    
  Which of the following would be an example of a variable cost for an amusement park?  
 Your Answer:  salary of park manager  
Correct Answer:  rest room supplies  
 
Question 32    Multiple Choice  0 of 1 point(s)    
  The first step in the development of an advertising campaign is:  
 Your Answer:  setting the advertising budget  
Correct Answer:  identifying the target market  
 

 Question 38    Multiple Choice  0 of 1 point(s)    
  What was the point of all of the pictures of retail spaces through the ages the instructor showed in class?  
 Your Answer:  That the design of retail spaces has changed dramatically over the centuries.  
Correct Answer:  That the activity taking place, rather than the historical period determines the structure of a retail space.  
 

Question 48    Multiple Choice  0 of 1 point(s)    
  According to your instructor, what contingency plan should you employ if the worst case scenario for a marketing plan would cause your business to become insolvent?  
 Your Answer:  Plan very carefully to make sure the worst case scenario doesn't happen.  
Correct Answer:  Don't implement the plan.  

Question 4    Multiple Choice  0 of 1 point(s)    
  Amazon.com where Marta can order books online and Waldenbooks, a store at the mall where she can purchase the same books, are both examples of:  
 Your Answer:  buying centers  
Correct Answer:  marketplaces  
 


  Question 5    Multiple Choice  1 of 1 point(s)    
  When a dentist's office manager orders latex gloves, disposable masks, file folders, and complementary toothbrushes to give to patients, he or she is most likely making a:  
 Your Answer:  straight rebuy  
Correct Answer:  straight rebuy  
 

Question 9    Multiple Choice  0 of 1 point(s)    
  Why would a company adopt an undifferentiated targeting strategy when there is so much evidence of the benefits of a differentiated strategy?  
 Your Answer:  to more efficiently focus on its CRM strategy  
Correct Answer:  to take advantage of economies of scale  
 
Question 19    Multiple Choice  0 of 1 point(s)    
  Ore-Ida's development of Funky Fries, microwaveable, single servings of Kool Blue, Cinna-Stick, Crunchy Rings, and Cocoa Crispers seasoned potatoes is an example of a _____ because consumers did not have to learn anything new to prepare and serve the potatoes.  
 Your Answer:  dynamically continuous innovation  
Correct Answer:  continuous innovation  
 
Question 26    Multiple Choice  0 of 1 point(s)    
  When Joe's Coffee Nook raised the price of latté, Joe noticed a substantial change in how many cups of latté he sold daily. When he reduced the price of a cup of latte, his sales of latté increased. From this information, you can assume the demand for cups of latté is:  
 Your Answer:  static  
Correct Answer:  elastic  
 
 Question 37    Multiple Choice  0 of 1 point(s)    
  When Harris Pool Chemical Co. purchased 144 buckets of chlorine tablets over the Internet from ChemTech, the company placed the order with the manufacturer and arranged for its transportation by truck to the store. ChemTech is an example of a:  
 Your Answer:  piggyback marketer  
Correct Answer:  wholesaling intermediary  
 


  Question 38    Multiple Choice  1 of 1 point(s)    
  A _______________ is how a target market perceives the store.  
 Your Answer:  store image  
Correct Answer:  store image  
 


  Question 39    Multiple Choice  0 of 1 point(s)    
  Which of the following modes of transportation would most likely be used to haul several tons of lettuce from California where it was grown to markets in New England?  
 Your Answer:  birdyback  
Correct Answer:  piggyback  
 


  Question 40    Multiple Choice  0 of 1 point(s)    
  A shoe department carries more than 55 different styles of white sandals. This is a description of the shoe department's:  
 Your Answer:  merchandise breadth  
Correct Answer:  merchandise depth  
 


  Question 41    Multiple Choice  1 of 1 point(s)    
  Changes in demographics reflect changes in the _____ environment.  
 Your Answer:  sociocultural  
Correct Answer:  sociocultural  
 


  Question 42    Multiple Choice  0 of 1 point(s)    
  Some Web sites are well-organized, have the information visitors are most likely to need readily available, and are easy to navigate. These well-designed Web sites realize customers want information that is convenient for them and illustrate how:  
 Your Answer:  gray marketing can influence product design  
Correct Answer:  quality concerns can influence marketing communications  
 

Question 45    Multiple Choice  0 of 1 point(s)    
  Which of the following is an example of a proactive approach to international marketing?  
 Your Answer:  Starbucks decides to enter Europe because there are no more good locations in the United States  
Correct Answer:  Gillette corporation creates a competitive advantage by investing in the high-volume machinery needed to supply the world with razor blades.  

 Question 10    Multiple Choice  0 of 1 point(s)    
  According to your instructor, what is the first step you need to take in order to develop a positioning strategy?  
 Your Answer:  Figure out where you were a year ago  
Correct Answer:  Figure out where you are now  
 
Question 13    Multiple Choice  0 of 1 point(s)    
  An independent grocer that wanted to know if cocktail and tartar sauce would sell better if the two products were moved from the salad dressing section of the store to a display next to the seafood selection could use _____ research.  
 Your Answer:  discriminating  
Correct Answer:  causal  
 
 Question 17    Multiple Choice  0 of 1 point(s)    
  The final stage in the product life cycle is:  
 Your Answer:  maturity  
Correct Answer:  decline  
 
 Question 26    Multiple Choice  0 of 1 point(s)    
  When operated at capacity, a company that markets silk-screened t-shirts can produce 75,000 shirts a year. Its variable and fixed costs are covered if each shirt is sold for $10. One year, the company found it could only sell 60,000 shirts at $10. To remain at full-production capacity, it decided to sell an additional 15,000 shirts for $8. What pricing method did the silk-screener use?  
 Your Answer:  capacity management  
Correct Answer:  price-floor pricing  
 
Question 27    Multiple Choice  1 of 1 point(s)    
  Which of the following is NOT a type of pricing objective?  
 Your Answer:  break-even  
Correct Answer:  break-even  
 
  Question 40    Multiple Choice  0 of 1 point(s)    
  _____ distribution is appropriate when demand is so large that exclusive distribution is inadequate, but selling costs, service requirements, or other factors make intensive distribution a poor match to many retailers' capabilities.  
 Your Answer:  [None Given]  
Correct Answer:  Selective  
 

Question 8    Multiple Choice  0 of 1 point(s)    
  Business buyers often develop _____, written descriptions of the quality, size, weight, color, features, quantity, training, warranty, service terms, and delivery requirements for the purchase.  
 Your Answer:  FAB lists  
Correct Answer:  product specifications  
 
Question 27    Multiple Choice  0 of 1 point(s)    
  Movie theaters typically have different ticket prices for children, adults, and senior citizens. This is a legal form of:  
 Your Answer:  psychological pricing  
Correct Answer:  price discrimination  
 
Question 41    Multiple Choice  0 of 1 point(s)    
  ISO 9000:  
 Your Answer:  concentrates on environmental management  
Correct Answer:  sets voluntary standards for quality management  
 

    Source: geocities.com/duehing